You are on page 1of 346

1

MANUAL PARA EXAMEN NACIONAL


DE RESIDENCIAS MDICAS
http://booksmedicos.org
2
3


MANUAL PARA EXAMEN NACIONAL
DE RESIDENCIAS MDICAS


Carlos Gonzlez Parra




















BENEMRITA UNIVERSIDAD AUTNOMA DE PUEBLA
Direccin General de Fomento Editorial

4


BENEMRITA UNIVERSIDAD AUTNOMA DE PUEBLA
Enrique Agera Ibez
Rector
Armando Valerdi y Rojas
Secretario General
Lilia Cedillo Ramrez
Vicerrectora de Extensin y Difusin de la Cultura
Ricardo Escrcega Mndez
Director Editorial














Primera edicin, 2005
ISBN: 968 863 855 2

Benemrita Universidad Autnoma de Puebla
Direccin General de Fomento Editorial
2 Norte 1404
Tel. 2 46 85 59
Puebla, Pue.
Miembro de la Cmara de la Industria Editorial Mxicana, registro 3087

Impreso y hecho en Mxico
Printed and made in Mexico


5
PRESENTACIN








Por razones muy diversas, el ingreso a la especializacin mdica es un hecho
trascendente en la vida de gran nmero de mdicos que adoptan esa opcin
vocacional, ya que les permite ampliar, diversificar y facilitar las posibilida-
des de desarrollo en el ejercicio de su profesin y les proporciona otros bene-
ficios de carcter social y econmico. De ah que miles de egresados de las
facultades y escuelas de Medicina de nuestro pas y algunos cientos de gra-
duados fuera de Mxico decidan participar en el Examen Nacional para As-
pirantes Mexicanos y Extranjeros a Residencias Mdicas que anualmente
efecta el Comit de Enseanza de Posgrado y Educacin Continua (CEPEC)
de la Comisin Interinstitucional para la Formacin de Recursos Humanos
para la Salud (CIFRHS).
Ante tan alta demanda, el propio CEPEC ha procurado mejorar cada vez
ms los diferentes aspectos que abarca el proceso de seleccin, con la finali-
dad de hacer ms expeditos los trmites administrativos, evitar largos y fre-
cuentes desplazamientos y asegurar la confiabilidad de los resultados. Para
cumplir con este propsito de la mejor manera posible, la Secretara Tcnica
de la CIFRHS aplica peridicamente encuestas a distintos grupos representati-
vos del universo de aspirantes para conocer sus opiniones, puntos de vista y
aportaciones o sugerencias, derivados tanto de sus propias necesidades o ex-
pectativas como de las situaciones que ms les afectan.
En la encuesta que se han aplicado para conocer la manera en que los m-
dicos se preparan para presentar el Examen Nacional las respuestas obteni-
das han resultado ser muy variadas: algunas, muy escuetas; en tanto que
otras, la mayora, han sido bastante objetivas y han expresado con claridad
las ideas y preocupaciones de los aspirantes. Sin embargo, en la encuesta ms
6
reciente apareci una respuesta inslita, sorpresiva y sorprendente, extraor-
dinaria por el valor de su contenido y el esfuerzo implcito: un libro! En efec-
to, el Dr. Carlos Gonzlez Parra no se limit a responder el cuestionario,
sino que generosamente quiso brindar el fruto de su propia experiencia y
proponer a sus compaeros un mtodo y una gua que a l le dieron resulta-
dos excelentes. As se configur este manual que, como un reconocimiento a
la nobleza idealista al esfuerzo y la dedicacin del doctor Gonzlez, el CEPEC
y la Secretara Tcnica de la CIFRHS ponen a la disposicin de quienes deseen
emprender el camino de la especializacin en alguna rama de la Medicina.
El manual comprende dos partes. La primera est dividida en tres captu-
los. En el denominado captulo 1, el autor analiza la situacin que enfrentan
quienes deciden presentar el Examen Nacional y con un lenguaje amistoso,
casi coloquial, expone la necesidad de asumir una metodologa de estudio
con el fin de evitar que se emprendan faenas desordenadas y encauzar siste-
mticamente el empeo laborioso que se requiere para prepararse en forma
adecuada.
Despus de un repaso breve de ciertos aspectos de la teora del conoci-
miento, as como de la teora del aprendizaje y de sus bases neurofisiolgicas
y psicolgicas, en el captulo 2 el doctor Gonzlez propone una tcnica de es-
tudio sencilla y prctica e invita a seguirla a quienes estn por experimentar
la prueba que l pas con xito.
La segunda parte del manual est constituido por un largo cuestionario
que, por medio de preguntas abiertas, conduce a la revisin de temas y aspec-
tos sobresalientes de 19 especialidades mdicas, quirrgicas y medicoquirr-
gicas. Las respuestas son concretas y breves; slo cuando el tema lo requiere,
son un poco ms extensas, pues explican en forma resumida los conceptos o
puntos bsicos que se analizan.
El mrito de la creacin y elaboracin de este manual pertenece plenamen-
te al doctor Gonzlez Parra; tambin es suya la responsabilidad de su conte-
nido. La informacin que ofrece est basada en la revisin cuidadosa y
detallada de la bibliografa que incluye en las pginas finales del documento.
Algunas respuestas podran parecer incompletas o imprecisas; sin embargo,
todas son plausibles y estimulan al lector a consultar obras ms extensas o li-
bros especializados.
7
El valor intrnseco de este trabajo, la fresca espontaneidad que le dio ori-
gen, lo prctico de la tcnica de estudio que propone, la informacin impor-
tante que contiene, la noble intencin que lo inspir, el singular y encomiable
esfuerzo de su autor y el sentimiento de solidaridad que lo anim a llevarlo a
cabo son las razones principales por las que el CEPEC y la Secretara Tcnica
de la CIFRHS decidieron editar y difundir este manual con el propsito de
ofrecerlo a los futuros candidatos a la especializacin como un auxiliar di-
dctico til que los impulse a prepararse mejor para superar la prueba del
Examen Nacional.
El CEPEC y la Secretara Tcnica de la CIFRHS agradecen profundamente
al doctor Carlos Gonzlez Parra su valiosa iniciativa y su contribucin desin-
teresada en apoyo de sus colegas, al mismo tiempo que hacen votos por su
xito como mdico residente y futuro especialista. Asimismo, expresan su
deseo de que al poner este manual en manos de sus destinatarios, les ayude a
lograr los altos objetivos que se han propuesto alcanzar.


Dr. Julio M. Cacho Salazar
Secretario Tcnico de la CIFRHS


8
9














PARTE I

10
11
CAPTULO 1

INTRODUCCIN








Estudios realizados con atletas olmpicos, msicos de nivel mundial y grandes maestros
de ajedrez demuestran que el rasgo que los une es la capacidad de motivarse ellos mismos
para llevar a cabo una rutina de entrenamiento implacable.
Motivacin y desempeo de las lites: Anders Ericson.
Expert performance: its structure and acquisition. Ameri-
can Psicologist; agosto de 1994.

Es claro que para alcanzar logros mayores en cualquiera de las disciplinas es determi-
nante la edad de inicio. No es lo mismo hablar de un atleta o de un violinista que co-
mienza su entrenamiento a los cinco aos que otros que lo hagan a los diez aos. El
inicio temprano ofrece una ventaja de por vida: los mejores alumnos de violn de la me-
jor academia de msica de Berln, todos ellos a principio de la veintena, haban dedica-
do diez mil horas de su vida a la prctica, mientras los alumnos de segundo nivel haban
alcanzado un promedio de unas siete mil quinientas horas.
Lo que al parecer separa a aquellos que logran un nivel competitivo ms elevado de
aquellos que poseen una capacidad aproximadamente igual, es el grado en que, tras un
inicio temprano, pueden perseguir durante aos y aos una ardua rutina de entrena-
miento. Y esa obstinacin depende de los rasgos emocionales el entusiasmo y la persis-
tencia ante los contratiempos por encima de todo lo dems.
La inteligencia emocional
DANIEL GOLEMAN

12
ENARM son las siglas para el Examen Nacional de Aspirantes a Residencias
Mdicas. Es un examen que se aplica en el mbito nacional a todos los mdi-
cos generales o estudiantes de Medicina en servicio social que desean realizar
una especialidad mdica dentro de la Repblica. Se compone de 700 reactivos,
dentro del los cuales 600 evalan conocimientos mdicos generales y confor-
man 90% de la calificacin y 100 que evalan tu capacidad para comprender la
literatura mdica en ingls y conforman 10% de la calificacin.
Las estadsticas siempre impactan. En este caso no es para menos. En la
versin XXVI del ENARM, que se aplic los das 21 y 22 de septiembre de
2002, el nmero de mdicos inscritos fue de 20,656, de los cuales 20,323 fue-
ron mexicanos y 333 extranjeros provenientes de 18 pases. Las plazas ofre-
cidas ese ao correspondieron a 22 especialidades de entrada directa dentro
de las cuales podemos encontrar desde Medicina Nuclear hasta Pediatra, pa-
sando por las medicoquirrgicas y Medicina Legal y del Trabajo. El nmero
de plazas por las que compitieron los aspirantes en 2002 fue de 4,470 para
mdicos nacionales y 90 para extranjeros.
Analizando hasta este punto los datos es interesante obsevar que cada uno
de los 20,656 aspirantes tena una probabilidad de 21.6% de entrar; es decir,
de aproximadamente 5 a 1. En otras palabras: de cada grupo de 10 amigos
que se presentaron, slo dos fueron aceptados. Estos clculos son generales,
pero varan de acuerdo con la especialidad elegida; por ejemplo, en 2001 la
especialidad de Medicina del Deporte tuvo un ndice de probabilidad de
11.28 a uno de ser aceptado, seguido de la de Medicina de Rehabilitacin
con un ndice de 10.02. Por otra parte, las especialidades de Medicina del
Trabajo y Anestesiologa tuvieron ndices tan favorables como 3.52 y 2.92,
respectivamente.
Ao con ao las tendencias no varan y podemos observar, por una parte,
que el nmero de mdicos aspirantes aumenta y que el nmero de plazas
ofrecidas permanece relativamente igual. Asimismo, podemos observar que,
por el contrario, el porcentaje de aceptados disminuye.
Por todo lo expuesto anteriormente, es claro que el ENARM es un reto; sin
embargo, que un aspirante apruebe y pase a formar parte de la lista de selec-
cionados no significa que sus problemas han terminado. Por el contrario, en
una segunda fase del proceso de seleccin competir con los otros seleccio-
13
nados dentro de su especialidad por obtener una plaza que convenga a sus in-
tereses y necesidades. Para algunos seleccionados, las caractersticas acad-
micas que un hospital ofrece para la formacin de sus mdicos residentes
podran ser un aspecto prioritario que influyan en esta decisin, en tanto que
para otros puede ser una prioridad cursar su especialidad dentro de una ciu-
dad especfica que generalmente es su ciudad natal.
Es entonces cuando nuevamente el resultado que se obtenga en el ENARM
cobra importancia, pues ste es uno de los parmetros que las instituciones
toman en cuenta para seleccionar a sus residentes.
Podemos concluir que para un aspirante es importante encontrarse en la
lista de seleccionados y tambin lo es obtener un buen lugar dentro de sta
para tener ms posibilidades de obtener una plaza que le convenga.
El presente manual est dirigido a todos aquellos mdicos generales que
aspiran a realizar una especialidad mdica y que, por tanto, desean presen-
tarse al ENARM. Tiene por objetivo brindar al aspirante un programa que in-
cluye algunas tcnicas de estudio adecuadas, informacin mdica relevante,
una bibliografa bsica y un plan de estudios que le ayuden a mejorar al m-
ximo sus capacidades acadmicas para lograr ser seleccionado y, asimismo,
obtener una buena calificacin que le permita aumentar las posibilidades de
realizar la residencia en la institucin o ciudad de su preferencia.
Si tu eres un aspirante y tu conviccin para este ao es estar incluido en
ese pequeo nmero de aceptados para realizar una residencia mdica, te in-
vito a plantearte la siguiente pregunta:
Cunto tiempo debo estudiar para aprobar un examen en el que se eva-
la lo que aprend durante siete aos de estudio?

Cada uno de nosotros responder esta pregunta de manera diferente y la
importancia de esta respuesta radica en la objetividad que le demos. Si ya
pensaste una respuesta, te invito a responder esta nueva pregunta:
Realmente crees que lo aprendido en siete aos pueda ser reaprendido
en dos, tres o cuatro meses?

Ahora seguro te diste cuenta que tu respuesta era insuficiente o, tal vez,
poco objetiva.
14
En la introduccin se hace referencia al rendimiento de grupos que pode-
mos llamar de lite y se explica que estas personas han alcanzado esos
grandes triunfos gracias a una combinacin de aptitudes dentro de las cuales
el coeficiente intelectual afortunadamente no ocupa el primer lugar. Asimis-
mo, se hace un claro nfasis en el hecho de que solamente con constancia e
invirtiendo grandes cantidades de tiempo es como un objetivo difcil puede
ser alcanzado e incluso sobrepasado.
Cul es entonces la mejor manera de calcular el tiempo y esfuerzo necesa-
rios para preparar el ENARM?
Supongamos que para preparar el examen un aspirante pudiera dedicar
exactamente el mismo tiempo que utiliz para estudiar y tomar clases en la
universidad. El estudiante de medicina promedio invierte cinco aos de su
vida en la facultad o escuela respectiva. Durante estos cinco aos, aproxima-
damente invierte siete horas diarias tomando clases e idealmente invierte al
menos una hora diaria extra para estudiar. Supongamos tambin que como
la mayora de sus compaeros, este estudiante hipottico slo realiza activi-
dades acadmicas de lunes a viernes. Si dividimos 365 das entre 7 obtene-
mos un total de 52 semanas en el ao. Esto significa que los das activos de
este estudiante, en un ao, son 313 que, multiplicados por 8 horas de estudio
y por cinco aos, nos da un total de 12,520 horas. Eso sin contar lo estudiado
en el internado y en el servicio social.
Ahora salgamos de la suposicin para entrar a la realidad. La mayora de
los textos sobre tcnicas de estudio nos dicen que sin un mtodo de estudio
adecuado estaremos desperdiciando 90% del tiempo empleado para estudiar.
Apoyando este hecho, los textos de neurofisiologa y psicologa afirman por
lo general que nadie utiliza ms de 20% de toda su capacidad mental y, por
ltimo, algo que no todo el mundo toma en cuenta: la nueva tendencia den-
tro del comportamiento humano es descubrir de qu manera nuestro status
emocional (por encima del cognitivo) determina gran parte de nuestro ren-
dimiento en todas las esferas, la acadmica incluida.
De lo anterior podemos inferir que utilizando los mtodos adecuados de
estudio, aprovechando de manera ptima los procesos mentales que nos ha-
cen recibir, procesar y retener la informacin (como son la lectura, la memo-
ria y la atencin) y adoptando alguna tcnica de meditacin, puedes reducir
15
esas 12,520 horas hasta en ochenta o noventa por ciento, digamos que 1,200
horas seran suficientes, en teora, para volver a estudiar todo lo aprendido en
la facultad de Medicina. Aunque de primera intencin suena descabellado, es
humanamente posible, pero no te preocupes, este no es nuestro objetivo, pues
no es necesario recursar la carrera de Medicina para aprobar el ENARM.
Ahora las buenas noticias. Puedes obtener un muy buen promedio de ho-
ras de estudio utilizando la frmula 6 x 7; es decir, 6 horas por 7 das de estu-
dio adecuado. Si estudiaras 5 meses diariamente y un promedio de 6 horas
diarias, obtendras un respetable promedio de 900 horas de estudio (75% de
lo necesario), si lo hicieras por 6 meses alcanzaras 1,080 horas y si, ideal-
mente, lo hicieras por 7 meses obtendras 1,260 horas de estudio. Ya sea que
estudies pocos meses y muchas horas (poco recomendable) o viceversa, debes
tener en cuenta que si quieres completar el estudio efectivo de toda la biblio-
grafa de base que sugiero en este manual, debes calcular de manera objetiva
cul ser el tiempo suficiente que requerirs para hacerlo. Cabe sealar que el
promedio de 6 horas que se sugiere puede variar de acuerdo con muchos fac-
tores. Esto se aborda con ms detalle en las secciones de planeacin que se en-
cuentran en el captulo 3.
Tal vez en este punto tu idea sea que un programa como ste es utpico,
imposible, y muchas cosas ms. Ante esto puedo decirte que si lo cumples al
pie de la letra, tienes 100% de posibilidades de ser aceptado en una residencia
en cualquier especialidad mdica.
Suena pretencioso, es cierto, pero el programa es factible y funciona. Lo
nico que se requiere para llevarlo a cabo es lo mismo que requerira cual-
quier proyecto ambicioso y de trascendencia: esfuerzo y disciplina. Al escri-
bir este programa no pretendo descubrir el hilo negro ni decir algo ignorado
por todos; simplemente hago hincapi en que nuestro mayor defecto no es la
ignorancia, sino la inaccin, y que pretendo mediante esta sencilla reflexin
recordarte que todos sabemos ya lo que debemos hacer y, sin embargo, no lo
hacemos.
As que, finalmente, te hago esta atenta invitacin: vulvete en este ao
una persona de alto rendimiento, dedica mucho tiempo a estudiar y s disci-
plinado. Puedo decirte que el ENARM no es nada fcil como puedes verlo en
las estadsticas que present al inicio del manual. Recuerda que competirs
16
contra otros miles de personas iguales a ti, que han recibido la misma prepa-
racin (acaso mejor?), o que tal vez slo compitas contra cien, pero pelean-
do por tres o cuatro plazas. Teniendo todo esto en contra, es un hecho que no
sera apropiado presentarse al ENARM sin haber tenido la preparacin ade-
cuada y si esto es la causa de un mal resultado no habr alguien a quien pue-
das culpar, excepto a ti mismo.
Nada vale lamentarse cuando los errores ya han ocurrido. Por esto es vital
aprovechar el tiempo presente con una visin a futuro situando en un primer
plano la preparacin para el ENARM. Si logras dejar tan solo por este tiempo
en segundo lugar amigos, diversiones o familia para concentrarte en el exa-
men, tendrs todas las posibilidades de aprobarlo y despus tendrs todo el
tiempo que quieras para continuar aquello que dejaste pendiente. Recuerda
entonces que nadie ms va a hacer esto por ti y ten en cuenta que en esto la
suerte no tiene mucho que ver; muy por el contrario, siempre se impone la
gran mxima que se dice ante cualquier examen: el que estudia pasa.
El presente manual se compone bsicamente de dos partes. La primera
comprende tres captulos: la introduccin, aspectos tcnicos sobre el estudio
y un programa de estudios exhaustivo en el que se pretende que el lector ha-
ga una seleccin adecuada de textos en los que se abarca la mayora de temas
que evala el ENARM y en los cuales se presentan slo conceptos concretos
que permiten aprovechar al mximo el tiempo de estudio dejando a un lado
los textos llenos de lo que todos conocemos como paja.
El segundo captulo est compuesto de diecinueve secciones de preguntas y
respuestas sobre temas que integran las cuatro especialidades bsicas y otras co-
mo psiquiatra, otorrinolaringologa y oftalmologa. Encontrars que las pregun-
tas son muy variadas, pero tambin especficas; algunas con enfoque a las
ciencias bsicas o bien a la clnica, con el objetivo de que en la lectura preliminar
tengas una idea clara sobre los conceptos que se preguntan y de qu manera se
preguntan, lo que facilita el reconocimiento de conceptos clave una vez que ini-
cies el estudio de la bibliografa. El orden que llevan las preguntas parece ser un
tanto arbitrario en ciertas secciones debido a que las preguntas relacionadas en-
tre s se ubican cercanas con el propsito de facilitar su asociacin.
17
En ningn momento este cuestionario es una gua rpida para presentar el
examen o una mera recopilacin de reactivos presentados con anterioridad
en otros cursos o en versiones anteriores del ENARM y resulta de muy poca
utilidad si no realizas el estudio de los temas de la manera que se sugiere,
pues vers que hay preguntas que, aunque son muy concretas, requieren un
conocimiento previo del tema. En algunos casos las preguntas tienen por fi-
nalidad lograr una asociacin rpida de todos los conocimientos previos que
requieras para contestarla y, por otro lado, tambin hay preguntas en las que
slo se busca agilizar la memorizacin de valores normales o de clasificacio-
nes que son de utilidad.

18
19
Figura 1. Forma en que todos pensamos que
nuestros ojos se mueven mientras desarro-
llan la lectura
CAPTULO 2

ASPECTOS TCNICOS

Nada puedes ensearle a un hombre.
Solo puedes ayudarlo a que lo descubra dentro de s mismo.
GALILEO


Vivimos en una era de informacin. La informacin es manejada en tales
cantidades y velocidades por sistemas sumamente complicados, que a veces
con slo or la palabra evocamos para ella varios significados adicionales:
imposible, inalcanzable, inconmensurable; en fin, algo de respeto. La infor-
macin es tan importante para una transnacional como lo puede ser para un
nio de primaria que presenta un examen de matemticas; por tanto, no lo es
menos para nosotros. Pero lo que es realmente importante para nosotros no
son las grandes cantidades de informacin que necesitamos para presentar un
examen, sino la manera en cmo vamos a recibir, procesar y almacenar esa
informacin. Yo voy a referirme a estos tres procesos con tres palabras
anlogas: lectura, concentracin y memoria. Y en ese orden comenzaremos

VISUALIZACIN Y LECTURA VELOZ

Probablemente al terminar de leer la
introduccin quisiste tirar el manual
por la ventana porque para tu gusto
plantea soluciones imposibles. Te
concedo un poco de razn. Mientras
escribo, tengo apilados frente a m los
libros de la bibliografa bsica que re-
comiendo y que forman un cmulo de
papel de al menos 40 cm de alto, di-
gamos, ms de 4,000 pginas. As que
esto se justifica porque debes apren-
20
Figura 2. Esquema que muestra el movimiento sal-
tatorio de los ojos cuando leemos, colocando un
dedo sobre el prpado mientras lo hacemos pode-
mos darnos cuenta de este movimiento.
der, sobre la marcha, a leer de manera que avances ms y comprendas ms;
te canses menos y te tome menos tiempo.
Por su naturaleza, este apartado no puede ser muy largo pues no voy a
describir las tcnicas y la infinidad de ejercicios para lograr la lectura veloz.
Slo me limitar a exponer los problemas bsicos que presenta la lectura
convencional y cmo podemos corregirlos. Desarrollar la lectura veloz re-
quiere un entrenamiento constante y diferente de la lectura que hars con
motivos de estudio, pero tambin es cierto que los beneficios que tendrs
aumentando tu velocidad de lectura van ms all del objetivo de leer para
preparar un examen.
Leer veinte pginas por minuto
significa terminar el libro La Guerra
y la Paz, de Tolstoi, en cuarenta o
cincuenta minutos, preparar una
exposicin leyendo el mismo tema
en 6 o 7 fuentes distintas en pocos
minutos o aprenderte el contrato
colectivo de trabajo en una semana
si lo necesitas.
Si le pides a alguien que haga un
dibujo que describa el movimiento
de los ojos cuando leemos, segu-
ramente dibujara una serie de l-
neas horizontales paralelas, algo como lo que se muestra en la figura 1. Mas esto
es totalmente incorrecto. Al leer, nuestros ojos efectan saltos de palabra en pa-
labra ya que para que una imagen pueda ser registrada e interpretada por el cere-
bro, debe permanecer fija en la retina por fracciones de segundo; este
movimiento es algo parecido a lo que se observa en la figura 2. Esto inmedia-
tamente nos lleva a la primera conclusin sobre el primer vicio de la lectura.
Si vamos a leer una pgina de una novela y sta tiene 300 palabras, nues-
tro ojo realiza 300 saltos, pues as fue educado para leer. Estas miradas de
saltos que tienen lugar cada vez que leemos generan algunas desventajas.
Cada movimiento es igual a trabajo muscular, y entre ms trabajo muscular,
el msculo se fatiga ms rpido llevndonos a los problemas frecuentes de
lectura: la vista cansada, dolor de cabeza, falta de concentracin, etctera.
21
Figura 3. Estructura que utilizamos habitualmente
para construir un apunte o un resumen.
Entonces, cul es la respuesta para este problema? Encuentro dos: leer
menos o mover menos los ojos cuando leemos. Oviamente tienes que hacer
un esfuerzo por aprender la segunda. Una tcnica adecuada de lectura es la
lectura en bloque (por mencionar alguna). En la bibliografa se mencionan al
menos dos textos con principios bsicos sobre lectura rpida con ejercicios de
lectura para que en un corto tiempo subas tu promedio de lectura de 250 pa-
labras por minuto a unas 350, lo
que ya es decir bastante.
Nunca pensamos que algo tan
simple como la lectura pueda
ser una de las tantas barreras
que nos impiden estudiar de
forma efectiva. Espero que aho-
ra tras esta breve explicacin te
des cuenta que puede ser real-
mente una gran debilidad y la
diferencia entre alcanzar una
gran meta o dejarla a medias. Si
piensas que es difcil desarrollar
esta habilidad, realmente no lo es; slo realiza los ejercicios correspondientes
y en poco tiempo vers resultados.
Otro aspecto interesante sobre la adquisicin de la informacin por la va vi-
sual es comprender cul es la manera ms fcil de construir un resumen con un
diseo que la mente reconozca ms fcilmente. Como nosotros aprendemos a
leer en forma lineal, y esta es la manera en que convencionalmente se nos pre-
senta cualquier tipo de informacin, es lgico pensar que la manera ideal de ha-
cer un buen resumen se parece a la presentada en la figura 3.
Nuevamente, esto es incorrecto. Para demostrarlo basta recordar el pro-
verbio chino que dice una imagen vale ms que mil palabras. Ciertamente,
es ms fcil recordar un cuadro de Dal que una descripcin escrita del mis-
mo. De hecho, el afirmar que las imgenes no impactan ms directamente al ce-
rebro sera como decir que el arte, la arquitectura, el diseo y la mercadotecnia
son algo inservible.
22
As pues, la mejor manera de hacer un resumen es aquella que no tiene
una forma lineal sino mas bien la de un mapa mental, que es algo parecido a
lo que se presenta en la figura 4.
Las caractersticas de este mapa
mental son muy simples. Al cen-
tro debe estar el concepto o te-
ma que se est estudiando y
dentro de las diferentes ramas
deben escribirse los conceptos cla-
ve que se relacionan entre s y
que dan sentido al tema.
Cmo reconocer un concepto
clave? Cuando nosotros recor-
damos la trama de una novela,
no releemos mentalmente la no-
vela; por el contrario, buscamos
palabras que expresen de mane-
ra corta y precisa aquello que sucedi. Cuando quieras recrear algn tema es-
tudiado, tu mente escoger las palabras que renen estas caractersticas ex-
plicativas y las utilizar. Estas palabras son los conceptos clave.
Las ventajas que ofrece un apunte de este tipo son muchas. Una de ellas es
poder adicionar informacin cuando se quiera sin tener que generar un nuevo
espacio o escribir todo de nuevo. Otra ventaja es que la informacin puede esta-
blecerse de manera jerrquica o bien relacionarse como t la entiendes. La realiza-
cin de este tipo de apunte es un proceso libre y creativo. Cada mapa mental
ser diferente, lo que facilita su aprendizaje; puedes incluir dibujos o colores si lo
quieres. El mapa mental provee un esquema ms amigable y fcil de utilizar; no
te ser ningn problema regresar a l para repasar.

ACTIVIDAD ELCTRICA Y MEMORIA

Seguramente t, al igual que muchos de nosotros, te has topado con una si-
tuacin similar a sta:
Figura 4. Un mapa mental debe ser construido de
manera grfica y usando slo conceptos clave
23
Comienzo a estudiar con entusiasmo y me coloco frente a los libros.
Puedo estudiar durante cuatro horas diarias. As lo hago por dos, tres, diez,
hasta catorce das, pero cuando termina ese tiempo y quiero recordar lo que es-
tudi el da 1, me es prcticamente imposible, por lo que me desanimo, o bien,
tengo que volver a comenzar desde el principio y pierdo mucho tiempo.
Ahora te invito a que tomes papel y lpiz y utilices tan slo cinco minutos
para realizar este ejercicio. Dibuja dos grficas en las que verticalmente se re-
gistrar la cantidad de informacin memorizada del 1 al 100% y de manera
horizontal el tiempo. En la primera, que llamaremos memoria durante el apren-
dizaje, el tiempo ser de dos horas, las cuales representan una sesin de estu-
dio. En la segunda, que denominaremos memoria despus del aprendizaje, el
tiempo se graficar por das hasta completar una semana. Ahora, haz tu pro-
pia descripcin de cmo crees que se comporta la memoria dentro de estos
dos periodos del aprendizaje.
La memoria durante el aprendizaje es similar a lo que se presenta en la figura
5. Esto quiere decir que la memoria durante el aprendizaje es mejor al inicio
y al final de ste. Tambin se ha graficado el grado de comprensin de lo le-
do y esto nos lleva a otra conclusin. Despus de un periodo constante de es-
tudio, la capacidad de comprensin no se ve afectada, pero lo que se lee ya
no se memoriza; de hecho, en algunos casos, tras cierto periodo de estudio
bajo condiciones adecuadas, podemos sentir que comprendemos mejor lo
que leemos como si hubiramos alcanzado un segundo aire y la prctica
ms comn es la de continuar mientras llevamos esa inercia. Esto es un error
pues slo conseguiremos terminar agotados y con un sentimiento de satisfac-
cin por haber abarcado mucho en poco tiempo y comprendido todo; mas
si en algunas horas queremos recordar lo estudiado, es casi seguro que no
podremos.
24

Por lo anterior, hay que recordar algunas verdades sobre la memoria.
Cuando el nivel de comprensin es ms o menos constante, la memoria fun-
ciona mejor al principio y al final del periodo de aprendizaje. Si se desea
mantener un nivel aceptable de memoria habr que encontrar el punto en
que se conjugue armnicamente con la comprensin. Para nuestra fortuna
ese tiempo ya se ha calculado y en promedio es de 20 a 40 minutos. Esto sig-
nifica que en una conferencia o una sesin de estudio de dos horas es reco-
mendable tener un ligero descanso al menos cada 40 minutos y as
lograremos que la curva sea ms uniforme durante todo el proceso. Esto
tambin nos da la ventaja de planear dos y hasta tres sesiones de dos horas de
estudio con un buen rendimiento (gradualmente alcanzars este ritmo de es-
tudio, ver seccin de planeacin). En la figura 5 se ha graficado el nivel de
memorizacin que logramos durante el aprendizaje si realizamos descansos
programados contra aquel que se realiza sin descanso.
C
a
n
t
i
d
a
d

d
e

i
n
f
o
r
m
a
c
i

n

m
e
m
o
r
i
z
a
d
a

1 hora 2 horas
Figura 5.
Descansos
Curva de memoria cuando descansamos cada 20 o 40 minutos durante la sesin de estudio
Curva de memoria cuando el aprendizaje no se interrumpe durante dos horas
Curva de memoria cuando el aprendizaje no se interrumpe por ms de dos horas
Nivel de comprensin durante el aprendizaje
25
La memoria despus del aprendizaje se parece a lo que puedes ver en la figura
6. Esto quiere decir que al terminar la sesin de estudio hay un momento
muy corto en que la memoria aumenta para luego caer rpidamente hasta
casi volver a su estado inicial (es decir, prcticamente olvidar todo) tan solo
en el primer da y siguiendo a ese nivel los das siguientes. El comportamien-
to de esta curva puede modificarse si tomamos en cuenta tres aspectos: 1) la
cada casi vertical de la curva de memoria durante las primeras horas pue-
de detenerse efectuando un repaso; 2) despus de este repaso la cada se hace
ms horizontal, es decir, ya no es tan rpida; 3) este efecto es acumulativo,
pues entre ms repasos se hagan, la curva se horizontaliza y da como resul-
tado un nivel constante de memoria. (Fig. 6)
La manera ms recomendable de efectuar los repasos es la siguiente: el
primer repaso debe ser bastante completo; por lo tanto, es mejor remitirse al
texto subrayado o bien a algn apunte definitivo sobre el tema pero asegu-
rndose que no se omite informacin importante. Los siguientes repasos con-
sisten en tomar una hoja en blanco y escribir todo lo que puedas recordar del
tema y luego compararlo con el texto subrayado o con tu apunte definitivo
para proceder a hacer las correcciones.
Memoria sin repaso
Memoria cuando se
programan los repasos
Tiempo
C
a
n
t
i
d
a
d

d
e

i
n
f
o
r
m
a
c
i

n

m
e
m
o
r
i
z
a
d
a

1 2
3 4 5
Figura 6.
Curva de la memoria cuando no se repasa a largo plazo
Curva de la memoria programando los repasos. 1 Fin del estudio. 2 Diez minutos
despus. 3 24 horas despus. 4 Una semana despus. 5 Un mes despus. Note que des-
pus de cada repaso la cada en la curva de memoria es menos pronunciada (olvida-
mos con menos facilidad).
26
Los repasos tambin deben programarse. La forma ms recomendable de
hacerlo es: el primer repaso, 10 minutos despus de terminada la sesin de es-
tudio y con una duracin de 10 minutos; esto mantiene constante el nivel de
memorizacin durante el primer da. Al cabo de este da se efecta el segun-
do repaso que debe durar tan solo dos a cuatro minutos; este repaso manten-
dr el nivel de memoria de esos conceptos por una semana. Transcurrida esta
semana efectuars otro repaso de dos minutos y, por ltimo, al cabo de un
mes realizars otro repaso de dos minutos. Una vez transcurrido este tiempo
la informacin quedar almacenada en la memoria a largo plazo, lo que sig-
nifica que ser tan familiar para ti como pueden serlo tu direccin o el nom-
bre de tus padres.
La planeacin formal y cuidadosa de los repasos es un aspecto que debes
tomar en cuenta ya que explica por qu debes disponer de tanto tiempo para
estudiar. Por ejemplo, habr das en que dediques tres o cuatro horas (con
descansos) a estudiar dos temas y sin embargo, para ese mismo da tendrs
programados: dos repasos de dos minutos sobre temas que leste hace un
mes, dos repasos de dos minutos por los temas de hace una semana, dos re-
pasos por los temas de ayer y veinte minutos para repasar los temas de hoy.
Esto har que inviertas por cada sesin de estudio 30 o 40 minutos extra en
repasos. Por tal motivo es recomendable que lleves una agenda de repasos.
Hasta este punto slo hemos hablado del mtodo para organizar y maxi-
mar la memorizacin de lo estudiado, y aunque suena bien, hay un aspecto
que puede hacernos an ms efectivos.
Como bien sabes, la actividad elctrica cerebral se registra mediante el
electroencefalograma y podemos encontrar varios patrones de actividad. De-
pendiendo de los ciclos por segundo (CPS) que componen cada patrn po-
demos encontrar ondas alfa, que van de los 7 a 14 CPS, ondas beta que van
de los 14 ciclos por segundo (CPS); en adelante y las ondas theta y delta, que
tienen menos de 7 CPS. Cuando estamos en estado de vigilar trabajamos con
ondas beta y cuando dormimos podemos estar en alfa, theta o delta.
Si a ti te preguntaran cmo crees que el cerebro trabaja mejor, en alfa o en
beta?; dicho de otra forma trabaja mejor cuando las ondas cerebrales presen-
tan ms CPS o cuando presentan menos CPS? La lgica nos dicta que un ce-
rebro ms activo trabaja mejor y, por consecuencia, en beta seremos ms
27
efectivos. En efecto, pero tambin existe una ley de la electricidad que dice
que el mejor circuito es aquel de menor impedancia o resistencia. Asimismo,
y paradjicamente, nuestro cerebro recibe y almacena de manera ms eficaz
la informacin mientras menos actividad elctrica presenta.
Aunque los periodos de baja actividad cerebral se asocian ms al estado de
dormido, podemos hacer que nuestra mente trabaje en condiciones muy si-
milares aun en un estado de despierto. Algunos textos describen este estado
mental como alfa o estado de flujo. Hablar del tema requerira otro volumen
para el presente manual y desarrollar el estado de flujo requiere un entrena-
miento mental especial que slo se logra con disciplina y dedicando mucho
tiempo a eso. Seguramente quieres saber por qu lo menciono. Pues la res-
puesta es simple. Puedes llegar a ser todo un gur mental si quieres, eso es se-
guro, pero para efectos de estudio confrmate con iniciar cada sesin de una
manera tranquila y seria, con tu mente lo mas alfa posible. La manera ms
conocida y menos comprendida de ayudarnos a permanecer en un estado
permanente de funcionamiento mental y emocional ptimo es la meditacin.
La meditacin es tan real que lleva miles de aos de existir y demostrar sus
beneficios. Tambin es interesante el hecho de que diferentes culturas han
desarrollado sus propios sistemas de meditacin: los hindes, el yoga; los ja-
poneses, el zen; los chinos, el tai chi. Hoy estas disciplinas slo son entendi-
das como mtodos de acondicionamiento fsico siendo que tienen como fin
principal el desarrollo mental y en el mejor de los casos, fsico y mental.
Dos tcnicas sencillas que te ayudarn a estar ms a tono y mejorar tu
rendimiento son:

Tcnica uno:
Cada maana, antes de despertar completamente, nos encontramos en un
estado alfa y es fcil conservar este estado de manera voluntaria por unos
minutos. De esta manera estaremos acostumbrando a nuestra mente a ese
estado o a esa sensacin. Al despertarte programa tu despertador para vol-
ver a sonar quince minutos despus en caso de que te quedes dormido.
Ahora cierra nuevamente los prpados y dirige los ojos hacia arriba e inicia
una cuenta regresiva del cien al uno a intervalos de dos segundos. Es im-
portante que dejes las prisas y la tensin a un lado tratando de disfrutarlo y
con plena confianza en que es algo til. Despus de todo realizarlo slo nos
toma alrededor de 200 segundos (tan solo tres minutos con 20 segundos). Si
28
lo realizas correctamente habrs llevado a tu mente a un estado alfa desde
el primer intento. Haciendo esto diariamente mejorars an ms tu memo-
ria y concentracin.

Tcnica dos:
Antes de tu sesin de estudio sintate en una silla cmoda con una postura
recta pero sin tensin, con los hombros ligeramente hacia atrs, al igual
que la barbilla y con piernas, rodillas y pies ligeramente separados. Coloca
las manos abiertas y con las palmas hacia arriba sobre tus muslos. Ahora
toma conciencia de tu respiracin y concntrate slo en ella. Aspira con-
tando del uno al diez, luego retn el aire contando del uno al diez y des-
pus exhala contando del uno al diez. Repite el proceso varias veces
procurando tener hacia ti mismo un sentimiento de bienestar.
Este es un ejercicio utilizado por los entrenadores de programacin neuro-
lingstica y se sabe que nos beneficia situando nuestro sistema nervioso en
un estado de alerta y haciendo que adquiera armona, todo con el simple
hecho de fijar la atencin en la respiracin. En el mbito de las funciones
mentales superiores nos ayuda a tener claridad en el razonamiento y aper-
tura en la creatividad.

Hay que precisar que para un practicante amateur de la meditacin estos
estados no son algo que tenga que sentirse conscientemente, aunque muchas ve-
ces s despierta sensaciones diversas. Esto vara con cada persona; cuando lo
intentes sabrs de qu estoy hablando. Tambin hay que tener en cuenta que
la meditacin solamente funciona mientras ms se practica y que es un me-
dio para conocernos y dominarnos. Yo te recomiendo que por lo menos te
decidas a intentarlo algunas veces a la luz de que no es una prctica esotrica
sino ms bien toda una gimnasia cerebral comprobada cientficamente. Des-
pus de todo, el hombre que se domina a s mismo es ms fuerte que aquel
que domina una ciudad.

INTELIGENCIA EMOCIONAL

Suponiendo que tienes cuatro aos, imagnate ante esta prueba: un seor est
ocupado haciendo una tarea y tu quieres que te d un bombn. El te dice que
29
si puedes esperar a que termine su trabajo te dar dos bombones; pero si no
puedes esperar slo te dar uno, aunque inmediatamente. Esto plantea un
desafo, sobre todo si realmente adoptas la mentalidad de un nio de esa
edad. Esta prueba, diseada por los psiclogos del desarrollo (Shoda, Mischel,
Peake), sirve para detectar la presencia de ciertas aptitudes en el nio como la
capacidad de contener las emociones y de retrasar el impulso. Estas herramien-
tas psicolgicas demuestran el autocontrol y la capacidad de perseguir objetivos
a pesar de las distracciones que tendr el individuo durante toda su vida. El se-
guimiento a catorce o quince aos de los nios que esperaron a que el adulto
terminara su labor, demostr que eran ms seguros, perseverantes y con una
muy buena capacidad para las relaciones humanas y el liderazgo. Por otro
lado, los nios que no esperaron eran inseguros y socialmente retrados; te-
nan sentimientos de insatisfaccin sobre sus logros y eran incapaces de so-
breponerse a la frustracin.
Este ao cada uno tiene frente a s su propia prueba de los bombones. Por
tanto, debes identificar los distractores que te ofrecen satisfaccin inmediata
pero que te hacen perder el tiempo y fijar tu atencin en un objetivo final ms
importante. Estos distractores (que pueden ser un trabajo de medio tiempo o
guardias en algn hospital) generalmente se convierten en una excusa si el re-
sultado que se obtiene no es favorable ya que podemos argumentar que no
nos alcanz el tiempo y muchas cosas ms. Lo cierto es que al final nadie
puede engaarse a s mismo.
Trata de incluir dentro de tus actividades diarias un momento para realizar
un poco de ejercicio aerbico no extenuante como saltar la cuerda o hacer
caminatas por el vecindario. Esto siempre es til y puedes hacerlo incluso en
tus descansos despus de cada 40 minutos de estudio, esto es increble para
relajarte y despejar la mente y te da nuevos bros para continuar la sesin.
Finalmente, procura estar en paz contigo mismo y con los dems. Quienes
se sienten ansiosos, deprimidos o enfurecidos no aprenden, y la gente que se
encuentra atrapada en esos estados de nimo no asimila la informacin de
manera adecuada ni la maneja bien. Un texto que considero provechoso y
que te dar tips al respecto es La inteligencia emocional de Daniel Goleman.

30
ECONOMA DEL RENDIMIENTO

Lgicamente, hay puntos bsicos en cuanto a la disciplina del estudio, lo
cierto es que casi nadie los respeta. Existen varios ejemplos de actitudes inco-
rrectas de estudio, una de ellas es la costumbre muy difundida de estudiar la
noche anterior al examen hacindolo hasta tarde, sacrificando horas de sue-
o y haciendo esfuerzos innecesarios. Esta prctica es incorrecta y general-
mente no da mejores resultados que las sesiones de estudio que respeten estos
principios bsicos.
Para tener un buen rendimiento es importante no sacrificar horas de sueo ni
tratar de estudiar slo por las noches hasta ser vencido por el sueo. Los
hombres que menosprecian las bondades del sueo y sus efectos reparadores
terminan pagando caro las consecuencias.
Si estudias hasta tarde, procura que no sea hasta quedarte dormido sobre
tus libros y primero haz un clculo de cul es tu hora promedio para dormir y
con base en esto planea tu sesin (ver adelante) calculando que terminars los
repasos cuando sea la hora de dormir. Si sigues estudiando cuando tu cuerpo
sabe que debe descansar, l simplemente comenzar a descansar y perders
tu nivel de concentracin, entendimiento y memoria, haciendo que al otro
da slo recuerdes un mnimo porcentaje de lo que deberas saber.
Evita en cada sesin los esfuerzos innecesarios. Henry Ford deca: Nunca estoy
de pie cuando puedo estar sentado y nunca estoy sentado si puedo estar acos-
tado. Esta es una muy buena leccin sobre economa del rendimiento y nos
deja claro cmo evitar esfuerzos innecesarios.
Cuando estudies, es importante verificar las condiciones fsicas del lugar
donde te encuentres. Es decir, que la altura de la mesa sea correcta (que lle-
gue a la altura de tus codos), que la silla sea firme y cmoda, que cuentes con
suficiente y adecuada iluminacin, de preferencia luz blanca que evita la fati-
ga visual y tener dos fuentes de luz, una directa (lmpara de escritorio) y otra
indirecta (iluminacin de la habitacin); que la temperatura y ventilacin de
la habitacin sean adecuadas y, por ltimo, que tu posicin para estudiar sea
apropiada, sentado de manera recta y no acostado en la cama como muchos
acostumbran.
31
CAPTULO 3

UN PROGRAMA DE ESTUDIOS

El triunfador tiene un justo sentido del tiempo; por consiguiente, no lo mal-
gasta. Sabe que hay una oportunidad para cada cosa y un momento para
cada actividad.

Un momento para ser agresivo y otro para ser pasivo,
un momento para estar juntos y un momento para estar solos,
un momento para luchar y otro para amar,
un momento para trabajar y otro para jugar,
un momento para llorar y otro para rer,
un momento para hacer frente y otro para retirarse,
un momento para hablar y otro para guardar silencio,
un momento para apremiar y otro para esperar.

Anlisis transaccional con experimentos Gestalt
MURIEL JAMES/DOROTHY JONGEWARD

El promedio de estudio diario puede extenderse desde 6 hasta 8 horas dividi-
das en sesiones dependiendo de tu capacidad o de qu tanto ests acostum-
brado a estudiar (o de qu tanto ya has perdido el hbito), incluso puedes
empezar con una o dos horas diarias. Al comenzar poco a poco, no te ser
pesado adoptar este nuevo hbito y disminuirn las probabilidades de que lo
abandones en poco tiempo. An as, te sugiero que dentro del primer mes al-
cances un promedio de 4 a 6 horas diarias de estudio como mnimo, pues en
este ao tu peor enemigo ser el tiempo.
Tal vez desde este momento preguntes por qu habra de utilizar incluso
los fines de semana? Las razones son simples. Todo en nuestras vidas debe
guardar un equilibrio y ciertamente todos nos merecemos un descanso de vez
en cuando, pero tambin es cierto que los hbitos adquiridos recientemente
con mucho esfuerzo pueden perderse con mucha facilidad. Por tanto, tu pro-
grama de estudio debe incluir sbados y domingos con un doble fin. El primero,
abarcar ms temas, y el segundo y ms importante, que no pierdas el ritmo y el
http://booksmedicos.org
32
efecto aditivo que logrars sobre el comportamiento de tu curva de memoria al
posponer los repasos por dos das. Al principio ser difcil y un poco incmodo,
pero cuando t mismo y los que te rodean comiencen a acostumbrarse al hecho
de que ests preparando tu examen, tu hbito se tornar ms fuerte y los de-
ms tratarn de amoldarse a ti (dentro de lo posible) para acordar las reunio-
nes, o en el caso de la familia, para respetar tu lugar de estudio.
La manera adecuada de abordar este tema es comenzar por conocer los
aspectos generales sobre la memoria, lectura, toma de apuntes, etc., mismos
que se expusieron en el captulo dos. Ahora hablaremos de la planeacin de
tus estudios, y tendremos en cuenta dos aspectos.

Planeacin de las sesiones de estudio

Es un ritual conocido actuar de la forma siguiente: preparar un lugar adecua-
do y silencioso para nuestra sesin de estudio, tener a la mano todo lo nece-
sario y decidirse a comenzar. En un siguiente paso nos disponemos a
enfrentarnos a un texto extenso leyendo lo ms que se pueda, hasta que el
cansancio indique que ya ha sido suficiente.
Este plan tiene errores que saltan a la vista. El hecho de acometer sobre un
libro extenso sin conocerlo provoca eventual y forzosamente un sentimiento
de rechazo a lo desconocido y de estar realizando una tarea sin fin, lo que
puede ser bastante molesto. Asimismo, comenzar a estudiar sin definir el
tiempo que se utilizar ni la cantidad de temas que se pretenden abarcar es
como abordar un barco sin rumbo. Estos dos errores en la sesin slo nos lle-
van al cansancio y a un sentimiento de frustracin, pues no importa lo mu-
cho que hayas avanzado, si ests ante un texto de mil pginas, siempre sentirs
que lo que hiciste fue insuficiente puesto que no fijaste una meta a corto plazo.
Un aspecto interesante descubierto en nuestra psique es la tendencia a
completar cosas. Si se nos presenta una serie de figuras bsicas incompletas
tendemos a imaginar lo que les falta o a interpretarlas como figuras en proce-
so de completarse. De igual forma, nuestra mente necesita tener trazadas las
directrices que indican el principio y el final de cualquier actividad para que
pueda ser realizada con mayor facilidad.
33
Una excelente planeacin de sesin consta de cinco puntos: Primer contacto
con el material de estudio, determinacin de tiempo y cantidad, anotacin de los cono-
cimientos sobre el tema, primera y segunda lectura, repaso y apuntes.
La manera correcta de hacer el primer contacto es la misma que utilizas
cuando ojeas una revista o un libro cuando vas a comprarlo. No debe tomar-
te ms de un minuto o dos y con esa ojeada rpida slo buscars percatarte
del contenido a modo general, de cules son los temas y subtemas, del conte-
nido de las tablas y cuadros, y cosas por el estilo. Esto te ayudar a tener una
perspectiva rpida sobre el tema y sabrs si es fcil de abordar o tomar ms
tiempo de lo que pensabas.
Despus del reconocimiento preliminar, ahora ya tienes una idea para de-
cidir cuntos temas abarcars y en cunto tiempo planeas hacerlo. Recuerda que es-
to es importante para que lleves una direccin definida en tu estudio. Para
este apartado debemos tomar en cuenta que nuestro objetivo es alcanzar un
promedio de 6 horas diarias de estudio; sin embargo, sera intil intentar ha-
cerlo desde el primer da.
No hay que olvidar que:
Si planeamos correctamente nuestros descansos en la sesin de estudio
fortaleceremos nuestra retencin.
Cada sesin de dos horas de estudio debe tener descansos cada 40 mi-
nutos en promedio.
Durante el da podemos planear dos o tres sesiones.
Es recomendable que durante las primeras dos semanas planees slo
una sesin al da y despus aumentes gradualmente el tiempo de estu-
dio o las sesiones.
No existe tiempo alguno predeterminado. El tiempo correcto es aquel
en que rindes mejor. Slo t puedes determinarlo; no hagas esfuerzos
innecesarios.
Trata de rendir al mximo; consentirte demasiado es perjudicial.
Entre sesiones trata de llevar a cabo actividades que dejen a la mente
descansar, como realizar una caminata, hacer ejercicio aerbico, con-
versar con un amigo, ver una pelcula, etctera.
34
Antes de comenzar necesitas fijar tu atencin en el tema. Lbrate de todo
pensamiento preocupante o que pueda llevarte a cualquier lugar que no sea
tu escritorio. Este es un buen momento para llevar a cabo el ejercicio de me-
ditacin nmero 2, descrito previamente. Despus de eso toma una hoja y
anota todo aquello que sepas sobre el tema que vas a leer. Este ejercicio har que
establezcas un marco mental adecuado como referencia. Encontrars que las
mismas dudas que te plantees en este momento o la posible falta de estruc-
tura sobre el tema se resolvern al momento de revisar el tema. Te apuesto
que recuerdas muchas de esas preguntas que durante toda tu vida estudiantil
no pudiste contestar a la hora del examen pero que comentaste con tus com-
paeros justo al salir del aula o bien las buscaste inmediatamente en el libro.
El esfuerzo que hiciste por recordar fue complementado por la llegada de la
informacin requerida; de esa forma, esta asociacin entra fcilmente en la
memoria a largo plazo. Al realizar las anotaciones previas ests generando
un proceso muy similar en el que te cuestionas, identificas tus puntos dbiles
y luego t mismo los corriges.
Ahora puedes comenzar con la primera lectura. Esta primera lectura debe
centrarse en todos aquellos detalles que se pasaron por alto en el reconoci-
miento preliminar. Ahora pondrs atencin en lo escrito en los prrafos, en
especial al principio y al final de ellos como tratando de reconstruir un rom-
pecabezas cuyas piezas son la informacin preliminar y la que ests ganando
en este momento. Trata de poner especial atencin en los apartados que con-
tengan resultados o conclusiones as como en todo lo resaltado con letras ne-
gritas o cursivas (itlicas).
La segunda lectura tendr como objetivo rellenar los ltimos espacios del
rompecabezas. En este punto seguramente habr muchas dudas sobre el asun-
to abordado pero tambin tienes una perspectiva mayor sobre l, de manera
que ahora eres un semiexperto del tema con ms herramientas para resolverlo
que si hubieras comenzado a leer de manera lineal; con esa informacin trata-
rs de solucionarlo y, por supuesto, conoces mucho ms del tema que cuan-
do comenzaste; si crees que no es cierto, puedes revisar las anotaciones que
hiciste al principio y comparar. Vers que es realmente interesante la diferen-
cia entre tus conocimientos previos y los que tienes hasta este momento. Pero
an no has terminado.
35
Para completar el mtodo y registrar todo en la memoria a largo plazo tie-
nes que proceder a hacer el primer repaso como ya se explic en la seccin de
memoria, y si no te quita tiempo o si lo necesitas para recordar algunos con-
ceptos de manera rpida, haz un resumen que guarde preferentemente carac-
tersticas similares a las del mapa mental ya descrito.
Cualquiera puede pensar que este mtodo suena a fraude, pues es lgico
que si lees algo y luego lo vuelves a leer y te lo preguntas a ti mismo obvia-
mente lo vas a saber. Es lo fantstico de este mtodo! Es como hacer trampa
sin hacerlo, pues siempre el resultado final es el mismo y se llama aprendizaje
a largo plazo. Sabemos que una persona ha aprendido cuando ha modificado
algn aspecto de su conducta de manera ms o menos permanente.

Planeacin a largo plazo

Comencemos por otro ejemplo. La versin 35 en espaol del Manual de
diagnstico clnico y tratamiento de Tierney tiene 1 537 pginas. Yo quiero ter-
minarlo en dos meses. En esos dos meses debo haberlo ledo, resumido y re-
pasado de manera efectiva. Al terminar habr aprendido casi todo su
contenido. Lograrlo en dos meses significa que tengo ocho semanas para ha-
cerlo; en otras palabras, debo avanzar aproximadamente 200 pginas por
semana y, por tanto, diariamente debo abarcar 30 pginas en promedio. No
suena como algo imposible verdad? Si considero que estar dedicando de 4
a 6 horas diarias para estudiar, treinta pginas resultan muy poco.
Utilizando adecuadamente todos los mtodos ya expuestos, bien podras
abarcar ms de 60 pginas con un alto nivel de comprensin y retencin. Lo
que disminuira el tiempo de dos meses a un mes.
Este es un ejemplo de cmo debes trazar tu propio plannig a largo plazo, de
manera precisa y detallada, explicndote a ti mismo los objetivos y nunca
perdiendo el sentido de la viabilidad; es decir, no pretendas leer 25 temas en
un da o avanzar 200 pginas y menos cuando apenas ests comenzando,
pues eso te resultar en frustracin inmediata. Plantate al principio metas
alcanzables y conforme vayas ganando habilidad plantate otras ms ambi-
ciosas, pero siempre procura planear de ms.
36
Planear de ms significa que dentro de tus planes siempre debe haber un es-
pacio que cubra cualquier atraso imprevisto e inevitable. Los problemas
siempre ocurren y no hay forma de preverlos. Tomndolos en cuenta no
abandonars tu rutina, si es que alguna vez tiene que verse interrumpida
(preciso que esto se refiere a imprevistos reales y no a la salida imprevista del
viernes por la noche ni cosas por el estilo).
Ahora bien, estos son los textos que te sugiero y la forma en cmo combi-
nar esta bibliografa con la parte 2 de este manual:
Lee primero la parte 2 (seccin de preguntas) sin intencin de memori-
zarlo, slo para tener una perspectiva de los temas.
Realiza el repaso completo y profundo de la bibliografa de base. Todos
los textos deben ser ledos, subrayados y resumidos, en otras palabras
ESTUDIADOS de pasta a pasta y al menos dos veces.
1. Current Medicine 2005 (o bien alguna versin del Diagnstico Clni-
co y Tratamiento, de Tierney y Papadakis en espaol).
2. NMS Internal Medicine de Wyers.
3. NMS Pediatrics de Dworkin.
4. NMS Surgery, de Jarrell.
5. NMS Obstetrics and, Gynecology de Beak.
Regresar a ste para contestar las preguntas tratando ahora de enten-
derlas y memorizarlas.
Cuando hayas alcanzado este punto ya habrs cubierto todas las reas
del examen, incluyendo la Anatoma, Patologa, Farmacologa, Fisio-
loga, Traumatologa, Otorrinolaringologa y Oftalmologa. La informa-
cin que encontrars en estos libros sobre estas materias es suficiente y te
sugiero no perder tiempo en textos muy amplios relacionados con ellas.
Como ejercicios examen, una vez que consideres que has abarcado casi
todos los temas sugiero que intentes contestar y repasar:
1. Repaso para el USMLE Paso1, de Barton
2. Repaso para el USMLE Paso 2, de Chan
Tambin es muy til realizar sesiones de preguntas contra reloj con un
formato de opcin mltiple que sea semejante al formato del ENARM.
Puedes sacar los reactivos de los NMS o de los pasos 1 y 2. Puedes en-
37
contrar los reactivos de todos los PAC en esta direccin de Internet:
www.drscope.com. Estas preguntas tambin son de gran utilidad por-
que estn formuladas con una orientacin epidemiolgica hacia la po-
blacin mexicana, justo como en el ENARM.
Es bueno tratar de contestar entre 100 y 150 preguntas en 50 minutos y
luego autocalificarte.
Hasta este punto puede ser que el resultado que obtengas todava no
sea muy bueno, pues las preguntas son difciles; pero debes tomar en
cuenta que esto es normal y que tanto en el ENARM, como en el USM-
LE o el MIR, las ms altas calificaciones suelen ser de 70. Al intentar
estas sesiones repetidamente tu calificacin mejorar poco a poco; eso
est garantizado, pero no esperes obtener 100, ya que realmente muy
pocos pueden lograrlo (ojal t seas uno de ellos).
Recuerda que este es un programa para terminarse en seis meses (pro-
medio) y que debes tomar en cuenta que no slo vas a ir leyendo lo co-
rrespondiente a ese tema y a ese da; es decir, tienes que cumplir cada
da los repasos programados. Como ya lo mencion, es muy til que
tengas una agenda para asignar a cada da los repasos necesarios.

Algunos aspectos que es necesario considerar

Un error frecuente y muy importante consiste en pensar que se debe repasar
mucho de la patologa comn y poco de las que llamamos enfermedades ra-
ras o poco frecuentes. Esto generalmente lo justificamos diciendo esto no
es del mdico general o nunca lo estudi en la facultad o eso no se usa
en la medicina mexicana. Todo esto no es verdad. En este manual encontra-
rs suficiente material para tener una idea cabal de muchas de estas enferme-
dades que debe conocer cualquier mdico general.
Cuando estudies sobre algunos padecimientos que a primera vista parecen
muy similares, trata de enfocarte slo en las diferencias, como pueden ser la
edad de inicio de la enfermedad (es la diferencia bsica entre la neumona y
la bronquiolitis, por ejemplo) o simplemente si es ms frecuente en hombres
o en mujeres pero guarda un esquema general de las similitudes. En el cues-
tionario encontrars preguntas que ejemplifican esto.
38
Para memorizar un concepto es importante establecer asociaciones entre la
imagen mental de ste y la manera en que es percibido por los sentidos. Esta
tcnica no es fcil de aplicar en Medicina. Pero digamos que hay que imaginar
cmo se palpa, se oye, se ve; por ejemplo, una lesin, o incluso cmo huele una
infeccin por Pseudomonas. El ejemplo clsico de esta asociacin es el compen-
dio de sndromes pleuropulmonares, en los que se involucran todos los sentidos
para recordar un concepto. Suena raro pero funciona. Est en los libros.
Mediante la repeticin de una pregunta o su planteamiento en contextos
diferentes se logra una mejor comprensin del concepto, al mismo tiempo
que se crea una asociacin ms poderosa que permite que quede almacenado
en la memoria de largo plazo. Por eso encontrars muchas preguntas pareci-
das en secciones diferentes de este manual. En l, todas las respuestas que
enumeran clasificaciones, tipos celulares o patologas estn ordenadas por
frecuencia o importancia y de mayor a menor.
Nunca dejes de preguntarte por qu tuve mal esa pregunta?, ni de buscar la
respuesta enseguida. As, una vez ms esta informacin quedar registrada en tu
memoria a largo plazo y posiblemente para siempre.
Es satisfactorio pasar de ser el hombre (o mujer) organizador a ser el hom-
bre (o mujer) de la accin.
Una gua fcil para automotivarte y evaluar si ests haciendo bien tu traba-
jo son las cinco reglas de oro de un buen trabajo:
1. Interesarse. Una condicin necesaria para el buen rendimiento es desarro-
llar un gusto por lo que se hace, encontrarle una razn de ser o ver el
beneficio a futuro que te traer.
2. Conocer y respetar el programa. Trata de hacer un buen programa, mismo
que se ajuste a tus necesidades y capacidades y sguelo al pie de la letra.
Evita caminar sin rumbo.
3. Hacer lo que se hace. No debes hacer dos cosas al mismo tiempo y, mu-
cho menos, pensar en dos cosas a la vez; esto se conoce como la ley de
la va nica. En el momento de estudiar concntrate en lo que ests ha-
ciendo y preocpate por lo dems cuando sea el momento de hacerlo.
Esta no es slo una regla de estudio, sino una regla de vida. Recuerda
que:
39
Cada da es una nueva vida para el hombre sabio; ayer ya se fue y ma-
ana todava no ha llegado; slo tienes el da de hoy.

4. Seguir un buen ritmo. El ritmo puede ser lento o apresurado. Lo malo
es que la lentitud es algo obsoleto y el tiempo es dinero. Todo aquel
que vive en el mundo moderno lo sabe. Entonces, tu ritmo debe ser
apresurado para lograr resultados dentro de los plazos convenidos.
5. Llegar hasta el final. Siempre permanecer es ms cmodo; todos los
cambios nos plantean dificultad. Qu cualidades son necesarias para
lograr el paso de lo conocido a lo desconocido? Sin duda, algo de va-
lor; eliminar las conductas inflexibles y estereotipadas; creer en lo que
se hace y en la capacidad de postergar el placer, entre otras.

Todo lo anterior comprende el programa de estudios que te propongo, en
adelante solo t y tu esfuerzo determinarn los resultados que obtengas. De
antemano te deseo suerte en este intento de aprobar el ENARM esperando que
lo que acabas de leer te sea til y contribuya a este propsito.
40
41












PARTE II
42
43
CARDIOLOGA




ANATOMA Y FISIOLOGIA CARDIOCIRCULATORIA.

De que capa germinal proceden el corazn, los vasos y las clulas
sanguneas?
Del mesodermo.

Cules son las estructuras cardiacas primitivas en el embrin?
Porcion craneal: Da origen a los arcos articos
Porcin caudal: Origina el ventrculo embrionico.
Bulbo cardiaco: Se divide en tercios. La porcin inicial origina el cuerpo
del ventrculo derecho o ventrculo primitivo. La porcin media origina el
conos cordis que constituir los tractos de salida del ventrculo derecho y
el ventrculo izquierdo. La porcion distal da origen al truncus arteriosus.

De que estructuras embrionarias proceden el nodo sinoauricular (NSA
o de Keith y Flack) y el nodo auriculoventricular (NAV o de Aschoff-
Tawara)?
El nodo SA se desarrolla a partir de estructuras del lado derecho y el no-
do AV de estructuras del lado izquierdo, esta es la razon por la que el
nodo SA se encuentra inervado por el vago derecho y del nodo AV por el
vago izquierdo.

Cul es el potencial de membrana en reposo de las clulas miocrdi-
cas?
-90mv.

Cules son las fases de la despolarizacin cardiaca?
Fase 0: Despolarizacin rpida: Apertura de canales rapidos de Na
+
.
44
Fase 1: Repolarizacin parcial: Inactivacin de los canales de Na
+
, aper-
tura de canales de K
+
, con salida de K
+
.
Fase 2: Meseta: Apertura de canales de Ca
++
. Con entrada del mismo.
Fase 3: Repolarizacin: Cierre de canales de Ca
++
. Persiste discreta sali-
da de K
+
.
Fase 4: Despolarizacin progresiva por entrada de Na
+
.



Dnde se encuentran ubicados el nodo SA y el nodo AV?
El nodo SA se encuentra situado en la unin de la vena cava superior
con la aurcula derecha y el nodo AV se encuentra situado en la porcin
posterior derecha del tabique interauricular.

Cmo se encuentran conectados el nodo SA y el nodo AV?
Se conectan gracias a tres fascculos de fibras de Purkinge, llamados de
Bachman (anterior), de Wenckebach (medio) y de Thorel (posterior).

A que se llama ciclo cardiaco?
Al conjunto de movimientos del corazn en el que se encuentra una fase
diastlica de llenado y una sistlica de expulsin. En cada ciclo cardiaco
se expulsan 60 a 70ml (algunos marcan de 70 a 90ml o bien el 65%)
promedio de sangre. Al final de la distole el volmen ventricular en pro-
medio alcanza 130 ml por tanto un promedio de 50ml se considera el vo-
lmen residual despus de la sstole.









45
Cul es la correlacin entre los eventos mecnicos, las presiones y los
fenmenos estetoacsticos del ciclo cardaco?

En que consiste el pronunciamiento de Frank Starling?
La energia de contraccin es proporcional a la longitud inicial de la fibra
muscular, dicho de otra forma mientras mas se distienda el miocardio (se
precargue) mayor ser el volmen sistlico ventricular, dando lugar a la
curva de FranK Starling. Es gracias a esta regulacin heteromtrica que
un corazn transplantado (no inervado) puede aumentar el gasto si las
demandas aumentan.

Qu relacin se expresa mediante la frmula de Poiseuille-Hagen?
Esta frmula expresa la relacin entre el flujo en un tubo largo estrecho,
la viscosidad del lquido y el radio del tubo. Su expresion matemtica es
la siguiente: F= (P
A
-P
B
)x(/8)x(1/n)x(r
4
/L).

Sistole
isovolumtrica
Contraccin isovolu-
mtricahasta alcanzar
80mmhg en aorta y
10mmhg en la pul mo-
nar
Duracin 0.5
seg
Cierre de vlvulas
AV que se abom-
ban hacia las au-
rculas
Primer Ruido o lub o S1
Fase de
expulsin
ventricular
Elevacin de la presin
hasta 120 en al VI y
hasta 25 en el VD. I
Al inicio es r-
pida enlentece
segun progresa
la satole.
Vlvulas AV se
traccionan hacia
abajo porlas
cuerdas tendino-
sas

Protodistole Descenso inicial de la
presin ventricular
Duracin
0.04seg
Termina con el
cierre de las vl-
vulas artica y
pulmonar
Segundo ruido o dub o S2
Relajacin
ventricular
isovolumtrica
Finaliza cuando la pre-
sin ventricular es me-
nor que en las
aurculas

Apertura ventricu-
lar de valvulas AV
permitiendo el
llemado
Tercer ruido o S3 por llenado ven-
tricular rpido.
Cuarto ruido o S4 se ausculta en
ventriculos rigidos o si hay presin
auricular alta, justo antes del S1.
F=flujo n= viscosidad r= radio del tubo L= longitud del tubo.
P
A
-P
B
= diferencia de presin en los extremos del tubo.

46
Qu relacin se expresa en la ley de Laplace?
La relacin entre la presin de distensin y la tensin. Se dice que la pre-
sin de distensin (P) es igual a la tensin (T) dividida por el radio del tu-
bo. P=T/r.

Qu parmetro hemodinmico se ve reflejado en la presin central pul-
monar?
La precarga. La presin central pulmonar mayor de 15 indica insuficien-
cia cardiaca izquierda y la menor de 15 indica hipovolemia.




Cuales son los principales mecanismos de control de la presin sangu-
nea?
La presin sangunea es resultado directo de multiplicar el gasto cardaco
por las resistencias perifricas. Entonces la TA se regula principalmente
mediante dos mecanismos imbrincados: 1) Los baroreflejos mediados
por el sistema nervioso simptico y 2) el sistema renina angiotensina al-
dosterona.
Quin se encarga de la regulacin de la TA a corto plazo?
Los baroreceptores en el arco artico y el seno carotdeo, disminuyen
sus estmulos aferentes hacia la mdula espinal cuando la presin cae;
esto provoca aumento de la actividad simptica y disminucin de la pa-
rasimptica. Como resultado hay vasoconstriccin y aumento en el gasto
cardaco.
Aqui introducir conceptos de hemodinamica
47
Cules son las acciones que genera la activacin de estos receptores?
1: Se encuentran en los vasos, su efecto es vasoconstrictor co-
ronario, renal, piel, mucosas, utero, vejiga, midriasis. Inhibe la li-
beracin de insulina
2: Se encuentran en las membranas prey post sinpticas del sis-
tema nervioso central y perifrico, vasos sanguneos, plaquetas y
leucocitos. A nivel del centro vasomotor provocan disminucin de
la presin arterial. Aumenta la secrecin de glndulas salivceas,
de insulina por los islotes de Langerhans. Disminuye la secrecion
de renina.
1: taquicardia, inotrpico positivo, aumento de la liplisis
2: causa relajacin del msculo ciliar del ojo, msculo esquelti-
co, bronquial, vesical, uterina. Vasodilatacin esplcnica y renal.
Incrementa la secrecin de renina en el parato yuxtaglomerular.
3 aumenta la liberacin de glucagon.
Los alfa tienen mayor afinidad a la noradrenalina y los beta a la
adrenalina.
1 y 2 a nivel cardiaco son inotropicos y cronotropicos positivos
Qu es el sistema renina angiotensina aldosterona?
Es el encargado de regular la presin sangunea a largo plazo. Los baro-
receptores renales conformados por el aparato yuxtaglomerular, regulan
la secracion de renina, esta transforma el angiotensingeno en angioten-
sina I y II provocando aumento de la presin sangunea.
Dnde se forma cada componente del sistema renina angiotensina al-
dosterona?
Angiotensingeno Se forma en hgado y circula en el plasma en la fraccin -2-
globulina.
Renina Se forma en las clulas del aparato yuxtaglomerular, localizadas
en la arteriola aferente del glomrulo
Angiotensina I Este decapptido se desdobla del extremo Terminal N del angio-
tensingeno. En el plasma en presencia de renina.
Enzima convertidora de
angiotensina
Est situada en las clulas endoteliales de todo el organismo
Angiotensina II La mayor parte de la transformacin es anivel pulmonar pero
tambin se realiza en otras partes del cuerpo.
48

Cuales son las acciones de la angiotensina II?
Produce vasoconstriccin arteriolar
Acta directamente sobre la corteza suprarrenal aumentando la
secrecin de aldosterona con retencin final de sodio.
Acta sobre las neuronas simpticas posganglionares facilitando
la liberacin de noradrenalina
Produce contraccin de las clulas mesangiales renales. Pero
principalmente causan constriccin en la arteriola EFERENTE,
manteniendo la TFG renal.
A nivel central aumenta la TA, aumenta la ingesta de agua e in-
crementa la secrecin de vasopresina, endotelina y ACTH.
Estimula la fibrosis, el PAI 1 y la formacin de superxidos


HIPERTENSIN

Cmo se realiza la exploracin de la TA (tensin arterial)?
Debe hacerse en reposo, evitando estados de tensin o nerviosismo, es
recomendable no haber ingerido en los 30 a 45 minutos previos, caf o
alcohol, ni haber fumado. Se coloca el manguito en el brazo a la altura
del corazn, y se aplica presin sobre la arteria humeral, hasta que se
impide la circulacin. La auscultacin se realiza sobre la flexura del codo.

A quienes se considera pre-hipertensos y como se manejan?
Toda persona que tiene PAS (presion arterial sistlica) de 120-139 o
PAD (presin arterial diastlica) de 80-89. Se les debe exhortar a modifi-
car su estilo de vida nicamente.

Cules son los estados 1 y 2 de la hipertensin?
Estado 1 es 140-159 de PAS y 90-99 de PAD
Estado 2 es >160 de PAS y >100 de PAD

Cules son las modificaciones en el estilo de vida recomendadas por el
JNC7?:
49
Reduccin de peso IMC 18,5-24.9 Kg/m
2

Dieta DASH Rica en frutas y vegetales, baja en
grasas saturadas.
Reduccin de sodio en la dieta Na+ no mas de 100mmol/da (2.4g
de Na o bien 6g de cloruro de Na)
Actividad fsica Ejercicio aerobico al menos 30
min/da
Moderacin en el consumo de al-
cohol
No ms de 30ml de etanol/da (dos
copas) en varones, 1 en mujeres.

Cules son causas de hipertensin seundaria?
Apnea del sueo
Causas inducidas o relacionadas a frmacos
Enfermedad renal crnica
Aldosteromismo primario
Enfermedad renovascular
Corticoterapia crnica y Sndrome de Cushing
Feocromocitoma
Coartacin de aorta
Enfermedad tiroidea o paratifoidea

A que se le llama hipertensin resistente?
Es el fracaso de conseguir los objetivos en pacientes que tienen adhe-
rencia a decuada a una pauta de 3 antihipertensivos a dosis tope que in-
cluya diurtico. Deben descartarse primero las causas de hipertensin
secundaria y luego revisarse las causas de hipertensin resistente.

Cules son las causas de hipertensin resistente?
1) Medida inadecuada de la presin arterial
2) Volumen plasmtico elevado y pseudotolerancia
Escaso volumen de sodio
Retencin de volumen por enfermedad renal
Terapia diurtica inadecuada
50
3) Inducida por frmacos
No adherencia o dosis inadecuadas
AINES, Inhibidores COX
Contraceptivos, corticoides, regaliz
4) Condiciones asociadas
Obesidad y excesivo consumo de alcohol.

Segn el JNC-7 que frmaco es la base del tratamiento en HAS (hiper-
tensin arterial sistmica) no complicada?
Debe usarse hidroclorotiazida sola o combinada.

Cul es el objetivo del tratamiento de la TA?
Mantener al paciente con 140/90. En los diabticos y pacientes renales
es de 130/80.

Cul es el manejo de las diferentes variantes clnicas de la hipertensin
arterial sistmica (HAS)?
La HAS grado I monoterapia y II deben recibir farmacoa asocia-
dos.
HAS con sndrome hipercintico: - bloqueadores.
HAS con sntomas de expansin del espacio extracelular: diurti-
cos.
HAS de predominio sistlico y edad avanzada: calcioantagonis-
tas.


Cules son las frmacos de eleccin en hipertensos con cardiopata is-
qumica?
En los pacientes con angina estable se usan betabloqueadores o cal-
cioantagonistas de accin larga. En pacientes con sndromes coronarios
agudos o posinfartados se unasn IECAS,, betabloqueadores (BB) y anta-
gonistas de la aldosterona (AA).

51
Cules son los beneficios de los antihipertensivos en los diabticos?
ECV e ictus Nefropata macroalbuminuria
ARA II + + +
IECA + + -
BB + - -
AA + - -
TIAZDICOS + - -
BBC + - -

Cul es la definicin del ATP III del sndrome metablico?
Se define como la presencia de 3 o mas de las siguientes:
Obesidad abdominal mayor de 102 en hombres y 89 en mujeres
Intolerancia a la glucosa (glucemia basal 110 mg/dl)
TA 130/85 mmHg
Triglicridos elevados 150mg/dl
HDL< 40mg/dl en hombres y <50mg/dl en mujeres

Cules son los nicos frmacos antihipertensivos que no modifican fa-
vorablemente la hipertrofia ventricular izquierda?
Los vasodilatadores directos hidralacina y minoxidil.

Cules son las consideraciones para la hipertensin en mujeres?
La TA y el riesgo de hipertensin aumentan con el consumo de
anticonceptivos orales
Por el contrario la terapia hormonal sustitutiva no causa hiperten-
sin
Las mujeres hipertensas embarazadas deben controlarse con me-
tildopa, betabloqueadores y vasodilatadores por la seguridad del
feto.
IECA y ARAII estn contraindicados en el embarazo.

Cules son ls principales familias de antihipertensivos?
52
Diurticos, betabloqueadores, inhibidores de la enzima convertidota de
angiotensina (IECA), antagonistas de receptores de angiotensina II
(ARAII), calcioantagonistas, bloqueadores alfa, antihipertensivos de ac-
cin central (agonistas 2).

Cules son las acciones de los betabloqueadores?
Inhiben competitivamente los efectos de las catecolaminas en los
receptores beta, disminuyendo la frecuencia cardaca y el gasto
cardaco.
Disminuyen los niveles de renina
Regulan a las barorreceptores para aceptar niveles ms bajos de
TA.
Generan liberacin de prostaglandinas vasodilatadores
Disminuyen el volumen plasmtico.
Disminuyen el flujo de salida simptico del SNC

Segn su mecanismo de accion como se dividen los betabloqueadores?
Cardioselectivos y no cardioselectivos.

Cules son los betabloqueadores con actividad simpaticomimetica in-
trinseca?
Son frmacos con actividad agonista parcial que adems de bloquear
el acceso de las catecolaminas, estimulan levemente al receptor adre-
nrgico produciendo menor enlentecimiento de la frecuencia cardiaca
con menor depresion de la conduccin aurculo-ventricular.Los farmacos
con estas propiedades son: pindolol, oxprenolol, celiprolol y acebutolol


Cul es el modelo de los bloqueadores beta generales?
El propanolol.

Cules son los bloquadores beta-1 selectivos?
El metoprolol, el esmolol y el atenolol.

Cules son los efectos adversos de los betabloqueadores?
53
Alteran el perfil lipdico aumentando los triglicridos, disminuyendo HDL.
Provocan aumento en las cifras de glucosa plsmtica. Pueden provocar
bloqueo AV, fenmeno de Raynaud, impotencia y depresin.

Cules son las dos acciones de la enzima convertidora de la angio-
tensina (ECA)?
La conversin de la angiotensina I en angitoensina II.
La degradacin de la bradicinina (un potente vasodilatador que
causa liberacin xido ntrico y prostaciclinas del endotelio vascu-
lar)
Disminuye la morbimortalidad en pacientes posinfarto.
Disminuye la remodelacin cardaca.

Cules son algunas indicaciones de los inhibidores de la ECA?
Posinfarto, como antihipertensivos en pacientes con diabetes mellitus e
hiperlipidemias y en la falla cardiaca crnica.
Cules son los efectos adversos de los inhibidores de la ECA?
Hipotensin, tos, fiebre, exantema, hiperkalemia y leucopenia.

Cules son algunas propiedades beneficosas de los IECA?
Incrementan la sensiblidad a la insulina por lo que son utiles en
diabeticos
disminuye la resistencia vascular periferica y la poscarga por lo
qus se utilizan en la insuficiencia cardaca.
Incremento del flujo renal y coronario
Disminuye la presin intraglomerular.

Por qu puede producir hiperkalemia el captopril?
Porque al disminuir la angiotensina II disminuye la aldosterona y se pro-
duce aumento de potasio, disminucin de sodio y prdida de agua.
Cules son las tres clases de bloqueadores de canales de calcio?
Fenilalkilaminas: verapamil.
Benzotiacepinas: diltiazem.
Dihidropiridinas: nifedipina, amlodipina, felodipino.
Cules son los canales de calcio mas conocidos?
54
Los L,T, N, P y R. Los que se encuentran a nivel cardiaco son los L.
Qu ventajas tienen la nicardipina y la amlodipina?
Tienen una menor interaccin con la digoxina y la warfarina.




Cules son las indicaciones de los bloqueadores de los canales de cal-
cio?
Las taquiarritmias supraventriculares,
Sndromes coronarios agudos.
Enfermedad vascular perifrica.
Su falta de efectos en el metabolismo de la glucosa, los lpidos los ha-
ce utiles en hipertensin con dislipidemia y diabetes
Enfermedad renal crnica.
Espasmo esofgico

Cules son los beneficios de los calcioantagonistas en la cardiopata hi-
pertensiva?
Reducen la hipertrofia ventricular izquierda, reducen las arritmias, los de-
fectos de llenado y de contractilidad. Los no dihidropiridnicos reducen el
riesgo de reinfarto.

Cules son los efectos adversos de los bloqueadores de los canales de calcio?
Mareo, constipacin, cefalea e hipotensin.

Cuales farmacos son inhibidores de vasopeptidasa?
Son farmacos capaces de inhibir la endopeptidasa neutra y la ECA lo que
incrementa los pptidos natriurticos e incrementa la vida media de otros
pptidos incluyendo la bradicinana y la adrenomodulina. El omapatrilat es
el primer farmaco de este grupo y se utiliza en hipertensin e insuficien-
cia cardiaca.

Como funcionan los farmacos antagonistas de receptores de angioten-
sina II (ARAII)?
Existen dos tipos de receptores de angiotensina, los AT1 y los AT2. A la
accin sobre los AT1 se deben la mayor parte de efectos conocidos de la
angiotensina. Se indican estos frmacos ante tolerancia a los IECA o
55
bien cuando los efectos adversos por acmulo de bradicinina (tos y an-
gioedema) son importantes.

Qu frmacos son vasodilatadores arteriales?
La hidralazina, el diazxido y el minoxidil.
Qu efectos adversos provoca la hidralazina?
Cefalea y sndrome lupus-like (sndrome similar al lupus)
Cules son los bloqueadores alfa generales?
La fenoxibenzamina y la fentolamina.
Qu son la clonidina y la metildopa??
Son agonistas alfa-2 que acta en el centro presor del tallo y por retroali-
mentacin negativa disminuyen el flujo simptico de noradrenalina cau-
sando disminucin en la TA. Secundariamente desplazan a la
noradrenalina de los receptores alfa perifricos impidiendo su accin en
el msculo liso vascular.
Qu es la prazocina?
Es un bloqueador alfa-1 de accin directa en arteriolas.
Cules son los antihipertensivos centrales (bloqueadores alfa centrales)?
La metildopa, la clonidina, el pentolinium, la prazocina y la guanabencina.
Cules son los antihipertensivos perifricos (bloqueadores alfa perifricos)?
La reserpina (depleta las vesculas de noradrenalina) y la guanetidina (in-
hibe la liberacin de noradrenalina).
Cul es la accin principal de los bloqueadores alfa?
Inhibir la funcin de la noradrenalina.
Cules son algunas indicaciones de los bloqueadores alfa?
Los sndromes coronarios agudos, el periodo posinfarto, las taquiarrit-
mias supraventriculares, el glaucoma y en la migraa.
Cul es el efecto de la prazocina?
La prazocina es un bloqueador alfa-1 selectivo, cuyo efecto consiste en
disminuir la tensin arterial sin afectar el gasto cardiaco.
Cules son los efectos adversos de los bloqueadores alfa?
Prdida de la libido.
56
Rebote cuando cesa su administracin debido a que provocan una
sobreexpresin de los receptores beta.
Anormalidades en los lpidos.
Incontinencia urinaria. Disminuyen el tono de los esfnteres y por eso
tienen utilidad en el prostatismo (terazosina)



Qu es una crisis hipertensiva?
Es una elevacin abrupta y sustancial de la presin arterial que amenaza
la vida o ciertas funciones. En general una TAD mayor de 120 se consi-
dera crisis hipertensiva. Las crisis pueden ser urgencias o emergencias.
Tambien algunos consideran hipertensin acelerada cuando aparecen
cefalea, visin borrosa y focalizaciones. Asimismo se considera hiperten-
sin maligna cuando hay edema de papila.

Qu es una emergencia hipertensiva y cuales son los objetivos de tra-
tamiento?
Es una crisis hipertensiva en la que se demuestra dao a organo blanco
progresivo principalmente en el sistema nervioso central, cardiovascular,
renal y en la retina.
Se busca reducir la TA 25% o bien alcanzar TAD 100-110 en una hora.

Qu familias de frmacos antihipertensivos intravenosos se utilizan para
tratar las crisis hipertensivas?
Vasodilatadores: nitroprusiato, nitroglicerina, diazxido, hidralazina, ni-
cardipina y enalaprilat.
Betabloqueadores: Esmolol, labetalol.
Alfabloqueadores: Fentolamina.
Bloqueadores ganglionares: camsilato de trimetafn
Agonistas dopaminrgicos: Fenoldopam.

Qu es el nitroprusiato de sodio y en que tipo de crisis est indicado?
57
Es un vasodilatador arterial y venoso de accin directa y corta (3-5min)
de eleccin para algunas emergencias hipertensivas. Un producto final
de su metabolismo es el cianuro. La intoxicacin por este se trata con
tiosulfato de sodio para producir un metabolito menos txico que es el
tiocianato. Se usa en dosis de 0.5 a 10 gammas.

Qu es la nitroglicerina y cuando se usa?
Es un nitrato orgnico, tiene efectos a nivel perifrico en los vasos veno-
sos y a nivel de las coronarias. En este caso, causa vasodilatacin veno-
sa reduciendo la precarga, el gasto cardiaco y secundariamente la TA.
Su tiempo de accin es de 3-5 minutos y la dosis es de 5-100g/min. Sus
principales efectos adversos son cefalea nusea y vmito. Tiene su prin-
cipal indicacin en casos de hipertensin y SICA del tipo IAM.

Cules son los frmacos de eleccin en las crisis hipertensivas por feo-
cromocitoma?
Se usan fentolamina y betabloqueadores en combinacin.
Segn el JNC 7 cuales son las dosis teraputicas de los siguientges
frmacos?
Frmaco Dosis (frecuencia) Frmaco Dosis (frecuencia)
Hidroclorotiazida 12.5-50mg/da (1) Valsartan 80-320mg/da (1)
Clortalidona 12,5-25 (1) Diltiazem 180-420 (1)
Furosemide 20-80 (2) Verapamil 80-320 (2)
Espironolactona 25-50 (1-2) Amlodipino 2.5-10 (1)
Metoprolol 50-100 (1-2) Felodipino 2.5-20 (1)
Propranolol 40-160 (2) Nifedipino ret 10-40 (1)
Captopril 25-100 (2) Prazocina 2-20 (2-3)
Enalapril 2.5-40 (1-2) Metildopa 250-1000 (2)
Losartan 25-100 (1-2) Hidralacina 25-100 (2)
Telmisartan 20-80 (1) Minoxidil 25-80 (1-2)

INSUFICIENCIA CARDIACA

Cul es la definicin de insuficiencia cardaca (IC)?
58
Es un sndrome clnico que resulta de cualqier desorden cardiaco funcio-
nal o estructural que impida el adecuado llenado y vaciado del corazn.
Las manifestaciones cardinales de la falla cardiaca son disnea y fatiga
que pueden limitar la tolerancia al ejercicio y retencin de lquidos que
puede condicionar congestin pulmonar o edema perifrico.




Como se clasifica la IC segn su evolucin?
Etapa A: En riesgo de insuficiencia cardiaca pero sin desorden
estructural
Etapa B: Con desorden estructural pero sin sntomas de IC.
Etapa C: Historia de sntomas de IC con dao estructural
Etapa D: IC terminal que requieremanejo especializado, como
inotrpicos, apoyo mecnico externo y finalmente transplante.

Cual es la clasificacin funcional de la IC de la NYHA?
Clase I: Sin limitacin de actividades, sin sntomas
Clase II: Con limitacin ligera, estn cmodos en reposo o con ac-
tividad ligera
Clase III: Marcada limitacin funcional, estn cmodos solo en re-
poso.
Clase IV: Confinado a la cama o silla, cualquier actividad fsica des-
encadena sntomas

Cules son algunas causas identificables de IC?
Muchas veces no puede identificarse un factor desencadenante pero
tambien pueden encontrarse desordenes sistemicos (hipertiroidismi, he-
mocromatosis, hypocalcemia) o antecedente de exposicin a cardiotxi-
cos (alcohol, cocana, antraciclinas o trastuzumab). Debe buscarse
tambin datos de inflamacin o infiltracin cardaca (sarcoidosis, ami-
loidosis).

Cual es la causa mas frecuente de IC?
La cardiopata isqumica es responsable hasta del 70% de la IC.

Cual es el marcador clinico ms confiable para diagnstico de IC?
59
La disnea de esfuerzo es un sntoma con especificidad del 17% pero
sensibilidad del 100%

Cuales son datos clnicos de IC en la exploracin fsica?
Reflujo hepatoyugular: una presin en hipocondrio derecho por 30 se-
gundos con elevacin de 4cm en la presion venosa.
Desviacin del choque de la punta: En paciente en decbito lateral iz-
quierdo. Es positivo cuando a la espiracin el choque rebasa la lnea me-
dio clavicular.
Ritmo de galope: En decubito lateral izquierdo, ausculte con la campana
del estetoscopio.
Ingurguitacin yugular. Medir en decubito supino a 45 grados con la
cabeza hacia la derecha. Medir desde el angulo de Louis.

Cmo se dividen las causas de la IC?
Se dividen en causas subyacentes y causas precipitantes. La causa sub-
yacente mas comn es la cardiopata isqumica, seguida de las cardio-
miopatas, cardiopatas congenitas, enfermedades valvulares y
finalmente la hipertensin.

Cuales son las formas clnicas principales de IC?
Sistlica y diastlica
De alto gasto y bajo gasto
Aguda y crnica
Derecha e izquierda
Antergrada y retrigrada


Cuales son los criterios de Framingham para la falla cardiaca?
Mayores:
3 pulmonares: edema pulmonar, disnea paroxstica nocturna y es-
tertores
3 de cuello: reflujo hepatoyugular, presion venosa>16cmH2O, dis-
tensin yugular
2 cardiacos: cardiomegalia y galope S3.
Menores
60
4 Respiratorios: capacidad vital a 1/3 de lo normal, disnea de
ejercicio, derrame pleural y tos nocturna.
Edema de extremidades
Hepatomegalia
Taquicardia mayor de 120lpm
Mayor o menor
Perdida de mas de 4.5kg despus de 5 das de tratamiento
El diagnstico requiere al menos un mayor y dos menores.

Qu tipo de IC causan las miocardiopatas y como se dividen?
Causan IC de tipo diastlico. Se estudian como primarias y secundarias.
Primarias son la miocardiopata restrictiva idioptica, la fibrosis endo-
miocrdica (enfermedad de Davies) y la miocardiopata eosinoflica (en-
fermedad de Lffler).
Secundarias son la amiloidosis, la hemocromatosis, las enfermedades
por atesoramiento y la sarcoidosis entre otras.

Cuales son los criterios ecocardiograficos de IC diastlica?
Tiempo de relajacin isovolumtrica (IVRT) mayor de 100.
La relacin entre el flujo transmitral precoz (ondaE) y el tardo (onda A,
sistole auricular). La relacion normal en jvenes es 2:1 y en mayores de
50 1:1. Una relacin E/A baja con IVRT prolongado corresponde a IC
diastlica
Cules son causas de IC de alto gasto?
Anemia, Fstulas arteriovenosas (sndrome de Osler-Weber-Rendu)
hipertiroidismo, beri-beri (deficiencia de tiamina), la enfermedad de Pa-
get, el sndrome de Albright, el mieloma mltiple y el embarazo.

Qu es la enfermedad de Paget?
Es una enfermedad en la que la resorcin y la formacin sea estn au-
mentadas provocando cortocircuitos intraseos, su causa es probable-
mente gentica o viral

Qu es el sndrome de Osler Weber Rend?
Es un desorden autosmico dominante tambin conocido como, telan-
giectasia hemorrgica hereditaria en la que hay epistaxis y sangrados in-
testinales frecuentes por malformaciones capilares y en el que puede
haber grandes fstulas arteriovenosas pulmonares.

Qu es el sndrome de McCune-Albright?
61
Tambin llamado simplemente sndrome de Albright, se caracteriza por la
triada de manchas caf con leche, displasia fibrosa polistica y disfuncin
endcrina que puede incuir tirotoxicosis, acromegalia., hirsutismo o galacto-
rrea. Hay que dirferenciarlo de la neurofibromatosis, eslcerosis tuberosa,
ataxia telangiectasia, sndrome de Fanconi e hiperplasia adrenal congnita
as como de tumores adrenales o gonadales entre otros.

Cules son las caractersticas clnicas de la IC de alto gasto?
Taquicardia con aumento de la presin del pulso
Latido hiperdinmico
Calor en manos y pis
En ocasiones soplo discreto en el pex y el borde esternal iz-
quierdo.




Cules son los frmacos que deben evitarse en IC?
Antiarrtmicos: Pueden ser cardiodepresores o proarrtmicos. Solo se
aprueba la amiodarona.
Calcioantagonistas: Aumentan el riesgo de eventos cardiovaaculares,
excepto el amplodipino.
AINES: pueden causar vasoconstriccin perifrica y retencin de sodio

Cuales son los 4 farmacos de uso recomendado en IC?
Diurticos, IECA, betabloqueadores y digitlicos. Su uso depende de las
caracteristicas clinicas predominantes.

En quienes pueden ser perjudiciales los IECA?
En aquellos pacientes que dada su patologa de base tienen muy dismi-
nuido el flujo renal y por tanto dependen de la regulacin del sistema re-
nina angiotensina aldosterona, en especial de la accin de la
angiotensina en la arteriola eferente. Al bloquear su produccin los pa-
cientes pueden caer en insuficiencia renal. Los pacientes con alto riesgo
de esta complicacion son los que presenta IC clase IV de la NYHA, los
pacientes hiponatrmicos, los que tienen estenosis de la arteria renal bi-
lateral o los consumidores de AINES (ya que prostaciclina, prostaglandi-
nas y tromboxanos tambin autoregulan favorablemente el flujo renal)

Cuales son las ventajas de los diureticos en IC?
62
Su efecto aparece en das u horas, a diferencia de la digital, betablo-
queadores o IECA. Cuyo efecto se establece eficazmente en semanas.
Deben usarse asociados

Deben usarse betabloqueadores en pacientes con IC clnicamente es-
tables?
Aunque hubiere una buena respuesta al uso de IECA, diurticos y digit-
licos, los betabloqueadores deben prescribirse (si no tienen contraindica-
cin) ya que disminuyen el riesgo de la progresin de la enfermedad, de
muerte sbita y de deterioro clnico.

Que valores hemodinamicos nos guian en el tratamiento de la IC agu-
da?
El Indice Cardiaco: se considera disminuido cuando es <2.2L/min/m
2

La presin capilar pulmonar en cua (se encuentra como PCWP en in-
gls): se considera baja si es <14mmHg y alta si es > 18-20mmHg.
Qu medicamentos se utilizan para aliviar los sntomas de la sobrecarga
(edema, ortopnea)?
Los diurticos.
Cul es el tratamiento ideal de la insuficiencia cardiaca con falla sistlica?
Digoxina, ms un inhibidor de la ECA (enzima de conversin de la an-
giotensina).
Cul es el tratamiento de la falla cardiaca diastlica?
Diurticos, ms -bloqueadores (que disminuyen la frecuencia cardiaca).
Cul es el tratamiento del edema pulmonar agudo?
Oxgeno, diurticos, vasodilatadores (nitroprusiato o nitroglicerina), sul-
fato de morfina, intubacin y presin positiva.
Cules son los medicamentos que se utilizan en la falla cardiaca congestiva?
Vasodilatadores: inhibidores de la ECA, hidralazina, isosorbide,
minoxidil y nitroprusiato de sodio.
Diurticos: diurticos de asa, tiazidas.
Inotrpicos: glucsidos cardiacos (digoxina y digitoxina), agonistas
-adrenrgicos (dobutamina) e inhibidores de la fosfodiesterasa
(amrinona y milrinona).
Cundo estn indicados los digitlicos?
63
Cuando existe insuficiencia cardiaca izquierda (no as en la derecha).
Cules son los beneficios del uso de digitlicos?
El tratamiento por 1-3 meses mejora la calidad de vida, la tolerancia al
ejercicio y reduce los sntomas, por tanto, una pauta para su uso es ini-
ciarlos ante una mediana respuesta al uso IECAs, diurticos o betablo-
queadores con persistencia de la sintomatologa.

Que medicamentos se utilizan para la fibrilacin auricular (FA) con IC?
Se utiliza rutinariamente digoxina si adems existe fibrilacin auricular.
Sin embargo los betabloqueadores controlan mejor la respuesta ventricu-
lar ante el estmulo auricular sobre todo durante el ejercicio.




Cmo funcionan los digitlicos?
Inhibiendo la actividad de la Na-K ATPasa. Esta accin a nivel car-
daco provoca aumento del inotropismo mediante el aumento de Na y
Ca intracelulares. A nivel de las fibras vagales sensibiliza a los baro-
receptores cardiacos que secundriamente disminuyen el flujo simpti-
co eferente del sistema nervioso central. A nivel renal disminuye la
resorcin tubular de sodio y el aumento consecuente de sodio a nivel
del tbulo distal causa inhibicin de la secrecin de renina.

A que dosis se usa comunmente la digoxina?
Se inicia y se mantiene a dosis de 0.125 a 0.25 mg diarios. En pacinetes
mayores de 70 aos se usan dosis bajas de 0.125, tambin en los pa-
cientes con insuficiencia renal y aquellos con poca masa muscular

Cuales son los efectos adversos de los digitlicos?
Arritmias cardiacas: ritmos ectpicos, de reentrada y bloqueos cardia-
cos. El mas importante es la taquicardia ventricular.
Sintomas gastrointestinales: anorexia nausea y vmito
Sntomas neurolgicos: anormalidades visuales, desorientacin y con-
fusin.
La toxicidad aparece en niveles sricos por arriba de 2 mg pero ocurre
con niveles menores si existen hipocalemia, hipomagnesemia e hipotiroi-
dismo.
64
.
Qu medicamentos propician la toxicidad digitlica?
El uso concomitante de quinidina, verapamilo, espironolctona, fecainida,
propafenona o amiodarona pueden incrementar la digoxina srica y favo-
recer su toxicidad. Si se usan estos frmacos solo hay que disminuir la
dosis de digoxina.
Cmo se maneja la intoxicacin por digital?
Prehospitalario: oxgeno, monitoreo cardiaco.
Hospitalario:
Guiar la terapeutica con niveles de digoxina.
Si la intoxicacion es leve solo suspender el medicamento
A las bradiarritmias inestables se les aplica atropina de 0.5 a 1
mg/dosis o marcapasos
En cao de contracciones ventriculares prematuras, bigemninismo
o trigeminismo inestables utilizar lidocana.
La taquicardia ventricular responde a:
Anticuerpos FAB contra digoxina (digibind)
Lidocana en bolos de 100mg o en infusin de 1-4mg/mg.
Fenitona sdica IV en bolos de 100mg c/5-10 minutos
hasta alcanzar una dosis de carga de 15mg/kg.
Magnesio en bolo de 2g y luego en infusin de 1 a 2g
tambin es til.
Evitar el uso de procainamida y bretilio.
Cules son los efectos cardiovasculares de la adrenalina?
Aumenta la frecuencia cardiaca y hace ms amplia la tensin arterial
(aumenta la sistlica y disminuye la diastlica).
Cules son los efectos cardiovasculares de la noradrenalina?
Mantiene la frecuencia cardiaca normal y tiene un efecto presor neto
pues aumenta igualmente la tensin arterial sistlica y la diastlica.

CARDIOPATA ISQUEMICA
Cuales son y como se dividen los factores de riesgo coronario?
No modificables
Sexo masculino
Edad (55 en hombres y 65 en mujeres)
65
Menopausia
Historia familiar de enfermedades cardiovasculares prematuras
(antes de 55 en hombres y antes de 65 en mujeres)
Modificables
Tabaquismo
Consumo de alcohol
Obesidad (IMC>30kg/m
2
)
Sedentarismo
Dislipidemia
Fibringeno
Anticonceptivos orales
Hipertensin
Diabetes Mellitus
Microalbuminuria o TFG<60ml/min

Cul es el perfil lipdico deseable en los pacientes diabticos?
LDL < 100mg/dl
TG < 150mg/dl
HDL > 45mg/dl

Cul es el papel de las lipoprotenas de alta densidad en la aterosclerosis?
Extraen el colesterol de la ntima de los vasos por lo que tienen un efecto
protector.
Cul es el ndice de la aterosclerosis entre el sexo masculino y el femenino?
4:1
Cul es la personalidad que se asocia a la aterosclerosis?
La personalidad tipo A.
Cules son los llamados nuevos factores de riesgo cardiovascu-
lar?
La homocistena, el fibringeno, el aumento de la reactividad de las pla-
quetas, la lipoprotena (a) y la hipercoagulabilidad se mencionan como
algunos nuevos factores de riesgo, de estos el mas importante es la ho-
mocistena.

66
Cules son los sndromes coronarios agudos?
Angina inestable (AI), infarto no Q y el infarto agudo del miocardio.

Qu otros nombres recibe el infarto no Q?
No transmural, no ST, sin elevacin ST (IAMNST).

Cul es la diferencia fisiopatolgica entre el sndrome AI/IAMNST y el
IAMST?
La angina inestable y el IAM NST son extensiones de un mismo sdrome
clnico en el que la base de la fisiopatologa es un imbalance entre el
aporte y la demanda de oxgeno, en estas la causa es la inestabilidad de
la placa que provoca un trombo no oclusivo. En el IAM ST el trombo
ocluye completamente la luz del vaso.

A qu se le llama angina?
Se llama angina al sndrome clnico en el cual hay discomfort o dolor de
pecho, mandbula, espalda, hombro o brazo. Esta molestia puede ser
igualmente de origen cardiaco, esofgico, de la pared torcica o de los
pulmones. Para caracterizarse correctamente deben interrogarse, cali-
dad, localizacin, duracin, factores precipitantes y atenuantes. Se divide
en angina estable e inestable.

Cul es la clasificacin de la angina de la Sociedad Cardiovascular Ca-
nadiense (CCSC)?
Clase I: Actividad fsica ordinaria sin angina. La angina solo ocurre en
actividad fsica extenuante.
Clase II: Angina en las primeras horas de la maana, al subir escaleras
mas de un piso, caminar 2 cuadras, despus de las comidas, en ambien-
te fro o bajo estrs.
Clase III: Limitaciones marcadas de la actividad fsica. Angina al caminar
una cuadra, o subir escaleras solo un piso.
Clase IV: Inhabilidad de cualquier actividad fsica por presencia de angi-
na

Cul es el cuadro clnico tpico de la angina estable?
Se presenta en pacientes mayores de 50 aos, con historia de males-
tar precordial descrito como opresivo y el puo cerrado sobre el pe-
cho. El dolor se irradia al hombro izquierdo y a ambos brazos. Es
desencadenado por el ejercicio y emociones excesivas y aliviado por
el descanso.
67

A qu se llama angina inestable?
Operacionalmente la angina inestable es aquella que se presenta de una
de estas tres maneras:
Angina en reposo: angina de mas de 20 minutos que haya sucedido du-
rante la semana previa a la valoracin.
Angina de reciente inicio: Angina de grado III de la CCSC que haya ini-
ciado al menos dos meses antes de la valoracin.
Angina en aumento: Angina ya conocida con patrn cambiante es de-
cir: mas frecuente, con mayor duracin o un menor umbral de presenta-
cin.
Algunos textos tambin consideran inestable la angina posinfarto.

Cul es la diferencia clnica y electrocardiogrfica entre angina inestable
y un infarto sin elevacin ST?
No existe diferencia clnica o electrocardiogrfica. Ambas pueden no te-
ner alteraciones en el ST o bien alteraciones inespecficas como un in-
fradesnivel. La diferencia es que el infarto sin elevacin ST s tiene
cambios enzimticos.

Cules son los cinco parmetros para realizar la estratificacin del ries-
go de la angina inestable?
La historia clnica, las caractersticas del dolor, los hallazgos clnicos, los
hallazgos electrocardiogrficos y los marcadores bioqumicos cardiacos.

Cul es el perfil de la angina inestable de alto riesgo?
Sntomas en aumento durante un periodo de 48hr
Angina de reposo en evolucin con duracin mayor a 20 minutos
Cambios clnicos probablemente secundarios a isquemia (edema
pulmonar, nuevo soplo, hipotensin, taquicardia o bradicardia) y
edad mayor a 75 aos
Angina de reposo con: cambios en el ST mayores a 0.05mv, nue-
vo bloqueo de rama o taquicardia ventricular sostenida.
Marcadores cardiacos francamente elevados.
Se considera de alto riesgo con la presencia de cualquiera de los
anteriores.

Cul es el perfil de la angina inestable de riesgo intermedio?
Historia previa de enfermedad arteriooclusiva como: IAM, EVC,
revascularizacin o uso de aspirina.
68
Angina de reposo mayor de 20 minutos que ha cedido per se o
angina <20min que cede con nitratos.
Inversin de la onda T mayor a 0.2 mv
Ondas Q patolgicas
Marcadores cardiacos medianamente elevados.
No debe existir ningun indicador de riesgo alto y al menos uno de
los anteriores

Cul es el perfil de la angina inestable de riesgo bajo?
Angina clase III o IV en ls dos semanas previas.
EKG sin cambis durante el episodio anginoso.
Marcadores normales
Sin indicadores de riesgo alto o intermedio


Qu pacientes deben manejarse con medidas de isquemia aguda?
Pacientes que al ingreso presenten:
Cambios en el ST o en la onda T
Angina en evolucin
Cambios hemodinmicos
Marcadores bioqumicos positivos
Prueba de esfuerzo positiva
Pacientes que se presentan sin cambios pero que los desarrollan al ser
observados de 4 a 8 hr.

Cules son las medidas para la isquemia aguda?
Antiagregantes plaquetarios: Acido acetil saliclico, tienopiridinas
(ticlopidina y clopidogrel) o antagonistas de la glicoproteina IIb/IIIa
Medidas antiisquemia: oxgeno, betabloqueadores, calcioantago-
nistas, IECA.
Anticoagulacin: heparina no fraccionada, heparina de alto peso
molecular.

Cul es la principal toxicidad de la ticlopidina?
La toxicidad hematolgica es la mas frecuente haciendo que este total-
mente contraindicada en pacientes con antecedentes de agranulocitosis
trombocitopenia o leucopenia.

Cmo es la farmacocintica del clopidogrel?
69
Se puede encontrar en el plasma despus de una hora de su administra-
cin, tiene vida media de 8 hr. Tiene un efecto antiagragante significativo
a los 2-3 das de uso pero su efecto mximo aparece hasta los 4 a 7 das
de uso y el efecto persiste hasta por 7 a 10 das despus del cese del
tratamiento.

Cundo estn recomendados los inhibidores de la GP Iib/IIIa?
Cuando persiste la sintomatologa despus de que se ha iniciado
terapia con ASA y tienopiridinas.
Cuando hay un perfil de alto riesgo
Cuando se planea realizar intervencin percutnea posteriormen-
te.

Cul es el mecanismo de accin de los antagonistas GPIIb/IIIa?
Eptifibatide y tirofiban: son antagonistas sintticos que imitan la secuen-
cia de aminocidos del fibringeno y tienen afinidad especfica por el re-
ceptor con vida media de 2 a 3 hr.
Abciximab: Es un fragmento FAB murino humanizado que acta contra
GPIIB/IIIA

En qu casos no se indican las pruebas de esfuerzo para el estudio de la
angina?
En la angina de Prinzmetal y en la angina inestable.

Cundo se considera una prueba de esfuerzo positiva en los casos de an-
gina estable?
Cuando presenta una depresin del intervalo ST mayor a 0.1 mV bajo la
lnea de base y con una duracin de 80 milisegundos a partir del punto J,
el segmento ST debe ser horizontal o descendente. Con menos frecuencia
se observa positividad del ST con criterios similares a los del infranivel
(0.5 a 10% de los casos).

Cules son las indicaciones de la arteriografa coronaria?
Pacientes con angina crnica estable.
Pacientes con angina inestable severamente sintomticos a pesar de la te-
raputica farmacolgica y que son prospectos para revascularizacin.
Pacientes sintomticos que no han sido diagnosticados propiamente.
70

Cul es la angina de Heberden?
La angina estable con caractersticas clsicas asociada a esfuerzo fsico.

Qu es la angina de Prinzmetal?
La angina variante de Prinzmetal e una forma de angina inestable
que se caraceriza por una elevacin cclica y transitoria de el seg-
mento ST que se resuelve comunmente sin progresar a IAM, es cau-
sada por espasmo de las coronarias y presenta respuesta a
nitroglicerina, nitratos de accin prolongada y a calcioantagonistas en
dosis altas (nifedipino 60 a 120 mg/d, verapamilo 240 a 480 mg/d y
diltiazem 120 a 360 mg/d)


Qu es el sndrome X coronario?
Es un sndrome de vasodilatacin coronaria insuficiente causada por
un tono vascular aumentado. En el que los pacientes presentan angi-
na o dolor de tipo anginoso con el ejercicio y presentan infradesnive-
les en el segmento ST en la prueba de esfuerzo pero coronarias
normales con flujo lento en la angiografa.

Cules son los dos signos clnicos de la angina?
El ritmo de galope y el soplo apical transitorio.
Cmo se evidencia el espasmo coronario en el episodio anginoso?
Por una elevacin concomitante del segmento S-T.

Cmo se clasifican los frmacos antianginosos?
Se clasifican en nitratos, beta-bloqueadores y bloqueadores de canales de
calcio.
Cmo actan los nitratos?
Los nitratos, como el dinitrato de isosorbide y la nitroglicerina, relajan el
msculo liso vascular por su conversin intracelular a nitritos y poste-
riormente a xido ntrico que activa a la guanilatociclasa aumentando los
niveles intracelulares de GMPc. El GMPc defosforila a las cadenas lige-
ras de miosina, lo que resulta en la relajacin del msculo liso vascular.
71

Cul es el cuadro clnico del infarto del miocardio?
20 a 60 % de los pacientes presenta agravamiento de una angina pre-
existente.
El dolor dura ms de 30 minutos; es retroesternal y opresivo.
El dolor se irradia al cuello, el trax anterior y posterior, la mandbu-
la, el epigastrio, los brazos y los hombros.
La tercera parte de los pacientes presenta disnea como sntoma aso-
ciado.
Aparece nusea y vmito en 40% de los casos y diaforesis en 50%.
Tambin pueden coexistir palpitaciones, debilidad generalizada, sn-
cope, dficit neurolgico, embolias perifricas y confusin mental.
Cules son las enzimas de valor diagnstico en el infarto agudo del mio-
cardio?
La fraccin MB de la creatincinasa (CKMB): cuando es mayor 16UI,
ms de 4% de la CK total, a las 6 horas. Tiene sensibilidad y especifi-
cidad mayores a 97%. Las determinaciones seriadas de CKMB son el
estndar de oro para diagnstico de infarto agudo del miocardio.
La creatincinasa (CK) total: es positiva en las primeras 4 a 8 horas y
alcanza su valor pico a las 24 horas en promedio.
La deshidrogenasa lctica (DHL): sobrepasa su valor normal en las
primeras 24 a 48 horas y alcanza su valor pico de 3 a 6 das
Trop T, I: tienen una sensibilidad de 50% en las primeras 2 a 4 horas
y de 75% despus de 4 horas. Son marcadores inequvocos de necro-
sis miocrdica an con un electrocardiograma normal.

72
Cundo es mas baja la sensibilidad de la de la CKMB?
La sensibilidad de la CKMB para detectar un IAM es muy baja antes
de 6hr y despues de 36hr de ocurrido el evento isqumico.

Cul es el marcador cardiaco mas util para el IAM temprano?
Las isoformas de la CKMB son el marcador mas util para IAM de 3 a
6 hr despues del inicio de los stomas. Las troponinas y la CKMB total
tienen baja sensibilidad en este periodo.

Con que entidades se hace diagnstico diferencial del IAM?
Hay que diferenciar el IAM de enfermedades como pericarditis, miocardi-
tis, diseccin artica aguda, embolismo pulmonar, y colecisttis aguda en-
tre otros.

Cmo se realiza el diagnstico definitivo electrocardiogrfico de IAM?
Al tomar el EKG debe existir un supradesnivel del segmento ST de mas
de 1mm, en dos derivaciones contiguas, a menudo con cambios en espe-
jo recprocos en las derivaciones contralaterales.

Cmo se clasifican los infartos segn el grupo de derivaciones afectadas?
V
1
, V
2
Septal
V
1
, V
2
,

V
3
Anteroseptal
V
2
, V
3
Apical
V
1
, V
2
,

V
3
, V
4
Anterior
V
1
, V
2
,

V
3
, V
4
,

V
5
, V
6
Anterior extenso
D
1
, AVL,V
5
, V
6
Lateral
D
1
, AVL Lateral alto
V
5
, V
6
Lateral bajo
D
II
, D
III
, AVF Inferior
V
1
, V
2
,

V
3
, V
4
,

V
5
, V
6,
DI, AVL

Anterior extenso en extensin lateral.

Cules son los cinco subgrupos de infartos segn la arteria responsa-
ble?
Anterior extenso con trastornos de conduccin: territorio de la
descendente anterior (DA)
Anterior extenso sin trastorno de la coduccin: DA media
Lateral: DA distal o bien la primera diagonal o la circunfleja.
73
Inferior extenso con cambios en espejo: territorio de la cronaria
derecha (CD) proxmal.
Inferior pequeo: CD distal.

Cules son los criterios para diagnosticar IAM en presencia de bloqueo
de rama izquierda?
Elevacin del ST>1mm concordante con el QRS en DII (5puntos)
Depresin del ST>1mm en V1, V2 y V3 (3 puntos)
Elevacin del ST mayor de 5 mm discordante con el QRS en DIII
y AVF (2 puntos)
Si se suman mas de 3 puntos la especificidad es mayor al 90%

Cul es la estratificacin del riesgo del IAM segn Killip y Kimball?








Cules son las medidas terapeuticas iniciales para el IAM?
Aspirina y teprapia antiplaquetaria: Aspirina (325mg vo seguidos de
150mg/d), Ticlopidina (500mg vo seguidos de 250mg c/12hr), clopidogrel
(300mg vo seguidos de 75mg/d).
Oxgeno
Nitroglicerina: 0.4mg sublingual, 10 a 20 mcg/min IV con aumentos de 5
a 10 mcg cada 5 a 10 min
Reperfusin.

Cules son las terapias de reperfusin aceptadas?
Fibrinolisis y la reperfusin por va percutnea.

Clase Caractersticas Mortalidad (%)
I Sin evidenca de ICC 5.1
II Crepitantes, ingurgitacin yuguglar y
3er ruido
13.6
III Edema pulmonar 32.2
IV Choque cardiognico 57.8
74
Cules son indicaciones precisas de fibrinolisis?
Clase I: En ausencia de contraindicaciones, pacientes dentro de las pri-
meras 12 horas desde el inicio de los sntomas con elevacin de ms de
0.1mv del segmento ST de dos derivaciones contiguas o bien con blo-
queo de rama izquierda nuevo .
Clase II: En presencia de un infarto posterior verdadero o si aun se en-
cuentra dentro de 24 hr del inicio de los sntomas.
Clase III: Nunca despus de 24 hr, ni cuando existan solo infradesniveles
ST sin comprobar que se trata de un infarto posterior verdadero.

En que situaciones se prefiere utilizar fibrinolisis?
Cuando el tiemo de evolucion es menor a 3 hrs
Cuando el tiempo desde la admisin hasta la angioplasta (door-
to-balloon) es mayor de 90 minutos.
Cuando no se dispone de equipo para angioplasta o el personal
no est suficientemente capacitado.
Si estas condiciones no se cumpen es preferible la angioplasta primaria.

Cmo acta la estreptocinasa?
Produciendo plasmina a partir de plasmingeno

Cules son las dos contraindicaciones absolutas de la trombolisis?
Hemorragia interna activa y evento vascular reciente.

Cules son las indicaciones precisas de angioplasta primaria?
Clase I:
En pacientes que cumplan los criterios electrocardiogrficos que
estn dentro de las primeras 12 hr de los sntomas, a menos de
90 minutos desde admisin hasta angioplasta (door-to-balloon).
Excepto aquellos que como ya se mencion tengan menos de 3hr
de evolucin y que no tenan angioplasta disponible en la hora si-
guiente.
En pacientes menores de 75 aos.
En aquellos inelegibles para fibrinolisis.
Clase III: Nunca en pacientes que se encuentran estables hemodinmica
y elcticamente a mas de 12 horas de evolucin.

En quienes se encuentra indicada la ciruga de puenteo de arterias co-
ronarias (Coronary artery bypass grafting-CABG) de urgencia?
75
Clase I:
En aquellos pacientes con angioplastia fallida que continuan ines-
tables
En pacientes con contraindicaciones para angioplasta o tromboli-
sis
En pacientes menores de 75 aos con las condiciones previas
Cuando la anatoma coronaria es adecuada para el procedimien-
to.

En quienes debe sospecharse infarto del ventrculo dercho (VD)?
Debe buscarse evidencia de infarto del VD en todos aquellos pacientes
que tengan infarto inferior y en los qe se presenta el sndrome clnico de
hipotensin, aumento de la presin venosa yugular y auscultacin pul-
monar normal.

Cuales son las manifestaciones elecrocardiograficas de infarto del VD?
Se debe realizar la toma de derivaciones derechas en especial la deriva-
cin V4R. Puede acompaarse de supradesniveles en el ST mayores a 1
mm en v1 a v3. Los signos electrocardiogrficos se resuelven en las 10
primeras horas de evolucin.

Cules son las medidas de tratamiento del infarto del VD?
Mantenimiento de la precarga, dismiucin de la poscarga del VD, soporte
inotrpico del VD y reporfusin temprana.

Cules son los dos principales tipos de complicaciones de un IAM?
Electricas: Taquicardia ventricular, taquicardia supraventricular, extrass-
toles ventriculares, firilacin ventricular, ritmos acelerados de la unin y
disfuncin sinusal entre otros.
Mecnicas: Ruptura de cuerdas tendinosas con disfuncin valvular mi-
tral, ruptura septal, ruptura de la pared libre y ruptura de aneurisma ven-
tricular.

Cules son los objetivos de la prevencin secundaria de un IAM?


76
Por cul mecanismo el captopril es un protector posinfarto contra insufu-
ciencia cardiaca secundaria?
Disminuye la dilatacin ventricular y la falla de bomba, ya que tambin
disminuye la remodelacin muscular patolgica; adems atena la acti-
vidad neurohumoral.

ALTERACIONES DE LA CONDUCCIN
Cunto dura la demora aurculo-ventricular normal?
100 milisegundos.
Cules son las principales perturbaciones de la conduccin?
El bloqueo sinoauricular, el bloqueo aurculo-venticular y el bloqueo in-
traventricular.
Cules son las principales manifestaciones de la disfuncin del seno?
Bradicardia sinusual extrema.
Paros sinisuales.
Bloqueos sinoauriculares.
Sndrome de bradicardia-taquicardia.
Fibrilacin auricular de respuesta ventricular lenta.
Ritmos de escape de la unin (nodal).
Cules son los tipos de bloqueo sinoauricular?
Bloqueo de salida sinoauricular de primer grado: el tiempo de conduccin
sinoauricular est aumentado, no se detecta por electrocardiografa
convencional; requiere registros intracardiacos.
Bloqueo de salida sinoauricular de segundo grado: es una falla intermitente
en la conduccin del impulso; se manifiesta por la ausencia intermi-
tente de ondas P.
Bloqueo de salida sinoaricular de tercer grado o completo: se caracteriza por
la falta de actividad auricular, o bien por la presencia de un marcapa-
so ectpico; slo se puede distinguir del paro sinusal mediante regis-
tros intracardiacos de la actividad sinusal.
Qu es el bloqueo aurculo-ventricular de primer grado?
Es un retraso en la conduccin evidenciado por un intervalo P-R aumen-
tado ms de 0.21 seg. Clnicamente se observa en los sujetos con tono va-
77
gal aumentado (atletas), pacientes que utilizan frmacos que causan bra-
dicardia sinusal como la digital, los bloqueadores de canales de calcio o
los simpaticolticos (beta-bloqueadores), en las miocardiopatas, en los in-
fartos de la pared inferior.
Qu es el bloqueo aurculo-ventricular de segundo grado, tipo Mobitz I?
Tambin se conoce como bloqueo de Wenckebach, ya que en este blo-
queo el intervalo P-R se prolonga progresivamente hasta lograr el blo-
queo de un impulso. Ocurre secundariamente a una afeccin nodal. La
pausa siguiente no se considera pausa compensatoria, pues es menor que
dos intervalos sinusales normales. Se reconoce fcilmente si el intervalo
P-R del nuevo impulso es claramente menor del que ocurri en el ltimo
impulso antes de la pausa. Se puede observar en la intoxicacin por cual-
quier frmaco que cause bradicardia sinusal, en los infartos de la pared
inferior y en los individuos con tono vagal aumentado. No requiere tera-
pia agresiva.
Qu es el bloqueo aurculo-ventricular de segundo grado, tipo Mobitz II?
Es un bloqueo secundario a un dao dentro del sistema His-Purkinge;
por tanto, se ubica infranodalmente. Se puede observar un P-R prolonga-
do constante, ya sea en relacin 2:1, 3:1, etc. Ocurre generalmente tras
los infartos nteroseptales o en los desrdenes esclerodegenerativos del
esqueleto fibroso del corazn. Es de gran importancia reconocerlo, ya
que evoluciona frecuentemente a bloqueo aurcilo-ventricular completo,
por lo que requiere implantacin de marcapaso.
Qu es el bloqueo aurculo-ventricular completo?
Es aquel en que observamos disociacin aurculo-ventricular y ritmo de
escape.
Cules son los tipos de bloqueo aurculo-ventricular completo?
De QRS normal: cuando el escape es de morfologa nodal, la frecuencia
es de 40 a 55 latidos por minuto; y se incrementa por el ejercicio o los
atropnicos; la lesin probablemente se ubica en el nodo aurculo-
ventricular.
De QRS ancho: cuando el escape es de morfologa ventricular y las fre-
cuencias menores a 40 latidos por minuto la lesin se encuentra en el
Haz de His o distal a ste.
78
Qu son las crisis de Stokes-Adams?
Son episodios de isquemia cerebral que se manifiestan como sncope se-
cundario a bloqueo A-V avanzado.
Cmo se trata la disociacin A-V?
Descontinuando cualquier agente que cause bradicardia sinusal, acele-
rando el ritmo con vagolticos (anticolinrgicos) o mediante la insercin
de un marcapaso si el ritmo de escape es insuficiente.
Qu es un escape nodal?
Es un QRS angosto, no precedido de onda P y que ocurre posteriormente
a una pausa diastlica mayor que la normal (0.64 segundos en lugar de
0.54); por tanto, se puede observar en los diferentes bloqueos, pausas y
bradicardias.
Cules son las caractersticas de un escape ventricular?
Son complejos QRS posteriores a una pausa anormal de morfologa se-
mejante a la que se observa en el bloqueo de rama avanzado y no prece-
dido de una onda P.
Qu es un bloqueo de rama?
Es un dao intrnseco de la conduccin del sistema intraventricular, ya
sea izquierdo o derecho, que se manifiesta por un QRS prolongado: ms
de 120 milisegundos, si es completo (avanzado), y de 100 a 120 milise-
gundos si es incompleto. El vector del QRS se orienta en direccin de la
regin miocrdica donde la despolarizacin est retardada.
Cules son las caractersticas del bloqueo de la rama derecha del Haz de
His?
Presenta complejos predominantemente positivos en V-1 tipo rSR y nega-
tivos en V-6 del tipo qRS; el vector del QRS se orienta hacia la izquierda.
Podemos observarlo en la enfermedad cardiaca congnita (septal) o ad-
quirida (valvular).
Cules son las caractersticas del bloqueo de la rama derecha del Haz de
His?
Presenta complejos predominantemente negativos en V-1 de tipo QS y
positivos en V-6 de tipo R; el vector del QRS se orienta hacia la derecha.
79
Se observa en la cardiopata isqumica, la hipertensin crnica, la car-
diomiopata o la enfermedad valvular artica severa.
Cules son las caractersticas del bloqueo parcial?
Es un bloqueo que slo afecta a uno de los fascculos de la rama izquier-
da del Haz de His, ya sea el anterior o el posterior (hemibloqueo), que
generalmente no prolonga la duracin del QRS pero s altera la direccin
del vector QRS.
Cules son los tipos de bloqueo bifascicular?
Bloqueo de la rama derecha del Haz de His y bloqueo fascicular iz-
quierdo posterior.
Bloqueo de la rama derecha del Haz de His y bloqueo fascicular iz-
quierdo anterior.
Bloqueo de la rama derecha del Haz de His completo.
La alternancia entre BRDHH y BRIHH es un signo de enfermedad tri-
fascicular.
Qu es el sndrome de Wolff- Parkinson-White?
Es un sndrome caracterizado por un intervalo P-R corto debido a que
existe empastamiento de la onda P en la rama ascendente del QRS, lo
que se conoce como onda delta. Tambin hay onda T invertida.

ALTERACIONES DEL RITMO.
Cmo se clasifican las arritmias?
En taquiarritmias y bradiarritmias.
Cules son la bradiarritmias con conduccin aurculo-ventricular normal?
La bradicardia sinusal y la pausa sinusal.
Cules son los ritmos pasivos?
Los escapes, la migracin del marcapaso, y los ritmos nodal e idioventri-
cular.
Cules son los focos miocrdicos que determinan el automatismo cardiaco?
Nodo sinusal o de Keith y Flack: determina el ritmo sinusal, su rango
de frecuencia normal es de 60 a 100 latidos por minuto.
Nodo aurculo-ventricular o de Aschoff-Tawara: determina el ritmo
nodal; su frecuencia va de 50 a 60 latidos por minuto.
80
Haz de His y fibras de Purkinge: cuando estos centros toman la fun-
cin de marcapasos se llama ritmo idioventricular. El ritmo es de 40 a
50 y menor de 40 latidos por minuto respectivamente.
Cules son las caractersticas del ritmo nodal?
La frecuencia cardiaca es de 50 a 60 latidos por minuto; la morfologa del
QRS es normal, o bien, semejante a la del bloqueo de rama y la onda P
es negativa en D-II, D-III y AVF y puede preceder o seguir al QRS
Cules son las caractersticas del ritmo idioventricular?
La frecuencia cardiaca es menor de 40 latidos por minuto, el complejo
QRS es ancho y mellado y hay ausencia de onda P.
Qu es el ritmo idioventricular acelerado?
Tambin se conoce como taquicardia ventricular lenta, ya que tiene fre-
cuencias de 60 a 120 pero con morfologa idioventricular. Ocurre en el
infarto del miocardio o durante la reperfusin.
Cmo se clasifican las extrasstoles supraventriculares?
En auriculares y complejos aurculo-ventricular de la unin.
De dnde provienen los complejos aurculo-ventricular de la unin?
Provienen de la parte superior del Haz de His, anterior a su bifurcacin.
Es mucho menos frecuente ver este tipo de extrasstoles que las auricula-
res o las ventriculares.
Cules son las caractersticas de las extrasstoles auriculares?
La onda P que se adelanta y tiene una morfologa diferente a la del com-
plejo sinusal y que puede presentarse positiva, negativa o difsica, sea o
no seguida del complejo ventricular, con un intervalo P-R ms corto y
que se observa mejor en D-I sugiere una extrasstole auricular.
Cules son las caractersticas de los complejos aurculo-ventricular de la
unin?
Son complejos QRS de morfologa normal, no precedidos de onda P. La
onda P aparece negativa y posterior al QRS en D-II, D-III y AVF (onda
P retrgrada). Se puede observar en cualquier enfermedad cardiaca o en
la intoxicacin por digitlicos.
Cul es el significado clnico de las extrasstoles supraventriculares?
Se observan en sujetos simpaticotnicos, hipertiroideos; en la insuficien-
cia cardiaca, en la fiebre reumtica activa, en la dilatacin auricular se-
81
cundaria a valvulopata mitral o tricuspdea, en la isquemia miocrdica,
en la intoxicacin por cafena, nicotina o alcohol y otras.
Cules son las caractersticas de las extrasstoles ventriculares?
Se puede observar un complejo QRS ancho (>0.14 segundos) no precedido
de onda P. El segmento ST y la T tienen una direccin opuesta al QRS. Las
extrasstoles ventriculares y los complejos sinusales pueden asociarse de dos
maneras: la unifocal exhibe complejos QRS de morfologa consistente y aco-
plamiento fijo al complejo sinusal; cuando el acoplamiento a dichos complejos
no es fijo y la periodicidad entre extrasstoles es independiente y constante se
sospecha un foco ectpico, lo que convierte al fenmeno en multifocal.
Cmo se clasifican los mecanismos de las taquiarritmias?
En desrdenes de la propagacin del impulso y desrdenes de la forma-
cin del impulso.
Cules son los desrdenes de la propagacin del impulso (reentrada)?
Cuando hay un bloqueo unidireccional en alguna parte del sistema elc-
trico cardiaco se puede producir la circulacin repetitiva de un impulso
causando una taquicardia sostenida. Las arritmias por reentrada pueden
iniciar y terminar por complejos prematuros y estimulacin rpida. La
respuesta de estas arritmias a la estimulacin puede ayudar a distinguirlas
de las ocasionadas por actividad desencadenada.
Cules son los desrdenes de la formacin del impulso?
Desrdenes por actividad incrementada y desrdenes por actividad des-
encadenada.
Cules son las caractersticas de los desrdenes por actividad incrementada?
Cuando alguno de los diferentes focos del miocardio especializado sufre
un incremento en su excitabilidad puede ganar actividad como marcapa-
so automtico. Esto puede ser a consecuencia de la accin de catecola-
minas endgenas, desrdenes electrolticos (hiperkalemia), hipoxia,
isquemia, efectos mecnicos y drogas (digital).
Cules son las caractersticas de los desrdenes por actividad desencadenada?
Requieren un cambio en la frecuencia cardiaca como factor desencade-
nante. Pueden ser causados por posdespolarizaciones tempranas que ocurren
en las fases 2 y 3 del potencial de accin. Se observan en la bradicardia,
82
la hipokalemia y, en general, en cualquier situacin que prolongue la du-
racin del potencial de accin. Las posdespolarizaciones tardas ocurren al
completarse la fase 3 del potencial de accin y se observan en los casos de
intoxicacin digitlica, hiperkalemia, hipercalcemia o de concentraciones
locales aumentadas de catecolaminas.
En qu se diferencian las taquiarritmias comunes?
Variables
Taquicar-
dia sinusal
Taquicardia
paroxstica
auricular
Fibrilacin
auricular
Aleteo au-
ricular
(flutter)
Taquicardia
ventricular
Frecuencia 100 a 200 169 a 190 160-90 140 a 160 100 a 230
Ritmo Regular Regular irregular regular Irregular
QRS Normal normal normal normal anormal
Actividad auri-
cular
P sinusal
Ausente o no
sinusal
ausente
Ondas de
aleteo
P sinusal
Relacin
P-QRS
Si
Encubierta
con RVR*
no
Encubierta
con RVR
No
Masaje carot-
deo
Se torna
lento
Sin respuesta
o se torna si-
nusal
Sin res-
puesta
Incremento
del blo-
queo
Sin respues-
ta
El QRS es normal siempre que no exista una alteracin concomitante en la conduccin
*RVR= Respuesta ventricular rpida
Qu efecto tienen las maniobras sobre el seno carotdeo?
Las maniobras vagales tienen una doble funcin teraputica y diagnsti-
ca. El masaje al seno carotdeo y la maniobra de Valsalva aumentan el
tono vagal disminuyendo la descarga del nodo sinusal y prolongando el
tiempo de conduccin del nodo A-V as como su refractariedad.
Cul es la respuesta de la arritmia por reentrada intranodal y de la taqui-
cardia auricular bloqueada a las maniobras vagales?
La primera muestran una disminucin abrupta de la frecuencia ventricu-
lar media, en tanto que en la segunda tambin disminuye la frecuencia
ventricular media y retorna al ritmo de base aunque la arritmia persiste.
Cul es el tratamiento para la fibrilacin auricular?
Depende de que se acompae o no de insuficiencia cardiaca. Si hay insu-
ficiencia cardiaca el tratamiento de eleccin es la digoxina (0.5mg IV ini-
cialmente): si no hay insuficiencia cardiaca se utilizan beta-bloqueadores.
83
Cul es el tratamiento de la taquicardia paroxstica supraventricular?
La adenosina; otras arritmias no reaccionan a este frmaco.
Cmo se trata la taquicardia venticular aguda?
El tratamiento se encuentra determinado por la presencia de deterioro
hemodinmico y la duracin de la arritmia. Si hay hipotensin, insufi-
ciencia cardiaca o angina debe realizarse cardioversin elctrica con 100
a 360 J. Si el paciente tolera la arritmia entonces se utiliza lidocana
(1mg/kg en bolo IV).
Cul es el tratamiento de la taquicardia ventricular sostenida?
La taquicardia ventricular sostenida es una variante crnica en la que hay
descargas de tres o ms latidos con duracin mayor a 30 segundos. El tra-
tamiento se lleva a cabo con antiarrtmicos o un desfibrilador-
cardiovertor implantable en los casos refractarios a frmacos.
Cules son los antiarrtmicos clase I (bloqueadores de canales de sodio)?
IA: hacen ms lenta la despolarizacin en la fase 0 del potencial de
accin: quinidina, procainamida, disopiramida y moricizina.
IB: acortan la repolarizacin de la fase 3: lidocana, mexiletina y feni-
tona.
IC: hacen marcadamente lenta la despolarizacin de fase 0: flecainida
y propafenona.
Cules son los antiarrtmicos clase II?
Actan como bloqueadores del adrenorreceptor beta: esmolol, metopro-
lol y propranolol.
Cules son los antiarrtmicos clase III?
Actan bloqueando los canales de potasio y son la amiodarona, el breti-
lio y el sotalol.
Cules son los antiarrtmicos clase IV?
Son los que actan bloqueando a los canales de calcio: verapamilo y dil-
tiazem.

VALVULOPATAS Y CARDIOPATAS CONGNITAS.

Cmo se llama el pulso de la insuficiencia artica?
84
Pulso colapsante, de corrigan o de martillo hidrulico.
A qu cardiopata se asocia un ritmo de cinco tiempos?
A la anomala de Ebstein.
A qu cardiopata se asocia el P
2
fijo?
A la comunicacin interauricular.
A qu cardiopatas se asocia el S-3?
Se asocia al galope ventricular, insuficiencia mitral y la insuficiencia ven-
tricular significativa.
Cules son dos causas de chasquido de eyeccin?
La vlvula semilunar estentica y la arteria pulmonar hipertensa disten-
dida.
Cules son causas de chasquido mesosistlico o telesistlico?
Prolapso de la vlvula mitral y prolapso de vlvula tricspide.
Cules son causas de S-2 permanente?
Todas las que involucran al ventrculo derecho como la estenosis pulmo-
nar, la comunicacin interauricular y el bloqueo de la rama derecha del
Haz de His.
Cundo son normales el soplo carotdeo y el soplo de Still?
En nios de 3 a 8 aos.
Cundo es normal un soplo pulmonar por flujo?
De los 6 a los 18 aos.
Cundo encontramos pulsus alternans?
Cuando la contractilidad cardiaca est disminuida o existen grandes de-
rrames pleurales.
Cundo aparece el pulso paradjico de Kussmaul?
La presin sistlica disminuye en 10 mm Hg durante la inspiracin. Se
encuentra en el taponamiento cardiaco.
Cules son los tres signos de la hipotensin ortosttica?
Disminucin de 20 mm Hg en la presin diastlica.
Disminucin de 10 mm Hg en la presin sistlica.
Aumento de 20 latidos por minuto en la frecuencia cardiaca.
Como se relaciona la prdida de volmen con la clnica?
85
20% (1000ml):taquicardia.
30%(1500ml):hipotensin ortosttica.
40%(2000ml):hipotensin supina y taquicardia.
50%(3000ml):choque.
Cules son las indicaciones de reparacin valvular?
Endocarditis fngica, falla cardiaca crnica y falla de la terapia antibitica.
Cules son las valvulopatas ms importantes y como se identifican clni-
camente?
Estenosis mitral: el soplo se localiza en el pex.
Regurgitacin mitral: el soplo se transmite hacia la axila izquierda y el
rea infraescapular izquierda.
Estenosis artica: el soplo se escucha en el segundo espacio intercostal
derecho, en las cartidas y en el rea interescapular superior.
Regurgitacin artica: el soplo se escucha mejor en el tercer o cuarto
espacio intercostal izquierdo sobre el borde esternal, y se relaciona
con un soplo apical mesodiastlico (soplo de Austin-Flint).
Estenosis y regurgitacin tricuspdeas: el soplo se percibe en el tercero o
cuarto espacio intercostal, a lo largo del borde esternal izquierdo.
Estenosis pulmonar: el soplo se irradia a la axila y espalda y se escucha
difusamente en el trax en los lactantes.
Cules son la 1 y 2 causas de regurgitacin tricuspdea?
Falla cardiaca izquierda y endocarditis por drogas administradas por las
va intravenosa.
Cules hallazgos clnicos acompaan a la estenosis artica?
Un soplo en la base, y electrocardiograma con depresin del segmento S-
T e inversin de la onda T.
Cul es la vlvula ms afectada por la cardiopata reumtica?
La mitral.
Qu soplos son mesosistlicos?
Los de la estenosis artica y de la estenosis pulmonar.
Qu soplos son holosistlicos?
Los de regurgitacin aurculo-ventricular y los de cortocircuito izquierda-
derecha a nivel ventricular.
Qu soplos son diastlicos tempranos?
86
Los de la insuficiencia artica o insuficiencia pulmonar.
Cmo afectan a los soplos la maniobra de Valsalva y la bipedestacin?
Hacen que la mayora de los soplos disminuyan en duracin e intensidad.
Los soplos de la cardiomiopata hipertrfica y del prolapso de la vlvula
mitral aumentan con la bipedestacin.
Cmo afecta a los soplos la posicin de cuclillas?
La mayora de los soplos aumenta en esta posicin, con la excepcin de
los soplos de la cardiomiopata hipertrfica y del prolapso de la vlvula
mitral que se tornan ms suaves.
Cmo afectan a los soplos los latidos ventriculares prematuros y la fibri-
lacin auricular?
Los soplos sistlicos de semilunares estenticas aumentan. Por el contra-
rio, los soplos sistlicos de la regurgitacin aurculo-ventricular disminu-
yen, se hacen ms cortos o no cambian.
Cmo afecta la respiracin a los soplos?
Todos los soplos correspondientes a estructuras de la circulacin derecha
aumentan con la inspiracin; tambin aumentan el S-3 y S-4. Todos los
soplos de la circulacin izquierda aumentan con la espiracin.
Qu soplos son mesodiastlicos?
El de la estenosis mitral.
El de Carey-Coombs (mesodiastlico apical que aparece en la fiebre
reumtica).
Los de flujo mitral aumentado, como sucede en la insuficiencia mi-
tral, los defectos ventriculares septales o la persistencia del conducto
arterioso.
El de la estenosis tricuspdea.
Los de flujo tricuspdeo aumentado como en los defectos septales au-
riculares, la insuficiencia tricuspdea y el drenaje venoso anmalo.
Qu soplos son continuos?
El de la persistencia del conducto arteriorso y el de la ruptura de un
aneurisma del seno de Valsalva.
Cul es el sndrome de Holt-Oram?
87
Es un defecto de tipo ostium secundum acompaado de deformidades
seas en los miembros superiores como la agenesia de pulgares.
Cul es la trada del taponamiento cardiaco?
Aumento de la presin venosa, disminucin de la presin arterial y pre-
cordio apagado (triada de Beck).
Cules son los factores predisponentes para la endocarditis?
Los procedimientos quirrgicos dentales, urolgicos y gastrointestinales;
el uso de drogas IV y lesiones preexistentes.
Cul es la endocarditis de Liebman-Sacks?
Es la endocarditis no bacteriana que se presenta en el lupus eritematoso
sistmico.
Cules son los signos clnicos de miocarditis?
Anormalidades de la conduccin, arritmias y falla cardiaca.
Cules son las manifestaciones inmunolgicas de la endocarditis bacte-
riana subaguda?
Manchas de Roth: citioides en la retina.
Ndulos de Osler: ndulos hipersensibles en la falange terminal (dife-
renciar de los de Heberden).
Lesiones de Janeway: manchas indoloras en las palmas y plantas.
Petequias y prpuras.
Glomerulonefritis proliferativa.
Cules son los criterios mayores de Duke para el diagnstico de endocar-
ditis?
Dos cultivos positivos para S. viridans, S. aureus o microorganismos
del grupo HACEK.
Evidencia de endocardio involucrado por ecocardiografa o la pre-
sencia de un nuevo soplo.
Cules son los criterios menores de Duke?
Predisposicin por dao valvular o por ser usuario de drogas IV.
Fiebre >38
Fenmenos vasculares, embolismo, aneurisma mictico.
88
Fenmenos inmunolgicos: manchas de Roth, ndulos de Osler y
glomerulonefritis.
Ecocardiografa que no es concluyente de un criterio mayor.
Evidencia menor de infeccin.
Cmo se interpretan los criterios de Duke?
Se considera diagnstico de endocarditis cuando hay dos criterios mayo-
res, tres menores y un mayor o cinco menores.
Cul es el tratamiento de la endocarditis de vlvula nativa?
El causante principal es S. viridans. El tratamiento se realiza con penicili-
na (12 a 18 millones) ms gentamicina (1g), o bien penicilina (12-18 mi-
llones) ms ceftriaxona (2 mg) durante 4 semanas.
Cul es el tratamiento de la endocarditis de vlvula artificial?
Vancomicina (15 mg/kg) ms gentamicina (1 mg) durante 14 das.

89
90
CIRUGA








Qu arteria irriga el polo superior de la tiroides?
La tiroidea superior, rama de la cartida externa.
Qu arteria irriga el polo inferior de la tiroides?
La tiroidea inferior, rama del tronco tirocervical, rama de la subclavia.
Qu arteria irriga el istmo de la tiroides?
La tiroidea imma, rama del arco artico.
Qu sntomas presenta la tiroides lingual?
Obstruccin y dificultad para hablar.
Cul es la infeccin de tejidos cervicales ms frecuente?
La angina de Ludwing.
Qu es la angina de Ludwing?
Es una infeccin de los espacios sublingual y submaxilar secundaria a in-
feccin de las races dentales.
Cul es el tratamiento quirrgico de la angina de Ludwing (AL)?
Se trata mediante drenaje por incisiones submentonianas bilaterales. De-
be hacerse da manera urgente cuando la va area est comprometida.
Cules son los agentes causantes de la AL y su tratamiento?
Estreprococos, estafilococos, bacteroides y fusobacterium. Se trata con
tres regmenes farmacolgicos: penicilina-metronidazol, ampicilina-
sulbactam o clindamicina-cefalosporina.
Cules son las zonas anatmicas en que se divide el cuello?
Zona I: comprende la base del cuello y es la entrada al trax.
Zona II: se encuentra por arriba de la base del cuello y abajo del ma-
xilar inferior. Las estructuras que se encuentran en esta zona son la
cartida, la yugular, la laringe, la trquea y el esfago. Una lesin
91
penetrante en esta zona requiere exploracin quirrgica sin importar
los datos clnicos.
Zona III: se encuentra por arriba del ngulo del maxilar inferior.
Cul es el tumor parotdeo ms comn?
El adenoma pleomrfico.
Cul es la tumoracin del cuello que sigue a los movimientos de deglu-
cin?
Las neoplasias tiroideas siguen generalmente este comportamiento.
Qu estructura se ocluye ms frecuentemente por sialolitiasis?
El conducto de Wharton.
Cul es la lesin esofgica benigna ms frecuente?
El leiomioma es la lesin ms frecuente y se presenta con dolor y disfagia.
El tratamiento es la reseccin extramucosa con abordaje por toracoto-
ma.
Qu es el divertculo de Zenker?
Es un divertculo que se produce por traccin en la parte alta del esfago
y se trata mediante miotoma del cricofarngeo.
Cul es el tratamiento profilctico antibitico para pacientes programa-
dos para ciruga dental?
Amoxicilina: 50 mg/kg, en nios; 2 g, en adultos.
Cules son los sndromes posgrastrectoma?
Sndrome de reflujo alcalino: es el trastorno ms frecuente, que cursa
con dolor epigstrico posprandial, nusea, vmito y prdida de peso.
Sndrome de asa aferente: es una obstruccin intermitente de la anas-
tomosis que causa sntomas gastrointestinales que se alivian nica-
mente con el vmito.
Sndrome dumping (de vaciamiento rpido): cursa con sntomas de
respuesta simptica por la llegada directa de lquido hiperosmolar al
intestino delgado causando distensin yeyunal.
Diarrea posgastrectoma.
Cul es el tratamiento para el sndrome de reflujo alcalino y el de asa afe-
rente?
Es quirrgico. Se debe realizar una Y de Roux.
92
Cul es el tratamiento del sndrome dumping?
Hay que administrar ocreotide y evitar un alto contenido de carbohidratos
en la dieta.
Cul es el cuadro clnico de una perforacin intestinal?
Dolor agudo, rigidez abdominal, rebote positivo y ruidos intestinales
apagados.
Cundo se debe sospechar una intususcepcin?
En pacientes de 2 meses a 2 aos que representan clicos que se alternan
con ondas peristlticas, una masa con forma de salchicha en la palpa-
cin abdominal, evacuaciones en jalea de grosella y generalmente se-
cundaria a una infeccin viral (verificar el antecedente de fiebre).
Cundo se debe sospechar un vlvulo en los pacientes peditricos?
Cuando hay vmito agudo de caractersticas biliares en pacientes que se
encuentran dentro del primer ao de vida.
Qu se observa en los pacientes con obstruccin a nivel gstrico?
Alcalosis, hipocloremia (por el vmito repetido) e hipokalemia.
Cmo se maneja la hemorragia del tubo digestivo alto secundaria a vri-
ces esofgicas?
Si no es aguda, se ligan las vrices; si es aguda se esclerosa.
Cul es el tratamiento de emergencia de la hemorragia del tubo digestivo
alto secundaria a vrices esofgicas?
El shunt o derivacin porto-cava.
Cul es el riesgo operatorio de la derivacin porto-cava segn la clasifica-
cin de Child?
A= 2%, B=10%, C=50%.
Qu es el procedimiento de Warren?
Es un shunt portosistmico (esplenorrenal).
Qu es el procedimiento de Sujiura?
Es la devascularizacin esofgica.
Cul es la mortalidad en casos de trauma heptico?
10-20% en lesiones parenquimatosas; 50% en lesiones vasculares.
Cmo se logra la hemostasia heptica?
Con empaquetamiento o compresin portal (Pringle).
93
De qu se componen los clculos biliares?
75% de colesterol y 10 a 25% de bilirrubinato de calcio.
De qu se componen las sales biliares?
cido quenodesoxiclico, cido desoxiclico, taurina y glicina.
Cul es el mtodo de eleccin para detectar clculos biliares?
El ultrasonido.
Cul es el mtodo de eleccin para diagnosticar clculos biliares?
La colecistografa oral.
Para qu se utiliza el paneo hepatobiliar con cido inmunodiactico (HIDA)?
Para diagnosticar colecistitis, quistes del coldoco y derrame biliar. Si se
usa asociadamente con colecistocinina (CCK) se puede estudiar la disci-
nesia biliar.
Dnde se ubica ms frecuentemente la obstruccin leo biliar?
En el ileon terminal (en la vlvula ileocecal).
Cmo son los clculos biliares de colesterol?
Grandes y suaves.
Cmo son los clculos biliares de pigmento?
Son suaves y de color verde o negro.
Con qu se asocian los clculos biliares de calcio?
Con inflamacin, infeccin o aumento de la beta-glucoronidasa.
Cules son causas frecuentes de colecistitis acalculosa?
Salmonelosis, poliarteritis nodosa, sepsis y trauma.
Cul es la triada de la colangitis?
Fiebre, ictericia y dolor. En ocasiones la coledocolitiasis puede presentar-
se con el mismo cuadro clnico, por lo que hay que hacer diagnstico di-
ferencial entre stas.
Cul es el tratamiento profilctico antibitico para pacientes programa-
dos para ciruga gastro-biliar y de colon?
Cefazolina (1-2 mg IV) ms metronidazol (0.5mg IV).
Cules son los dos abscesos abdominales ms comunes?
El subfrnico y el subheptico.
Cules son las caractersticas clnicas de los abscesos?
Fiebre en agujas despus de 2 semanas de posoperatorio.
Dolor y masa abdominal.
94
Cul es la antibioterapia emprica para el traumatismo penetrante abdo-
minal?
Se aplica un doble esquema de metronidazol-amikacina.
Cul es la causa ms frecuente de hemorragia retroperitoneal en un trau-
matismo?
La fractura plvica.
Cules son los principales ligamentos plvicos ginecolgicos?
Ligamento cardinal: fija el cuello a la pared pelviana y la parte supe-
rior vaginal. Las arterias uterinas discurren a travs el ligamento car-
dinal.
Ligamento terosacro: se extiende del cuello al sacro.
Ligamento ovrico: es un ligamento avascular que se extiende del
ovario al mesoslpinx.
Ligamento redondo: une al tero con la trompa y discurre por el ca-
nal inguinal para terminar en los labios mayores.
Cules son las paredes del conducto inguinal?
Anterior: oblicuo externo.
Superior: el ligamento formado por el oblicuo interno y el transverso.
Posterior: fascia transversalis.
Inferior: Ligamento inguinal.
Cundo se presenta una hernia directa?
Cuando existe una debilidad del piso del canal inguinal.
Qu estructura se encuentra daada en las hernias femorales?
El ligamento de Cooper.
Cules son la primera y segunda causas de obstruccin intestinal en ni-
os?
Adherencias y hernias.
A los cuntos meses se presenta la hernia inguinal?
A los 6 meses.
Cul es la ciruga ms frecuente en pacientes peditricos?
La reparacin de la hernia inguinal.
De qu lado se localizan ms frecuentemente las hernias?
60% del lado derecho, 30% del izquierdo, 10-15 % bilateral.
95
Cul es el procedimiento quirrgico ms utilizado para reparar las hernias
directas?
El de Bassini, que sutura al mismo tiempo la fascia transversalis y el li-
gamento inguinal al tendn conjunto.
Con cul procedimiento se reparan las hernias femorales?
Con el de McVay, que sutura el tendn conjunto y la fascia transversalis
al ligamento de Cooper.
Cul es el tratamiento de las hernias encarceladas?
Es quirrgico y se debe realizar dentro de las primeras 24 horas por el
riesgo de isquemia.
Cul es el diagnstico diferencial de la fiebre posoperatoria (recordar las
3 w)?
Wind: pulmonares de 1-3 das.
Water: vas urinarias de 3-5 das.
Wound: herida infectada de 5-8 das.
Cul es el tratamiento emprico de la celulitis?
Dicloxacilina o cefalexina IV; cefalexina oral a altas dosis (750-1,000
mg).
Cul es el cuadro clnico de la fascitis necrosante?
Bulas hemorrgicas.
Toxicidad sistmica progresiva.
Descarga ftida.
Aire en los tejidos.
Cul es el tratamiento antibitico en los casos de fascitis necrosante, gan-
grena gaseosa y choque txico esptreptoccico?
Penicilina sdica cristalina (24,000,000 U) ms clindamicina (900 mg).
Cul es el cuadro clnico de la fascitis necrosante (miositis por clostri-
dios)?
Ocurre en heridas profundas y contaminadas.
A las 48 horas se encuentra dolor y crepitacin.
En un extendido se observan bacilos esporulados grampositivos.
Cul es el sitio ms frecuente para el desgarro de la aorta?
Distal, a la salida de la subclavia izquierda.
96
Cules son los hallazgos clnicos en la claudicacin intermitente?
Dolor en la pantorrilla que ocurre mientras el paciente camina y que cesa
cuando el paciente se mantiene en reposo.
Cules son los hallazgos clnicos en el dolor de origen isqumico del
miembro plvico?
Dolor en el pie, incluyendo la cara plantar; se exacerba al elevar el pie,
por lo que el paciente duerme con el pie en el piso. Puede presentarse ne-
crosis y requerirse la amputacin.
En caso de claudicacin intermitente, con qu padecimientos debe hacerse
el diagnstico diferencial?
El dolor lumbar o la claudicacin neurgena.
Cul es el diagnstico diferencial del dolor isqumico del miembro plvico?
La neuropata diabtica y la artritis.
Ante la sospecha de gangrena, con qu padecimiento se debe hacer el
diagnstico diferencial?
Las lceras del pie diabtico o la osteomielitis subyacente.
Cules son los ndices brazo-tobillo y qu nos indican?
1: normal.
0.5: sugiere claudicacin intermitente.
0.2: sugiere origen isqumico del dolor.
<0.2: inminencia de amputacin.
Cules son las tres complicaciones ms frecuentes de la arteriografa?
Debidas al contraste: nefrotoxicidad y alergia.
De origen arterial: trombosis y embolismo.
En el sitio de puncin: hematoma y pseudoaneurisma.
Cul es el mtodo diagnstico de eleccin para un aneurisma disecante de
la aorta?
La ecografa transesofgica.
Cul es la triada del sndrome de Leriche?
Claudicacin en muslo y en el glteo.
Pulso femoral ausente.
Impotencia sexual.
97
Cul es la diferencia entre la isquemia mesentrica aguda y la crnica?
Aguda: aparece como un dolor exquisito y fuera de proporcin con
los hallazgos fsicos (no hay abdomen agudo, rigidez, etc.)
Crnica: la angina intestinal provoca dolor posprandial que provoca
en el paciente miedo a comer con la consecuente prdida de peso.
Cmo se conoce el sndrome de la mesentrica superior?
Como sndrome del cors.
Cundo est indicado el tratamiento quirrgico de una obstruccin caro-
tdea?
Cuando es de 60 a 90%, sin importar si existe o no sintomatologa.
Dnde se observa el signo del cascarn (eggshell)?
En el aneurisma de la aorta abdominal .
Cundo est indicado el tratamiento quirrgico del aneurisma de la aorta
abdominal?
Cuando es mayor de 5 cm, an si es asintomtico.
Cul es la triada de Virchow?
Aumento de la coagulabilidad.
Disminucin de la velocidad del flujo de la sangre.
Trauma.
Cul es el cuadro clnico de la trombosis venosa superficial y su tratamiento?
Presencia de vena indurada, roja y dolorosa.
El tratamiento es conservador (medias, elevacin de miembros). La
ciruga se indica nicamente por razones estticas, dolor recurrente o
episodios de sangrado.
Cul es el cuadro clnico de la trombosis venosa profunda y su tratamiento?
Ocurre despus de procedimientos de ortopedia (50-70%) y ciruga ab-
dominal (30%). Se presenta con edema, signos de Homans y Orlow posi-
tivos. Puede acompaarse de embolismo pulmonar. El tratamiento se
lleva a cabo con heparina, warfarina y filtro intracaval.
Cul es la diferencia entre phlegmasia alba dolens y phlegmasia cerlea dolens?
Phlegmasia alba: se obstruyen las venas ilaca y femoral. Por contigi-
dad se afecta la arteria femoral y causa su obstruccin, por lo que se
presenta extremidad fra, plida y sin pulso.
98
Phlegmasia cerlea: se obstruye exclusivamente el retorno venoso. Se
observa una extremidad edematizada, dolorosa, morada y caliente.
Cul es la complicacin ms frecuente de la ciruga laparoscpica?
El neumotrax. Se identifica por la aparicin de hipotensin sbita, desa-
turacin y aumento en la presin ventilatoria. Puede ser secundario a de-
fectos en el diafragma, en el espacio retropleural o en el espacio
retroperitoneal.

99
100
DERMATOLOGA


Cules son las capas de la piel?
Epidermis: crnea, granulosa, espinosa y basal.
Dermis: papilar, reticular y profunda.
Tejido celular subcutneo.
Cul es la causa de aparicin del acn?
El aumento en la produccin de andrgenos, que aumentan la produc-
cin sebcea.
Cul es el tratamiento del acn?
Comedolticos, antiqueratolticos, inhibidores de las glndulas sebceas
por va sistmica (isotretinona) y antiandrognicos sistmicos (dexame-
tasona o anticonceptivos hormonales).
Cul es la diferencia entre leucoplaquia y candidiosis?
La leucoplaquia es una lesin blanca que no puede ser removida frotn-
dola (a diferencia de la candidiosis) y que tiene de 2 a 6% de probabilida-
des de desarrollar cncer.
Cul es la incidencia del cncer en la eritroplaquia?
90% de los casos evolucionan a cncer.
Cules son la distribucin y antecedentes de la dermatitis atpica?
En los adultos aparece en las reas flexoras de los miembros.
En los lactantes, en las reas facial y extensoras de los miembros.
Existe el antecedente de asma o alergia en el sujeto o en la familia.
Es comn la presencia de obscurecimiento infraorbitario.
Cul es el tratamiento de la dermatitis atpica?
Medidas generales y corticoides tpicos.
Qu es el eritema polimorfo?
Es una enfermedad inflamatoria de etiologa infecciosa o secundaria a
frmacos que presenta lesiones cutneas en diana (en blanco de tiro, en
iris), que son lesiones con anillos concntricos y una prpura central,
principalmente en las porciones distales de los miembros, aunque tam-
bin se pueden ver en las palmas y en la cara.
101
Qu es el sndrome de Stevens-Johnson?
Es una forma grave del eritema polimorfo en la que se involucran la mu-
cosa oral, la faringe y la regin anogenital.
Qu es el eritema nodoso?
Es una enfermedad inflamatoria de la piel y del tejido subcutneo, carac-
terizada por la presencia ndulos rojos dolorosos principalmente en la re-
gin pretibial. Las causas ms frecuentes son las infecciones
estreptoccicas y la sarcoidosis.
Cmo es la lesin del lquen simple?
En placas y con huellas de rascado; se trata con esteriodes tpicos.
Cul es el cuadro clnico de la psoriasis?
Se observan escamas plateadas sobre placas rojas en los codos, las rodi-
llas y el cuero cabelludo; pueden coexistir artritis y onicolisis.
Cul es el tratamiento de la psoriasis?
Para la variedad localizada se utilizan queratolticos y para la variedad gene-
ralizada se emplean metotrexate y rayos ultravioleta.
Cul es el cuadro clnico de la pitiriasis rosada?
Se observa la placa herldica (lesin confluente de gran tamao localizada
en el pecho) y existe descamacin perifrica. El agente etiolgico es el vi-
rus HHV7.
Cul es el diagnstico diferencial de la pitiriasis rosada?
La lesin es similar a la de la sfilis secundaria, por lo que hay que hacer
prueba de reagina (RPR) y tambin debe diferenciarse de la tinea corporis
mediante una prueba con KOH.
Cul es la diferencia entre psoriasis, dermatitis seborreica y caspa?
Psoriasis: placas eritematosas delimitadas, no confluentes, con des-
camacin.
Dermatitis seborreica: hay eritema generalizado o confluente con
descamacin.
Caspa: no hay eritema; la descamacin est presente.
Cul es el tratamiento de la dermatitis seborreica?
Shampoo con alquitrn o pitirona de zinc diariamente.
102
Ketoconazol en solucin tpica cada tercer da. No se usa por su ac-
cin antimictica sino ms bien por otra no especificada).
Cmo es la lesin de la queratosis seborreica?
Se observan ndulos grandes como empotrados.
Cul el el tratamiento de la roscea?
Deben evitar cualquier alimento que cause vasodilatacin como el al-
cohol o los alimentos condimentados.
Cundo se presenta el eritroderma macular?
Es el signo dermatolgico que acompaa al sndrome de choque txico.
Cul es el diagnstico diferencial de las lesiones hipopigmentadas?
Vitiligo, piebaldismo, tinea versicolor, albinismo oculocutneo, psoriasis y
sfilis secundaria, entre otras.
Qu es la tinea versicolor?
Mculas hipo o hiperpigmentadas en la parte superior del tronco, causa-
das por Malasezia furfur. Debido al tipo de lesin, hay que diferenciarla
del vitiligo.
Qu es el vitiligo?
Es la prdida de melanocitos que se presenta en 1% de la poblacin y
puede relacionarse con hipertiroidismo, hipotiroidismo, anemia pernicio-
sa, diabetes mellitus y enfermedad de Addison.
Qu es el mal del pinto?
Es una treponematosis no transmitida sexualmente causada por T. Cara-
teum; es endmica de las reas rurales de Amrica Latina. En la lesin
inicial en la piel provoca una coloracin lila o negra seguida de atrofia de
la zona e hipopigmentacin.
Cul es la diferencia entre la enfermedad de Bowen y la queratosis actnica?
Las dos presentan una placa con vesculas color rojo. La queratosis act-
nica miden de 0.2 a 1 mm y en la enfermedad de Bowen alcanzan de 1 a
3 cm.

103
ENDOCRINOLOGA



A qu se llama retroalimentacin positiva?
Es el mecanismo por el cual una hormona acta estimulando a la misma
glndula que la secreta a aumentar la secrecin de la propia hormona.
Qu es la retroalimentacin negativa?
Es un mecanismo por el cual la hormona secretada estimula a la propia
glndula para que disminuya la secrecin de dicha hormona.
Cul es la diferencia entre el diagnstico de diabetes mellitus y el de into-
lerancia a la glucosa?
En la prueba con 75 g de glucosa, el diagnstico de diabetes se hace con
un valor de 200 de las 0 a las 2 horas. La intolerancia a la glucosa se
diagnostica cuando el valor de 200 se presenta ms all de las 2 horas.
Para qu sirve la determinacin de hemoglobina glicosilada (HbA
1
C)?
El nivel de HbA
1
C indica cul ha sido el promedio de glucemia en las l-
timas 8 a 12 semanas. Es til para valorar si hay un buen control de la
glucemia.
Para qu se utiliza la determinacin de fructosamina srica?
Para evaluar la glucemia de las ltimas 2 semanas.
Cul es el riesgo de padecer diabetes mellitus cuando los progenitores son
diabticos?
5-10%
Cul es el riesgo de padecer diabetes mellitus cuando el padre es el afectado?
El riesgo es cinco veces mayor y est relacionado a HLADR4
Qu es la diabetes juvenil de inicio tardo (tipo MODY, por sus siglas en
ingls)?
Es una hiperglucemia leve con resistencia a la cetosis causada por los ge-
nes MODY 1, 2, 3. Su nombre proviene de las siglas en ingls para Matu-
re Onset Diabetes of the Young, que quiere decir, diabetes juvenil de inicio
tardo.
104
Qu es el fenmeno de Somogy?
Es una hiperglucemia de rebote cuando se sobreutiliza la insulina. Al
causar hipoglucemia, sta activa los mecanismos contrarreguladores co-
mo el glucagon, la hormona de crecimiento, el cortisol y las catecolami-
nas, que causan hiperglucemia.
Por qu la hiperinsulinemia causa resistencia a la insulina?
Porque aumenta la sntesis de grasas y el transporte de las mismas por
VLDL haciendo de stas el principal sustrato energtico en lugar de la
glucosa, lo que disminuye la necesidad de utilizar insulina en la clula.
Cules son los tipos de insulina disponibles?
Tipo
de insulina
Ejemplo
Inicio
de la accin
Pico
de accin
Duracin
de la
accin
Va de
administracin
Accin ultracorta Insulina lispro 0-15 minutos 1-2 hr 3-5 horas SC, IV
Insulina aspart 10-12 minutos 40-50 min 4-6 horas SC, IV
Accin rpida Insulina regular 30-60 minutos 2-4 hr 6-8 horas SC, IV
Accin
Intermedia
Insulina NPH 1-2 horas 6-12 hr 18-24 horas SC
Insulina lenta 1-3 horas 6-12 her 18-24 horas SC
Accin
Prolongada
Insulina ultralenta 3-4 horas 4-12 hr 12-18 horas SC
Insulina ultralarga 4-6 horas Ninguno 18-30 horas SC

Cmo se lleva a cabo la terapia convencional con insulina?
15 unidades de lenta o NPH, divididas en dos dosis.
Cules son los frmacos hipoglucemiantes de eleccin?
Las sulfonilureas (glibenclamida) que actan estimulando la liberacin de
insulina desde las clulas beta. Todava son los frmacos de eleccin, se-
guidos por las biguanidas (metformina).
Cmo acta la metformina?
Inhibiendo la gluconeognesis heptica.
Qu es la clorpropamida?
Es una sulfonilurea de primera generacin con efecto hipoglucemiante que
tambin tiene un uso teraputico en la diabetes inspida, ya que potencia
el efecto de la hormona antidiurtica.
Qu es la troglitazona?
105
Es un hipoglucemiante del grupo tiazolidinediona, de primera generacin,
que acta disminuyendo el estado de resistencia a la insulina en los dia-
bticos.
Qu se requiere para desarrollar cetoacidosis?
Un estado de hipoinsulinemia, ya que ste favorece la movilizacin de
los depsitos de grasa; tambin hiperglucagonemia, que favorece la oxi-
dacin de los cidos grasos en el hgado.
Cmo se valora la respuesta al tratamiento de la cetoacidosis?
Slo con el PH y el anion gap calculado.
Cmo se lleva a cabo el tratamiento con insulina en la cetoacidosis?
Primero se debe usar de manera intravenosa continua a travs de una
bomba de infusin, a dosis de 0.1 U/kg, despus de una dosis de carga en
bolo de 0.1U/kg y segn las necesidades del paciente. Posteriormente,
cuando se haya administrado 75% de la dosis por bomba de infusin, se
procede a iniciar dosis intermitentes IV o subcutneas.
Cules son las hormonas que producen los diferentes lbulos de la hipfisis?
Lbulo anterior: hormona estimulante del tiroides (TSH, tirotropina),
hormona adenocorticotrpica (ACTH, corticotropina), hormona de
crecimiento (GH, somatotropina), hormona foliculoestimulante
(FSH) y hormona luteinizante (LH).
Lbulo intermedio: hormona estimulante de los melanocitos (MSH,
melanotropina); gamma-lipotropina (gamma-LPH), pptido similar a
la corticotropina (CLIP) y otros derivados de la proopiomelanocorti-
cotropina (POMC).
Lbulo posterior: vasopresina (antidiurtica, ADH) y oxitocina.
Qu es un macroadenoma hipofisario?
Es un tumor de la hipfisis, que mide ms de 10 mm.
Qu protenas son subproductos de la proopiomelanocorticotropina
(POMC)?
La beta-endorfina, ACTH, MSH y la metencefalina, entre otras.
Cules son los principales antagonistas de la hormona liberada de gona-
datropina (GnRH)?
106
La prolactina, la dopamina, la ACTH y el cortisol.
Cmo acta la dopamina?
Acta sobre la eminencia media para inhibir la secrecin de hormona li-
berada de gonadatropina (GnRH).
Cmo afecta la noradrenalia a la GnRH?
Tiene un efecto estimulante sobre el rea preptica y favorece la secre-
cin de GnRH.
Cul es el mecanismo por el cual los anlogos de GnRH inhiben a la FSH
y la LH?
Proveen un estmulo potente y continuo diferente al estmulo fisiolgico
(por pulsos) de la GnRH humana.
Qu hormonas son inhibidas por el cortisol?
La ACTH y el factor liberador de cortisol (CRF).
Cules son los cambios metablicos en la acromegalia?
Intolerancia a la glucosa: por las propiedades antiinsulnicas de la
hormona de crecimiento.
Hiperfosfatemia: por su accin directa sobre la reabsorcin de fosfato.
Ms de 10 ng/ml de GH.
Aumento del factor insulnico de crecimiento -1 (IGF-1) en el hgado.
Cul es el sndrome clnico de la enfermedad de Graves?
Hipermetabolismo, signos oculares anormales y edema pretibial inusual.
Cul es la fisiopatologa de la enfermedad de Graves?
Es un trastorno autoinmunitario en que los linfocitos T producen anti-
cuerpos a algunos antgenos tiroideos. La inmunoglobulina estimulante
de tiroides, sustancia estimulador tiroideo de larga accin (LATS o TSI),
es un anticuerpo que se fija al receptor de la hormona estimulante de ti-
roides (TSH) e incrementa su crecimiento y funcionamiento.
Cmo se obtiene el ndice de tiroxina libre?
Es el producto del valor de la proporcin de captacin de hormona tiroi-
dea (THBR) multiplicado por la concentracin srica total de tiroxina.
Constituye una estimulacin de la concentracin de T4 libre
.
Cmo se encuentran los ndices tiroideos en la enfermedad de Graves?
107
Aumento de T
4
.
Aumento de T
3
RU.
Aumento de T
3
.

Cul es la diferencia entre enfermedad de Graves y la tirotoxicosis sin hi-
pertiroidismo?
Slo en la primera estn amentados el ndice de toroxina libre (ITL) y la
captacin de yodo.
Qu es la tormenta tiroidea?
Es una exacerbacin intensa de la tirotoxicosis en la que se encuentra hi-
permetabolismo, exceso de la respuesta suprarrenal, sudacin y fiebre.
De no tratarse, puede ocasionar la muerte por colapso cardiovascular. Se
trata con: propranolol para evitar las complicaciones cardiacas, con una
solucin intravenosa saturada de yoduro potsico, para evitar la liberacin
de hormona tiroidea, y con propiltiouracilo, para bloquear la produccin
de dicha hormona.
Cul es la diferencia de accin entre el propiltiouracilo (PTU) y el metimazol?
El PTU inhibe la conversin perifrica de T-4 a T-3 e inhibe la yodacin
de los grupos tirosil y la agrupacin de yodotirosinas para formar T3 y T4
en la tiroides. El metimazol slo tiene accin en la glndula y carece de
accin perifrica.
Cules son los tipos de bocio difuso?
Por deficiencia de yodo.
Tiroiditis aguda supurativa y tiroiditis subaguda
Tiroiditis crnica (de Hashimoto)
Cul es la forma ms frecuente de bocio?
La nodular.
Cul es la diferencia entre tiroiditis de Hashimoto y tiroiditis nodular?
Las dos tienen T-4 y T-3 (T3RU) normales y son asintomticas, pero la
primera es un bocio difuso.
Qu cambio produce el hipertiroidismo en la citologa hemtica?
Granulocitopenia.
Cules son los bocios txicos que requieren tratamiento quirrgico?
El multinodular o enfermedad de Plummer. Es ms benigno en muje-
res mayores de 50 aos.
108
El bocio difuso de Graves.
Cules son los bocios no quirrgicos?
Coloide: la causa es la falta de yodo; los pacientes se encuentran euti-
roideos.
Tiroiditis aguda (bacteriana): slo hay dolor y se observa eutiroidis-
mo.
Tiroiditis subaguda de Quervain (viral): los pacientes se encuentran
hipertiroideos.
Tiroiditis crnica de Hashimoto (autoinmune): existe eutiroidismo.
Tiroiditis fibrosa de Riedel: puede presentar sntomas obstructivos.
A qu se le llama sndrome del eutiroideo enfermo (euthyriod sick syndro-
me)?
Es una variedad de padecimientos agudos o crnicos no tiroideos que se
asocian con pruebas de funcin tiroidea anormales en pacientes eutiroi-
deos, especialmente en los de edad avanzada. Las variedades principales
son: el sndrome de T
4
aumentada, sndrome de T
4
disminuida y el sndrome de
T
3
baja.
Para qu sirve determinar la relacin cloruro/fosfato?
Es un indicador ms confiable que la hipofosfatemia para apoyar el diag-
nstico de hiperparatiroidismo cuando es mayor de 33.
Por qu mecanismo elevan la calcemia los tumores no seos?
Producen un pptido similar a la paratohormona.
Por qu mecanismo aumenta la calcemia la sarcoidosis?
Produce 1, 25 dehidroxocolecalciferol (1,25-DHC).
Por qu mecanismo se produce aumento del calcio en la hipercalcemia hi-
pocalcirica familiar?
Por un defecto gentico no existe el sensor de calcio en las paratiroides ni
en el tbulo.
Cules son las funciones de la paratohormona?
Remueve el calcio del hueso, aumenta la resorcin renal de calcio, au-
menta la produccin de 1, 25 DHC y aumenta la fosfaturia.
Cules son los efectos generales del hiperparatiroidismo?
Hipercalcemia, hipofosfatemia, hiperfosfaturia.
109
Cules son las principales causas de hiperparatiroidismo?
80% de los casos se debe a un adenoma solitario y 10-15%, a hiperplasia
de las cuatro glndulas.
Dnde se secreta la calcitonina?
En las clulas parafoliculares (tambin llamadas clulas claras o clulas
C) de la tiroides.
En dnde acta la calcitonina?
Acta en dos niveles: a nivel seo impide la resorcin, y a nivel renal
aumenta la calciuria. El efecto neto es una disminucin del calcio srico.
Cmo se calcula la carga filtrada de calcio?
Es el 40% del calcio srico multiplicado por la TFG.
Cul es el tratamiento para la hipercalciuria?
Diurticos tiazdicos.
En qu enfermedad se encuentran niveles elevados de calcitonina?
En el carcinoma medular tiroideo.
Cules son tres estmulos importantes para la secrecin de hormona re-
ductora del calcio (calcitonina)?
Los beta-adrenrgicos, los niveles elevados de gastrina (como en el sn-
drome de Zollinger-Ellison) y los niveles altos de calcio.
Qu es el sndrome de hueso hambriento?
Tras la paratiroidectoma se presenta hipocalcemia (e hipofosfatemia)
debido a la entrada excesiva de calcio a los huesos que se encontraban
descalcificados secundariamente al exceso de paratohormona.
Cul es el nivel normal de fsforo srico y en qu proporcin se une a pro-
tenas?
El nivel normal va de 2.8 a 4 mg/dl y el 12 % a protenas (en compara-
cin al calcio, que se une en 50%)
Cul es la diferencia entre osteomalacia y osteoporosis?
En la primera no se encuentra reducida la matriz sea y en la segunda s.
Cmo se hace el diagnstico de osteopenia?
Slo se hace por densitometra, que debe evidenciar un cambio >1 y <2.5
desviaciones estndar en la medicin de la densidad sea.
Cmo se hace diagnstico de osteoporosis?
110
Cuando la medicin de la densidad sea muestra una disminucin de la
densidad >2.5 desviaciones estndar.
Qu frmacos son inhibidores de la actividad osteoclstica?
Bifosfonatos, nitrato de galio, plicamicina, calcitonina.
Cul es la histologa de la corteza de las glndulas suprarrenales?
Capa glomerulosa externa, que produce mineralocorticoides.
Capa fasciculata intermedia, que produce glucocorticoides.
Capa reticularis interna, que produce andrgenos y estrgenos.
Cules son las horas pico y valle de la produccin del cortisol?
6:00 am y 8-12 pm
Qu produce la mdula suprarrenal?
Catecolaminas.
Cmo se clasifican los esteroides?
Esteroides C19 (17 KS) o DEHA o andrgenos
Esteroides C21 (17 OHCS), que son mineralocorticoides o anlogos
de la aldosterona.
Esteroides C21 17- hidroxicorticosteroides (17OHKS), que son glu-
cocorticoides o anlogos del cortisol.
Cmo se llaman las vas que producen mineralocorticoides y glucocorti-
coides y dnde se encuentra el defecto para la sntesis de las enzimas invo-
lucradas?
Son las vas 17-desoxy y la 17-hidroxy, respectivamente. El defecto
se encuentra en el brazo corto del cromosoma 6.
Cules son las diferentes acciones del cortisol?
El cortisol es principalmente catablico en la mayora de tejidos, pero
anablico en el hgado.
Qu caractersticas conforman la insuficiencia suprarrenal?
Insuficiencia de cortisol (glucocoriticoide): ocasiona pobre tolerancia
al estrs, anorexia, prdida de peso, hiperpigmentacin de la piel, hi-
poglicemia e hipotensin.
Insuficiencia de aldosterona (mineralocorticoide): causa hiponatre-
mia, hiperkalemia, hipovolemia, azoemia y acidosis.
Qu es la prueba de cosintropina y para qu se usa?
111
Es una prueba de estimulacin de ACTH, que se usa para hacer el diag-
nstico diferencial de la insuficiencia adrenal. El resultado positivo
(normal) es un aumento de 2 veces sobre el nivel base del cortisol plasm-
tico (>20 g/dl) y un aumento de los 17 OHKS urinarios, despus de la
administracin del medicamento. Este resultado descarta enfermedad de
Addison (insuficiencia suprarrenal primaria).
Qu es la prueba de metirapona?
La inhibicin de la secrecin pituitaria de ACTH por la produccin
adrenal de cortisol se bloquea con la metirapona, frmaco que bloquea la
11-beta hidroxilacin (paso ltimo en la produccin de esteroides). En los
individuos con deficiencia de ACTH esta no se eleva. En los individuos
normales o con Cushing pituitario se eleva la ACTH, se eleva el 11-
desoxicortisol, disminuye el cortisol, y los 17-OHKS urinarios se elevan.
En la hiperplasia adrenal no hay cambio en los 17-OHKS urinarios, ni en
la produccin de cortisol.
Cul es la diferencia entre la enfermedad y el sndrome de Cushing?
La enfermedad es de causa pituitaria; la causas del sndrome son varias,
de origen extrapituitario.
Cul es la incidencia de las etiologas del sndrome de Cushing?
15% es adrenal, 15% es extraadrenal (incluye el medicamentoso) y 70%
es pituitario.
Cul es el protocolo para estudiar el sndrome de Cushing?
Demostracin de hipercortisolismo.
Determinacin de ACTH para descartar una causa pituitaria.
Uso de tomografa axial compaturizada (TAC) o resonancia magn-
tica nuclear (RMN) para la localizacin del adenoma.
Cules son las pruebas para la deteccin de hipercortisolismo?
Niveles de cortisol a las 8:00 am (30 g); (nivel base >5.0g/dl).
Prueba de supresin con dosis altas de dexametasona.
Deteccin de cortisol libre (17OHKS) en orina de 24 horas el cual
debe ser mayor de 100 g.
Cmo se interpreta la prueba de supresin con dosis altas de dexametasona?
Se inyecta 1 mg a las 23:00 horas.
112
Se mide el cortisol srico a las 8:00 de la maana.
Un resultado normal es <3g. En los estados de hipercortisolismo es
>3g e incluso a veces >10g.
Qu otra utilidad tiene el la prueba de supresin con dosis altas de dexa-
metasona?
Es una prueba para diferenciar el sndrome de Cushing (en el que la su-
presin por retroalimentacin negativa es mnima) de los tumores adre-
nales o de los estados de produccin ectpica de ACTH.
Cmo afecta el sndrome de Cushing a la citologa hemtica?
Produce eosinofilia.
Cules son los hallazgos clnicos en el coma mixedematoso?
EKG de bajo voltaje, creatincinasa (CK) elevada, PCO2 elevada,
PO2 baja.
Hiperglucemia, debido al efecto antiinsulnico anablico de los este-
riodes.
El hematocrito menor a 30% y la hiponatremia son reflejo del secues-
tro de lquidos por actividad mineralocorticoide.
Cmo se evala la prueba de supresin estndar en 48 horas (bajas dosis)?
Se utilizan 0.5 mg de dexametasona cada 6 horas por 48 horas.
El resultado normal debe ser un cortisol srico <2g.
Para qu sirve la prueba de dosis bajas?
Se realiza con 8 dosis de 0.5 mg de dexametasona cada 6 horas. (una va-
riante es una dosis nica de 1 mg a las 11 de la noche y recoleccin de
muestra de sangre a las 8 de la maana). Sirve para descartar sndrome
de Cushing si es positivo (disminucin del cortisol srico a <5g y el uri-
nario de 24 hr a <25g).
Cul es el estudio de eleccin para diagnosticar el sndrome de Cushing
primario y pituitario?
TAC o RMN.
Qu drogas inhiben la sntesis adrenal de los esteroides?
La metirapona, el trilostane y la aminogluteimida.
Cules son los antagonistas de los esteroides?
113
Prednisona, tamoxifn, estrgenos, leuprolide, flutamida y la aminoglu-
teimida.
Cmo acta el tamoxifn?
Se une a los receptores de estrgenos sin activarlos y los bloquea.
Cmo actan los estrgenos (dietiletilbestrol)?
Actan inhibiendo la produccin de hormona LH. Por consecuencia, s-
ta no acta sobre las clulas de Leydig y se detiene la produccin de tes-
tosterona.
Cmo actan el leuprolide y la groserelina?
Son anlogos hormona liberadora de hormona luteinizante que se unen a
los receptores hipofisarios causando su desensibilizacin y, por ende,
disminuyendo la FSH y LH y, finalmente, la sntesis de andrgenos y es-
trgenos.
Cmo acta la flutamida?
Se une al receptor de andrgenos y aumenta el efecto inhibitorio de la
testosterona en la secrecin de LH (causa aumento de LH y testostero-
na).
Cmo acta la aminogluteimida?
Acta inhibiendo el paso de conversin adrenal de colesterol a pregneno-
lona y la aromatizacin perifrica (en el tejido graso principalmente) de
androstenediona a estrgeno.
Qu drogas actan inhibiendo a la hipfisis?
La ciproheptadina, que es un agonista de la serotonina y la bromocriptina,
que es un agonista de la dopamina.
Qu es el sndrome de Conn?
Es el hiperaldosteronismo primario.
Cul es la diferencia entre el sndrome de Conn y el aldosteronismo se-
cundario?
En el primero, los niveles de renina son bajos.
Cmo se toma la muestra para determinar los niveles de aldosterona?
Debe tenerse cuidado de reponer previamente el potasio; si no se hace, la
hipokalemia causa hipoaldosteronismo y un resultado normal.
Cul es el tumor del 10%?
114
El feocromocitoma: 10% bilateral, 10% maligno, 10% mltiple, 10% fa-
miliar.
Qu es el sndrome de Nelson?
Es un adenoma pituitario secundario a adrenalectoma total bilateral.
Qu pruebas de laboratorio se usan para hacer el diagnstico de feocromo-
citoma?
Niveles de metanefrinas y cido vanililmandlico (VMA) en orina.
Niveles fraccionados de catecolaminas en el plasma.
Cmo se hace la prueba de supresin en caso de feocromocitoma?
Se utilizan bloqueadores ganglionares como la clonidina o el pentoli-
nium. Cuando existe un tumo productor de catecolaminas, stas no dis-
minuyen.
Qu neoplasia se conocen como tumores endocrinos?
A los tumores de los islotes del pncreas. Se pueden dividir en no funcio-
nantes, cuyos sntomas pueden ser principalmente obstructivos y funcio-
nantes que son: insulinoma, vipoma, somatostatinoma, gastrinoma,
glucagonoma, tumor carcinoide, tumor secretor de ACTH. Estos pueden
quedar incluidos en los sndromes de neoplasia endocrina mltiple (NEM).
Qu es el sndrome de Zollinger-Ellison?
Es el gastrinoma. Puede estar localizado en el pncreas y en la pared duo-
denal. Se asocia a otras anomalas como los adenomas paratiroideos, ti-
roideos, ovricos e hipofisarios. El cuadro dominante es el de una
ditesis ulcerosa pptica agresiva de localizacin atpica (distal al bulbo
duodenal) refractaria a tratamiento mdico y quirrgico.
Cules son las pruebas de provocacin para el gastrinoma?
Calcio 5 mg/kg/hr IV durante 3 horas. Causa incremento notable de
gastrina.
2 U de secretina/kg/hr IV rpida: causa aumento paradjico.
Comida de prueba: existe un aumento menor del 50 % (el nivel ya es-
t elevado).
Cul es el tratamiento del gastrinoma?
Localizado: se utilizan bloqueadores H2, octretida o gastrectoma
total.
Metastsico: estreptozocina ms 5-fluoruracilo.
115
Cmo se presenta el insulinoma?
Se presenta con episodios de hipoglucemia que deben diferenciarse de
trastornos neurolgicos.
Qu es la triada de Whipple?
Glusemia <40 mg/dl, sntomas de la hipoglucemia y remisin de los sn-
tomas administrando glucosa.
Cmo se realiza el diagnstico de insulinoma?
Tras una dosis de tolbutamida en ayuno, la hipoglucemia persiste con ni-
veles altos de insulina medida por radioinmunoanlisis de insulina (IRI>
20 U/ml). En casos de diagnstico difcil, se hace una prueba de supre-
sin del pptido C.
Qu es el pptido C?
La cadena de preproinsulina de 23 aminocidos, que se pliega para for-
mar la proinsulina, se escinde en dos partes: el pptido C y la insulina. Al
liberar insulina, las clulas beta tambin liberan una cantidad equimolar
de pptido C.
En qu consiste la prueba de supresin de pptido C?
Se utiliza insulina sinttica para disminuir el nivel srico de insulina por
retroalimentacin negativa. Como es imposible distinguir entre la insuli-
na externa y la pancretica, se utiliza el nivel de pptido C para ver si la
ltima ha disminuido.
Cul es el tratamiento del insulinoma?
Es principalmente quirrgico; como tratamiento mdico se utilizan el
diazxido y la estreptozocina.
Qu otros nombres tiene el vipoma?
Sndrome de Verner Morrison, DAHA (diarrea acuosa, hipokalemia
aclorhidria), DAHH (diarrea acuosa, hipokalemia, hipoclorhidria), cle-
ra pancretico.
Cules son las funciones del pptido intestinal vasoactivo (VIP)?
Estimula la secrecin intestinal de agua y electrolitos; provoca tambin la
relajacin del msculo intestinal, la vasodilatacin de vasos sanguneos
perifricos y la inhibicin de la secrecin gstrica cida.
Cul es la triada del somatostatinoma?
116
Diabetes, esteatorrea y litiasis vesicular.
Cmo se presenta el glucagonoma?
Es un tumor de clulas alfa muy poco frecuente. Produce diabetes, hi-
poaminoacidemia, hipolipidemia, anemia normo-normo. Su caractersti-
ca distintiva es la erupcin rojo-parduzca que aparece en las
extremidades, llamada eritema migratorio necroltico.
Cul es el sndrome de Wermer?
Es la neoplasia endocrina mltiple tipo I (MEN-I) o adenomatosis endo-
crina mltiple tipo I.
Cul es el sndrome de Sipple?
Es la neoplasia endocrina mltiple tipo II (MEN-IIA) o adenomatosis
endocrina mltiple tipo II.
En quines se debe sospechar la MEN-I?
En quienes tengan sntomas que sugieren gastrinoma o tumor pituitario.
En quines se sospecha la MEN-IIA?
En personas que son hijas de pacientes con diagnstico de carcinoma
medular tiroideo, hiperaldosteronismo o feocromocitoma.
Cul es la presentacin clnica del somatostatinoma?
Diabetes, esteatorrea, clculos biliares y aclorhidria.
Cul es la presentacin clnica del glucagonoma?
Diabetes mellitus, anemia, prdida de peso y rash.

117
GASTROENTEROLOGA


Qu enzimas degradan las protenas de la dieta?
La pepsina gstrica y las proteasas pancreticas.
Qu nutrientes se absorben en el leon proximal?
cido flico, hierro y calcio.
Qu nutrientes se absorben en el leon terminal?
cidos biliares y vitamina B12.
Cules son dos causas de leo paraltico?
Hipokalemia y ciruga abdominal reciente.
Dnde se produce la secretina?
Se produce en la porcin superior del intestino delgado y su funcin con-
siste en aumentar la secrecin de bicarbonato por el pncreas.
Dnde se produce la colecistocinina (CCK)?
Se produce en la porcin superior del intestino delgado y provoca la con-
traccin de la vescula biliar, la secrecin de enzimas pancreticas y po-
tencia la accin de la secretina.
Qu producen las clulas parietales oxnticas?
Acido clorhdrico (HCl) y factor intrnseco.
Qu producen las clulas parietales cimgenas?
Pepsingeno.
Qu producen las clulas antrales en forma de botella?
Secretan gastrina, que aumenta la produccin de cido clorhdrico.
Qu es la gastritis Tipo A?
Es la gastritis del fondo asociada a anticuerpos anticlula parietal, niveles
altos de gastrina y anemia perniciosa.
Qu es la gastritis Tipo B?
Es la gastritis del antro y presenta anticuerpos antigastrina; la lesin se debe
a reflujo duodenobiliar asociado a cncer gstrico.
Qu es la enfermedad de Mntrier?
Es la gastritis hipertrfica.
118
Cmo se hace el diagnstico diferencial de la gastritis?
La gastritis produce dolor ms intenso despus de la ingesta de alimentos;
en los casos de lcera, el dolor disminuye tras la ingesta de alimentos pa-
ra recurrir 2-3 horas despus.
Cmo se realiza el diagnstico de sndrome de Zollinger-Ellison?
Tras una prueba de estimulacin con secretina, la gastrina alcanza valo-
res de 200 U.
Cules son las causas de la lcera duodenal?
Clulas parietales hipersensibles a la gastrina (hiperacidez) y la presencia
de H. pylori, que debilita la mucosa.
Dnde aparece la mayora de las lceras duodenales?
En la primera porcin del duodeno.
Cul es el patrn del dolor en la lcera duodenal?
El dolor se alivia con la ingesta de alimentos slo para recurrir algunas
horas ms tarde.
Cules son las causas de la lcera gstrica?
Antiinflamatorios no esteroidesos, reflujo biliar e hipersecrecin gstrica.
Dnde aparece la mayora de las lceras gstricas?
En el cuerpo y en la unin de las clulas parietales del cuerpo; las de gas-
trina en el antro.
Qu condiciones se asocian al cncer gstrico?
Anemia perniciosa, gastritis atrfica y la presencia de H. pylori.
Cmo se manifiesta el linfoma gstrico?
Se presenta con dolor y su estirpe histolgica es de tipo no Hodgkin.
Qu enfermedades no infecciosas se presentan con diarrea, sangre y cli-
cos?
El sndrome hemoltico urmico (SHU) y la colitis ulcerativa crnica
inespecfica (CUCI).
Qu enfermedades se presentan con heces formadas, sangrado y sin dolor?
Divertculo de Meckel, poliposis juvenil y colitis eosinoflica.
Cul es la causa ms frecuente de hemorragia del tracto digestivo alto
(HTDA)?
Enfermedad cido-pptica (EAP): 75%.
119
Vrices esofgicas: (5-15%).
Cules son las causas ms frecuentes de hemorragia del tracto digestivo
bajo (HTDB)?
Aguda: diverticulosis y angiodisplasia (1er. y 2 lugar en el anciano).
Crnica: hemorroides y cncer.
Jvenes: divertculo de Meckel, enfermedad inflamatoria.
Cmo se corroboran la HTDA y la HTDB?
HTDA: mediante una sonda nasogstrica o endoscopa superior.
HTDB: mediante el tacto rectal, anoscopia o rectosigmoidoscopia.
Cul es el diagnstico diferencial de la hemorragia del tacto digestivo
(HTD)?
Sangrado con dolor: isquemia, inflamatoria, aneurisma roto.
Sangrado sin dolor: divertculo de Meckel, diverticulosis, angiodis-
plasia, hemorroides.
Sangrado con diarrea: infeccin, causa inflamatoria.
Sangrado con dolor rectal: fisura hemorroides.
Sangrado con estreimiento: neoplasia.
(Otros datos como la edad o el tiempo de evolucin son importantes
para el diagnstico).
Cul es la diferencia entre la poliposis juvenil y el divertculo de Meckel?
Ambos padecimientos producen sangrado del tubo digestivo, indoloro y
con heces bien formadas. El primero aparece entre los 2 y los 12 aos y el
segundo, antes de los dos.
Cmo es el sangrado en la poliposis, en la colitis eosinoflica y en el di-
vertculo de Meckel?
En los primeros hay estras de sangre y en el ltimo, melena abundante.
Cundo est indicado el tratamiento quirrgico de la hemorragia del trac-
to digestivo alto?
Cuando requiere hasta 5 unidades de sangre en 24 horas.
Cmo se tratan las hemorroides de primero y segundo grados?
Con ligadura por arriba de la lnea dentada, con fotocoagulacin o con
escleroterapia.
120
Cmo se tratan las hemorroides de tercero o cuarto grados?
Con hemorroidectoma.
Cul es el tratamiento mdico de las fisuras anales?
Sediluvios y nitroglicerina tpica al 0.2%
Qu sangrado se detecta por angiografa?
El menor de 0.5ml/min.
Qu sangrado se detecta por gammagrafa?
El menor de 0.05ml/min.
Con qu dato de laboratorio se sospecha hemorragia del tacto digestivo
(HTD)?
Con un ndice BUN/creatinina de 36. (BUN= nitrgeno ureico en sangre).
Cul es el tratamiento mdico de la hemorragia activa por vrices esof-
gicas?
Vasopresina ms nitroparche.
Somatostatina u ocretida.
Cul es el tratamiento de eleccin de la hemorragia por varices esofgi-
cas?
La escleroterapia.
Cul es la clasificacin endoscpica del sangrado por lcera pptica?
lcera con sangrado visible (recidiva de 85%).
lcera sin sangrado visible (55%).
lcera con cogulo centinela.
Cules son las indicaciones del tratamiento quirrgico de la hemorragia
por enfermedad cido-pptica?
Inestabilidad hemodinmica.
Prdida de ms de 30% del volumen sanguneo en 24 horas.
Necesidad de 5 paquetes globulares o ms en 24 horas.
Cules son la lceras de Cushing y de Curling?
Cushing: lcera secundaria a dao intracraneal.
Curling: lcera secundaria a estrs por quemaduras de ms de 35% de
la superficie corporal.
Cmo se presentan ms frecuentemente las angiodisplasias?
En mayores de 40 aos.
121
En el colon derecho.
Como manchas rojo cereza.
Qu parte del colon se afecta ms en la colitis isqumica?
El ngulo esplnico.
Cul es la diferencia entre rectorragia y hematoquezia?
La primera es sangre color rojo brillante; la segunda, de rojo vinoso.
Qu es la acalasia?
Es un trastorno motor en el que disminuye la peristalsis esofgica. Al
mismo tiempo, se encuentra aumentada la presin del esfnter esofgico
inferior.
Cul es el cuadro clnico de la acalasia?
Disfagia para slidos y lquidos, dolor torcico, regurgitacin, tos noc-
turna y prdida de peso, de muchos aos de evolucin.
Cul es la imagen radiolgica tpica de la acalasia?
En pico de ave.
Cul es el tratamiento de la acalasia?
Esofagotoma modificada de Sller.
Toxina botulnica.
Cul es el tratamiento del espasmo esofgico difuso?
Bloqueadores de los canales del calcio y nitratos.
Cules son las causas principales de la perforacin esofgica?
Iatrognica en 50% y traumtica en 20%
Dnde se localiza con ms frecuencia la lcera gstrica?
En el cuerpo del estmago.
Cul es el diagnstico diferencial de la apendicitis?
Gastroenteritis aguda, adenitis mesentrica, divertculo de Meckel y en-
fermedad de Crohn. En mujeres jvenes debe considerarse los tumores
ovricos, quistes ovricos rotos y enfermedad inflamatoria plvica (EIP).
En pacientes mayores se debe descartar la diverticulitis, hernia encarce-
lada, carcinoma cecal y trombosis mesentrica.
Dnde se localizan principalmente los tumores carcinoides del intestino?
Apndice, leo y recto.
Cules son los cuatro tipos de plipo, en orden de frecuencia?
Hiperplsico, hamartomatoso, inflamatorio o edematoso y neoplsico.
122
Cul es el nico plipo con potencial maligno y que debe resecarse?
El neoplsico (ms el velloso).
Con qu se diagnostican los plipos?
El primer estudio debe ser un enema baritado contrastado con aire libre.
El de eleccin es la colonoscopia.
Cul es la diferencia entre el sndrome de Peutz-Jeghers y la poliposis ju-
venil?
Ambos tienen plipos de tipo hamartomatoso. En el sndrome de Peutz-
Jeghers tambin hay manchas hipopigmentadas en la boca, y los plipos
se extienden a lo largo del tubo digestivo; en cambio, la poliposis juvenil
slo aparecen en el recto.
Cul es la diferencia en el tratamiento del plipo ssil y el pedunculado?
El primero requiere colectoma parcial; el segundo, slo polipectoma
endoscpica.
Qu es la poliposis adenomatosa familiar (PAF) y cul es su gen?
Es una enfermedad donde generalmente hay ms de 100 plipos, que
desarrollan cncer 100% de las veces. La presencia de C5PAF aumenta el
riesgo de cncer del colon.
Qu es el sndrome de Gardner?
Es una variante de la poliposis adenomatosa familiar, asociada con os-
teomatosis, quistes epidermoides y fibromas de la piel; sus genes son el
5q21 y 5q22.
Qu es el sndrome de Turcot?
Es la poliposis adenomatosa familiar (PAF) con tumores malignos del
sistema nervioso central.
Cules sndromes de poliposis son autosmicos dominantes?
El sndrome de Peutz-Jeghers (SPJ), la poliposis juvenil difusa y la
poliposis adenomatosa familiar (cromosoma 5).
Cules son las complicaciones de los sndromes de poliposis?
Todos pueden causar intususcepcin y sangrado.
Los adenomatosos (SPJ, PAF) pueden causar un estado de prdida
de protenas.
La PAF evoluciona a cncer en 100% de los casos.
Cul es la diferencia entre el tratamiento de la PAF y el SPJ?
123
En la primera se requiere colectoma total y en la segunda slo se extirpa
un segmento o los plipos.
Qu es el sndrome de Trousseau?
Es la coagulacin intravascular diseminada trombtica crnica en pa-
cientes con carcinomatosis. Es ms frecuente en el cncer del pncreas y
del estmago.
Cmo se realiza la evaluacin inicial de diverticulitis?
Con tomografa y radiografa del abdomen y del trax, que revelarn aire
subdiafragmtico.
Cul es el tratamiento de la diverticulitis?
Dieta baja en residuo y metronidazol o cefalosporinas de segunda o terce-
ras generacin.
Cul es el diagnstico diferencial de la enfermedad de Crohn?
Linfoma, TB, Yersinia, enteritis por radiacin.
Cules son las manifestaciones extraintestinales de la enfermedad de
Crohn?
Fiebre, malestar, eritema nodoso, pioderma gangrenoso, episecleritis,
uvetis, nefrolitiasis y afeccin hepatobiliar.
Cules son los hallazgos radiolgicos en la enfermedad de Crohn?
lceras profundas (collar button), signo de la cuerda y reas respetadas.
Cules son los hallazgos anatomopatolgicos en la enfermedad de Crohn?
Inflamacin transmural, engrosamiento de la pared del intestino, granu-
lomas y una imagen con reas respetadas y afectadas en forma alterna.
Qu anticuerpo se asocia a la enfermedad de Crohn?
El anticuerpo anti-Saccharomices cerevisiae (ASCA).
Qu medicamentos se utilizan para tratar le enfermedad de Crohn?
Sulfasalazina a dosis de 3-4 gr/da e infliximab.
Qu es el infliximab?
Es un anticuerpo quimrico que se une al factor de necrosis tumoral-alfa.
Cules son las manifestaciones extraintestinales de la colitis ulcerativa
crnica inespecfica?
Artitis asimtrica, sacroiletis, eritema nodoso, pioderma gangrenoso,
uvetis y colangitis esclerosante.
124
Cules son los hallazgos anatomopatolgicos en colitis ulcerativa crnica
inespecfica (CUCI)?
Microabscesos de las criptas de Liberkun.
Cules son los anticuerpos encontrados en enfermos de CUCI?
Anticuerpos anticitoplasma de neutrfilo y anticuerpos antisacaromices.
Cundo se indica el cido 5-aminosaliclico para pacientes con CUCI?
Cuando los pacientes son alrgicos a la sulfasalazina.
Cules son los cambios anatomopatolgicos encontrados en el esprue celaco?
Atrofia de vellosidades, hipetrofia de las criptas, cambios cuboidales e in-
filtracin extensa de la lmina propria.
Cules son las complicaciones de la enfermedad celaca?
Linfoma, lceras, estenosis, dermatitis herpetiforme y esprue colgeno.
Cules son los antgenos asociados a la enfermedad celaca?
HLA-B8, HLA-DW3 y el antgeno de linfocito B.
Cul es el cambio anatomopatolgico en la enfermedad de Whipple?
Macrfagos PAS positivos (contienen a los bacilos).
Cules son las pruebas para evaluar la funcin sinttica del hgado?
Colesterol, protenas sricas, tiempo de protrombina y glucosa.
Cules pruebas se utilizan para evaluar la funcin depurativa del hgado?
Bilirrubina indirecta y amonio.
Cules tests se utilizan para evaluar la funcin excretora del hgado?
Determinacin de bilirrubina directa, aspartato-aminotransferasa (AST) y
gamma-glutamiltranspeptidasa (gamma-GTP).
Qu prueba se utilizan para evaluar la extensin del dao heptico?
La determinacin de aspartato-aminotransferasa (AST) y alanina-
aminotransferasa (ALT).
Cules son los patrones en el estudio de las pruebas del funcionamiento
heptico?
Bilirrubinas: 30-40 (y se mantienen) en enfermedad obstructiva; 50 en
adelante en enfermedad hepatocelular.
AST y ALT: son indicadores de dao hepatocelular. Mayor que 500
indica dao viral o alcohlico, 100-2000, dao por drogas. Mayor
125
que 4000, indica dao por acetaminofn o por sustancias hepatotxi-
cas.
Relacin AST/ALT: mayor que 2 indica dao alcohlico; mayor que
3 indica dao por enfermedad heptica.
ALP: Es un indicador de obstruccin biliar. Obstruccin: fosfatasa al-
calina (FA) 3-10 veces el valor normal; ALT y AST, poco eleva-
das.Enfermedad parenquimatosa: AST, ALT 3 a 10 veces lo normal y
FA poco elevada. ndice GT/ALP mayor que 5 indica hepatopata
alcohlica.
Colesterol: Marcadamente disminuido, en hepatitis y cirrosis; marca-
damente aumentado, en procesos obstructivos intra y extrahepticos
y en la cirrosis biliar primaria.
-globulina:aumentada en la enfermedad heptica crnica.
Cules son las alteraciones de laboratorio de la ictericia obstructiva?
Bilirrubinas altas, AST mayor de 300 y ALP aumentada tres veces lo
normal.
Cules son las alteraciones de laboratorio de la hepatitis alcohlica?
AST/ALT mayor de 3, ALT mayor de 300.
Cules son las alteraciones de laboratorio en el alcoholismo crnico?
Gamma-GTP/ALP mayor de 5 (tambin se usa para evaluar la toxicidad
por anticomiciales).
Cundo se utiliza la determinacin de gamma-GTP?
Para el diagnstico de enfermedad heptica cuando coexisten embarazo,
enfermedad sea, en los nios (porque tienen normalmente elevada la
ALP por el alto recambio seo) y en el alcoholismo oculto.
Cmo se presenta la hepatitis crnica?
Enzimas elevadas durante ms de 6 meses, malestar, fatiga y vasculitis.
Cmo se hace la distincin entre la hepatitis B crnica activa y el estado
de portador asintomtico?
En una biopsia se observa inflamacin portal con necrosis progresiva
cuando es hepatitis crnica activa.
Qu enfermedades se asocian a la hepatitis B?
La poliarteritis nodosa y la glomerulonefritis.
126
Qu enfermedades se asocian a la hepatitis C?
La porfiria cutnea tarda y la crioglobulinemia.
Por qu se administra espironolactona a los pacientes con ascitis?
Porque el metabolismo heptico est disminuido y la depuracin de la
aldosterona est afectada; por lo tanto, su efecto se prolonga y el paciente
tiende a retener sodio y agua.
Por qu la hipokalemia provoca encefalopata portosistmica?
Porque junto con la alcalosis que produce disminuye la excrecin renal
de NH
3
Qu es el sndrome hepatorrenal?
Es una disfuncin renal secundaria a vasoconstriccin sostenida de los
vasos renales en pacientes cirrticos en fase avanzada y con ascitis. Apa-
rece en la insuficiencia heptica terminal con azoemia, sodio urinario ba-
jo, sodio srico bajo e hipotensin arterial.
Qu significa un gradiente de albmina suero-ascitis >1.1?
Que hay mas protenas en el suero que en el lquido de ascitis; por tanto,
la presin onctica est conservada y la causa de la fuga de lquido es un
aumento en la presin hidrosttica portal.
Qu significa un resultado con protenas totales <1 en lquido de ascitis?
Riesgo aumentado para peritonitis bacteriana espontnea.
Cul es el cuadro clnico de la peritonitis bacteriana espontnea?
El paciente con ascitis presenta fiebre, dolor abdominal y una cuenta de
ms de 250 PMN en el citolgico del lquido de ascitis.
Para qu sirven el USG, la TAC y la angiografa en el estudio de la pato-
loga heptica?
USG: investigacin de lesiones parenquimatosas.
TAC: estudio de lesiones que involucran el parnquima y estructuras
adyacentes.
Angiografa: estudio de lesiones parenquimatosas extensas.
Cul es la presentacin clnica del quiste y del absceso hepticos?
El primero, con escasos sntomas focales (masa y dolor). En el segundo,
predominan los sntomas sistmicos de inicio subagudo y por varias se-
manas (fiebre, malestar, anemia, sepsis y malnutricin); puede haber do-
127
lor pleurtico derecho y hepatomegalia, as como dolor en el cuadrante
superior derecho.
Cules son causas de absceso heptico bacteriano?
Colangitis, apendicitis, diverticulitis. Cuando el foco es abdominal pre-
dominan los gram-negativos y cuando es extraabdominal los gram-
positivos.
Cmo se presenta el absceso heptico amibiano?
Con sntomas de absceso; en la imagen se observa solitario y en 90% de
los casos se ubica del lado derecho.
Cules son las manifestaciones clnicas de la cirrosis?
Encefalopata, varices esofgicas y hemorragia del tracto digestivo alto,
malnutricin, ascitis, red venosa colateral y esplenomegalia.
Qu enfermedades hepatobiliares tienen como sntoma inicial el prurito?
La cirrosis biliar primaria (90% en mujeres) y colangitis esclerosante
(80% en hombres)
Cul es el tratamiento de la cirrosis biliar primaria?
Esencialmente sintomtico y con cido ursodesoxiclico.
Cules son los cambios anatomopatolgicos en la deficiencia de alfa-1-
antitripsina?
Se observan glbulos PAS positivos en las reas portales.
Por qu aparecen galactorrea, masculinizacin, feminizacin y sndrome
de Cushing en la cirrosis?
Porque en el hgado se depuran los esteroides y se produce la globulina fi-
jadora de hormonas esteroideas. Por tanto la actividad esteroide aumenta.
Cul es la causa principal de la hipertensin portal?
La cirrosis heptica.
A qu presin portal aparecen las vrices esofgicas?
20 mm Hg. La presin normal es de 5-6mmHg.
Cul es el tratamiento farmacolgico de la hemorragia del tracto digesti-
vo alto secundaria a vrices esofgicas?
Vasopresina y nitroglicerina: por su efecto vasodilatador sistmico
reducen la presin portal.
128
Somatostatina y ocretida: son vasodilatadores esplcnicos.
Metoclopramida y pentagastrina: aumentan el tono del esfnter eso-
fgico inferior, que disminuye la circulacin a ese nivel.
De cunto es el cociente amilasa-creatinina que indica la posibilidad de
pancreatitis?
Mayor de 5.
Cules son los signos radiolgicos de la pancreatitis?
El del colon cortado y el del 3 invertido.
Qu es la prueba de Nardi?
Es un espasmo biliar con elevacin secundaria de la amilasa, que se pro-
duce con la administracin de narcticos. Se utiliza para demostrar la
existencia de pancreatitis recidivante.
Cul es la etiologa principal de la pancreatitis crnica?
El alcoholismo.
Qu cambios radiolgicos son propios de la pancreatitis crnica?
Calcificaciones e imagen de red de lagos.
Cul es el estndar de oro para diagnosticar insuficiencia pancretica?
Se hace una prueba con colecistocinina y se miden las enzimas y el bicar-
bonato que se producen despus de su aplicacin.
Cules son las manifestaciones del sndrome de insuficiencia pancretica?
Malnutricin.
Esteatorrea: las grasas no se emulsifican para su absorcin.
Hemorragia: por absorcin deficiente de vitamina K.
Raquitismo: por deficiencia de vitamina D.
Anormalidades neurolgicas: Nictalopa secundaria a deficiencia de
vitamina A.
Cules son los cuatro procedimientos quirrgicos para el tratamiento de
la pancreatitis crnica?
Colangiografa pancretica retrgrada endoscpica (ERCP), Puestow,
Duval y Child.
Cul es la presentacin clnica del pseudoquiste pancretico?
Ocurre tras un caso de pancreatitis. Cursa con dolor crnico, fiebre poco
elevada, leucocitosis mnima cifras altas de amilasa y masa palpable.
Aparece en 5% de los casos.
129
Cul es la localizacin ms frecuente del cncer de pancretico?
La cabeza del pncreas; se presenta como ictericia indolora.
Qu se encuentra detrs de la primera porcin del duodeno y de la cabeza
del pncreas?
La vena porta.

130
131
GINECOLOGA Y OBSTETRICIA

Cul es la causa ms frecuente de quiste vaginal en la infancia?
El urter ectpico. El quiste vaginal es dependiente del tejido ureteral que
deriva de las estructuras de Wolff (mesonfricas) en especial del conducto
de Gartner.
Cul es el orden de aparicin de los caracteres sexuales secundarios en la
mujer?
Telarquia, pubarquia, tasa mxima de crecimiento y menarquia.
Cundo se considera pubertad precoz?
Cuando la aparicin de los caracteres sexuales secundarios ocurre antes
de los 9 aos.
Cundo ocurre el pico de estradiol y cmo se evidencia clnicamente?
El pico de hormona luteinizante ocurre en el da 14 del ciclo; dos das an-
tes (en el da 12), ocurre el pico de estradiol que se evidencia por un au-
mento en la temperatura.
A qu niveles ocurren las anomalas que explican la amenorrea de la ado-
lescente?
Anatmico (Mlleriano): sndrome de Mayer Rokitansky Kunster
Hauser e himen imperforado, aplasia vaginal y uterina y septos vagi-
nales.
Hipotalmico: sndrome de Kallman [disminucin de la hormona li-
beradora de gonadotropinas (GnRH) y anosmia].
Gonadal: sndrome de Turner (45XO), sndrome de Swyer (disgene-
sia gonadal pura 46xy).
Cundo deben estudiarse la pubertad retrasada y la amenorrea primaria?
Cuando hay ausencia de caracteres sexuales secundarios despus de
los 13 aos.
Cuando hay ausencia de menarca despus de los 16 aos.
Qu frmacos inhiben la dopamina?
Fenotiacinas, butirofenonas, tricclicos, metildopa, reserpina, metoclo-
pramida, cimetidina y otros.
132
Cul es el efecto de la inhibicin dopaminrgica sobre la hipfisis?
Aumenta la secrecin de prolactina.
Por qu la hiperprolactinemia es causa de amenorrea?
Porque inhibe directamente a la hormona liberadora de gonadotropinas
(GnRH), y tiene accin inhibitoria sobre el ovario.
En quin debe sospecharse la presencia de endometriosis?
En cualquier mujer con infertilidad, dispareunia y amenorrea.
Cul de las tres infecciones vaginales ms frecuentes presenta mayor sin-
tomatologa?
La candidiosis.
En qu porcentaje son asintomticas la vaginosis bacteriana y la tricomo-
niasis?
En 50%.
Qu se observa en las laminillas de muestras en estos padecimientos:
Candidiosis: hifas y pseudohifas (preparada con KOH)
Vaginosis bacteriana: linfocitos escasos y clulas clave.
Tricomoniasis: leucocitos abundantes y parsito flagelado mvil.
Qu diferencia hay entre menorragia y metrorragia?
En las dos el sangrado es prolongado y abundante, pero en la menorragia
ocurre a intervalos regulares y en la metrorragia es irregular.
Cules son tres causas de ciclos anovulatorios?
La inmadurez del eje hipotlamo-hipfisis, la disfuncin del eje hipotlamo-
hipfisis y la falla de la retroalimentacin por aumento del estradiol.
Cules son las causas de aumento del estradiol?
El embarazo, la falla del metabolismo y la produccin ectpica (adenoma
o mucho tejido adiposo).
Qu causas producen disfuncin del eje hipotlamo-hipfisis?
Hiperprolactinemia.
Stress y ansiedad.
Prdida de peso.
Trastorno limtrofe (borderline).
Anorexia nervosa.
133
Cul es el tratamiento mdico de la hemorragia uterina disfuncional
(HUD) aguda?
Dosis altas de estrgenos ya que estimulan la cicatrizacin de los vasos
sangrantes.
Cul es el tratamiento quirrgico de la hemorragia uterina disfuncional
aguda?
Dilatacin y curetaje.
Cul es el tratamiento de la HUD crnica (hiperplasia endometrial)?
Progestina (10mg/dl), por su accin antimittica.
Cul es la triada de la endometriosis?
Dismenorrea, dispareunia e infertilidad
Cmo se hace el diagnstico de endometriosis?
En orden de importancia, primero con la historia clnica y la exploracin
fsica. En algunos casos se requiere laparotoma exploradora.
Cules son los hallazgos a la exploracin fsica en la endometriosis?
Nodularidad en el ligamento terosacro, a la palpacin rectovaginal.
Masa anexial en el ligamento redondo, a la palpacin vaginal.
Qu marcador tumoral se eleva en la endometriosis?
El CA125.
Cul es el tratamiento mdico de la endometriosis?
Progestina y estrgeno para producir la atrofia del tejido ectpico.
AINES, 2 a 3 das antes de iniciar cada periodo, para aliviar la dis-
menorrea.
El danazol tambin causa atrofia del tejido endometrial, pero tiene
efectos antiestrognicos.
Las progestinas de depsito, como la medroxiprogesterona, causan
cambios deciduales en el tejido ectpico que alivian los sntomas.
En qu padecimientos est indicado el uso de danazol?
Endometriosis y mastopata fibroqustica.
Cul es el tratamiento de la endometriosis severa?
Quirrgico.
Cules son los exmenes endocrinolgicos utilizables para el estudio de
las amenorreas?
134
Niveles de prolactina y TSH.
Niveles matutinos de cortisol.
Niveles de dehidroepiandrosterona y testosterona.

T
3
y T
4.

Cul es el diagnstico diferencial del dolor plvico ginecolgico?
Clico: cuando existe un rgano hueco obstruido; puede originarse
en el intestino, la vescula, el urter y el apndice.
Sbito: perforacin o isquemia.
Lento: por inflamacin u obstruccin (salpingitis, apendicitis, obs-
truccin).
Localizado: anexos o focal del tero.
Irritacin peritoneal: aumenta con los movimientos abdominales y
generales; aumenta con los movimientos intestinales o vesicales; es
generalizado.
Masa anexial dolorosa: embarazo ectpico, quiste ovrico y masa in-
flamatoria.
Vmito precoz: apendicitis, colecistitis, salpingitis, pielonefritis.
Vmito tardo: obstruccin intestinal.
Cul es el tratamiento de la dismenorrea severa?
Inhibidores de las prostaglandinas y un ciclo de anticonceptivos orales.
Cules son los pasos en el estudio de la dismenorrea?
Citologa hemtica y velocidad de sedimentacin globular (VSG) pa-
ra descartar causas infecciosas.
Exudado vaginal y pruebas especficas para descartar enfermedades
de transmisin sexual (ETS).
USG plvico para descartar quistes ovricos.
Laparotoma exploradora para establecer el diagnstico de endome-
triosis o para descartar otra patologa.
Qu son las clulas clave?
Son clulas epiteliales vaginales rodeadas de restos de gram-negativos en-
contradas en la secrecin vaginal de pacientes con vaginosis bacteriana.
Qu serotipos de Chlamydia causan tracoma?
Los serotipos A y C.
135
Qu serotipos de Chalmydia causan la conjuntivitis de inclusin?
El D y K.
Cul es el sndrome de Fitz-Hugh-Curtis?
Es una perihepatitis secundaria a enfermedad plvica infecciosa (EPI) y
causada por Chlamydia o gonococo.
Cules son las contraindicaciones de contraceptivos hormonales en muje-
res >35aos?
Diabetes, tabaquismo, obesidad y migraa.
Cules son las contraindicaciones generales de los contraceptivos hormo-
nales?
Historia de cncer endometrial o mamario, hipertensin arterial, antece-
dente de enfermedad vascular cerebral (EVC) y de estados trombticos.
Cules son los estmulos estrognicos que predisponen al cncer endome-
trial?
Menarca temprana, menopausia tarda, obesidad, sndrome de Stein Le-
venthal, terapia estrognica sin progestina, tumores secretantes de estr-
geno
En qu proporcin se desarrolla el cncer endometrial a partir de la hi-
perplasia endometrial?
1 a 14 % de los casos desarrolla cncer.
Cules son los tipos de hiperplasia endometrial?
La hiperplasia simple y la adenomatosa. La adenomatosa puede ser con
atipia o sin ella.
Cul es el tratamiento de la hiperplasia endometrial?
Se administra una progestina cada 10 a 12 das durante 3 a 6 meses y
se monitorea con biopsias cada 3 meses.
Si la hiperplasia persiste se somete a la paciente a histerectoma.
Cul es el tratamiento de la hiperplasia endometrial en las adolescentes?
Se administra estrgeno y progestina durante 6 meses.
Cundo se positivizan las deteminaciones de la fraccin beta de la gona-
dotropina corinica humana (HGC) en caso de embarazo?
136
Se puede detectar la fraccin beta de la gonadotropina corinica humana
(HGC) despus de 7 das de la concepcin y 7 das antes de la fecha espe-
rada de menstruacin.
Cundo una prueba sin esfuerzo es negativa?
Cuando no hay aceleraciones en el ritmo cardiaco que duren ms de 15
segundos durante un periodo de 20 minutos.
Qu es la reactividad?
Es un aumento de 15 a 20 latidos por minuto en la frecuencia cardiaca fe-
tal basal ante un estmulo que dure 15 o 20 segundos.
Cul es la causa ms frecuente de hiporreactividad no patolgica?
Durante el sueo fetal puede haber periodos de hasta 40 minutos de hi-
porreactividad.
Cmo se define el sufrimiento fetal?
Como un aumento en la frecuencia cardiaca del feto acompaado de una
disminucin en el pH fetal.
Cules son los resultados de la hipoxia fetal prolongada?
Acidosis respiratoria que evoluciona a metablica y disminucin de la
frecuencia cardiaca fetal.
Cmo se interpreta la variabilidad disminuida de la frecuencia cardiaca
fetal (FCF)?
Como un pobre control autonmico sobre la FCF, lo cual es de mal pro-
nstico para el producto.
Qu exmenes estn indicados desde la semana 15 a la 20 del embarzo?
Determinacin de alfa-fetoprotena para la deteccin de defectos del tubo
neural, niveles de gonadotropina corinica para deteccin de sndrome de
Down y determinacin de estriol no conjugado para detectar trisoma 18.
Cules son dos signos de embarazo ectpico encontrados en la explora-
cin fsica?
Masa anexial y dolor a la movilizacin del crviz.
Por qu las infecciones bacterianas son un factor de riesgo de parto pre-
trmino?
Las bacterias producen prostaglandinas que estimulan al miometrio.
Cul es el agente etiolgico ms comn de la infeccin puerperal grave?
137
Clostridium.
Cul es el agente de la infeccin puerperal sin hemlisis?
Estreptococo grupo B.
Cul es la incidencia de hemorragia del tercer trimestre del embarazo?
Con un episodio previo sucede en 1 de 250. El abruptio placentae ocurre 1
en 120.
Cul es la incidencia del aborto recurrente?
Si es aislado, 2%. Si es recurrente, de 25 a 30%.
Cul es la triada del embarazo ectpico?
Amenorrea, manchado (sangrado) y dolor.
Cmo se clasifica la hipertensin durante el embarazo?
En hipertensin crnica complicada con el embarazo e hipertensin in-
ducida por el embarazo.
Cules son los subtipos clnicos de la hipertensin inducida por embarazo?
Preeclampsia: cuando existe compromiso renal y se detecta proteinu-
ria.
Eclampsia: cuando hay compromiso del SNC y ocurren convulsio-
nes.
Sndrome HELLP: presencia de hemlisis, enzimas hepticas eleva-
das y plaquetas bajas.
Cundo se diagnostica la hipertensin en el embarazo?
Cuando existe un aumento de 30 mm Hg en la sistlica y de 15 mm
Hg en la diastlica en relacin con valores previos antes de las 20 se-
manas de gestacin (SDG).
Cuando se detecta una cifra aislada de 140/90 despus de las 20
SDG
Cuando hay un aumento de 20 mm Hg en la tensin arterial media
(TAM) en relacin a valores previos o un valor total de 150 mm Hg
en la TAM.
Cmo se diagnostica la proteinuria?
Normalmente hay una excrecin de protenas de 0.15 g en 24 horas.
Cuando las protenas en orina alcanzan 3 g se habla de proteinuria.
Cmo se diagnostica la presencia de edema?
138
Con el signo de godete o la ganancia de 2,300 g en una semana.
Cules son los criterios de severidad de la preeclampsia?
Presin sistlica de 160 mm Hg y diastlica de 110 mm Hg.
Proteinuria mayor a 5 g/24horas.
Creatinina srica mayor de1.4.
Convulsiones.
Edema pulmonar.
Menos de 10,000 plaquetas.
Disfuncin heptica, dolor epigstrico o ruptura de la cpsula.
Trastornos vasculares como espasmo retiniano y anemia microangioptica.
Trastornos fetoplacentarios como RCIU o sufrimiento fetal.
A qu se llama isoinmunizacin Rh?
Es la produccin de anticuerpos maternos ante la exposicin a clulas fe-
tales que contienen antgenos del grupo Rh como el Cc, Dd y Ee. Estos
anticuerpos atacan a los eritrocitos fetales y prodecen diferentes grados
de hemlisis e incluso hidrops fetalis.
Qu es la eritroblastosis fetal?
Es la presencia de hemlisis fetal secundaria a anticuerpos anti-d mater-
nos, anemia secundaria a hemlisis, obstruccin umbilico-portal fetal que
simula hipertensin portal, cese de las funciones hepticas normales, ca-
da de la presin coloidosmtica fetal, que causa edema.
Cmo se trata la isoinmunizacin Rh?
Con la administracin de gamma-globulina anti-d dentro de las primeras
72 horas posparto.
Cmo se diagnostica la intolerancia a la glucosa en el embarazo?
A las 24 o 28 semanas debe haber al menos dos prueba de tolerancia a la
glucosa con valores alterados.
Por qu la diabetes en el embarazo causa polihidrammios?
Porque la hiperglucemia produce poliuria en el producto.
Cules son las dosis de insulina para tratar la diabetes gestacional?
Inicial 15 a 20 U. En la primera mitad del embarazo se calcula 0.5
U/kg. En la segunda mitad del embarazo se calcula a 0.7 U/kg.
139
Se administran 2/3 de la dosis total en la maana. La dosis matutina
puede estar compuesta por 2/3 de insulina intermedia y 1/3 de insu-
lina rpida.
Se administra el 1/3 restante por la noche. Esta dosis se compone de
rpida y lenta.
Por qu la insulina no afecta al producto?
Porque debido a su peso molecular mayor de 6,000 no cruza la barrera
placentaria.
Cul es la incidencia de la enfermedad trofoblstica?
Benigna: 1 en 1,200; maligna: 1 en 20,000.
Cundo se habla de neoplasia gestacional metastsica de bajo riesgo?
Cuando los niveles de gonadotropina corinica beta son menor de 40,000
y la evolucin es menor a 4 meses.
Qu es una mola incompleta?
Es una variedad de enfermedad trofoblstica en la que hay presencia de
un feto y membranas. Las vellosidades corinicas se encuentran edema-
tizadas y el cariotipo es 69XXX.
Cul es la ttrada de la enfermedad trofoblstica?
Sangrado en el primer trimestre
Preeclampsia
Hipertiroidismo clnico (por reaccin cruzada de la -HGC en exceso)
Nusea vmito, dolor abdominal.
Cundo est indicado utilizar quimioterapia e histerectoma simple en la
enfermedad trofoblstica?
En la neoplasia trofoblstica gestacional no metastsica y en la que cursa
con metstasis de bajo riesgo.

140
141
HEMATOLOGA







Cules son los diferentes sitios donde se lleva a cabo la hematopoyesis
durante la vida fetal?
0 a 12 semanas en el saco vitelino.
12 a 24 semanas en el hgado.
A partir de las 24 semanas en la mdula sea.
Cules son los ndices eritrocitarios?
Hemoglobina corpuscular media (HCM)= Hemoglobina (Hb)/Hemates
(RBC).
Concentracin de hemoglobina corpuscular media (CMCH)= Hb/
hematocrito (Hto).
Volumen corpuscular medio (VCM)= Hto/RBC.
Qu pruebas se usan en los estudios hematolgicos?
Ferritina: 15-150 g/dl. Es la principal protena para el almacenaje
de hierro; se usa para el diagnstico de deficiencia de fierro o de su
exceso, como en la hemocromatosis.
Hierro srico: 50-150 mg/dl. Para el diagnstico diferencial de las
anemias. Bajo en las ferropnicas, elevado en las hemolticas.
Hierro medular: 50-60%. es el estndar de oro para el diagnstico de
ferropenia.
Haptoglobina: es un indicador de hemlisis crnica; su nivel dismi-
nuye en las reacciones postransfusionales.
Fosfatasa alcalina leucocitaria: para el diagnstico diferencial entre la
leucemia mielgena crnica y la reaccin leucemoide.
Capacidad de fijacin de transferrina: 300-360 g/dl.
142
Saturacin de transferrina: 30-50%.
Citometra de flujo: se utiliza para el diagnstico de las leucemias,
mielodisplasias y los linfomas; para medir la actividad sinttica de
DNA en los tumores y para el conteo de subpoblaciones de linfocitos.
INR: 2-3. Es otra forma de informar el tiempo de protrombina (TP)
para medir la actividad de los factores dependientes de la vitamina k
(va extrnseca) y monitorear la accin de los cumarnicos.
Cules son causas de anemia microctica?
Falla en la sntesis del hem como en la falta de hierro, la sideroblasto-
sis o la inflamacin crnica.
Falla en la sntesis de globina, como en las talasemias o en las enfer-
medades por hemoglobinas anormales.
Cmo se clasifican las anemias macrocticas?
Con mdula megaloblstica, como en las deficiencias de vitamina B-
12 y cido flico.
Sin mdula megaloblstica como en las hepatopatas, el hipotiroi-
dismo y la enfermedad de mdula sea.
Cules son las causas de anemia normo-normo?
Anemia aplsica.
Hemorragia.
Hemlisis.
Insuficiencia renal crnica.
Cules son las anemias por trastornos del metabolismo eritrocitario?
Los defectos de la va de Embden-Meyerhof.
Los defectos en el shunt hexosa-monofosfato (deficiencia de glucosa-
6-fosfato-deshidrogebasa) (DG6PD).
Cules son las anemias por trastornos congnitos de la membrana eritro-
citaria?
Eliptocitosis hereditaria: transtorno de herencia autosmica domi-
nante, eliptocitos, escasa o nula hemlisis, esplenomegalia ocasional.
Esferocitosis hereditaria: heredada como rasgo dominante, con he-
mlisis franca e ictericia.
143
Qu datos se observan al microscopio en las enfermedades siguientes?
Talasemia: microcitosis y poiquilocitosis.
Anemia sideroblstica: sideroblastos en anillo (depsitos de fierro mi-
tocondrial vistos con azul de Prusia).
Anemia perniciosa: macrocitosis, polimorfonucleares hipersegmen-
tados; en la mdula sea se observan clulas con asincrona citoplas-
ma-ncleo.
Chdiak Higashi: Inclusiones granulomatosas gigantes en los neutr-
filo.
Anemias microangiopticas (coagulacin intravascular diseminada
(CID) prpura trombocitopnica trombtica (PTT), vlvula cardia-
ca): esquistocitos.
Cules son causas de anemia microctica y cuenta baja de eritrocitos?
Anemia ferropnica y talasemia alfa.
Cules son causas de anemia microctica y cuenta de eritrocitos normal o
alta?
La talasemia beta menor.
Cules son las alteraciones de laboratorio en la ferropenia?
Ferritina menor de 15, hierro srico menor de 60, capacidad de fijacin
de transferrina mayor de 360, saturacin menor de 20%.
Cules son las alteraciones de laboratorio en la anemia microctica hipo-
crmica?
VCM menor de 80 fl, CMHC menor de 30g/dl, Hb menor de 11, Hto
menor de 38, capacidad de transporte de hierro aumentada, hierro srico
disminuido.
Cules son las alteraciones de laboratorio de la anemia de la enfermedad
crnica?
Capacidad de transporte de hierro baja, hierro srico bajo anemia nor-
moctica normocrmica.
Cul es el tratamiento de la anemia de la enfermedad crnica?
Eritropoyetina.
Qu es la anemia perniciosa?
Es una anemia macroctica con: polimorfonucleares hipersegmentados,
mdula hipercelular, eritropoyesis inefectiva, hemlisis intramedular que
144
causa deshidrogenasa lctica menor de 500 (normal de 100-2,450), glosi-
tis atrfica y sntomas neurolgicos. Se debe a un dficit de vitamina B-
12 que puede ser por una dieta insuficiente, o bien, por deficiencia de se-
crecin de factor intrnseco por las clulas parietales gstricas, como se
observa en ciertos tipos de gastritis, como la atrfica tipo A, que tiene an-
ticuerpos anticlula parietal.
Qu complicacin grave puede acompaar a la anemia perniciosa?
La degeneracin de las astas anteriores medulares.
Qu es la anemia aplstica?
Es una falta de clulas madre en la mdula sea.
Cul es la diferencia en el tratamiento de la eliptocitosis y la esferocitosis?
En la esferocitosis es necesaria la esplenectoma; en la eliptocitosis puede
ayudar, pero generalmente no es necesaria si la hemlisis y la anemia son
leves.
Qu es la deficiencia de glucosa-6-fosfato-deshidrogenasa (DG6PD)?
Es una alteracin en la va de la pentosa fosfato para producir NADPH que
active la glutatin-reductasa. El glutatin reducido acta como antioxidan-
te en los eritrocitos eliminando todos los radicales O
2
.
Dnde observamos los cuerpos de Heinz?
En la deficiencia de glucosa-6-fosfato-deshidrogenasa.
Por qu se observan clulas mordidas en la DG6PD?
En los individuos con bazo activo, ste extrae de los hemates los cuerpos
de Heinz (estadios avanzados) y les da esa apariencia.
Qu frmacos desencadenan hemlisis en pacientes con DG6PD?
Primaquina, salicilatos, sulfas, nitrofurantona.
Cules son las variedades de la DG6PD?
La tipo A, que se da en Asia y frica
La tipo B, que es el tipo mediterrneo.
Qu es la deficiencia de piruvatocinasa (DPK)?
La piruvatocinasa produce ATP en los eritrocitos; si ste falta, se inacti-
van las bombas sodio-potasio.
Cul es la diferencia en el tratamiento de la DG6PD y de la DPK?
En la primera no se indica la esplenectoma; en la segunda s.
145
Cmo est formada la hemoglobina?
La hemoglobina es una protena con un peso molecular de 64450 forma-
da por cuatro subunidades, cada una de ellas contiene un hem conjugado
con un polipptido. El hem es un derivado de la porfirina y los polippti-
dos se llaman cadenas alfa, beta, gamma, etctera.
Cules son los pasos en la formacin del hem?
cido delta aminolevulnico porfobilingeno hidroximetilbilano
URO III coproporfiringeno protoporfiringeno protoporfirina
hem.
Qu es la metahemoglobina?
Es la hemoglobina cuya fraccin de hierro se encuentra en estado frrico
en lugar de ferroso.
Qu sistema evita la sobreproduccin de metahemoglobina?
El sistema reductasa NADH-metahemoglobina.
A qu se llama anemias por sntesis defectuosa de hemoglobina?
Son alteraciones genticas de la molcula de Hb que se demuestran por
cambios en las caractersticas qumicas, en la movilidad electrofortica o
en otras propiedades.
Cules son las hemoglobinas del adulto?
HbA (cadenas alfa-2-beta-2), HbA2 (alfa-2-delta-2), HbA1c. Todas son
normales. La ms abundante es la HbA.
Cules son las hemoglobinas fetales?
Hemoglobina F (alfa2-gamma2).
Cules son las hemoglobinas embrionarias?
Gower 1(z
2

2
) y Gower 2 (alfa2-psilon2).
Cul es el error gentico que da origen a la alfa-talasemia?
La delecin del gen que codifica la cadena alfa.
Cul es el error gentico que da origen a la beta-talasemia?
Una anormalidad en la trascripcin para codificar la cadena beta.
Cuntos genes existen para la codificacin de la cadena alfa?
Cuatro.
Cmo se le llama al estado en que existen slo tres genes para la cadena alfa?
Portador asintomtico.
146
Cmo se le llama al estado en el que existen slo dos genes para cadena alfa?
Rasgo de alfa talasemia.
Por qu la beta talasemia causa hemlisis?
Porque el exceso de cadenas se precipita en el eritrocito y produce su li-
sis.
Qu pasa con el exceso de cadenas beta en la alfa talasemia?
Forman la hemoglobina H tambin conocida como beta-4.
Cmo se llama el procedimiento cuando hay slo un gen de cadena alfa?
Enfermedad de hemoglobina H.
Cul es la diferencia entre la Hb de Bart y la Hb H?
En la primera hay 4 genes afectados de la cadena alfa y en la segunda
tres. En ambas hay hemoglobina gamma-4F. En la primera hay hidrops fe-
talis y muerte in tero. En la segunda, el producto llega a trmino.
Qu hemoglobinas aumentan en la beta-talasemia?
La HbA
2
(alfa2-delta2) y la HbF (alfa2-gamma2).
Qu es la anemia de Cooley?
Es la beta-talasemia mayor. Cursa con hepatoesplenomegalia, anemia
hemoltica severa, hiperplasia medular, hiperostosis facial y absorcin
aumentada de fierro, que causa hemocromatosis.
Cul es el fenotipo de la anemia de Cooley?
Beta-beta. Aparece a los seis meses con hemlisis severa e hiperplasia
medular.
Cul es el tratamiento de las talasemias?
cido flico, transfusiones y transplante de mdula sea.
Cules son los otros nombres de la anemia de clulas falciformes?
Anemia drepanoctica, drepanocitosis hereditaria, enfermedad de la he-
moglobina S.
Cul es el cuadro clnico de la anemia de cdulas falciformes?
El cuadro est dominado por dos puntos principales:
La anemia grave, con ictericia y bilirrubinemia de 2-4 mg/dl, y que
se torna en verdaderas crisis aplsticas tras alguna infeccin viral.
Las lesiones vasooclusivas, a causa de que la membrana eritrocitaria
anormal muestra una adherencia aumentada al endotelio. Esto causa
147
dolor a diferentes niveles, como los huesos largos y de manos y pies
(sndrome mano-pie); puede causar necrosis avascular del carpo y
metacarpo, as como autoesplenectoma. Es causa de trax y abdo-
men agudos. En otros casos pueden ocluirse vasos intracraneales que
causen hemiplejia o una variedad de dficits neurolgicos.
Qu es la hemoglobina S?
Es la sustitucin de valina por cido glutmico en la sexta posicin de la
cadena beta de la globina, que la hace menos soluble en oxgeno.
Cul es el mecanismo fisiopatolgico para desarrollar clulas falciformes?
Al desoxigenarse la hemoglobina, se aglutina y se hace rgida, lo que de-
forma a la clula y la vuelve falciforme (sickle cell).
Hasta cundo est protegido el infante contra la hemoglobina S?
Hasta los seis meses cuando todava tiene hemoglobina F.
Qu medicamento aumenta la hemoglobina F?
La hidroxiurea.
Qu infecciones son ms frecuentes en los enfermos que padecen esta
anemia?
Las infecciones causadas por encapsulados, como H. influenza o Salmonella.
Qu provoca el Parvovirus B19 en los pacientes con anemia de clulas fal-
ciformes?
Puede provocar crisis aplsticas.
Cul es el tratamiento de la anemia de clulas falciformes?
La esplenectoma y los hematnicos carecen de valor. Se aplican transfu-
siones en caso de anemia grave y se utilizan medidas sintomticas.
Cul es el tratamiento profilctico antibitico para pacientes con anemia
de clulas falciformes?
Amoxicilina: 125 mg/da.
Qu es la anemia de Fanconi?
Es la anemia aplstica congnita.
A qu edad se manifiesta clnicamente la anemia de Fanconi?
Entre los 3 y los 8 aos.
Cul es el cuadro clnico de la anemia de Fanconi?
Piel pigmentada, retraso en el crecimiento, anormalidad renal, anormali-
dades seas como la agenesia del pulgar, radio o metacarpianos, hemo-
globina F y macrocitosis.
148
Hasta cundo est presente la hemoglobina F?
Hasta los 9 o 12 meses; generalmente hacia los 6.
Qu factores causan anemia aplstica?
El benceno, el cloramfenicol, las sulfas y la radiacin.
Qu organismos pueden causar supresin de mdula sea?
El virus del dengue, parvovirus humano B19, virus de Epstein-Barr, virus
de la inmunodeficiencia humana y el citomegalovirus.
Cul es la dosis diaria de fierro para tratar la anemia ferropriva?
6 mg/kg da.
Cul es la dosis en la terapia con cido flico?
De 5 a 10 mg/da.
Qu refleja el hallazgo de ferritina srica baja?
Que el hierro medular est bajo.
Qu es la anemia sideroblstica?
Es la anemia por la deficiente utilizacin de Hierro para la formacin de Hb.
Qu factores causan anemia sideroblstica?
Deficiencia de piridoxina (vitamina B-6) como en el alcoholismo.
Intoxicacin por plomo.
Defecto de la delta-aminolevulnicosintetasa.
Isoniacida.
Qu se observa en un frotis de sangre perifrica y de mdula sea el caso
de la anemia sideroblstica?
En la sangre perifrica se observan clulas diana policromatfilas (en
blanco de tiro).
En la mdula sea se observan hiperplasia eritroide y sideroblastos en
anillo. El diamante del anillo es un cmulo de mitocondrias sobre-
cargadas de fierro. Existe eritropoyesis inefectiva.
Cul es el tratamiento de la anemia sideroblstica?
Se trata con transfusiones y eritropoyetina.
Qu porcentaje de las anemias sideroblsticas desarrollan leucemia aguda?
10 %
Cules son la causas de la sobrecarga de Fe en las talasemias?
La absorcin est aumentada.
149
Las transfusiones frecuentes causan hemocromatosis (diabetes bron-
ceada) y afectan el hgado, pncreas, corazn, pulmones y piel.
Cmo se transporta la vitamina B-12 en el plasma?
Por medio de la transcobalamina.
Cmo se administra el tratamiento con vitamina B12?
Una dosis de carga de 1,000 mg y una dosis mensual de 100 mg.
Cul es el sndrome de Diamond Blackfan?
Es la aplasia pura de clulas rojas.
Cul es el aumento de hemoglobina (Hb) y de hematrocito (Hto) al admi-
nistrar un concentrado eritrocitario?
Aumenta 1 g/dl de Hb y 3% de Hto.
Cundo se usan glbulos rojos lavados?
Cuando hay reaccin anafilctica, urticaria no prevenible por antihista-
mnicos y disminucin de IgA.
Cul es el lmite inferior de neutrfilos?
1,500. Se considera neutropenia grave cuando existen menos de 500.
Qu infecciones son frecuentes ante una neutropenia de 500 a 100?
Mucocutneas principalmente.
Para qu se usa la prueba de aglutinacin a la ristocetina?
Cuando las coagulopatas son debidas a deficiencia del factor vW la
prueba de ristocetina resulta baja.
Cul es la neoplasia infantil ms comn?
La leucemia linfoblstica que representa 80% de las leucemias aguda
(LLA).
Cmo se clasifican las leucemias agudas en nios?
Leucemia linfoblstica aguda (LLA) y leucemia mieloblstica aguda
(LMA).
Cmo se clasifican las leucemias crnicas en nios?
Solo existen las no linfocticas.
Cules son las LLA?
La leucemia linfoblstica aguda (de clula B, CALLA positivo), el linfo-
ma de Burkitt y la leucemia de clulas T.
Cules son las leucemias mieloblsticas agudas (LMA)?
La clasificacin comprende 7 tipos celulares, lo que depende de la etapa
de maduracin de la clula mieloide.
150
Cules son las leucemias linfocticas crnicas (LLC)?
La clasificacin comprende cuatro tipos, en los que domina la linfocitosis
de clulas maduras.
Cul es el tratamiento de la LLC?
Clorambucil (alquilante), bajas dosis de radiacin y fludarabina (antime-
tabolito).
Cules son las leucemias mielocticas crnicas (LMC)?
Son leucemias que tienen la fosfatasa alcalina baja, neutrfilos maduros
aumentados y cromosoma Filadelfia (Ph) positivo.
Cul es el cuadro clnico dominante en las leucemias?
Fallo la mdula sea, que afecta todas las lneas celulares.
Fallo del sistema reticuloendotelial.
Dolor seo.
Sitios aislados involucrados como el sistema nervioso central (SNC).
De que clulas se conforman las LLA?
84% de precursores de clulas B, 1% de clulas B, 15 % de clulas T.
Cules son marcadores de buen pronstico en los casos de ALL?
Poliploidia mayor de 56 cromosomas.
Gen TEL-AML1.
Cules son marcadores de mal pronstico en los casos de ALL?
Pseudodiploidia.
Hipoploidia menor de 46 cromosomas.
Cules son marcadores para LLA Pre-B?
El antgeno CALLA positivo y el TdT positivo.
Cules son marcadores para clulas B?
El antgeno CALLA negativo y el TdT negativo.
Cules son las fases de la leucemia mieloctica crnica (LMC)?
Crnica y acelerada.
Cmo acta la prednisona en las leucemias?
Como un citotxico, gracias a su metabolito la prednisolona.
Qu es la enfermedad de Hodgkin?
Es un grupo de cnceres de origen celular incierto, caracterizados por la
presencia de clulas de Reed-Stemberg (clulas gigantes binucleadas), que se
151
divide fundamentalmente en cuatro tipos: de predominio linfoctico, con
esclerosis ganglionar, de celularidad mixta y con deplecin de linfocitos.
Cul es la incidencia de la enfermedad de Hodgkin?
Presenta una incidencia bimodal segn la edad de aparicin; tiene un pi-
co en los adultos jvenes de 10 a 25 aos (predomina la esclerosis gan-
glionar y es ms benigna), y otro en los adultos mayores de 50 aos
(predomina la celularidad mixta y es ms agresiva).
Cul es la clasificacin de Ann Arbor para la enfermedad de Hodgkin?
I: se involucra slo una regin de linfticos (I) o un solo sitio extralin-
ftico (I
E
).
II: se involucran dos regiones de linfticos (II) o extralinfticos (II
E
)
de un solo lado del diafragma.
III: se involucran mltiples regiones de linfticos (III) o extralinfti-
cos (III
E
) (a ambos lados del diafragma o el bazo (III
B
).
IV: hay una enfermedad diseminada linftica o extralinftica.
Cmo se subclasifica la enfermedad de Hodgkin?
Segn los sntomas que presente, se divide en A para los asintomticos y
en B para los sintomticos.
Cules son los sntomas B?
Fiebre de 38C, sudores nocturnos, prdida inexplicada de 10 % de peso.
Cmo se presenta el linfoma de Burkitt?
Como una masa abdominal o masa en el maxilar inferior.
Morfolgicamente cmo se clasifica el linfoma no hodgkiniano?
Linfoblstico: parecido a L1 y L2 (de tipos celulares T, derivados del
timo).
No linfoblstico: parecido a L3 (de tipos celulares B, derivados de la
mdula sea).
Cmo se presenta el linfoma de clulas T?
Como una masa mediastinal anterior y derrame pleural.
Cmo se presenta el linfoma de clulas B?
Como una masa abdominal.
Cul es la forma ms frecuente de presentacin de la enfermedad de Ho-
dgkin?
La adenopata cervical.
152
Cules son los efectos secundarios del tratamiento de la enfermedad de
Hodgkin?
Hipotiroidismo, esterilidad, nausea y vmito.
Cul es el tratamiento del linfoma de Hodgkin diseminado primario
(IIIA, IIIB, IV)?
La quimioterapia combinada: mecloretamina, vincristina, procarbazina y
prednisona (programa MOPP).
Cul es el tratamiento para el linfoma de Hodgkin diseminado recidivante?
La combinacin de adriamicina, bleomicina, vinblastina y decarbazma
(ABVD).
Cul es el tratamiento de la enfermedad de Hodgkin en etapas I y II?
La radioterapia.
Qu son las gammopatas monoclonales?
Son discrasias de clulas plasmticas caracterizadas por la proliferacin
desordenada de un clon de clula plasmtica (encargado de la produccin
de una Ig para un antgeno especfico) y por la presencia en la sangre o en
la orina de una Ig electrofortica o estructuralmente homognea, es decir,
monoclonal.
Cul es el orden en que aparecen las diferentes globulinas en una electro-
foresis?
Se evidencian por picos. El primer pico es el que representa la albmina y
es la fraccin ms grande. Los siguientes son alfa1, alfa2, beta y gamma.
Cules son las gammopatas sintomticas ms frecuentes?
El mieloma mltiple, la macroglobulinemia de Waldestrm, la amiloido-
sis sistmica primaria no hereditaria y la enfermedad de cadenas pesadas.
Cul es el pronstico para cada una de las siguientes enfermedades?
Leucemia linfoblstica aguda: la quimioterapia cura la mayora de los
casos.
Leucemia mieloblstica aguda: la quimioterapia slo cura hasta la
mitad de los casos; 2/3 de los casos curan con transplante de mdula
sea.
Leucemia mieloctica crnica: el adulto sobrevive 2 aos; los ms j-
venes 9 meses en promedio.
153
Linfoma no-Hodgking: la quimioterapia y la radioterapia son curativas.
Enfermedad de Hodgkin: a) la etapa I tiene 90% de curacin; b) la
etapa II: 60 a 70%.
Qu neoplasias dan eritrocitosis?
El hipernefroma, el quiste renal, el hemangioblastoma y el hepatoma.
Cules son causas de eritrocitosis relativa?
El sndrome de Gaisbock (eritrocitosis por estrs) y por disminucin del
volumen plasmtico.
Cules son causas de policitemia con mdula autnoma?
La policitemia vera y la mielofibrosis.
Cules son los criterios mayores para el diagnstico de policitemia vera?
El aumento en la masa eritrocitaria.
La saturacin de oxgeno mayor de 92% (para descartar la hipoxia
como causa de la eritrocitosis).
La esplenomegalia.
Cules son los criterios menores para el diagnstico de policitemia vera?
La leucocitosis.
La trombocitosis.
El aumento de la fosfatasa alcalina leucocitaria.
El aumento de la vitamina B-12 srica.
A qu se le llama agranulocitosis?
A un cuadro constituido por cifras de neutrfilos menores de 500, fiebre,
postracin, faringitis necrtica y formas celulares inmaduras (insuficien-
cia medular).
Cmo se hace el diagnstico de leucemia mieloide crnica?
Con la presencia del cromosoma Filadelfia y niveles bajos de fosfatasa
alcalina.
Qu es la hemoglobinuria paroxstica nocturna?
Es una debilidad de las clulas madre para la lisis por complemento faci-
litada por la seudoacidosis del sueo.
Qu enfermedades hematolgicas se pueden tratar con esplenectoma?
La beta-talasemia mayor de Cooley, cuando las transfusiones son
mayores a 250 ml/k/ao.
154
Los defectos de la membrana eritrocitaria: eliptocitosis, esferocitosis.
Las anemias autoinmunes, por medicamentos, neoplasia o enferme-
dad de la colgena.
Deficiencia de piruvatocinasa mas no en la DG6PD.
La prpura trombucitopnica idioptica.
Cul es el cuadro clnico de la esferocitosis hereditaria?
Malestar, dolor abdominal, ictericia, anemia y esplenomegalia; existen
lceras en los miembros plvicos y clculos biliares. El tratamiento es la
esplenectoma.
Cmo se hace el diagnstico de esferocitosis?
Se deben observar esferocitos, reticulocitos aumentados y pruebas de fra-
gilidad osmtica positivas.
Qu otro nombre reciben los factores K dependientes?
Complejo protrombina.
Cmo acta el factor de VonWillebrand?
Hace un puente entre el colgeno vascular y las plaquetas. Tambin dis-
minuye la prdida de factor VIII.
Cules son las reacciones que inician y terminan la cascada de la coagula-
cin?
La va extrnseca inicia con el factor VII activado; la va intrnseca, con la ac-
tivacin del factor VII. La serie de reacciones que inician en las dos vas
culminan con la activacin del factor II (protrombina) para formar trom-
bina, que acta sobre el fibringeno para formar fibrina.
Cules son los estmulos que activan a las plaquetas?
La disminucin de prostaciclina: en endotelio sano produce PGI
2
que
eleva el nivel intraplaquetario de AMPc (que inhibe la liberacin de
grnulos plaquetarios). Cuando el endotelio se daa, la PGI
2
ya no se
une a los receptores plaquetarios y favorece la activacin.
El colgeno expuesto: al daarse el subendotelio, las plaquetas auto-
mticamente se adhieren al colgeno mediante receptores especficos,
los cuales median la liberacin de grnulos plaquetarios que contie-
nen ADP y serotonina. Estos factores median la activacin de la glu-
coprotena IIa/IIIb (receptores plaquetarios de fibrina); mediante esta
activacin pueden formarse puentes de fibrina entre las plaquetas.
155
La sntesis aumentada de tromboxano: la estimulacin plaquetaria
provocada por trombina, colgeno y ADP provoca que la enzima ci-
clooxigenasa produzca prostaglandina H
2,
que en un segundo paso es
transformada en tromboxano A2, esencial para la formacin rpida
del tapn hemosttico.
Cules son los tres mecanismos anticoagulantes?
La antitrombina III: se fija a todas las proteasas del sistema de coagu-
lacin e inhibe su accin.
La trombomodulina: junto con la trombina forma el complejo trom-
bina-trombomodulina, que activa a la protena C (y a su cofactor, la
protena S), que inactiva a los factores V y VIII.
La plasmina (fibrolisina): la trombina y el activador tisular del plas-
mingeno favorecen la formacin de plasmina, que es el componente
activo del sistema fibrinoltico cuya funcin es lisar a la fibrina y al
fibringeno.
Sobre qu factores acta la va antitrombina-heparansulfato?
IX, X, XI y XII.
Sobre que factores acta la protena CS?
Sobre el V y VII activados.
Cules son los antiagregantes plaquetarios?
La aspirina: acta inhibiendo a la ciclooxigenasa plaquetaria (con la
consecuente falta de tromboxano A
2
) de manera irreversible; por tan-
to, las plaquetas pierden su capacidad de agregacin durante toda su
vida media (7 das).
La ticlopidina y el clopidogrel: inhiben la va del ADP que inhibe la
activacin del receptor de fibrina.
El dipiridamol: es un vasodilatador coronario que se usa combinado
con aspirina. Su accin consiste en inhibir a la fosfodiesterasa plaque-
taria con el consecuente aumento de AMPc plaquetario.
Los inhibidores de la glucoprotena IIa/IIIb: el abciximab, eptifibati-
de y tirofibn conforman este grupo y actan unindose a la gluco-
protena IIa/IIIb y asi evitan la formacin de puentes de fibrina entre
las plaquetas.
156
Cul es el mecanismo de accin de los cumarnicos?
Una vez que ha sido utilizada como cofactor, la vitamina k se transforma
en epxido y requiere de la enzima vitamina k-epxido-reductasa para reuti-
lizarse. La acenocumarina, el dicumarol y la warfarina bloquean esta en-
zima, y afectan, por tanto, a todos los factores K-dependientes.
Cul es el mecanismo de accin de la heparina?
Aumenta la afinidad de la antitrombina III por todas las proteasas que
inhibe y potencia su accin.
Cmo se clasifican los trastornos de la coagulacin?
Trastornos vasculares.
Enfermedades de las plaquetas.
Trastornos hereditarios de la coagulacin.
Trastornos adquiridos de la coagulacin.
Cules son enfermedades hemorrgicas por defecto de los vasos?
Escorbuto, sndrome hemoltico-urmico (SHU), prpura de Henoch-
Schnlein (PHS), prpura senil, prpura por esteroides, hemorragia-
telangiectasia hereditaria.
Cul es el tiempo de tromboplastina parcial (TTP) activado normal?
30-40 segundos.
Cul es el tiempo de protrombina normal?
12-15 segundos.
Cul es la forma de presentacin ms comn de la hemofilia A?
Hemartrosis.
Cul es el tratamiento de la Hemofilia B?
PFC o purificados de complejo protrombina que contienen factores II,
VII, IX y X.
Qu enfermedades aumentan el TTP sin historia de sangrado?
Deficiencia de factor XII, deficiencia de prekalicrena y deficiencia de ci-
ngeno de alto peso molecular.
En qu sndrome se encuentra tiempo de sangrado prolongado con todo lo
dems normal?
En el sndrome de Glanzman.
157
Cmo se presenta la enfermedad de VonWillebrand (EVW)?
Con estigmas de deficiencia plaquetaria (sangrado mucocutneo) y es-
tigmas de deficiencia de factores (hemartrosis), pues afecta la funcin de
las plaquetas y del factor VIII.
Cmo se presentan las pruebas de laboratorio en la EVW?
TTP prolongado: lo que indica anormalidad de la va extrnseca.
Tiempo de sangrado aumentado: porque no se forma el puente pla-
queta-colgeno.
Prueba de coagulacin con ristocetina alterada.
Cundo se indica el PFC?
Cuando el tiempo de protrombina es mayor de 15 el tiempo de trombo-
plastina parcial es mayor de 40, en casos de sobredosis de anticoagulante
oral y en caso de hemofilia B.
Qu contienen los crioprecipitados?
Fibringeno, factor V, factor VIII y factor XIII.
Cundo se indican los crioprecipitados?
Cuando el fibringeno es menor de 100 g/dl
Cul es la dosis de los crioprecipitados?
7 a 10 mg/kg de peso.
Qu porcin del factor VIII se encuentra en los concentrados?
Porcin VIII procoagulante.
Qu porciones del factor VIII se encuentran en el crioprecipitado?
VIII procoagulante y VIII antignico.
Cul es el tratamiento de la enfermedad deVon Willebrand (EVW)?
Crioprecipitados con ambas partes del factor VIII.
Qu medicamento aumenta el factor VIII?
La desmopresina (DDAVP).
En qu leucemia se utiliza rutinariamente la heparina?
En la promieloctica.
Cmo se monitorean las heparinas de bajo peso molecular?
Con el factor X activado.
Cul es la actividad y vida media del concentrado de factor VIII?
1 U/K aumenta 2% y dura 12 horas.
158
Cul es la actividad y vida media del concentrado de factor IX?
1 U/k aumenta 1% y dura 24 horas.
Cules son los estados de hipercoagulabilidad?
Se encuentran en este estado los pacientes con medicacin anticonceptiva
oral, cncer, deficiencia de las protenas C, S y antitrombina III, y en los
que hay presencia del factor V de Leiden.
Cmo acta la heparina?
Potenciando la accin de la antitrombina III.
Cmo acta la warfarina?
Alterando la sntesis de factores K-dependientes.
Cmo se evala la actividad de la warfarina?
Con el INR y el T.P.
Cul es la dosis de carga de la estreptocinasa?
250,000 U IV durante 30 minutos.
Qu es la enfermedad de Rendu-Osler-Weber?
Es la ataxia-telangiectasia hereditaria hemorrgica relacionada con los
genes HHT 1 y 2.
Cul es el mnimo de plaquetas viable en el preoperatorio?
50,000.
Cules son causas de trombocitopenia?
La disminucin de la produccin: vitamina B-12 disminuida, infiltra-
cin medular, congnita.
El secuestro: hiperesplenismo.
La destruccin acelerada: uremia, drogas (heparina), prpura trom-
btica trombocitopnica y prpura trombtica idioptica.
Por qu la uremia causa disfuncin plaquetaria y cul es su tratamiento?
Porque altera los complejos de factor VIII; se corrige con hemodilisis y
crioprecipitados.
Qu es la prpura trombtica idioptica (PTI)?
Es una trombocitopenia autoinmune en la que aparece IgG antiplaqueta-
ria que provoca la destruccin de las plaquetas en el bazo, con una forma
aguda (nio) y una crnica (adulto), sin esplenomegalia.
Qu es la prpura trombtica trombocitopnica (PTT)?
159
Es una anemia hemoltica microangioptica de causa autoinmune, rpi-
damente progresiva y fatal; se acompaa de sntomas neurolgicos y su
lesin patognomnica son los depsitos hialinos en los vasos.
Qu es el sndrome hemoltico-urmico (SHU)?
La PTT y el SHU no son dos entidades distintas; ms bien son una pro-
gresin de la misma enfermedad. Mientras en la PTT se destacan los da-
tos neurolgicos, en el SHU se agrega insuficiencia renal. Por tanto, en el
SHU se encuentra anemia hemoltica microangioptica, trombocitopenia
e insuficiencia renal y pruebas de coagulacin normales.
Cul es el tratamiento de la PTI?
Se usan prednisona o gamma-globulina slo cuando las plaquetas son
menores de 20,000.
Cules son los tratamientos para la PTT y la PTI?
PTT: en la forma complicada se utiliza metilprednisolona o predni-
sona ms gamma-globulina. En la forma no complicada, solamente
esteroides.
PTI: esplenectoma.
Cundo ocurre la PTI aguda?
De los 2 a los 6 aos, secundaria a infeccin viral.
Qu es el sndrome de Evans?
Es la anemia hemoltica autoinmune que acompaa a la PTI.
Cmo se presenta la prpura anafilactoide?
La prpura de Henoch-Schnlein se presenta como un exantema morbili-
forme que afecta la mitad inferior del cuerpo, acompaado de poliartral-
gias y edema que afecta tobillos, rodillas, caderas y codos. Hay afeccin
renal en 50% de los casos, que causa hematuria y proteinuria.
Cul es el tratamiento de la prpura anafilactoide?
Esteroides e inmunosupresores (azatioprina y ciclofosfamida).
Qu se debe pensar ante la presencia de anemia, trombocitopenia y prue-
bas de coagulacin alteradas?
La posibilidad de coagulacin intravascular diseminada (CID).
Cundo est indicada la reposicin de factores en la CID?
Cuando el fibringeno es menor de 75 mg/dl se inicia el uso de criopre-
cipitados.
160
Qu se debe pensar ante la presencia de anemia, trombocitopenia y prue-
bas de coagulacin normales?
Se debe hacer el diagnstico diferencial entre PTT y SHU.
Cundo se utiliza la transfusin de plaquetas y cunto las aumenta?
Cuando la cuenta es de 5,000 a 10,000 y aumentan 10,000 por concen-
trado (en un hombre de 70 kg).
Cmo se calcula el aumento plaquetario esperado?
Incremento = (# de concentrados X 5,000)/ (peso/10)
A qu se le llama refractariedad a plaquetas?
A un aumento de las plaquetas de slo 4,000 o 5,000 despus de 1 hora
de administrar el concentrado plaquetario.
Qu se utiliza en el paciente refractario a plaquetas?
Inmunosupresin, gamma-globulina anti-D e IgG intravenosa.
Cules son los sndromes mieloproliferativos?
Policitemia vera, eritrocitosis secundaria, mielofibrosis, trombocitemia
primaria.
Cules son el primero y segundo pasos en el estudio de la eritrocitosis
primaria?
Buscar signos de hipovolemia para descartar eritrocitosis esprea.
Saturacin Hb mayor de 92% para descartar que sea secundaria a hi-
poxia.
Cul es la presentacin clnica del hiperesplenismo?
Anemia, leucopenia, trombcitopenia, esplenomegalia y reticulocitosis.
Cul es el tratamiento del hiperesplenismo primario?
Quirrgico; los esteroides no sirven.
Qu enfermedad provoca autoesplenectoma?
La anemia de clulas falciformes.
Cul es el mecanismo principal del hiperesplenismo secundario?
La hipertensin portal; la causa que ocupa el segundo lugar es la trombo-
sis de la vena esplnica.
Cmo se manifiesta la incompatibilidad de grupo ABO?
Coagulacin intravascular diseminada, necrosis tubular aguda y muerte.
161
Cules son los tipos de reacciones transfusionales?
Hemoltica o por incompatibilidad A, B, O, Rh
No hemoltica: se presenta con una reaccin febril causada por anti-
cuerpos del receptor contra leucocitos del donador.
Cul es el tratamiento de la reaccin alrgica transfusional?
Difenhidramina (25 mg. I.V.), epinefrina (0.5 ml. I.V)., hidrocortisona
(100 mg. I.V.).

162
163
INFECTOLOGA








Cul es el tratamiento del herpes zoster?
Aciclovir: 800 mg 5 veces al da durante cinco das.
Cul es el diagnstico diferencial de la faringitis por estreptococo beta-
hemoltico del grupo A?
Mononucleosis infecciosa, faringitis viral gorronea, infeccin por Myco-
plasma pneumoniae o por Clamydia pneumoniae.
Hasta que temperatura se considera fiebre de origen viral?
De 40.6 C a 41.1 C.
A partir de qu temperatura se considera fiebre de origen bacteriano?
A partir de 41.1 C.
Cul es la conjuntivitis viral ms frecuente y cul su tratamiento?
Es la fiebre faringoconjuntival por adenovirus tipo 3, y se aplican gotas
de sulfas y compresas para prevenir una complicacin bacteriana.
Qu enfermedades infecciosas provocan sntomas de resfriado y exantema?
El sarampin, el sndrome retroviral y la mononucleosis infecciosa.
Qu es la enfermedad de Paget?
Es una enfermedad de los huesos producida por paramixovirus (moquillo
canino), que causa destruccin y reparacin excesiva con deformidad re-
sultante.
Cul es el diagnstico diferencial de la enfermedad de Paget?
El mieloma mltiple, las metstasis seas y la ostetis fibrosa qustica.
Qu es la enfermedad de Kawasaki?
Es una vasculitis de etiologa incierta, aunque presumiblemente infeccio-
sa, que afecta a pacientes menores de 4 aos. Se diagnostica con la pre-
sencia del criterio principal y 4 de 5 adicionales.
164
La presencia de fiebre por 5 das es el criterio principal. Los siguientes 5
criterios son:
Sndrome extrapiramidal.
Cambios en las mucosas; incluye xerostoma y lengua en fresa.
Eritema, edema o descamacin de la piel de las extremidades.
Exantema cutneo.
Linfadenopata cervical.
Cules son las principales complicaciones de la enfermedad de Kawasaki?
Trombocitosis y aneurismas cardiacos.
Cules son los ocho tipos principales de herpesvirus?
Virus simple tipo 1 (HSV-1), HSV-2, virus de la varicela-zoster (tipo 3),
virus de Epstein-Barr (VEB), virus de la mononucleosis infecciosa (tipo
4), citomegalovirus (CMV, tipo 5), virus de la rosola (exantema sbito
HHV-6), el virus relacionado a la rosola (HHV-7) y el relacionado al
sarcoma de Kaposi (HHV-8).
Cmo se presenta la mononulceosis infecciosa?
Con linfocitosis atpica, retinitis y prueba monospot positiva.
Qu signos neurolgicos y no neurolgicos aparecen en la mononucleosis
infecciosa?
Parlisis de Bell, meningitis asptica, encefalitis, neumonitis, miocarditis,
faringitis, fiebre, linfadenopata, esplenomegalia y exantema.
Qu alteraciones de laboratorio se presentan en la mononucleosis infec-
ciosa?
En 50% de los casos aparece monocitosis y en 10%, linfocitosis atpica y
transaminasas elevadas.
Cul es el diagnstico diferencial de la mononucleosis infecciosa?
Faringitis exudativas, infeccin por citomegalovirus (CMV), rubola y
toxoplasmosis.
Cmo se presenta la infeccin por CMV?
Con linfocitosis atpica y alteracin de las pruebas hepticas y, clnica-
mente, con linfadenopata generalizada, fiebre y exantema.
Cul es el estndar de oro para hacer el diagnstico de infeccin por
CMV?
El cultivo viral.
165
Cul es el cuidado que deben seguir los contactos de pacientes con varicela?
La varicela es transmisible desde 48 horas antes de que aparezca el exan-
tema y deben vacunarse todos los contactos inmunodeprimidos.
Cul es el antiviral de eleccin para todas las infecciones comunes por
herpes?
El aciclovir.
Cules son los antivirales anlogos de la guanosina?
El aciclovir, famciclovir y ganciclovir. Actan compitiendo con la desoxi-
guanosinatrifosfatasa viral (dGTP) como sustrato para la DNA-polimerasa,
lo que causa una terminacin prematura de la cadena de DNA viral.
Cundo est indicado utilizar ganciclovir?
Aunque tiene el mismo espectro de accin que el aciclovir, slo est
aprobado en la terapia de la retinitis citomeglica.
Cul es la principal indicacin del famciclovir?
El herpes zoster agudo.
Qu es el foscarnet y cundo est indicado?
Es un fosfonoformato derivado del pirofosfato (no es un anlogo de puri-
nas o pirimidinas), que a pesar de su amplia actividad antiviral in vitro,
slo est aprobado para el tratamiento de las infecciones virales resisten-
tes al aciclovir y al ganciclovir que se presentan en pacientes infectados
con HIV.
Cules son los principales antivirales utilizados en las infecciones respira-
torias?
La amantadina y la rimantadina estn indicadas como agentes profi-
lcticos en las infecciones por virus de la influenza A y pueden ser
utilizadas junto con la vacuna. Estn indicadas en pacientes no vacu-
nados y en contactos o personas con riesgo elevado de infeccin.
La rivabirina es un anlogo de la guanosina, que tiene un amplio es-
pectro antiviral contra virus RNA y DNA, pero que se utiliza princi-
palmente en infecciones por el virus sincicial respiratorio, influenza A
o B y la fiebre de Lassa.
Cul es la conducta a seguir para contactos de enfermos de hepatitis A?
Todos los contactos domiciliarios deben ser vacunados; el periodo de in-
cubacin es de 25 a 30 das.
166
Cules son las pruebas serolgicas para infeccin por el virus de la hepati-
tis B?
HBsAg: el antgeno de superficie es el primer marcador serolgico
que se hace positivo y tambin el ms confiable; puede aparecer tan
temprano como los primeros 14 das de infeccin, aunque normal-
mente el promedio es de 27 a 41 das. Se positiviza de 7 a 26 das
despus de la aparicin de las anormalidades bioqumicas.
Anti-HBsAg: la presencia del anticuerpo contra el antgeno de superficie
en ausencia de HBsAg indica recuperacin, ausencia de infectividad
e inmunidad para futuras infecciones por el virus de la hepatitis B.
HBeAg: el antgeno e indica un estado de replicacin viral en que el
paciente es altamente infeccioso.
Anti-HBeAg: El anticuerpo contra HBeAg aparece cuando el ant-
geno desaparece y es un marcador de buen pronstico y resolucin de
la infeccin aguda.
Anti-HBcore: La fraccin total de este anticuerpo se positiviza en la in-
feccin aguda y persiste durante toda la vida. La fraccin IgM es la nica
altamente especfica; est presente en el periodo en que los antgenos e y
el de superficie han desaparecido, pero todava no han aparecido los anti-
cuerpos (periodo de ventana del core).
Qu es el interfern?
Es una familia de glicoprotenas inducidas naturalmente, que interfiere
con la habilidad de los virus para infectar clulas. Existen tres tipos sinte-
tizados por tecnologa de DNA recombinante: alfa, beta y gamma. Ac-
tualmente se utiliza el interfern alfa-2b para el tratamiento de la
hepatitis B y C, as como para tratar la leucemia de clulas peludas y el
sarcoma de Kaposi.
Qu infecciones causan neutropenia?
Influenza A o B, hepatitis por virus sincicial respirtatorio (VSR), rubola,
varicela , por virus de Epstein-Barr (VEB), salmonella, brucella, tulare-
mia.
Qu microorganismos afectan frecuentemente al paciente neutropnico?
Aspergillus, Cndida, E. coli y Pseudomonas.
167
Cules son las manifestaciones neuromusculares en la infeccin por HIV?
Es ms comn la meningitis por toxoplasma que por criptococo.
Cuando su aparicin es muy temprana, la etiologa es el propio HIV.
Ocurre polirradiculitis tipo Guillain-Barr secundaria a infeccin por
HIV y citomegalovirus (CMV).
La miopata puede ser por HIV (proximal) y por el uso de AZT (so-
bre todo si hay afeccin renal).
Cul es la manifestacin endocrina ms frecuente en la infeccin por
HIV?
La insuficiencia suprarrenal.
Cul es el estndar de oro para hacer el diagnstico de infeccin por HIV?
El cultivo viral.
Qu tipo de anlisis son el ELISA y Western blot?
Son anlisis cualitativos para detectar anticuerpos contra el HIV. El ELI-
SA es til para hacer la deteccin (debe ser positivo en dos muestras dife-
rentes) y el Western blot se utiliza para hacer la confirmacin.
Qu prueba se usa para confirmar el Western blot, segn el protocolo pa-
ra diagnstico de infeccin por HIV?
Un Western blot que informa un resultado dudoso debe confirmarse por
reaccin de cadena de la polimerasa (PCR).
Qu pruebas se utilizan para hacer diagnstico en el producto de una ma-
dre infectada por el HIV?
ELISA a las 18 semanas de vida extrauterina, la deteccin del antgeno
P-24 por PCR y cultivo directo a partir de linfocitos.
Cules son las cuatro categoras definitorias de inmunodeficiencia (cate-
gora C)?
Infeccin bacteriana grave recurrente.
Infeccin por oportunistas.
Sndrome de desgaste
Enfermedad neurolgica progresiva.
Qu profilaxis con antibitico se usan en los pacientes con SIDA, segn
sus determinaciones de subpoblaciones de linfocitos CD
4
?
Menos de 300CD
4
: profilaxis contra Criptococcus y Cndida con flu-
conazol.
168
Menos de 200CD
4
: dar profilaxis contra Pneumocystis carinii con tri-
metoprima-sulfametoxazol o dapsona (100 mg durante tres semanas).
75-100 CD
4
: dar profilaxis contra Mycobacterium avium-intracellulare
con claritromicina 500 mg y rifabutina 300 mg (opcional).
Menos de 50 CD
4
: dar profilaxis contra Citomegalovirus con ganci-
clovir.
Cul es el agente ms comn del absceso cerebral en pacientes con SIDA?
Toxoplasma.
Cul es el agente ms comn de la retinitis en enfermos de SIDA?
Citomegalovirus.
Cul es el agente ms comn de la leucoplaquia peluda oral en casos de
SIDA?
El virus de Epstein-Barr.
Cul es el agente ms comn de la meningitis en pacientes con SIDA?
Criptococcus neoforman.
Qu se usa para el tratamiento del herpes en pacientes infectados por el VIH?
Aciclovir seguido por foscarnet.
A qu se asocia la infeccin por citomegalovirus?
A la aparicin de sarcoma de Kaposi (se observa como ndulos cutneos
o farngeos).
Cul es la imagen radiolgica de la neumona por Pneumocystis carinii?
La imagen es la de un infiltrado en alas de mariposa.
Qu frmacos se usan para tratar la infeccin por Pneumocystis carinii?
Sulfametoxazol-trimetoprima, dapsona y pentamidina en aerosol.
Cules son los dos tipos de antivirales utilizados en casos de infeccin por
VIH?
Los anlogos de nuclesidos y los inhibidores de proteasa.
Cules antivirales son anlogos de nuclesidos?
La zidovudina (AZT), la didanosina (ddI), la zalcitabina (ddC), la es-
tavudina (d4T) y la lamivudina (3TC).
Cules son los antivirales inhibidores de la proteasa?
Los antivirales saquinavir, ritonavir, indinavir, nelfinavir y amprenavir
son inhibidores de la proteasa aprobados, que suelen usarse en conjunto
con la combinacin zidovudina-lamivudina.
169
Cundo est indicado iniciar la terapia con AZT?
Cuando las cuentas virales son mayores de 5,000 copias o la cuenta de
CD4 es menor a 500 o existe alguna enfermedad definitoria de SIDA.
Cules son las dosis de AZT por grupo de edad?
Nios: 180 mg/m
2
/6horas.
Adulto: 1,500 mg/24horas para SIDA y 500 mg/24horas cuando es
portados de VIH con CD4 menores de 500.
Los principales efectos secundarios son la neutropenia en 48% de los ca-
sos y la toxicidad hematolgica en 26%.
Cul es el segundo frmaco aprobado para el tratamiento antiviral en la
infeccin por HIV?
La didanosina (ddI) es el antiviral recomendado en las infecciones resis-
tentes a AZT. Su principal efecto adverso es la pancreatitis.
Cul es la penetracin al sistema nerviosos central de los principales anti-
virales?
El AZT tiene una penetracin de 70% y la ddI de 25%.
Cules son los frmacos de primera lnea para el tratamiento de la infec-
cin por el VIH?
Los anlogos nuclesidos. El primero, el AZT.
Cules son dos frmacos anlogos no nuclesidos?
La nevirapina y el delavirdine.
Qu antivirales causan neuropata?
La vincristina, zalcitabina y didanosina.
Cundo est indicado el uso de la zalcitabina?
En terapia de combinacin con AZT o como monoterapia en pacientes
que no toleran el AZT.
Cules son los principales efectos txicos de los anlogos nuclesidos?
AZT: toxicidad medular, leucopenia y anemia.
Didanosina y zalcitabina: pancreatitis.
Estavudina: neuropata perifrica.
Cules son las principales bacterias encapsuladas?
H. influenzae e meningococo, neumococo y Salmonella.
A qu grupo de edad afectan ms los encapsulados?
A los pacientes comprendidos entre los 3 meses y los 2 aos.
170
Cules son las principales enfermedades causadas por cocos gram-
positivos?
Infecciones estafiloccicas, estreptoccicas y neumoccicas.
Qu pacientes son susceptibles de infecciones estafiloccicas?
Los usuarios de drogas IV, los diabticos y los que presentan infecciones
de tejidos superficiales.
Qu enfermedades son causadas por estafilococos?
La foliculitis, los abscesos, los furnculos y las infecciones de tejidos
blandos (ntrax estafiloccico).
Cul es la principal etiologa de la osteomielitis?
S. aureus es el causante de 60% de las infecciones.
Cules son los sndromes provocados por toxinas bacterianas?
El sndrome de choque txico, el sndrome de piel escaldada y el sndro-
me de intoxicacin alimentaria.
Qu es el sndrome de choque txico?
Es un sndrome provocado por la toxina TSST-1 producida por estafilo-
cocos; es ms frecuente en adultos, de aparicin sbita, con fiebre alta,
vmito, diarrea, exantema, conjuntivitis y descamacin de las palmas y
las plantas. Pueden ocurrir hipotensin e insuficiencia renal aguda.
Cul es el diagnstico diferencial de la intoxicacin alimentaria?
La intoxicacin alimentaria causada por estafilococos inicia en el transcur-
so de las primeras 6 horas (siempre en menos de 12 horas), y tpicamente
se observa cuando los alimentos se han dejado enfriar al aire libre o con-
tienen productos como crema o mayonesa. La intoxicacin por Bacillus
creus tiene un inicio mas lento y generalmente se debe a la ingesta de
arroz frito contaminado. Los sntomas de intoxicacin por Clostridium
aparecen entre 8 y 16 horas despus de la ingesta y el dato principal es la
diarrea acuosa.
Qu es el gonococo?
Es un diplococo gram-negativo, reniforme, intracelular, oxidasa-positivo
como todas las Neisserias.
Cules son las principales infecciones causadas por bacilos gram-positivos?
El erisipeloide, la listeriosis la nocardiosis y el carbunco.
171
Qu es la nocardiosis?
La nocardiosis o micetoma es una enfermedad provocada por actinomi-
cetos aerobios, que se puede presentar con 4 variedades clnicas: celulitis,
sndrome linfocutneo, actinomicetoma y neumona.
Cul es el tratamiento del micetoma?
Diaminodifenilsulfona y sulfametoxazol-trimetoprima (SMZ- TMP) du-
rante dos aos.
Cules son las principales enfermedades causadas por bacilos gram-nega-
tivos?
Las infecciones causadas por enterobacterias, por Haemophilus, por
Pseudomonas, la salmonelosis, la shigelosis, la brucelosis, el clera, la tu-
laremia y la peste.
Cules son los vectores de la tularemia y la peste?
La garrapata y la pulga, respectivamente.
Cul es el tratamiento emprico para cualquier tipo de diarrea?
Los fluoroquinolonas.
Cul es la fisiopatologa de la diarrea en la infeccin por vibrio cholerae y
ETEC?
Ambos producen toxinas que inactivan el GMPc en la pared intestinal.
Cules son los agentes causales de gastroenteritis infecciosa con diarrea
no inflamatoria?
Infeccin por Clostridium, Escherichia coli enterotoxignica, estafiloco-
cos e infestacin por Giardia lamblia.
Cules son los agentes causales de gastroenteritis con diarrea de tipo in-
flamatorio?
Shigella, Salmonella, Campylobacter y E. coli enteroinvasiva.
Cundo se debe sospechar que el paciente padece brucelosis?
Cuando hay fiebre de bajo grado, diaforesis, agotamiento, cefalea, estre-
imiento, artralgias y, en algunos casos, adenopatas y esplenomegalia. Al
interrogatorio se encuentra inicio insidioso y antecedente de haber consu-
mido carne posiblemente infectada o leche y queso no pasteurizados.
Cul es el tratamiento de la brucelosis?
Se trata con un doble esquema que puede ser doxiciclina y rifampicina o
sulfametoxazol-trimetoprima (TMP-SMZ) y rifampicina.
172
Cules son los hallazgos de laboratorio en pacientes que padecen brucelosis?
Las reacciones de aglutinacin se vuelven positivas despus de dos a tres
semanas de ocurrida la infeccin. Un ttulo mayor de 1:160 es positivo.
El anlisis inmunolgico de IgG o IgM especficas es ahora el mtodo de
eleccin.
Cundo se debe sospechar de fiebre tifoidea?
Cuando se encuentran sntomas correspondientes a una gastroenteritis
que inicialmente presenta diarrea (en sopa de chcharos), que posterior-
mente desarrolla estreimiento. Al mismo tiempo hay ataque al estado
general y sntomas de faringitis. Puede aparecer el exantema clsico en el
trax y abdomen. La meseta de todos estos sntomas coincide con la me-
seta de la fiebre. El hallazgo patognomnico de la enfermedad es la bra-
dicardia relativa.
Qu es la bradicardia relativa?
Normalmente, por cada grado que aumenta la temperatura por arriba de
38.3, el pulso se eleva 10 latidos por minuto. Cuando no ocurre una ta-
quicardia proporcional a la fiebre, se habla de bradicardia relativa.
Cul es el tratamiento de la fiebre tifoidea?
El de primera eleccin es el (TMP-SMZ) seguido por ceftriaxona y cipro-
floxacino.
Cules son las alteraciones de las pruebas de laboratorio en la salmonelosis?
El diagnstico se basa en un hemocultivo positivo en los primeros 10 das
de inicio de la fiebre. Existen leucopenia y anemia normoctica. La reac-
cin de Widal no es confiable porque se positiviza con la vacunacin o con
la infeccin por cualquier especie de Salmonella y se negativiza si se usa
previamente cloramfenicol o ampicilina.
Cundo se debe sospechar shigelosis?
Cuando el cuadro clnico es de fiebre, tenesmo y dolor abdominal; ade-
ms se encuentra evidencia de sndrome disentrico (leucocitos y eritroci-
tos en las heces). Frecuentemente se complica con un cuadro de
deficiencia de disacaridasas transitoria y sndrome de Reiter.
En qu medio se cultivan la Salmonella y la Shigella?
En el medio SS.
173
Cules son las principales enfermedades causadas por bacilos anaerobios?
Las infecciones por clostridios, por bacteroides y la actinomicosis.
Qu es la actinomicosis?
Es una enfermedad indolente, lentamente progresiva, causada por gr-
menes gram-positivos anaerobios del gnero Actinomices. En la lesin
producida por la actinomicosis se observan grnulos sulfurosos que tras-
pasan todos los planos tisulares hasta drenar al exterior.
Cmo es la lesin del actinomicetoma?
Es una lesin supurativa, granulomatosa, filante, que forma tractos fistu-
losos, y al microscopio se observa infiltracin extensa de neutrfilos.
Cules son las enfermedades provocadas por clostridios?
Los clostridios son bacilos anaerobios gram-positivos. Las principales in-
fecciones causadas por clostridios son la mionecrosis por clostridios
(gangrena gaseosa), el ttanos y el botulismo.
Qu es el ttanos?
Es la enfermedad causada por la toxina que produce el Clostridium Tetani
(tetanospasmina). Es secundaria a una herida profunda contaminada. Se
presenta con rigidez del cuello, hormigueo en el sitio de la inoculacin y
disfagia. Posteriormente existe trismus, irritabilidad, hiperreflexia, opist-
tono y convulsiones.
Cul es el manejo de las heridas para hacer la prevencin del ttanos?
En caso de una herida menor limpia debe aplicarse la primera dosis de
toxoide tetnico (esquema de tres dosis). En caso de una herida profunda
y contaminada con material que probablemente contenga anaerobios,
deben aplicarse toxoide e inmunoglobulina antitetnica.
Cmo actan los antibiticos bacteriostticos?
Inhiben la replicacin y el crecimiento bacterianos y favorecen la elimi-
nacin de los organismos remanentes por el sistema inmune; por tanto, si
la administracin del frmaco se hace antes de que el organismo elimine
todos los grmenes, puede ocasionarse un segundo ciclo de infeccin.
Cules son los mecanismos de resistencia microbiana a los antibiticos?
Mutacin del DNA.
Transferencia de DNA por plsmidos (factor R extracromosmico).
174
Modificacin de receptores especficos del frmaco en el microorga-
nismo.
Inactivacin enzimtica del frmaco.
Qu antibiticos inhiben la sntesis de folato?
Las sulfas: sulfadiazina de plata, sulfisoxazol, sulfametoxazol, sulfasala-
zina, etctera.
Qu medicamentos inhiben la reduccin de folato?
La pirimetamina y el trimetoprim.
Qu medicamento inhibe la sntesis y la reduccin de folato?
El cotrimoxazol.
Por qu son bacteriostticas las sulfas?
Las coenzimas derivadas del cido flico se requieren para la sntesis de
cidos nucleicos y, por tanto, para la replicacin celular. Las bacterias
son impermeables al folato y deben sintetizarlo a partir de cido para-
aminobenzaico (PABA), pteridina y glutamato.
Cules son los efectos adversos de las sulfas?
Cristaluria, kerncterus, Stevens-Johnson, reaccin leucemoide, granuloci-
topenia, poliarterisis nodosa, vasculitis y otras.
Qu antibiticos actan como inhibidores de la sntesis de la pared celular
bacteriana?
Los betalactmicos, la vancomicina y la bacitracina.
Cmo se clasifican los betalactmicos?
En penicilinas, cefalosporinas, carbapenems y monobactams.
Cul es el sitio de accin de los betalactmicos?
Actan inhibiendo el ltimo paso de la sntesis de la pared celular; es de-
cir, en la transpeptidacin del peptidoglicano.
Cules son las penicilinas antipseudomonas?
La ticarcilina, carbenicilina, azlociclina, mezlociclina y piperacilina.
Cul es el espectro antibacteriano de las cefalosporinas de primera gene-
racin?
Actan sobre todos los gram-positivos meticilinorresistentes, Proteus, E.
Coli y Klebsiella.
Cul es el espectro de las cefalosporinas de segunda generacin?
Tienen la misma cobertura que las de primera generacin y un espectro
extendido a gram-negativos, Haemophilus infuenzae y Enterobacter.
175
Cul es el espectro de las cefalosporinas de tercera generacin?
Su mayor efecto es sobre bacilos gram-negativos, H. infuenzae y entero-
bactar as como Serratia marcesens. La cefotaxima y la ceftriaxona son de
eleccin en casos de meningitis y la ceftazidima, en infeccin por Pseu-
domonas.
Cul es el espectro de las cefalosporinas de cuarta generacin?
Actan sobre estafilococos y estreptococos meticilinosensibles y gram-
negativos como Enterobacter, E. Coli, Klebsiella, Proteus y Pseudomonas.
Cules son las cefalosporinas que pueden utilizarse por va oral?
Primera generacin: cefalexina, cefadroxil y cefradina.
Segunda generacin: cefuroxima acetil y cefaclor.
Tercera generacin: ceftibuten y cefixima.
Cul es el betalactmico con mayor espectro antibacteriano?
La imipenem-cilastatina. Puede ser de eleccin como terapia emprica en
infecciones que amenazan la vida y cuya etiologa no est bien definida.
Cules son las indicaciones para el uso de vancomicina?
Infeccin grave por gram-positivos betalactamasa positivos.
Infeccin grave por gram-positivos en los pacientes alrgicos a los be-
talactmicos.
Infeccin por Clostridium o estafilococo, que amenaza la vida.
Profilaxis en procedimientos dentales.
Como antibitico en pacientes con implante de vlvulas cardiacas.
Cules son los efectos adversos de los betalactmicos?
Reacciones de hipersensiblidad y disfuncin plaquetaria.
Qu antibiticos actan inhibiendo la sntesis de protenas?
Las tetraciclinas, los aminoglucsidos, los macrlidos, el cloramfenicol y
la clindamicina.
Qu antibiticos actan sobre la subunidad 30s?
Las tetraciclinas y los aminoglucsidos.
Qu antibiticos actan sobre la subunidad 50s?
El cloramfenicol y los macrlidos.
Cmo actan las fluoroquinolonas?
Actan inhibiendo la replicacin del DNA bacteriano e interfiriendo en
la accin de la DNA-girasa (topoisomerasa II).
176
Qu antibiticos se consideran antispticos de las vas urinarias?
La metenamina y la nitrofurantona.
Cul es el efecto adverso ms importante del cloramfenicol?
La supresin de la mdula sea.
Qu medicamentos causan disfuncin renal?
Los aminoglucsidos y la anfotericina B.
Cules son las principales enfermedades infecciosas transmitidas por vec-
tores?
Piojo: tifo epidmico (Rickettsia prowazecki).
Pulga: tifo epidmico y peste.
caro: Rickettsia pustulosa y tifo de los matorrales.
Mosquito: paludismo, fiebre amarilla, encefalitis de San Luis, encefa-
litis equina del oeste y encefalitis de California.
Garrapata: enfermedad de Lyme, fiebre manchada de las Montaas
Rocallosas, tularemia.
Triatoma: enfermedad de Chagas.
Mosca: tripanosomiasis africana (enfermedad del sueo)
Cul es el mtodo de deteccin de tuberculosis por tincin?
Auramina-rodamina.
Cul es la tincin confirmatoria para tuberculosis?
Kynyoun o Ziehl-Nielsen.
Qu es un complejo de Gohn?
Es un foco primario de infeccin tuberculosa calcificado.
Qu es un complejo de Ranke?
Es un foco tuberculoso primario calcificado unido a un ganglio parahiliar
calcificado.
Cunto se inyecta de PPD y cunto tiene de tuberculina para efectuar la
prueba de Mantoux?
Se inyectan 0.1 ml, que equivalen a U de tuberculina.
Qu es un convertidor reciente?
Una persona menor de 35 aos, con una ganancia de 10 mm en la
prueba de Mantoux en los ltimos dos aos.
Una persona mayor de 35 aos, con una ganancia de 15 mm en la
prueba Mantoux en los ltimos dos aos.
177
Cules son los cuatro grupos que deben recibir profilaxis contra la tu-
berculosis?
Cuando se sospecha o se conoce infeccin por HIV.
Todos los contactos de personas con TB.
Todos los convertidores recientes.
Cualquiera que padezca una enfermedad inmunosupresora como la
diabetes mellitus o tumores bajo terapia con frmacos inmunosupre-
sores.
Cmo se lleva a cabo la profilaxis de la tuberculosis?
Con Isoniacida durante 9 meses.
Cmo acta la isoniacida?
Inactiva la enzima responsable de ensamblar los cidos miclicos de la pa-
red externa del microorganismo. Es activa contra los microorganismos
intracelulares y contra los que se encuentran en el cseum. Su concentra-
cin en el lquido cefalorraqudeo es prcticamente igual a la plasmtica.
Cmo acta la rifampicina?
Bloquea la trascripcin interfiriendo con la subunidad-beta de la RNA-
polimerasa dependiente del DNA; por tanto, inhibe la sntesis de RNA y
suprime el paso inicial. Es bactericida de microorganismos intra y extra-
celulares. Frecuentemente se usa en la profilaxis de contactos intradomi-
ciliarios de pacientes con meningitis meningoccica o por H. influenzae.
Se distribuye uniformemente en todos los rganos y fluidos, incluyendo
el lquido cefalorraquideo.
Cules son los efectos adversos de la rifampicina?
Tie de color anaranjado la orina, las heces y otras secreciones. Puede
causar ictericia en pacientes con enfermedad heptica, alcohlicos y con
edad avanzada.
Cul es el efecto adverso ms importante del etambutol?
Causa neuritis ptica que produce disminucin de la agudeza visual y
prdida de la habilidad para discriminar entre los colores verde y rojo.
Cules son las principales enfermedades causadas por espiroquetas?
Las treponematosis, la fiebre recurrente, la enfermedad de Lyme y la en-
fermedad por araazo de gato.
178
Cules son los falsos positivos para VDRL?
Paludismo, lepra, embarazo, edad avanzada, mononucleosis infecciosa,
hepatitis C, enfermedades del tejido conectivo y la fiebre.
Cundo se encuentran ttulos altos de VDRL?
En la sfilis secundaria; pueden ser mayores de 1:32.
Cundo se hace positivo el VDRL en caso de sfilis?
A las seis semanas.
Cmo son los ttulos de VDRL en la sfilis tarda?
Muy bajos o incluso negativos.
Cul es el tratamiento antibitico para la sfilis?
Penicilina sdica cristalina o benzatnica.
Doxiciclina 100 mg cada 12 horas o tetraciclina (500 mg c/6hr VO
durante 14 das).
Qu enfermedades infecciosas se tratan con doxiciclina?
Las provocadas por Chlamydia, Brucella y la sfilis.
Cules son las caractersticas de las diferentes etapas de la sfilis?
Sfilis primaria: la lesin bsica es el chancro; hay linfadenopata re-
gional.
Sfilis secundaria: hay lesiones en la piel, mucosas, huesos, hgado,
SNC y linfadenopata generalizada.
Sfilis terciaria: la lesin bsica son los gomas en los huesos, hgado y
en las vsceras. Tambin existe enfermedad cardiovascular caracteri-
zada por aortitis y neurosfilis.
Cules son los tipos clnicos de la neurosfilis?
Neurosfilis sintomtica (5%) y neurosfilis asintomtica (15%).
Meningovascular: puede ser de la corteza (signos menngeos, convul-
siones) o basal (parlisis de nervios craneales).
Parenquimatosa: causa demencia paraltica secundaria a la sfilis.
Tabes dorsal.
Qu es la tabes dorsal?
Es una forma de degeneracin progresiva del parnquima de las colum-
nas posteriores de los ganglios sensitivos y de las races nerviosas. El pri-
mer signo y ms comn es un dolor fulgurante en las piernas seguido por
179
inestabilidad en la marcha, ataxia, alteraciones sensoriales y prdida de
los reflejos tendinosos.
Qu es la pupila de Argyl-Robertson?
Es una pupila irregular y pequea, que reacciona de modo normal a la
acomodacin pero no a la luz.
Qu agente produce el mal del pinto?
El Treponema carateum.
Qu agente causa la fiebre recurrente?
La Borrelia recurrentis.
Qu pruebas de laboratorio se positivizan en la fiebre recurrente?
La reaccin de Weil-Felix y el VDRL.
Qu etapas clnicas tiene la leptospirosis?
Septicmica, inmunitaria e ictrica.
Cules son las etapas de la enfermedad de Lyme?
I: resfriado y eritema migratorio.
II: enfermedad generalizada y enfermedad neurolgica en la que ocu-
rre parlisis de Bell (meningitis asptica).
III: artritis y molestias musculoesquelticas.
Cules son las principales enfermedades parasitarias?
Las protozoosis y las helmintiasis.
Cul es la lesin caracterstica de la amibiasis intestinal?
La lcera amibiana.
Cules son las formas clnicas de la amibiasis intestinal?
Asintomtica, colitis disentrica y no disentrica.
Cmo se toman las muestras de heces para hacer el diagnstico de ami-
biasis?
Se toman tres muestras, que deben examinarse dentro de los primeros
treinta minutos despus de la toma.
Cul es el diagnstico diferencial del absceso heptico amibiano?
Lesiones por equinococo, cncer y absceso pigeno.
Cmo se observa el ultrasonido del absceso heptico?
Como lesiones redondas no homogneas, de transicin abrupta y centro
hipoecico.
180
Cmo se ve la tomografa axial computarizada del absceso heptico?
Como lesiones de baja densidad, redondas, bien definidas y de estructura
no homognea.
Cules son los amebicidas tisulares?
La emetina y la dehidroemetina
Cmo inicia el ciclo de la leishmaniasis?
Con la reproduccin de amastigotes en los macrfagos de algn mamfe-
ro, llamados cuerpos de Donovan.
Qu se observa en el frotis de un enfermo de leishmaniasis?
Amastigotes flagelados intracelulares.
Qu se observa en el cultivo del tejido afectado?
El parsito en fase de promastigote flagelado.
Cul es el tratamiento de la leishmaniasis?
Estibogluconato de sodio o pentamidina ms anfotericina B.
Para qu sirve la pirimetamina?
Para el tratamiento del paludismo y la toxoplasmosis.
Para qu sirve la pentamidina?
Para tratar la enfermedad del sueo, y para la profilaxis y el tratamiento
de la neumona por Pneumocystis carinii.
Cmo se presenta la toxoplasmosis?
Clnicamente, con fiebre, linfadenopata cervical, esplenomegalia, exan-
tema y retinocoroiditis.
Cmo se hace el diagnstico de toxoplasmosis?
Obteniendo ttulos cuatro veces mayores de los normales de IgM, o con
un ttulo simple alto de 1:60.
Cul es el tratamiento antimicrobiano de la toxoplasmosis?
Pirimetamina (acompaada de cido folnico) ms sulfadiazina.
Qu es la enfermedad de Chagas?
Es la tripanosomiasis americana, causada por el Trypanosoma cruzi. Suele
observarse ms en nios pequeos y se presenta con fiebre, linfadenopa-
tas, hepatoesplenomegalia y edema facial, y puede haber meningoence-
falitis y convulsiones. La forma crnica tiene pocos sntomas y causa
cardiomiopata, megacolon y megaesfago.
181
Cul es el vector de la enfermedad de Chagas?
La chinche picuda o Triatoma reduvidae.
Cmo se hace el diagnstico de enfermedad de Chagas?
Se puede lograr la observacin del parsito dentro de la capa leucoci-
taria en un centrifugado de sangre.
Se puede observar el parsito en una tincin de Giemsa de capas
gruesas de sangre.
Para la etapa crnica slo sirven el cultivo (Nicolle-Novi-McNeal) y
el xenodiagnstico.
Cul es el tratamiento de la etapa crnica de la enfermedad de Chagas?
Se utiliza nifurtimox como antiparasitario y amiodarona para la insufi-
ciencia cardiaca congestivo-venosa, ya que los pacientes no toleran bien
la digoxina.
Cul es la enfermedad del sueo y quin la provoca?
Es la tripanosomiasis africana, provocada por Trypanosoma brucei gam-
biense y rodeniense.
Cules son las etapas de la enfermedad del sueo?
Etapa hemolinftica: fiebre, linfadenopata y exantema papuloso.
Etapa meningoenceflica: insomnio, trastornos motores y sensitivos.
Cul es el tratamiento de la enfermedad del sueo?
La suramina y la eflornitina. La profilaxis se lleva a cabo con pentamidina.
Cules son las variedades clnicas de la cisticercosis?
La neurocisticercosis, la subcutnea, la muscular y la oftalmocisticerco-
sis.
Cules son las variedades de la neurocisticercosis?
Ventricular, parenquimatosa, subaracnoidea, racimososa y de la mdula
espinal.
Cules son los mtodos de eleccin para hacer el diagnstico de laborato-
rio de esquistosomiasis?
Kato Katz y la bsqueda del parsito en orina de 24 horas.
Qu es la larva migrans cutnea?
Es la uncinaria del perro y del gato, Ancylostoma brasiliense y Ancylos-
toma caninum.
182
Cul es el tratamiento de la esquistosomiasis y de la paragonimiasis?
El prazicuantel.
Cul es el tratamiento de la fasciolosis?
El triclabendazol.
Cul es el tratamiento de la uncinariasis, la ascariasis y la enterobiosis?
El albendazol.
Cmo se hace el diagnstico de enterobiosis?
Con el mtodo de Graham.
Cmo se hace el diagnstico de filariasis en el laboratorio?
Observando microfilarias en sangre perifrica tomada con periodicidad
nocturna. El frote de gota gruesa se utiliza para la deteccin y los frotes
delgados se utilizan para evaluar la morfologa; se tien con Giemsa.
Tambin se utiliza el mtodo de concentracin de sangre de Knott o la
filtracin de membrana.
Cul es el tratamiento de la filariasis?
La dietilcarbamacina.
Cul puede ser la prdida diaria de hemoglobina en la infestacin por un-
cinarias?
1 mg al da en la infestacin por Necator americanus
3 mg al da en la infestacin por Ancylostoma duodenale
Cul es el cuadro clnico de la oncocercosis?
Ndulos subcutneos, deterioro visual y ceguera.
Cmo se hace el diagnstico de estrongiloidosis?
Observando larvas filariformes en las heces o en el lquido duodenal (por
aspirado o cpsula de Beal).
Cul es el tratamiento de la estrongiloidosis y la oncocercosis?
La ivermectina.
En qu se debe pensar si se observa un granuloma eosinoflico en el ojo?
En la toxocariosis.
Cul es el mejor medio de cultivo para los hongos?
El de Sabouraud.
Cul es el mecanismo de accin de los siguientes antimicticos?
Ketoconazol: inhibe la biosntesis de ergosterol y bloquea la desmeti-
lacin del sitio C14.
183
Anfotericina B y nistatina: alteran la permeabilidad de la membrana
al formar complejos con ergosterol.
Griseofulvina: acta contra la formacin de microtbulos.
Flucitocina: acta sobre el DNA y el RNA.
Cundo est indicado el uso de los siguientes antimicticos?
Griseofulvina: en todas las micosis, excepto la tinea capitis y la oni-
comicosis. El itraconazol y el fluconazol han reemplazado amplia-
mente a la griseofulvina para el tratamiento de las dermatofitosis
sistmicas.
Ketoconazol: en la infeccin por Malassezia furfur, que causa la tia
versicolor.
Fluconazol: en todas las micosis mucocutneas como la candidiosis.

184
185
INMUNOLOGA Y REUMATOLOGA








Qu elementos conforman la inmunidad innata-inespecfica?
Componente celular: neutrfilos, monocitos (en la sangre) y macr-
fagos (en los tejidos).
Componente humoral: protenas del complemento, reactantes de fase
aguda y las diferentes citocinas.
Qu elementos conforman la inmunidad adquirida-especfica?
Componente celular: linfocitos.
Componente humoral: inmunoglobulinas.
Cules son los reactantes de fase aguda y cmo se interpretan?
A excepcin de la protena Creactiva (CRP), los reactantes de fase aguda
no se usan para detectar inflamacin. Slo hay que tener en cuenta que
sta puede ser la causa de que reactantes se eleven cuando se estn estu-
diando por otra razn; por ejemplo, si se desea demostrar la presencia de
la enfermedad de Wilson con niveles de ceruloplasmina, habr que des-
cartar que no exista un estado inflamatorio previo que provoque un falso
positivo.
CRP: se eleva hasta 100% cuando existe dao tisular severo.
Fibringeno. Se eleva 200-400%.
Alfa-1-antitrispsina: se eleva 200-400%.
Haptoglobina: se eleva 200-400%.
Ferritina: se eleva 50%.
Ceruloplasmina.
Glicoprotena alfa-1-cida.
186
Qu causan las deficiencias de complemento C5, C6, C6, C7 y C8?
Propician infecciones rpidas producidas por Neisserias.
Qu causa el dficit de C3?
Infecciones por gram-negativos entricos.
Cmo se reconoce lo extrao de lo propio?
Gracias a los productos del complejo principal de histocompatibilidad
(MHC), que son un conjunto de genes que codifican los antgenos leuco-
citarios humanos y que se encuentran codificados en el cromosoma 6.
Qu son las citocinas?
Son protenas solubles diferentes de las inmunoglobulinas, secretadas por
monocitos y linfocitos y que regulan la magnitud de la respuesta inflama-
toria e inmunitaria.
Qu son los eicosanoides?
Son cidos insaturados, de 20 carbonos, precursores de la sntesis de las
prostaglandinas.
Qu son y como actan las prostaglandinas?
Son mediadores locales de la inflamacin. Son producidas y eliminadas
en el mismo sitio de accin, por lo que no hay concentraciones significa-
tivas en la sangre.
Qu citocinas median la inmunidad natural?
Interleucina (IL)-1, IL-6, IL-8 y el interferon-gamma (IFN-gamma).
Qu citocinas median la inflamacin alrgica?
IL4: estimula directamente a la IgE.
IL3, 4 y 9: se encuentran en las clulas cebadas.
IL3 y 5: actan sobre el factor estimulante de colonias granulocito-
monocito (GM-CSF), que estimula la produccin de eosinfilos.
Cmo se conforman las inmunoglobulinas o anticuerpos?
Tienen dos cadenas pesadas y dos ligeras, cada una de las cuales tiene re-
giones constantes y regiones variables.
Cmo reconocen los antgenos a los linfocitos B y a los linfocitos T?
Los linfocitos B tienen inmunoglobulinas de superficie y el linfocitos T
(RCT) tiene receptor de clula T.
Cmo se conforman los receptores de clulas T?
Por dos cadenas (alfa-beta o gamma-beta) y la molcula CD-3. Se conoce
entonces tambin como complejo RCT-CD3.
187
Qu respuestas median las clulas TH1 y TH2?
La respuesta celular patgeno-especfica y la hipersensibilidad por inter-
leucina 4, respectivamente
Qu significa CD?
Grupo de diferenciacin (Cluster of differentiation).
Qu es un anticuerpo completo?
Es el que puede aglutinar eritrocitos y desencadenar el complemento a
partir de C9.
Cules son anticuerpos completos e incompletos.
GI es un anticuerpo completo; la IgM es un anticuerpo incompleto.
Qu anticuerpo se detecta en la prueba de Coombs?
La inmunoglobulina G (IgG).
Qu son los antgenos leucocitarios humanos A, B y C (HLA-A,B y C)?
Los genes del MHC-I forman 3 loci que son el HLA A, B y C. Se encuen-
tran en la superficie de todas las clulas nucleadas y en las plaquetas.
Qu son las HLA D, DR y DQ?
Los genes del MHC-II se distribuyen en varias subregiones: HLA D, DR
y DQ. Slo se encuentran en la superficie de las clulas B, los macrfa-
gos, las clulas dendrticas, las clulas de Langerhans y las clulas T acti-
vadas.
Cmo elimina un macrfago a un micoorganismo extra e intracelular?
Al extracelular, lo fagocita. Para eliminar al intracelular necesita ingerirlo
y despus ser activado para destruirlo. (Esto explica la hipersensibilidad
retardada).
Cules son las caractersticas de las inmunoglobulinas?
IgM: es la primera que se produce tras la inmunizacin primaria; se
encuentra en forma monomrica o pentamrica y slo en la sangre.
Puede tener cadena J. Su peso molecular: 950-1150 KDa. Son anti-
cuerpos contra muchos grmenes gram-negativos. Las grandes mol-
culas pueden activar el complemento.
IgG: es la que aparece principalmente tras la reactivacin inmunitaria
(memoria); generalmente es monomrica y se encuentra en el plasma
y extravascular. Protege principalmente frente a bacterias, virus y to-
188
xinas. Su peso molecular: 150 KDa. Es la nica que atraviesa la ba-
rrera placentaria.
IgA: la clase secretora se encuentra en las mucosas, el tracto gastro-
intestinal y respiratorio y en sus productos (saliva, lgrimas, calostro,
etc.). Se le identifica en forma de monmero o dmero. Puede tener
cadena J y Sc. Peso molucular: 160-400 KDa. Protege contra Bruce-
lla, Corynebacterium dipheriae y poliovirus.
IgD: se utiliza como marcador para clulas B maduras. Se encuentra
en forma monomrica. Pero molucular: 175 KDa.
IgE: es un anticuerpo reagnico, estabilizador y anafilctico. Se en-
cuentra en los tractos respiratorio y gastrointestinal. Se encuentra
aumentada en suero en las enfermedades atpicas y parasitarias y en
el mieloma monoclonal de IgE. Pero molecular: 190 KDa.
Cmo se evalan las inmunoglobulinas?
La IgG, IgA e IgM se encuentran en grandes cantidades en el suero.
Se pueden medir por inmunoelectroforesis.
La IgE se encuentra en cantidades mnimas y debe usarse un estudio
sensible como el RAST.
Cules son las reacciones de hipersensibilidad segn la clasificacin de
Gel y Coombs?
Tipo I: un antgeno (Ag) se fija a un anticuerpo (Ac) de tipo IgE y
mediante su fraccin FC ste se fija a los mastocitos y basfilos, libe-
rando factores vasoactivos e inflamatorios. Incluye las reacciones
atpicas y la anafilaxia sistmica.
Tipo II: es la reaccin citotxica cuando el anticuerpo se fija a una
clula o a haptenos que se encuentran en su superficie. Ocurren en las
anemias hemolticas Coombs-positivas, en la prpura trombocitop-
nica idioptica, el pnfigo, sndrome de Goodpasture.
Tipo III: es la reaccin que involucra complejos inmunes Ac-Ag que se
depositan en vasos y tejidos y activan as el complemento y la liberacin
de enzimas proteolticas lisosomales y factores de permeabilidad. Ocurre
en la enfermedad del suero, poliarteritis, glomerulonefirtis aguda y en
la membranoproliferativa crnica, etctera.
189
Tipo IV: involucra la presencia de linfocitos T sensibilizados que se
activan al contacto con el Ag especfico (hipersensibilidad retardada).
Se presentan en los granulomas, la dermatitis por contacto, la neu-
momitis por hipersensibilidad, etc.
Cmo se clasifican las reacciones a drogas?
En esperadas e inesperadas.
Cmo se clasifican las reacciones inesperadas?
En reacciones de intolerancia, idiosincracia e hipersensibilidad.
Qu es una reaccin farmacogentica?
Es una variacin de la respuesta farmacolgica producida por factores
hereditarios. Por tanto, se puede considerar como causa de reacciones de
idiosincracia.
Cules son las reacciones farmacogenticas ms comunes?
La reduccin de la actividad de la warfarina causada por la disminucin de
la afinidad de fijacin de la warfarina a su receptor y la hipertermia maligna
que se produce al administrar combinadamente un relajante muscular y un
anestsico inhalado que actan sobre los canales de rianodina y provocan
una liberacin exagerada de calcio que causa contracciones extremas, una
tasa metablica elevada y finalmente, hipertermia.
Cul es la diferencia entre una reaccin de Arthus y una prueba de hiper-
sensibilidad retardada?
Ambas son secundarias a la administracin intradrmica de un Ag. La reac-
cin de Arthus es el resultado de la unin del Ag con un Ac que ya estaba en
el sitio, lo que provoca una respuesta a las 4-6 horas con infiltrado inflamato-
rio consistente en neutrfilos. La prueba de hipersensiblidad retardada debe
interpretarse hasta las 48 horas y es positiva cuando es menor de 5mm. El in-
filtrado que se genera consiste en clulas mononucleares.
Cules son las manifestaciones de la reaccin de hipersensibilidad tipo III
por complejos inmunes?
Fiebre, examina, linfadenopata y artralgias.
Cules son las manifestaciones del sndrome de vasculitis por hipersensi-
bilidad?
Artritis, urticaria, angioedema y fiebre.
190
Cul es la diferencia entre la anafilaxia y la urticaria-angioedema?
Ambas se asocian a la deficiencia de inhibidor del complemento C1. La
anafilaxia es una variedad mas grave de la reaccin tipo I, en la que
adems de haber angioedema existe colapso circulatorio.
Cules son los tratamientos de primera y segunda lnea en los casos de
anafilaxia?
La epinefrina y la reanimacin con lquidos.
Cul es el tratamiento en los casos de urticaria-angioedema?
Los antihistamnicos.
Cules son los tres usos principales de los antihistamnicos?
El tratamiento de reacciones alrgicas, de las nuseas y el mareo y como
somnferos.
Cules son los signos clnicos de la rinitis alrgica?
La presencia de plipos, epistaxis, mucosa plida y ojeras.
Cules son los sntomas de la enfermedad del suero?
Urticaria, angioedema, fiebre, artritis y eritema multiforme. Todas pue-
den aparecer entre una y dos semanas despus de la exposicin al ant-
geno. Las formas graves presentan vasculitis (neuropata y nefrosis).
Cmo se clasifican las inmunodeficiencias primarias?
Deficiencia de clulas B (anticuerpos).
Deficiencia de clulas T (predominante de clulas T a combinada de
clulas T y B).
Trastornos fagocticos (de la actividad microbicida y del movimiento
celular).
Trastornos del complemento (de sus componentes y de sus protenas
reguladoras).
Qu tipo de inmunodeficiencia causa el alcoholismo?
Disfuncin de los neutrfilos.
Qu es el sndrome de Chdiak-Higashi?
Es un defecto de la actividad microbicida, autosmico recesivo, con albi-
nismo parcial, tendencia a la malignidad linforreticular, infeccin piognica
severa, fotofobia, linfadenopata, hepatoesplenomegalia e inclusiones
granulares gigantes en los neutrfilos.
191
Qu microorganismos se consideran menos virulentos y son encontrados
como patgenos en los defectos de la fagocitosis?
S. aureus, Pseudomonas, Haemophilus y Aspergillus.
Cules son los microorganismos extracelulares o encapsulados encontra-
dos como patgenos en la deficiencia de clulas B?
S. pneumoniae, S. pigenes, H. influenzae.
Qu es la enfermedad de DiGiorge?
Es la aplasia tmica, que se acompaa con deficiencia de clulas T, hipo-
calcemia, faxcies caracterstica, anomalas del arco artico y enfermedad
cardiaca.
Qu es la enfermedad de Wiskott-Aldrich?
Es la deficiencia de clulas T y B, que produce eczema y trombocitopenia.
Qu es el sndrome de ataxia-telangiectasia?
Es la deficiencia de clulas T y B, asociadas con dermatitis y deterioro
neurolgico.
Cmo se saca la cuenta absoluta de neutrfilos?
Multiplicando el nmero de leucocitos totales por el porcentaje de neu-
trfilos.
Cmo se saca la cuenta de CD4?
Multiplicando el nmero de leucocitos totales por el porcentaje y por el
porcentaje de linfocitos por el porcentaje de linfocitos teidos para CD4.
Qu infecciones provocan linfocitosis?
La infeccin por Bordetella pertusis, la mononucleosis infecciosa y la hepa-
titis.
Qu infecciones provocan neutropenia?
La hepatitis, la fiebre tifoidea y la influenza.
Qu frmacos provocan neutropenia?
Las fenotiacinas, la fluoxetina y los antitiroideos.
En qu enfermedades se encuentran granulomas eosinoflicos?
En la enfermedad de Hand-Schller-Christian y la histiocitosis X.
Por qu el piroxicam y la indometacina son ms dainos a la mucosa gs-
trica?
Porque son COX-1 selectivos.
192
En donde se encuentran los receptores COX-2?
En las clulas inflamatorias.
Qu es la fiebre reumtica?
Es una enfermedad inflamatoria multisistmica en la que los anticuerpos
producidos contra las cepas M3 y M18 del estreptococo beta-hemoltico
del grupo A (EBHGA) atacan a tejidos del propio organismo que tienen
epitopos similares. Clnicamente se presenta entre 1 y 3 semanas despus
de la infeccin bacteriana.
Cules son los criterios de Jones para hacer el diagnstico del ataque ini-
cial de la fiebre reumtica?
Criterios mayores:
Poliartritis: aparece hasta en 75% de los pacientes y afecta las grandes
articulaciones; es migratoria, dolorosa y se acompaa de fiebre.
Carditis: ocurre en 50% de los casos; se evidencia por un soplo pan-
sistlico mitral.
Corea: aparece en 10 a 30% de los pacientes y dura entre 1 y 3 sema-
nas.
Eritema marginado.
Ndulos subcutneos.
Criterios menores:
Clnicos: fiebre y artralgias.
Laboratorio: reactantes de fase aguda (velocidad de sedimentacin
globular y protena C reactiva) elevados.
Intervalo P-R prolongado.
El diagnstico se hace al encontrar: 2 criterios mayores o 1 mayor y 2
menores y, adems, ttulos elevados de anticuerpos anti-EBHGA o un
cultivo positivo para ste.
Cules son los sntomas ms frecuentes de la fiebre reumtica?
Dos que aparecen principalmente en los adultos (eritema marginado y
carditis) y dos que aparecen principalmente en nios (ndulos subcut-
neos y corea de Sydenham).
Qu son los ndulos de Aschoff?
Son grandes clulas multinucleadas que tienen cmulos de colgeno en
su interior. Se les encuentra en el miocardio en caso de fiebre reumtica.
193
Cul es el tratamiento de la fiebre reumtica?
Salicilatos: para la artritis y la fiebre.
Esteroides: si la carditis es grave.
Clorpromazina o haloperidol: para la corea de Sydenham.
Qu es la artritis reumatoide?
Es una enfermedad crnica inflamatoria, con afectacin articular y sis-
tmica, con factor reumatoide positivo y asociada a HLA DR4.
Qu es el pannus?
Es la lesin inflamatoria deformante de la sinovial que ocurre en los pa-
cientes que padecen artritis reumatoide.
Cul es la localizacin de los ndulos reumatoides?
Yuxtaarticular.
Cules son las manifestaciones clnicas de la artritis reumatoide (AR)?
Sinovitis: afecta las articulaciones grandes de manera simtrica y que
respeta las interfalngicas distales (a diferencia de la osteoartritis),
con afectacin de tejidos sinoviales periarticulares (tendones y liga-
mentos) que causa sndrome del tnel del carpo, inestabilidad C1-C2
y tendinitis del manguito rotador (hombro).
Manifestaciones extraarticulares principales: ndulos reumatoides,
queratoconjuntivitis sicca, afeccin del pulmn y del corazn, vascu-
litis, etctera.
Cules son los criterios para hacer el diagnstico de AR?
Artritis de tipo inflamatorio, afeccin poliarticular afeccin simtrica, que
involucra las manos; factor reumatoide positivo y cambios radiolgicos su-
gerentes. El diagnstico se realiza con la presencia de 4 criterios de siete.
Cul es el tratamiento de eleccin de la AR?
El metotrexate.
Qu es el etanercept?
Es un receptor artificial del factor de necrosis tumoral alfa.
Qu es el infliximab?
Es un anticuerpo quimrico contra los receptores del factor de necrosis
tumoral alfa.
Qu frmacos son antiinflamatorios de vida media larga?
La indometacina, el sulindaco y el piroxicam.
194
Cmo actan el acetaminofn y la fenacetina?
Disminuye la ciclooxigenasa nicamente en el SNC; por tanto, tienen
efecto analgsico y antipirtico, pero no antiinflamatorio.
Cules son los efectos adversos de las sales de oro?
La dermatitis, los cambios en las mucosas, el sndrome nefrtico y la
agranulocitosis.
Para qu se usa la D-penicilamina?
Se usa como antiinflamatorio, como quelante en la intoxicacin por me-
tales pesados y para el tratamiento de la cistinuria.
Cules son los efectos adversos de la D-penicilamina?
Son principalmente dermatolgicos; en ocasiones hay nefritis y anemia
aplstica.
Cules son los efectos adversos del metotrexate?
lceras orales, nusea, citopenias, cirrosis y sndrome de neumona aguda.
Qu es artritis reumatoide juvenil y cuales son sus variedades?
Es la que inicia entre 1 y 4 aos de edad, ms comn en nias, y sus va-
riedades son: poliarticular, pauciarticular y sistmica.
Qu es la enfermedad de Still?
Es la artritis reumatoide juvenil de inicio sistmico, con examema morbi-
liforme, fiebre y hepatoesplenomegalia. No es lo habitual que se presente
con artrirtis de grandes articulaciones.
Cmo se presenta la artritis reumatoide juvenil (ARJ) pauciarticular?
Sin fiebre, con iridociclitis y anticuerpos antincleo y positivo; esqueleto
axial respetado.
Cmo es la ARJ poliarticular?
Hay cinco o ms articulaciones de gran tamao y simtricas afectadas,
anticuerpos antincleo y anticuerpos anti-DNA positivos.
Cmo est el factor reumatoide en la ARJ y cul es el tratamiento?
El factor reumatoide es negativo y el tratamiento se lleva a cabo con ci-
do acetilsaliclico con ASA.
Cules son los once criterios para hacer el diagnstico de lupus eritemato-
so sistmico (LES)?
Eritema malar.
Eritema discoide.
195
Fotosensibilidad.
lceras orales.
Artritis.
Serositis.
Hematolgicos: linfopenia (menor de 1,500), leucopenia (menor de
4,000), trombocitopenia (menor de 100,000).
Renales: cilindros, menor de 5g de protenas en orina de 24 horas
(3+).
Sistema nervioso central: convulsiones.
Inmunolgicos: clulas LE, VDRL positivo, anticuerpos Anti-DNA;
anticuerpos Anti Sm, protena P-ribosomal (marcador para demen-
cia).
Anticuerpos antincleo positivo: aparece en todas las enfermedades
del tejido conectivo (95% en LES).
Cules son los usos que tiene la determinacin de anticuerpos Anti-DNA?
Es altamente especfica para el diagnstico de lupus y es un marcador
para medir su actividad.
Sirve para diferenciar entre LES y lupus eritematoso discoide (LED).
Es una prueba positiva junto con la determinacin de anticuerpos an-
tincleo en caso de artritis reumatoide juvenil poliarticular.
Cules son las pruebas que sugieren la presencia de anticuerpos antifosfo-
lpido?
El anticoagulante del lupus.
IgG o IgM anticardiolipina.
VDRL falso positivo.
Tiempo de protrombina (TP) y tiempo de tromboplatina parcial
(TTP) prolongados, que no corrigen con una mezcla 1:1 de plasma.
Pruebas de coagulacin especiales como: antibeta-2 glicoprotena,
veneno viperino de Russel, TTP diluido activado y tiempo de san-
grado con caoln.
Cmo se diferencian por laboratorio el LES y LED?
En el ltimo, los anticuerpos anti-DNA son casi siempre negativos.
196
En qu enfermedades aparecen los anticuerpos SSA (Rho) y SSB (L-alfa)?
En el lupus y el sndrome de Sjgren.
En el lupus cutneo y la fotosensibilidad por lupus (SSA).
En el lupus neonatal y el bloqueo cardiaco congnito.
Con qu enfermedades se relacionan los HLA DR2 y DR3?
Se asocian a LES y sndrome de Sjgren sin artritis, respectivamente.
Cmo se trata la nefritis por lupus?
Con altas dosis de esteroides en bolo (pulsos) y citotxicos (azatioprina y
ciclofosfamida).
Qu es la artropata de Jacoud?
Es la deformidad articular provocada por el lupus eritematoso sistmico.
Para qu sirven los antiinflamatorios no esteroideos en el tratamiento del
LES?
Para calmar la fiebre, el dolor articular y la serositis.
Para qu sirven los antimalricos en los casos de LES?
Para tratar la enfermedad renal y el lupus refractario.
Cules son las variantes clnicas de la esclerosis sistmica progresiva (es-
cleroderma)?
Hay una variante limitada, una difusa y la enfermedad mixta del tejido
conectivo.
Cules son los anticuerpos que se relacionan con la variante difusa de la
esclerisis sistmica progresiva?
Los Scl-70 y los anticuerpos antinucleolar.
Cules anticuerpos se relacionan con la variante limitada de la esclerosis
sistmica progresiva?
Los anticuerpos anticentrmeros.
Cules anticuerpos se relacionan con la enfermedad mixta del tejido co-
nectivo?
Los Anti U
1
RNP.
Cul es el diagnstico diferencial de la escleroderma?
La escleroderma localizada (morfea o linear), la fascitis eosinoflica y el
sndrome de eosinofilia-mialgia.
197
Cul es la lesin anatomopatolgica que se encuentra en casos de esclero-
sis sistmica progresiva?
Hay un dao en el endotelio de los vasos por hiperplasia de la ntima.
Cules son los anticuerpos antifosfolpido?
Anticoagulante lpico (aumenta el TTP).
Anticuerpos anticardiolipina.
Se acompaan de pruebas positivas falsas para sfilis (VDRL).
Cul es la diferencia entre polimialgia y polimiositis?
Las dos se presentan con dolor muscular, pero la primera no causa debi-
lidad muscular proximal.
Cules son los criterios para hacer el diagnstico de polimiositis?
Fraccin MM de la creatincinasa elevada, debilidad muscular proximal,
biopsia que demuestre la existencia de lesin inflamatoria del msculo y
la electromiografa anormal.
Cules son los signos clsicos de la dermatomiositis?
Edema periorbitario, sufusin heliotrpica (color violceo de los prpa-
dos), signo de Gottron (manchas escamosas sobre las articulaciones inter-
falngicas proximales y metacarpofalngicas), signo de la V del cuello
y signo del chal.
Qu se encuentra en una biopsia de un paciente con polimiositis?
Un infiltrado dominado por linfocitos T CD8.
Qu se encuentra en la biopsia de un paciente con dermatomiositis?
Infiltrado muscular en el que dominan los leucocitos T CD4 y linfocitos
B; es una vasculitis de pequeos vasos.
Qu enzimas se miden en los casos de dermatomiositis?
Fraccin MM de la creatincinasa la aldolasa y mioglobina.
Cules son los anticuerpos que se pueden encontrar en los pacientes con
dermatomiositis?
Especficos: anticuerpos antisintetasa (jo-1), anticuerpos de recono-
cimiento de partculas (Mi-2).
No especficos: anticuerpos antincleo de patrn especular (anti-
Hep2), anti-Rho, anti-la, PM-SCL, KU.
198
Cules son las seis formas clnicas de la polidermatomiositis?
Polimiositis asociada a malignidad, con cuerpos de inclusin, dermato-
miositis, juvenil y mezclada con sndrome de Reiter, sndrome de
Sjgren, LES o AR.
Cules son las caractersticas de la polimiositis con cuerpos de inclusin?
Debilidad distal y proximal, ausencia de autoanticuerpos y cambios va-
cuolares (cuerpos de inclusin).
Cules son las manifestaciones electromiogrficas de la polimiositis?
Potenciales de alta frecuencia, polifsicos y fibrilacin.
Cules enfermedades pueden provocar descamacin periarticular?
La artritis gotosa, la artritis psorisica y la dermatomiositis.
Cules son los cinco hallazgos radiogrficos clsicos de la osteoartritis?
Formacin de quistes subcondrales, formacin de hueso nuevo (osteofi-
tos), esclerosis sea, estrechamiento del espacio articular y carencia de
osteoporosis.
Qu son las espondiloartropatas seronegativas?
Son artritis inflamatorias diferentes de la artritis reumatoide (AR). Las
principales son: espondilitis anquilosante, artritis reactiva (sndrome de
Reiter) y artritis psorisica. El factor reumatoide es tpicamente negativo.
Cules son las manifestaciones clnicas de las espondiloartropatas?
Afectacin del esqueleto axial: sacroilitis. El esqueleto axial es respe-
tado en la artritis reumatoide (con excepcin del cervical).
Entesis: inflamacin del sitio de implantacin de los tendones y liga-
mentos al hueso.
Manifestaciones extraesquelticas: la uvetis y conjuntivitis son co-
munes (a diferencia de la escleritis y episcleritis que acompaan a la
artritis reumatoide). Tambin aparecen uretritis, colon irritable y
examema psoriasiforme.
Asociacin a HLA-B27 y ausencia de factor reumatoide (FR) positivo.
Qu es la artritis reactiva?
Tambin conocida como sndrome de Reiter, es una espondiloartropata
que cursa con uretritis, lceras mucocutneas y artritis.
199
Qu microorganismo se asocian al sndrome de Reiter?
Chlamydia y Shigella.
Cules son las manifestaciones clnicas de la artritis psorisica?
Las ms significativas son la sacroilitis asimtrica y la onicolisis as como
la afectacin de las articulaciones interfalngicas distales, sin ndulos
reumatoides. Tiende a remisiones ms completas, frecuentes y rpidas
que la AR. El FR es negativo.
Cmo se presenta la espondilitis anquilosante?
En varones jvenes con dolor de espalda y restriccin de los movimientos
de la columna por afeccin axial.
Cules son los frmacos de eleccin para tratar las espondiloartropatas y
la artritis reumatoide juvenil?
Los antiinflamatorios no esteroideos.
En qu consiste el sndrome de cola de caballo?
Se observa frecuentemente en la espondilitis anquilosante (EA). Existe
incontinencia urinaria nocturna, arreflexia del aquleo, impotencia e hi-
poestesia vesical. Tambin se acompaa de citica.
Cundo es positivo el signo de Lasegue?
Cuando se pruduce dolor al levantar la pierna ms de 60.
Cul es el diagnstico diferencial de la espondilitis anquilosante (EA)?
En la EA suele haber citica y la columna vertebral de bamb. Se debe
hacer con la hernia de disco intervertebral o con la hiperostosis vertebral
idioptica difusa.
Qu es la maniobra de Schober?
Es una prueba que se utiliza para evaluar la limitacin de la flexibilidad
lumbar. En un sujeto en bipedestacin, se marca la piel en un punto a ni-
vel de la V lumbar y se marca otro 10 cm arriba, y se pide al sujeto que se
incline hacia adelante. En un sujeto normal, los puntos se separan ms de
5 cm. La separacin menor de 3 cm indica limitacin del movimiento.
Cul es la diferencia entre osteofito y sindesmofito?
Los sindesmofitos son calcificaciones que aparecen en las caras nterola-
terales de las vrtebras y que con el tiempo forman un puente unidas por
osteofitos; son calcificaciones triangulares que se desarrollan a varios mi-
lmetros de la unin del disco vertebral.
200
Cul es la diferencia entre espondilosis y espondilolistesis?
La espondilosis es la interrupcin de la pars interarticularis de la vertebra y
la espondilolistesis se refiere al desplazamiento de una vrtebra sobre otra.
Cundo se indica el 5-ASA (olsalazina o mesalamina) para pacientes con
CUCI?
Cuando los pacientes son alrgicos a la sulfasalazina.
Cul es la diferencia entre el fenmeno y la enfermedad de Raynaud?
Ambos son provocados por un espasmo de las arteriolas de los dedos,
que causa cianosis y palidez. El fenmeno es unilateral y constante; la
enfermedad es bilateral y progresiva.
Cules enfermedades pueden presentar el fenmeno de Raynaud?
La esclerosis sistmica, el sndrome de CREST, el lupus eritematoso sis-
tmico y la enfermedad mixta del tejido conectivo.
Cules son los principales frmacos inmunosupresores no selectivos?
En metotrexato, la ciclofosfamida, la azatioprina y la prednisona.
Cules son los frmacos inmunosupresores selectivos?
La ciclosporina y el tacrolimus (FK506).
Cules son los efectos inmunosupresores del metotrexato?
Disminuye la actividad celular y humoral al inhibir la proliferacin de c-
lulas T en fase S.
Qu es la azatioprina?
Es un frmaco anlogo de la mercaptopurina y antagonista de la sntesis
de purinas.
Cules son los efectos inmunosupresores de la azatioprina?
Es citotxico y disminuye la inmunidad celular.
Cmo acta la ciclosporina sobre las clulas T?
Inhibe su activacin.
Cmo acta el metotrexato sobre las clulas T?
Inhibe la formacin de clulas T activadas (en fase S).
Cules son las acciones de la ciclosporina A y de la ciclofosfamida?
La primera es inmunomoduladora y la segunda cititxica.
Cules son los frmacos citotxicos ms utilizados?
Los antimetabolitos (azatioprina y metotrexato) y la ciclofosfamida.
http://booksmedicos.org
201
Qu se debe adicionar al tratamiento, en los casos de uso prolongado de
esteroides?
Vitamina D y calcio.
Qu son las vasculitis?
Son enfermedades que cursan con inflamacin de los vasos, a menudo
segmentaria, y que pueden involucrar venas y arterias de cualquier cali-
bre. Sin embargo, la mayora de las vasculitis se debe a inflamacin arte-
rial, lo que compromete el riego sanguneo y lleva a la necrosis del rea
que nutre el vaso afectado.
Cules son las principales vasculitis?
Prpura de Henoch-Schnlein: es la prpura palpable, desencadena-
da por reacciones de hipersensibilidad, y cuyo dato anatomopatolgi-
co caracterstico son los depsitos de IgA. Los vasos ms afectados
son las vnulas poscapilares que se infiltran con neutrfilos, por lo
que tambin se llama prpura leucocitoclstica.
Eritema nodoso: es una inflamacin de los capilares de la dermis pro-
funda, que causa lesiones dolorosas, tumefactas, profundas y rojas en
los brazos y piernas.
Poliarteritis nodosa: las arterias musculares de mediano tamao son
las afectadas, y la lesin causa ruptura de la lmina elstica interna
produciendo aneurismas posinflamatorios.
Arteritis de clulas gigantes: la fisiopatologa es similar a la poliarteri-
tis nodosa, pero el proceso ms bien es limitado a las arterias del r-
bol carotdeo extracraneal.
Arteritis de Takayasu: los vasos de gran calibre y sus ramas estn
afectados por un infiltrado linfocitario que lleva a la estenosis posin-
flamatoria y a la disminucin del pulso.
Cul es el cuadro de la crioglobulinemia tipo II (mixta) y con qu se rela-
ciona?
Prpura palpable, artritis y crioglobulinas elevadas. Se relaciona con el
VHC.
Qu es la polimialgia reumtica?
Es una enfermedad que cursa con dolor y rigidez del hombro y la cadera,
velocidad de sedimentacin globular elevada, en personas mayores de 50
202
aos, con fiebre, malestar y prdida de peso. Se asocia a arteritis de clu-
las gigantes.
Qu es la fibromialgia?
Es una enfermedad no inflamatoria caracterizada por dolor difuso acom-
paada de un transtorno del sueo del tipo de movimientos oculares r-
pidos (REM, etapa IV). Existen mltiples puntos de dolor como el
soboccipital, trapecio, glteo, a 2 cm de los epicndilos, en la segunda
unin costocondral, etc. La velocidad de sedimentacin globular es nor-
mal. El tratamiento se encamina a mejorar la calidad del sueo mediante
amitriptilina o relajantes musculares.
Qu enfermedades reumatolgicas se presentan con dermatitis y artritis?
La artritis por gonococo causa el sndrome periartritisdermatitis.
La artritis de Lyme causa artritis, dermatitis, sntomas neurolgicos y
cardiacos.
La enfermedad de Still presenta un examema morbiliforme evanes-
cente.
La dermatomiositis presenta exantema sobre las articulaciones (signo
de Gottron).
El sndrome de Sjgren presenta lesiones purpricas en las pantorri-
llas y los tobillos.
Cules son los pasos para estudiar y precisar la naturaleza de una artritis?
Determinar si el dolor proviene de la articulacin o de estructuras
adyacentes.
Determinar si es inflamatoria; es decir, si cursa con rigidez matutina
de ms de una hora de duracin.
Observar si la afeccin articular es simtrica, en cuyo caso habr que
sospechar lupus eritematoso sistmico, artritis reumatoide, dermato-
miositis y esclerosis sistmica progresiva.
Si la afeccin es asimtrica se debe pensar en sndrome de Reiter o
espondilitis anquilosante.
Cul es el componente principal del lquido articular?
El cido hialurnico. Su presencia se puede reconocer por el signo de la he-
bra, ya que le confiere una viscosidad elevada al lquido articular. Al adi-
203
cionar cido actico se aglutina (prueba del cogulo de mucina). Cuando
la artritis es inflamatoria estas pruebas son negativas.
A qu se llama deformidad en ojal?
A la flexin de la articulacin interfalngica proximal por lesin de los
extensores, presente en la enfermedad inflamatoria.
A qu se le llama deformidad en cuello de cisne?
A la flexin de la articulacin interfalngica distal y metacarpofalngica,
con extensin de la interfalngica proximal.
Cules son las principales enfermedades articulares causadas por micro-
cristales?
La gota y la pseudogota.
Qu es la gota?
Es una artritis aguda, recidivante, de las articulaciones perifricas, resultan-
te del depsito de cristales de urato monosdico procedente de lquidos
corporales hiperuricmicos sobresaturados en tendones, articulaciones y
cerca de ellos. Puede hacerse crnica y deformante.
En qu tejidos se produce el urato?
En todos aquellos en que exista la enzima xantina-oxidasa, principalmen-
te el hgado y la mucosa del intestino delgado.
Cul es el nivel normal del cido rico srico?
7 mg/dl.
Cul es la secuencia de los cambios metablicos que transforman el IMP
hasta urato?
IMP hipoxantina xantina urato.
Cmo se presenta el episodio agudo de gota?
Puede ser causado por un traumatismo mnimo, por un abuso diettico o
alcohlico, por una ciruga. Se presenta con artritis, descamacin y pruri-
to posinflamatorios. Cuando afecta al primer ortejo se llama podagra.
Cules son los factores que inducen la hiperuricemia secundaria?
Diurticos: tiacidas.
Frmacos citotxicos: ciclosporina.
Estados de recambio aumentado de purinas: trastornos mieloprolife-
rativos, mieloma mltiple, hemoglobinopatas.
204
Estados de excrecin disminuida de urato: enfermedad renal crnica,
lactacidemia, cetoacidemia, probenecid a dosis subteraputicas.
Enfermedades crnicas: diabetes, hipotiroidismo e intoxicacin por
plomo.
Cules son las causas de aumento de excrecin renal de cido rico?
La quimioterapia, el sndrome de Lesch-Nyhan y la gota.
Cmo se trata la hiperexcrecin de cido rico?
Con hidratacin y alcalinizacin de la orina a un pH mayor de 7.5
Qu es la pseudogota?
Es una artritis con depsitos de cristales de monofosfato de sodio con bi-
rrefringencia positiva.
Qu esteroide se usa intraarticularmente y a qu dosis?
Triamcinolona, 30-40 mg.
Cmo acta la colchicina?
Inhibiendo la quimiotaxis celular hacia los cristales de urato.
Cmo se decide usar alopurinol o un uricosrico en los casos de hiperuri-
cemia?
Se determina el cido rico en la orina de 24 horas, si este es menor de
800, se usa un uricosrico; si es mayor de 800, se usa alopurinol.
Cules son agentes uricosricos?
El probenecid y la sulfinpirazona disminuyen la resorcin tubular de ci-
do rico si se utilizan a dosis teraputicas. A dosis inferiores bloquean la
secrecin tubular. El probenecid inhibe la secrecin tubular de penicilina,
por lo que en ocasiones se usa para potenciar su efecto.
Cules son los msculos del manguito rotador del hombro?
Supraespinoso, infraespinoso, redondo menor y subescapular. La patolo-
ga del manguito rotador es causa frecuente de dolor de hombro. En estos
casos el dolor se desencadena con el movimiento hacia arriba y la rota-
cin interna del brazo.
Qu es el hombro de Milwaukee?
Es una lesin del hombro causada por cristales de hidroxiapatita, caracte-
rizada por cambios radiogrficos como un gran derrame, sinovitis proli-
ferativa, depsito mineral e inestabilidad ligamentosa.
205
Cmo se identifica las lesiones ligamentarias de la rodilla?
Debemos verificar si existe deslizamiento de la tibia sobre el fmur, lo
que se conoce como signo de la gaveta. Si el deslizamiento es anterior est
afectado el ligamento cruzado anterior, si es posterior, est afectado el li-
gamento cruzado posterior.

206
207
INTOXICACIONES Y ACCIDENTES







Cul es el principal sitio de obstruccin de de vas areas en los menores
de un ao?
La laringe.
Cul es el cuadro clnico cuando existe obstruccin de las vas areas infe-
riores?
Sibilancias y asimetra de los ruidos respiratorios.
Qu es la metahemoglobina?
Es la hemoglobina oxidada a su estado frrico; es incapaz de transportar
oxgeno.
Cules son las sustancias que pueden causar metahemoglobinemia?
Benzocana, anilinas, nitritos, gases de xido nitroso, nitrobenceno, dap-
sona, piridio y otras.
Cules son los datos clnicos de la intoxicacin por monxido de carbono?
El monxido de carbono se une a la hemoglobina con una afinidad 250
veces mayor que el oxgeno. Las vctimas experimentan cefalea, vrtigo,
dolor abdominal y nusea cuando aspiran una mezcla de 10 a 20%. Las
concentraciones mayores prodecen confusin, disnea y sncope. Cuando
las concentraciones llegan a 50 o 60% hay hipotensin, convulsiones y
coma.
Qu sistema evita la sobreproduccin de metahemoglobina?
El sistema reductasa-NADH-metahemoglobina.
Cules son los pasos para iniciar la desintoxicacin oral?
Adsorcin con carbn activado, utilizndolo en proporcin de 5 a 12
veces la cantidad calculada de lo ingerido.
Induccin del vmito con jarabe de ipecacuana o lavado gstrico.
208
Catrticos que actan disminuyendo el tiempo de trnsito y, por con-
secuencia, la absorcin del txico. Se utilizan citrato o sulfato de
magnesio.
Cmo se trata la intoxicacin por cido acetilsaliclico (ASA)?
Se alcaniliza con HCO
3
y se adjunta cimetidina en el evento agudo para
disminuir la actividad de la citocromo-P450-oxidasa, que al metabolizar
el ASA, da origen a sus metabolitos txicos.
Qu niveles sricos de ASA producen los diversos grados de intoxicacin?
Intoxicacin leve: 30 mg/dl provocan al principio alcalosis respirato-
ria y luego acidosis metablica.
Intoxicacin moderada: 100-150 mg/dl a partir de este nivel hay que
inducir el vmito y hospitalizar.
Intoxicacin grave: ms de 150 mg/dl.
Cundo es txico el acetaminofn?
Cuando, en dosis altas, su metabolito, la N-acetilbenzoquinoneimina, exce-
de al glutatin que la inactiva y produce necrosis heptica.
Cules son las fases de la intoxicacin por acetaminofn?
Fase 1: en los primeros 30 o 60 minutos, hay nusea y vmito, sin
prdida de la conciencia.
Fase 2: en las primeras 24 a 48 horas, hay alteracin de las pruebas
de funcionamiento heptico.
Fase 3: en los siguientes 3 a 5 das hay confusin y estupor secunda-
rios al dao heptico.
Por qu hay alcalosis respiratoria en los casos de la intoxicacin por ace-
taminofn?
Porque aumenta la sensibilidad del centro respiratorio a la acidosis, lo
que provoca hiperventilacin.
Por qu puede haber tambin acidosis metablica en la intoxicacin por
acetaminofn?
Porque se inhibe el ciclo de Krebs.
Por qu se debe administrar bicarbonato para tratar la intoxicacin por
acetaminofn?
Porque a un pH urinario menor de 8, el acetaminofn y sus metabolitos
txicos se eliminan ionizados en la orina.
209
Cul es el antagonista del sulfato de magnesio y del potasio?
El gluconato de calcio.
Cul es el cuadro clnico causado por la sobredosificacin de ansiolticos?
Depresin respiratoria, hipotensin y coma.
Cul es el cuadro clnico causado por la intoxicacin grave por narcti-
cos?
Depresin respiratoria, vasodilatacin perifrica, pupilas puntiformes y
muerte.
Cundo aumenta la protoporfirina?
Cuando disminuye el fierro y cuando existe intoxicacin por plomo.
Cules son los sntomas de cafeinismo?
Se relacionan con la ingestin de ms de 500 mg al da y son de tipo car-
diovascular, gastrointestinal, nerviosismo, agitacin e insomnio.
Cules son los tres grupos de sntomas producidos por la intoxicacin por
rganofosforados?
Sntomas muscarnicos, nicotnicos y del sistema nervioso central.
Cules son los sntomas muscarnicos?
Diaforesis, sialorrea, pupilas puntiformes, epfora, broncoconstriccin,
aumento de la secrecin bronquial, espasmo abdominal, vmito, diarrea
y bradicardia.
Cules son los sntomas nicotnicos?
Taquicardia, fasciculaciones y espasmos musculares que afectan el dia-
fragma y los dems msculos respiratorios.
Cules son las manifestaciones en el sistema nervioso central (SNC)?
Cefalea, fatiga, vrtigo, ansiedad, confusin, convulsiones, depresin del
centro respiratorio y coma.
Cmo actan fisiopatolgicamente los rganofosforados?
Inhibiendo la acetilcolinesterasa, la principal va de exposicin es la piel.
Los sntomas inician 2 a 3 horas despus de la intoxicacin.
Cul es el primer paso en el tratamiento de la intoxicacin con rganofos-
forados?
Establecer una va area permeable.
Cul es la dosis de Ipecac para tratar la intoxicacin con rganofosfora-
dos?
15 a 30 ml.
210
Cul es la dosis del sulfato de atropina para tratar la intoxicacin con r-
ganofosforados?
1 mg. IV cada 20-30 minutos hasta la desaparicin de los sntomas.
Qu olor tienen los productos de intoxicacin por arsnico y fsforo?
A ajo.
Cmo causan dao los lcalis concentrados?
Producen necrosis por licuefaccin penetrante.
Cmo causan dao los cidos concentrados?
Produce necrosis por coagulacin.
Dnde se puede encontrar el etilenglicol?
El etilenglicol es un solvente (alcohol) incoloro, inodoro y de sabor dulce
que se puede encontrar en pinturas, plsticos, explosivos, extintores, flui-
dos hidrulicos, limpiadores de parabrisas y preparaciones anticongelan-
tes.
Cul es el efecto general del etilenglicol?
Es un depresor potente del SNC; es muchas veces ms potente que el
etanol.
Qu efectos comparten la intoxicacin por carbamacepina y la causada
por anfetaminas?
Hipertensin, convulsiones y nistagmo.
Qu productos son susceptibles de ser eliminados por la orina?
Todos los de volumen de distribucin bajo (menor de 100)
Qu medicamentos causan intoxicacin por opiceos?
El difenoxilato y el dextrometorfn.
Cul es la trada de la intoxicacin por opiceos?
Miosis, coma y depresin respiratoria.
Cul es la dosis de la naloxona (antagonista opiceo)?
100-200 mEq/kg/dosis
Cules son las complicaciones producidas por los opiceos?
leo, arritmias y edema pulmonar no cardigeno.
Qu medicamentos son anticolinrgicos?
La belladona, butilhioscina y atropina.
Cules son los sntomas anticolinrgicos?
Sequedad de ojos y mucosas, taquicardia, retencin urinaria, estrei-
miento, leo y midriasis.
211
Cul es el antdoto de los anticolinrgicos?
La fisostigmina.
Qu es el succmero?
Tambin conocido como DMSA (cido meso-2,3-dimercaptopurnico),
se utiliza para tratar la intoxicacin por plomo.
Qu medicamentos son simpaticomimticos?
Los anticongestivos o antigripales como la fenilefrina, efedrina, propano-
lamina, nafazolina y la oximetazolina.
Qu se debe sospechar ante la presencia de sntomas de neuroestimula-
cin sbita en un nio con catarro?
Intoxicacin por antigripales simpaticomimticos alfa y beta.
Cundo se presenta intoxicacin por talio?
Cuando hay ingestin de raticidas.
Cul es la clnica de la intoxicacin por talio?
Dolor abdominal, estreimiento, neuropata y encefalopata.
Cules son las fuentes de intoxicacin por plomo?
Los talleres de loza de barro vidriada y las fundiciones.
Cul es al cuadro clnico de la intoxicacin por plomo?
Encefalopata e hipertensin endocraneal.
Qu intoxicaciones cursan con encefalopata?
La de plomo y la de talio.
Cules son los quelantes del plomo?
Edatato clcico disdico (EDTA), succmero, dimercaprol y D- penici-
lamina.
Cul es el cuadro clnico de la intoxicacin por hierro?
Dolor tipo clico, vmito, diarrea y hemorragia gastrointestinal.
Cmo acta el cianuro?
Es un inhibidor de la citocromo oxidasa.
Cmo se clasifican las quemaduras?
Grado1: epidermis
Grado 2A: epidermis y dermis papilar
Grado 2B: epidermis y dermis reticular.
Grado 3A: epidermis, dermis y tejido celular subcutneo (TSC).
Grado 3B: epidermis, dermis, TSC, msculo y hueso.
212

213
URONEFROLOGA








Cules son las cinco variedades clnicas de las glomerulopatas?
Sndrome nefrtico agudo.
Glomerulonefritis rpidamente progresiva.
Sndrome nefrtico.
Anormalidades asintomticas del sedimento urinario (proteinuria y
hematuria).
Glomerulonefritis crnica.
Cules son los rangos de proteinuria en orina de 24 horas?
3 cruces en el examen general de orina (EGO), resultado significati-
vo, pero no patolgico.
300 mg: normal.
500 mg:significativo pero no patolgico.
Menos de 1 g: tubular.
Ms de 1 g: glomerular.
3.5 g: sndrome nefrtico.
960 mg/m
2
/da: sndrome nefrtico.
Qu es la hematuria?
Se considera hematuria cuando se hallan ms de 3 a 5 eritrocitos por
campo de gran aumento en una muestra de sedimento. Los falsos positi-
vos pueden deberse al consumo de vitamina C, betabel y mioglobina en
la dieta.
Para qu se utiliza la prueba de ortotoluidina?
Para demostrar hematuria detectando la presencia de hemoglobina en la
orina cuando no pueden identificarse eritrocitos en el sedimento.
214
Cundo se habla de macroalbuminuria y microalbuminuria?
Macroalbuminuria es cuando la tasa de excrecin de albmina rebasa los
300 mg/da y microalbuminuria cuando va de 30 a 300 mg/da.
Cundo se considera que existe oliguria?
Cuando el volumen urinario es menor de 300 ml/m
2
/da o menor de 0.5
ml/kg/hora.
Cules son las causas de la proteinuria?
La proteinuria secundaria a ortostatismo.
La nefritis tbulointersticial.
La glomerulonefritis.
Qu es el sndrome nefrtico?
Es un sndrome en el que hay falla renal aguda que se desarrolla en va-
rios das, oliguria, hematuria y cilindros de eritrocitos, edema en las par-
tes declives e hipertensin arterial.
Qu es el sndrome nefrtico?
Es un sndrome integrado por la presencia de proteinuria (ms de 3.5g
por cada 1.73 m
2
en 24horas), hipoalbuminemia menor de 3, edema, co-
lesterol ms de 200, lipiduria, ditesis trombtica y disminucin de 10 a
30% en la velocidad de filtracin glomerular (VFG).
Cul es el cuadro clnico de la nefritis interstiticial?
Presencia de protenas especiales, como la protena de Tamm-Horsfall,
beta 2-microglobulina y albmina. Se puede encontrar en pacientes que
sufren nefritis causada por frmacos o enfermedades inflamatorias crni-
cas. Hay fiebre, exantema mculopapular transitorio, insuficiencia renal
aguda (IRA), hematuria y piuria.
Qu frmacos pueden causar nefritis intersticial?
La penicilina, la meticilina y el alopurinol, algunos diurticos y antiin-
flamatorios no esteroideos.
Cul es el hallazgo caracterstico en el examen general de orina (EGO) de
pacientes que padecen nefritis intersticial?
Hay eosinofiluria.
Cul es el cuadro clnico de la glomerulonefritis?
Se encuentra desde proteinuria glomerular (ms de 1 g/24horas) hasta
proteinuria nefrtica (ms de 3.5gr/24horas), IRA y sndrome nefrtico.
215
Cmo se clasifican las glomerulonefritis (GN) por su anlisis serolgico?
GN por anticuerpos, anticitoplasmas de neutrfilo (ANCA): granu-
lomatosis de sndrome de Wegener, Churg-Strauss.
GN antimembrana basal glomerular: sndrome de Goodpasture.
GN de complejos inmunes: GN del lupus, GN posinfecciosa, nefro-
pata por IgA. GN crioglobulinmica y GN membranoproliferativa.
Cules son las capas de la barrera de filtracin glomerular que separan a
la sangre del filtrado glomerular?
El endotelio capilar glomerular fenestrado.
La capa de clulas mesangiales.
La embrana basal.
El epitelio de clulas viscerales (podocitos).
Cul es la lesin anatomopatolgica que se encuentra en las siguientes en-
fermedades glomerulares?
GN membranosa: depsitos subepiteliales electrodensos.
GN mesangial proliferativa: proliferacin de las clulas mesangiales.
GN membranoproliferativa: reduplicacin o engrosamiento de la
membrana basal y proliferacin de la matriz mesangial.
GN rpidamente progresiva: se encuentran lesiones en semiluna en
el espacio de Bowman.
GN proliferativa difusa: existe hipercelularidad mesangial y endote-
lial, depsitos electrodensos subepiteliales, infiltrado de neutrfilos y
monocitos y depsitos de C3 en la membrana basal.
Cules son las caractersticas de la GN posinfecciosa?
La lesin anatomopatolgica caracterstica es la GN proliferativa difusa,
cuya presentacin clnica tpica es la del sndrome nefrtico.
Cules son las caractersticas clnicas y de laboratorio de la glomerulone-
fritis membranoproliferativa?
Puede ser idioptica o secundaria a lupus eritematoso sistmico (LES), crio-
globulinemia o infeccin crnica. Se presenta con sndrome nefrtico, hipo-
complementemia de C3 y evolucin a la enfermedad renal terminal.
Cules glomerulopatas cursan con hipocomplementemia C3?
La glomerulonefritis membranoproliferativa y la posinfecciosa (lesin
proliferativa difusa).
216
Qu es el sndrome de Alport?
Es una nefritis hereditaria que puede ser autosmica dominante o ligada
a X. Cursa con piuria, hematuria, falla renal progresiva y prdida auditi-
va neurosensorial.
En qu consiste la enfermedad de cambios mnimos?
Es una enfermedad glomerular tambin conocida como nefrosis lipoidea
o sndrome nefrtico de lesin nula. Es la causa principal de sndrome
nefrtico en nios; cursa con sndrome nefrtico, hipertensin y azoemia.
El tratamiento se lleva a cabo con esteriodes, citotxicos (ciclofosfamida,
clorambucil) y ciclosporina.
Cul es el diagnstico diferencial del sndrome de Goodpasture?
Granulomatosis de Wegener, sndrome de Churo-Strauss, prpura trom-
bocitopnica trombtica (PTT), crioglobulinemia, vasculitis necrosante,
enfermedad del legionario y prpura de Henoch-Schnlein.
Cmo se diagnostica la nefropata por IgA (enfermedad de Berger)?
Por una biopsia de piel que demuestre depsitos de IgA (no por el nivel
srico de IgA).
Cules son los dos tipos de nefropata diabtica?
Nodular y difusa. La difusa es ms frecuente; se presenta con un ensan-
chamiento mesangial generalizado.
Dnde se observa y cmo es la lesin de Kimmelstiel-Wilson?
Se observa en la nefropata diabtica nodular. Se encuentran depsitos de
material Schiff-positivo en la periferia de los glomrulos.
Cmo es la lesin de la glomeruloesclerosis?
Existe hialinizacin de la arteriola y lesin de Kimmelstiel-Wilson.
Cul es la velocidad de filtracin glomerular (VFG) normal?
125 ml/min.
Cmo se obtiene el ndice de falla renal?
Sodio urinario (UNa)/ Creatinina urinaria (UCr)/Creatinina plasmtica
(PCr).
Cmo se calcula la fraccin excretada de sodio (FENa)?
(UNa/PCr) / (UCr/PCr)
217
Cmo se evalan el ndice de falla renal y la fraccin excretada de sodio
(FENa)?
Cuando son menores que 1, la falla es prerrenal; cuando son mayores
que 1 la falla es renal (necrosis tubular aguda).
Cmo se interpreta el sodio urinario?
Cuando es menor que 20, la falla es prerrenal, y si es mayor que 20, es
renal o posrenal.
Cmo se evala el ndice BUN/Creatinina en suero?
Mayor que 20 es prerrenal; menor que 10, es renal o necrosis tubular
aguda (NTA).
Cundo se habla de NTA?
Cuando se encuentran estigmas de insuficiencia renal aguda, FENa ma-
yor a 1% y sedimento con cilindros granulosos pigmentados.
Cundo se habla de insuficiencia renal aguda y crnica con base en los va-
lores de creatinina?
La insuficiencia renal aguda tiene valores de creatinina plasmtica de 1.5
a 3. Cuando la creatinina es mayor que 3 se habla de insuficiencia renal
crnica.
Cundo se habla de falla renal terminal?
Cuando la depuracin de creatinina es menor que 5.
Cul es el cuadro clnico de la insuficiencia renal crnica (IRC)?
Hiperazoemia, sndrome urmico, prurito de difcil control, hipertensin
arterial, isostenuria, osteodistrofia renal y rin pequeo.
Qu es el sndrome urmico?
Ocurre despus de la falla renal y cursa con aliento urmico, escarcha
urmica, malestar general, vmito, alteraciones del sensorio y derrame
pericrdico.
Cmo afecta la insuficiencia renal el metabolismo del calcio y del fsforo?
Al disminuir la velocidad de filtracin glomerular (VFG) por debajo
de 25% de lo normal, se deteriora la excrecin de fsforo. La hiper-
fosfatemia causa hipocalcemia, ya que el fosfato es un quelante del
calcio. La hipocalcemia aumenta la secrecin de hormona paratiroi-
dea, que tiene un efecto fosfatrico y normaliza el fsforo srico.
218
Por la conversin de 25-hidroxicolecalciferol a 1-25-hidroxicolecal-
ciferol, lo que disminuye la absorcin intestinal de calcio y da como
resultado una mineralizacin anormal del hueso u osteomalacia.
Cmo se monitoriza el fsforo cuando existe la insuficiencia renal?
Con el producto calcio-fsforo, que debe ser menor de 60-70, para evitar
calcificaciones metastsicas en vasos sanguneos, tejidos blandos, pulmo-
nes y miocardio.
Qu es la osteodistrofia renal?
La hipocalcemia funcional secundaria a la hiperfosfatemia y la disminu-
cin del 1, 25-DHC provocan niveles altos de hormona paratiroidea
(PTH), estimulan un alto recambio seo con resorcin osteoclstica y le-
siones subperisticas, lo que causa lesiones en sal y pimienta, princi-
palmente en clavculas y falanges.
Qu frmacos se usan para fijar el potasio cuando existe insuficiencia re-
nal crnica (IRC)?
Carbonato de calcio, sulfato de potasio e hidrxido de aluminio.
Cules son las restricciones dietticas para los pacientes que padecen
IRC?
Protenas menos de 1g/da y Na 1-2 g/da. Cuando la VFG es menos de
10-15, el potasio se restringe a menos de 1g/da, y el sodio a 60-70
mEq/da.
Cules son las indicaciones para dilisis?
Sndrome urmico, PH menor de 7.20, hiperkalemia de 7, sobrecarga de
lquidos, sntomas neurolgicos y en algunos casos, creatinina de 10 y
BUN mayor de 100.
Qu clulas conforman el aparato yuxtaglomerular?
Las clulas yuxtaglomerulares, la mcula densa y las clulas en encaje.
Dnde se encuentran las clulas yuxtaglomerulares?
En la pars de la arteriola aferente del glomrulo.
Dnde se encuentra la mcula densa?
La rama ascendente gruesa del asa de Henle alcanza al glomrulo de la
nefrona de la cual se origina y pasa cerca de las arteriolas aferente y efe-
rente; en este punto el epitelio del tbulo se modifica para dar lugar a la
mcula densa.
219
Dnde se produce y para qu sirve la renina?
Se produce en las clulas yuxtaglomerulares. Su funcin consiste en ex-
traer la angiotensina I de su precursor, el angiotensingeno.
Dnde se producen el angiotensingeno y la enzima de conversin de la
angiotensina (ECA)?
El angiotensingeno se produce en el hgado y la ECA en el endotelio
vascular.
Cules son los receptores de angiotensina II?
Los receptores AT
1A
se encuentran en las paredes de los vasos sangu-
neos, donde ejercen su efecto presor. Los receptores AT
1B
se encuentran
en la corteza suprarrenal, donde promueven la secrecin de aldosterona.
Un exceso de angiotensina II regula en forma descendente a los recepto-
res vasculares, y en forma ascendente, a los receptores adrenales.
Qu enfermedades cursan con desensibilizacin a la angiotensina?
Los pacientes con cirrosis no metabolizan la angiotensina II para inacti-
varla, los pacientes con un estado prolongado de deplecin de sodio
siempre tienen niveles altos de A-II. Por tanto ante este exceso los recep-
tores se regulan en forma descendente provocando un estado refractario.
Cmo acta la prostaciclina en el eje renina-angiotensina-aldosterona?
Es un contrarregulador de la angiotensina II.
A qu se llama retroalimentacin tbulo-glomerular?
Al fenmeno regulado por la mcula densa, en el que una reduccin del
flujo dentro del tbulo aumenta la velocidad de filtracin glomerular
(VFG) y viceversa. La VFG se ajusta por constriccin o dilatacin de la
arteriola eferente.
Qu fenmeno explica la relacin directa entre la resorcin tubular de
agua y la de sodio?
En el fenmeno de equilibrio glomerulotubular, un aumento de la VFG
produce aumento de la resorcin de solutos y, en consecuencia, de agua,
principalmente en el tbulo proximal; es particularmente prominente pa-
ra el sodio.
Cmo se realiza la prueba para la determinacin de renina?
Se utiliza una dosis media-alta de furosemida. Lo normal es que despus
de la deplecin de volumen la renina aumente. El nivel no se eleva cuan-
220
do hay un estado hiperproductor de renina como en el hiperaldostero-
nismo secundario.
Cul es la mejor prueba para detectar hiperaldosteronismo primario?
El ndice de aldosterona/renina tomado en bipedestacin.
Cmo se realiza la prueba para la determinacin de aldosterona?
Se hace que el paciente lleve una dieta de 150 miliequivalentes de sodio
por algunos das. Luego se le administra solucin salina. La respuesta
normal resulta en una elevacin de la aldosterona. sta no se eleva cuan-
do existe hiperaldosteronismo.
Cul es la osmolaridad urinaria normal?
700-1,400 m0sm/kg.
Qu osmolaridad urinaria se alcanza en la diabetes inspida?
200 m0sm/kg.
Cul es el tratamiento de la diabetes inspida nefrognica?
Las tiazidas, porque disminuyen el volumen urinario y aumentan la concen-
tracin de sodio en el fluido tubular lo que produce orina hiperosmolar.
Cul es el tratamiento en la diabetes inspida central?
Desmopresina y clorpropamida, que potencian la secrecin de hormona
antidiurtica hormona antidiurtica (ADH).
Cmo se realiza la prueba para determinar hormona antidiurtica (ADH)?
Se restringen los lquidos al paciente y luego se le administra un anlogo
de la vasopresina (desmopresina). La respuesta normal es que la densidad
urinaria se eleve (ms de 1.020), pues se evita la salida de agua. En la
diabetes inspida la densidad urinaria no se eleva tras la deprivacin de
lquidos, pues no hay ADH que evite la salida de agua; sin embargo, la
densidad urinaria (que se encuentra muy baja) se eleva slo para alcanzar
el nivel normal cuando se administra un agente que emule la funcin de
la ADH.
Qu medicamentos aumentan la secrecin de ADH?
Clorpropamida, carbamacepina, vincristina y clofibrato.
Cmo afectan la excrecin de calcio los diurticos de asa y las tiacidas?
Los diurticos de asa aumentan la calciuria, y las tiacidas la disminuyen.
Cules son dos indicaciones de las tiacidas?
Para producir hipocalciuria cuando hay cristales de oxalato y para pro-
ducir orina hiperosmolar en los casos de diabetes inspida.
221
Cmo acta la espironolactona?
Es un competidor activo de la aldosterona en los sitios intracelulares del
tbulo colector donde estimula el recambio Na-K, por lo que el Na no se
reabsorbe y s se reabsorben potasio e hidrgeno.
Cmo actan la amilorida y el triamterene?
Bloquean los canales de Na y disminuyen el recambio Na-K en el tbulo
colector.
Cmo actan los diurticos osmticos?
Actan en el tbulo contorneado distal arrastrando agua por su efecto
osmtico.
Cules son las indicaciones de los diurticos osmticos?
Hipertensin intracraneana, falla renal por choque hipovolmico e into-
xicaciones por frmacos.
Cmo actan los diurticos de techo alto?
Inhiben la bomba Na-K-Cl en la rama ascendente gruesa del asa de Henle.
Cul es el cuadro clnico de la hipertensin renovascular?
Inicio antes de los 20 y despus de los 50 aos.
Refractariedad al tratamiento.
Hipocalcemia.
Soplo abdominal.
Renina elevada, en especial despus de una prueba con captopril.
Cules son las dos formas clnicas de la infeccin de vas urinarias (IVU)?
Infeccin de las vas urinarias bajas: aumento de la frecuencia urina-
ria, disuria, dolor suprapbico, orina con mal olor y turbia y urgencia
miccional. Estos sntomas, en cualquier combinacin, se observan en
la uretritis, prostatitis y cistitis.
Pielonefritis aguda: dolor en el flanco, fiebre, malestar y sintomatolo-
ga urinaria baja.
Para qu sirve la prueba de los nitritos en orina?
El microorganismo patgeno habituales de vas urinarias (Proteus) rompe la
urea formando nitritos; la presencia de stos es indicativa de infeccin.
Cul es el tratamiento de las infecciones urinarias no complicadas?
Amoxicilina: 3g VO durante tres das, o TMP-SMZ 320 mg/1.6g durante
tres das.
222
Cul es el tratamiento de la pielonefritis y de la IVU recidivante?
Debe tratarse por catorce das con trimetoprim-sulfametoxazol (TMO-
SMZ) (80 mg/400 mg) dos veces al da o con una cefalosporina de pri-
mera generacin.
Cul es el tratamiento de la prostatitis?
Debe tratarse durante 14 das con medicamentos que permanezcan en el
tejido prosttico, como el trimetoprim o la carbenicilina.
Por qu se justifica el uso de eritromicina en los casos de prostatitis no
bacteriana?
Porque como agentes etiolgicos pueden estar implicados ureaplasmas,
micoplasmas y chlamydias que no se evidencian por los estudios con tin-
cin o cultivo habituales.
Por qu los antibiticos no alcanzan fcilmente la prstata?
Por el epitelio prosttico especializado y porque el lquido prosttico tie-
ne un pH cido (6.4).
Qu medicamentos se usan para tratar la prostatodinia (espasmo vesical)?
Bloqueadores alfa como la terazocina (1 a 10 mg) o la doxazocina (1 a 8
mg).
Cul es el tratamiento de la hiperplasia prosttica benigna (HPB)?
La reseccin transuretral (RTU).
Cules son las 4 vas que integran el reflejo de miccin?
I Corteza-puente: promueven la continencia.
II Puente-sacro: promueven la miccin.
III Sacro-vejiga: coordina el reflejo de miccin (detrusin y relaja-
cin).
IV Corteza-sacro: promueve la continencia.
Cmo se gobierna el reflejo de miccin?
Lo controla el cerebro, pero se coordina en el puente.
Cules desrdenes pueden alterar el mecanismo de miccin?
La hiperreflexia del detrusor, la arreflexia del detrusor y la disinergia de-
trusor-esfnter.
Quin inhibe al msculo detrusor?
La corteza cerebral.
223
Cules son los patrones de disfuncin del vaciamiento (vejiga neurognica)?
Hiperreflexia del detrusor: vejiga neurognica hipertnica secundaria
a lesiones suprasacras que causan una capacidad de almacenamiento
disminuida de la vejiga y prdida de orina en decbito.
Arreflexia del detrusor: secundaria a lesiones de la cauda equina que
causan aumento de la capacidad de almacenamiento y retencin uri-
naria.
Disinergia del esfnter externo y del detrusor: produce contraccin del
esfnter externo mientras se contrae el detrusor, lo que causa una obs-
truccin funcional.
Cul es el tratamiento en los casos de vejiga neurognica?
Anticolinrgicos que inhiben al detrusor, o bien, agonistas alfa que actan
disminuyendo el espasmo prosttico, cervical y uretral. Tambin debe
existir un reentrenamiento vesical.
Cmo funcionan el cuello vesical y la uretra?
Tienen receptores alfa que median la contraccin y receptores beta que
median la relajacin.
Cul es la causa ms comn de incontinencia en la mujer?
La disfuncin del ngulo uretrovesical o cistocele. Tambin puede haber
incontinencia si hay disfuncin del esfnter o uretra hipermvil.
Qu factores causan incontinencia caracterizada por urgencia?
La hiperreflexia del detrusor y la inestabilidad de la uretra.
Cul es el cuadro clnico de la incontinencia femenina verdadera?
Hay prdida de orina en cualquier caso en que la presin abdominal e in-
travesical rebasen a la presin de cierre del esfnter uretral; por ejemplo al
toser.
Cul es el protocolo para estudiar la incontinencia femenina?
Primero se practica un examen general de orina para descartar infeccin.
Despus se realizan las pruebas urodinmicas y luego, la cistoscopa.
Cul es el tratamiento mdico del cistocele?
Los ejercicios de Kegel dan buenos resultados en 75% de los casos; tam-
bin se utilizan anticolinrgicos para distender la vejiga y simpaticomi-
mticos para dar ms tono al esfnter.
224
Cul es el tratamiento quirrgico del cistocele?
La uretropexia retropbica, que consiste en elevar la vejiga y la uretra fijn-
dolas al pubis. Tambin se utiliza la colporrafia anterior.
Cules estudios son los indicados en caso de trauma ureteral y renal?
Primero la urografa excretora y luego la TAC.
Menciona dos causas de trauma ureteral.
En primer lugar, las heridas penetrantes, y en segundo, la yatrogenia.
Cul es la causa ms frecuente de trauma vesical?
La fractura plvica.
Cules son los signos clnicos del traumatismo uretral?
Sangre en el meato y prstata elevada.
Cmo se evalan radiolgicamente el trauma vesical y el uretral?
Con uretrografa retrgrada y con cistografia de llenado mximo, respec-
tivamente.
Cmo se hace el diagnstico de ruptura testicular?
Con el cuadro clnico y el ultrasonido.
Cules son los tumores testiculares de tipo seminomatoso?
El clsico, anaplsico y espermatoctico.
Cules son los tumores testiculares no seminomatosos?
El carcinoma embrionario, teratoma, tumor del saco de Yolk y coriocar-
cinoma.
Qu marcadores sirven para hacer el diagnstico de los tumores testicula-
res no seminomatosos?
La alfa feto protena y la gonadotropina corinica humana.
Cul es el tratamiento de :
Tumor testicular no seminomatoso: ciruga con exploracin retrope-
ritoneal.
Tumor testicular seminomatoso: radioterapia.
Tumor testicular seminomatoso en etapa III y IV: cisplatino, etopsi-
do y bleomicina.
Cundo se debe sospechar agenesia de las vesculas seminales?
Cuando hay disminucin del volumen eyaculatorio y disminucin en su
contenido de fructosa.
225
Cul es la inervacin simptica del pene?
Le dan inervacin las races dorsales inferiores y lumbares superiores
(nervio hipogstrico).
Cul es la inervacin ms importante para que ocurra ereccin?
La parasimptica.
Cules son los estudios de laboratorio tiles en los casos de impotencia?
Los niveles de testosterona y un estudio completo que permita descartar
cualquier enfermedad sistmica.
En qu etapa del sueo ocurre la ereccin?
En la del sueo REM.
Cmo se descarta una causa vascular de la impotencia?
Se descarta si se obtiene una buena respuesta al instilar papaverina, fento-
lamina o Prostaglandina E.
Cmo acta el sildenafil?
Es un inhibidor selectivo de la fosfodiesterasa.
Cul es el cuadro clnico de la aplasia de clulas germinales (clulas de
Leydig) y la obstruccin del conducto deferente?
Ambas se presentan con volumen eyaculatorio normal, contenido normal
de fructosa en el semen y azoospermia.
Cunto tarda un espermatozoide en llegar del epiddimo al conducto defe-
rente?
12 a 21 das.

226
227
NEUMOLOGA







Dnde deben auscultarse los ruidos traqueales?
En el hueco supraesternal.
En qu consiste la respiracin de Cheyne-Stokes?
La frecuencia respiratoria sube y baja, con periodos de apnea interme-
dios. Aparece en la insuficiencia cardiaca izquierda grave o en la enfer-
medad neurolgica.
Cmo se calcula la presin alveolar de oxgeno (PAO
2
)?
Se necesita calcular primero la presin inspirada de oxgeno (PIO
2
) mediante
la frmula PIO
2
=FIO
2
x (presin atmosfrica (760 mmHg)- pH
2
O).
Entonces PAO
2
= PIO
2
(PaO
2
/RQ).
RQ= Cociente respiratorio, que tiene un valor de 0.8
Para qu sirve la PAO
2
?
Calculando la PAO
2
se puede calcular la diferencia alvolo-arterial de O
2
.
Utilizando la diferencia alvolo-arterial, la PaO
2
, la PaCO
2
y la respuesta
a la administracin de O
2
al 100%, se puede diferenciar entre los meca-
nismos bsicos de hipoxemia.
Cul es la capacidad de difusin del CO
2
(DLCO
2
) normal?
25 ml/min/mm Hg.
Cul es la PaO
2
que alcanza una persona normal al recibir una fraccin
inspirada de oxgeno FIO
2
al 100%?
Hasta 670 mmHg.
Cules son los mecanismos bsicos de la hipoxemia?
Disminucin de la PIO
2
: PaO
2
disminuida, PaCO
2
normal o dismi-
nuida
Hipoventilacin: PIO
2
disminuida, PaCO
2
aumentada.
228
Anormalidad en la difusin: PaO
2
disminuida, PaCO
2
normal o dis-
minuida, gradiente A-a aumentada, con respuesta al O
2
al 100%.
Incompatibilidad V/Q: PaO
2
disminuida, PaCO
2
normal o disminuida,
gradiente alvolo-arterial aumentada, con respuesta al O
2
al 100%.
Derivacin: PaO
2
disminuida, PaCO
2
normal o disminuida, gradien-
te alvolo-arterial aumentada, sin respuesta al O
2
al 100%.
Cules son las indicaciones de apoyo ventilatorio?
Frecuencia respiratoria mayor de 35x, PaCO
2
mayor de 55 y PaO
2
me-
nor de 70.
Cul es el nivel de hemoglobina insaturada con el que clnicamente se
presta la cianosis?
Ms de 5g/dl.
Cmo se calcula si es correcta la relacin aporte consumo de O
2
?
Midiendo la saturacin de oxgeno de la sangre venosa mezclada.
Qu es la capacidad vital?
Es la suma de la reserva inspiratoria ms la reserva espiratoria y el volu-
men corriente.
Qu tipos de espacio muerto existen?
El espacio muerto anatmico es el aire de las vas areas altas que no
llega a los alveolos; aproximadamente slo los primeros 350 ml de
cada 500 ml de aire inspirado comprenden la ventilacin alvolar.
El espacio muerto total es la suma del espacio muerto anatmico ms
el fisiolgico. El espacio muerto fisiolgico comprende todos aquellos
estados en los que la ventilacin no se encuentra equilibrada con el
flujo sanguneo (alvolos no perfundidos).
En condiciones normales ambos espacios muertos son equivalentes. Esta
equivalencia se pierde cuando aparecen estados patolgicos como los al-
volos sobreventilados o hipopoerfundidos.
Cmo se calcula el espacio muerto anatmico?
Mediante curvas de inhalacin de nitrgeno, o bien, calculando la dife-
rencia entre la ventilacin total minuto y la ventilacin alveolar minuto.
Cmo puede calcularse el espacio muerto total?
Mediante la ecuacin de Bohr, la cual indica que el volumen ventilatorio
(V
T
) multiplicado por la PCO
2
del gas espirado equivale a la PCO
2
arte-
229
rial multiplicada por la diferencia entre el volumen ventilatorio y el espa-
cio muerto (V
D
) ms la PCO
2
del aire inspirado, ms la PCO
2
del aire es-
pirado, multiplicado por el V
D;
es decir, P
E
CO
2
x VT= PaCO
2
x (V
T
- V
D
)
+ P
I
CO2 x V
D.

En la ecuacin el resultado de P
I
CO
2
+ V
D
es despreciable, por tanto,
puede ignorarse.
Cules son los signos de hipoxemia intensa?
Taquicardia, taquipnea, confusin, ansiedad, cianosis, hipertensin,
arritmias y temblores.
Cules son los signos de hipercapnia?
Disnea, cefalea, papiledema y asterixis.
Cmo es la relacin FEV1/FVC (volumen espiratorio forzado del primer
segundo/capacidad vital forzada) en la obstruccin y la restriccin?
En la primera menos de 70% en la segunda ms de 70%.
Cules son los datos de la espirometra en los casos de obstruccin de las
vas areas superiores?
Disminucin del volumen tanto a la inspiracin como a la espiracin.
Cul es el indicador principal de supervivencia cuando existe enfermedad
pulmonar?
El volumen espiratorio forzado del primer segundo (FEV1).
Para qu es til la determinacin de la capacidad pulmonar total (TLC) y
de la capacidad vital (VC)?
Para detectar patrn restrictivo.
Cul es el diagnstico diferencial del neumotrax espontneo?
El infarto agudo del miocardio, la tromboembolia pulmonar, la neumo-
na, la EPOC y el asma.
Cules son los cinco tipos de derrame pleural?
Empiema, hemotrax, quilotrax, trasudado y exudado.
Cules son los tres mecanismos fisiopatolgicos del exudado?
El aumento de la presin hidrosttica, la disminucin de la presin onc-
tica y el aumento de la presin negativa intrapleural.

230
Con que datos se clasifica al derrame pleural como exudado?
Variable Total empiema Relacin derrame/suero Observaciones
Protenas Ms de 2.9 0.5
DHL Ms de 250 0.6
Colesterol Ms de 45 0.3
Glucosa Ms de 60 1 Se encuentra en todos
Glucosa Menos de 60 0.5 TB, malignidad, AR, LES
Amilasa Ms de 2 LES, pancreatitis
Amilasa Menos de 1 Todas
Si los valores son menores de stos, se trata de un trasudado

Qu sndrome corresponde a la neumona lobar?
El sndrome de condensacin pulmonar.
Qu sndrome corresponde a la neumona intersticial o bronquiolitis?
El sndrome de rarefaccin pulmonar.
Qu sndrome corresponde a la bronconeumona o neumona lobulillar?
Los estertores difusos, que no forman un sndrome.
Cules son los agentes etiolgicos de la neumona en los pacientes mayo-
res de 60 aos inmunocompetentes?
S. pneumoniae, virus, H. influenzae, aerobios Gram negativos y S. aureus.
Cules son las neumonas por grupo de edad, su agente etiolgico y su
tratamiento?
Neumona/Edad Etiologa Tratamiento
Neonatal
Virus: citomegalovirus y herpes
simple
Amplicilina + gentamicina
o vanco + gentamicina
(penicilinorresistente)
Bacterias: EBHGA coliformes
Otras: chlamydia
1-3 meses
Chlamydia y virus sincicial respira-
torio
Eritomicina 10 mg/kg
1-24 meses
Virus sincicial respiratorio, S.
Pneumoniae, H. influenzae, Chlamy-
dia, Mycoplasma
Cefuroxima 50 mg/kg IV
3 meses - 5 aos Los mismos, sin Haemophilus Eritomicina, claritromicina
5-18 aos Micoplasma y virus Claritromicina, azitromi-
cina
231
Viral en adultos Influenza, parainfluenza, Virus
sincicial respiratorio
Zanamivir 10 mg inhala-
do, amantadina 100 mg
Adultos mayores
de 8 aos, Adquie-
ra en la comuni-
dad
Sin comorbilidad: micoplasma,
chlamydia, viral raramente S.
Pneumonias
Azitromicina 500 mg
claritromicina 500 mg
eritromicina 1 gr
fluoroquinolonas
Fumador: Spneumonias, H. in-
fluenza, Moraxella
Alcohlico: S pneumoniae, anae-
robios, coliformes
Aspiracin: anaerobios
Drogas IV: aureus
Pjaros: psitacosis
Conejos: tularemia
Hospitalaria no
neutropnica
Falla orgnica: coliformes Para cualquier coliformes,
S. Pneumoniae y anaero-
bois: imipenem/cilastatina
500 mg o meropenem 1 g o
piperacilina/tazobactam +
ciprofluoxacino
Ventilacin mecnica: coliformes,
pseudomonas, S. Aureus
Obstruccin: anaerobios
Esteroides crnicos: cndida
Historia neutrop-
nica
Cualquiera de las adquiridas en el
hospital y aspergillus o cndida
Seguir los criterios para
neutropenia y fiebre

Cul es el agente etiolgico ms frecuente de la neumona en los pacientes
neutropnicos?
Lysteria monocitogenes.
Cules son les tres agentes de la neumona lobar?
S. pneumoniae, Klebsiella pneumoniae, H. influenzae.
Cul es el tratamiento de la neumona por anaerobios?
Penicilina sdica cristalina: 1 a 2 millones de U. Cada 6 horas.
Clindamicina: 600 miligramos IV cada 6 horas hasta mejorar, y luego
300mg.
Cules son los datos clnicos de la neumona intersticial?
Crepitacin, patrn obstructivo (FEV1/FVC menor de 70%) e imagen de
vidrio despulido.
232
Cules son los criterios de mal pronstico en los pacientes que padecen
neumona?
Edad mayor de 65 aos, ms de 30,000 leucocitos, hipotensin, tempera-
tura mayor de 38.3 C y que est casado por estafilococo, neumococo
Legionella.
Cules son los agentes causales de la bronquiolitis obliterante (BOOP)?
El virus sincicial respiratorio y Mycoplasma pneumoniae.
Cul es el pH del empiema?
Menor a 7.4
Cul es la epidemiologa del absceso pulmonar?
Ocurre principalmente sujetos propensos a la aspiracin (cualquiera que
curse con estado de conciencia alterado).
Cul es la imagen radiolgica del absceso pulmonar?
La cavidad solitaria de pared gruesa con consolidacin perifrica y nivel
hidroareo.
Cules son los segmentos pulmonares ms afectados por el absceso pul-
monar y la neumona por aspiracin?
Los segmentos posteriores de los lbulos superiores y el superior y basal
de los inferiores.
Cules son las enfermedades cavitarias pulmonares?
La granulomatosis de Wegener, la tuberculosis, la micosis, el cncer y el
infarto pulmonar.
Cul es la imagen radiolgica de la neumona necrosante?
Mltiples cavidades dentro de un rea de condensacin.
Qu drogas causan fibrosis pulmonar?
El busulfn, la bleomicina, la nitrofurantona, la ciclofosfamida, las sales
de oro y el metotrexate.
Qu otros nombres recibe la neumonitis por hipersensibilidad?
Alveolitis alrgica extrnseca o respuesta pulmonar al polvo orgnico. Es
una reaccin de hipersensibilidad tipo III.
A qu se le llama neumona eosinoflica (Sndrome de Leffler)?
Es una neumona por hipersensibilidad en la que hay un infiltrado eo-
sinoflico pulmonar y eosinofilia perifrica. Los agentes casuales pueden
233
ser parsitos (filarias, nematelmintos), frmacos (hidralazina, nitrofuran-
tona) y hongos (Aspergillus), aunque la mayora son idiopticas.
Cules son los cambios histopatolgicos producidos por el asma?
Denudacin del epitelio, depsitos de colgena bajo la membrana basal,
edema de las vas respiratorias e hipertrofia del msculo y de las glndu-
las bronquiales.
Cules son dos signos de obstruccin respiratoria en los casos de asma?
La sibilancias y la fase espiratoria prolongada.
Cul es el asma de los nios y de los adultos?
El asma alrgico y el intrnseco, respectivamente.
Cules son los signos de laboratorio en los casos de asma?
Alcalosis respiratoria y aumento de la A-a DO
2.

Qu se observa en el anlisis del esputo de un asmtico?
Espirales de Crushman y cristales de Leyden.
En qu se debe pensar un paciente joven tiene signos y sntomas de asma
despus de la actividad fsica?
Se debe sospechar que el paciente tiene asma inducida por el ejercicio. El
diagnstico se hace con una prueba teraputica positiva con salbutamol.
Cul es el tratamiento de eleccin para el asma inducida por el ejercicio?
90% de los pacientes responde a los agonistas beta-2 inhalados. Una se-
gunda opcin es la teraputica combinada de cromoln sdico y bromuro
de ipratropio.
Cul es el flujo espiratorio medio (FEM) en una exacerbacin moderada
de asma?
50 a 80.
Qu frmacos aceleran la depuracin de teofilina?
La rifampicina, la fenitona, los barbitricos y la nicotina. Todos aumen-
tan la actividad del citocromo P-450; por tanto, aceleran la depuracin de
cualquier frmaco cuya va de eliminacin sea principalmente la heptica.
Qu frmacos disminuyen la depuracin de teofilina?
La cimetidina, los macrlidos, las quinolonas y los anticonceptivos ora-
les. Todos disminuyen la actividad del citocromo P-450.
234
Cmo acta la teofilina?
Inhibe la accin de la adenosina que se libera como segundo mensajero
para causar broncoconstriccin. Slo se usa en el tratamiento crnico, ya
que fcilmente causa convulsiones y arritmias.
En quines se contraindica la teofilina?
En pacientes con prostatismo, ya que puede causar un efecto alfa que
aumente los sntomas.
Cul es la dosis de salbutamol para nebulizar?
2.5 mg
Cul es la dosis de prednisona para tratar los casos de exacerbacin asm-
tica?
0.5-1 mg/kg/da
Qu frmacos son mucolticos?
La acetilcistena, la alfa-dornasa y el yoduro potsico.
Qu son los leucotrienos?
Son los productos de la va de la 5-lipooxigenasa del metabolismo del ci-
do araquidnico. La 5-lipooxigenasa se encuentra en todas las clulas de la
serie mieloide y es un potente quimiotctico de neutrfilos y eosinfilos.
Cules son los efectos de los leucotrienos en el rbol bronquial?
Promueven la constriccin bronquial, aumentan la permeabilidad endo-
telial y promueven la secrecin de moco.
Cules son los inhibidores de los leucotrienos?
Zafirlukast y montelukast.
Qu es el zileutn?
Es un bloqueador de la 5-lipooxigenasa.
Cundo se usan el zileutn y el montelukast?
Slo se usan para la profilaxis del asma; no tienen utilidad en el ataque
agudo. Con ellos logra una disminucin marcada en el uso de beta ago-
nistas, disminucin de los sntomas y aumento del flujo espiratorio pico
matutino.
Cundo est presente el EPOC Tipo A?
Es un paciente disneico, soplador rosado, enfisematoso, menor de 60
aos, con CO
2
bajos.
235
Qu es el EPOC Tipo B?
En un paciente tosedor, azul abotagado, bronqutico crnico, menor de
60 aos, con O
2
muy bajo y CO
2
lato.
Cmo se define clnicamente la bronquitis crnica?
Es una neumopata con eventos de tos productiva durante 2 a 3 meses,
cuando menos, en dos aos consecutivos.
Cules son dos signos radiolgicos patognomnicos de enfisema pulmo-
nar?
Bulas parenquimatosas y vejigas subpleurales.
En quin se debe sospechar la presencia de fibrosis qustica?
En pacientes jvenes con EPOC, bronquitis crnica, insuficiencia pan-
cretica (o pancreatitis crnica) e infertilidad.
Cules son los datos de la espirometra en los pacientes que padecen
EPOC?
Hay un aumento del volumen residual y un volumen pulmonar total
normal o elevado.
Con qu parmetro se diagnostica EPOC grave?
Con un FEV1 menor de 1L.
Qu es la fibrosis qustica?
Tambin conocida como mucoviscidosis, es un trastorno que afecta al
brazo largo del C17 (banda q31). Es la mas grave de las enfermedades
genticas mortales que afecta a las glndulas exocrinas principalmente
del pncreas y del aparato respiratorio, y se presenta con la trada:
EPOC, insuficiencia pancretica exocrina y cifras anormalmente altas de
electrolitos en el sudor.
Qu sugiere un predominio de linfocitos T en el lavado bronquioalveolar?
Sarcoidosis o neumonitis por hipersensibilidad.
Qu se encuentra en un lavado bronquioalveolar en los casos de fibrosis
pulmonar?
Predominio de neutrfilos, eosinfilos y macrfagos.
Cul es la imagen radiolgica de la enfermedad intersticial pulmonar?
Infiltrados en vidrio molido, retculonodulares y en panal de abejas.
236
Cules son los hallazgos en la espirometra de quienes padecen enferme-
dades restrictivas?
Disminucin del volumen residual y disminucin del volumen pulmonar
total.
Qu es la sarcoidosis?
Es una enfemedad granulomatosa, no caseificante, sistmica, con infil-
trado pulmonar, linfadenopata mediastinal, lesiones cutneas, lesiones
oculares, hipercalciuria, hipercalcemia, parlisis de Bell, eritema nodoso,
uvetis y arritmias.
Cmo se diagnostica la sarcoidosis?
Por biopsia de las lesiones o biopsia transbronquial.
Qu es la granulomatosis de Wegener?
Es una enfermedad que cursa con necrosis de pequeos vasos y con gra-
nulomas no caseificantes alrededor, que afecta ms a las vas respiratorias
superiores y que cursa tambin con vasculitis de pequeos vasos.
Qu se usa para hacer el diagnstico de granulomatosis de Wegener?
c-ANCA; tiene una especificidad de 90%.
Qu es el sndrome de Churg-Strauss?
Es una vasculitis idioptica de arterias de pequeo y mediano calibre,
que ocurre en pacientes con asma.
Cul es el tratamiento de la granulomatosis de Wegener y del sndrome de
Churg-Strauss?
1 mg/kg de prednisona ms 2 mg/kg de ciclofosfamida.
Qu es el sndrome de Goodpasture?
Es la enfermedad rin-pulmn causada por la presencia de anticuerpos
antimembrana basal. Causa depsitos lineales de anticuerpos IgG en los
glomrulos.
Con qu otro nombre se conoce a la histiocitosis X?
Granuloma eosinoflico o granulomatosis de clulas de Langerhans (del
pulmn y del hueso).
A qu corresponden las masas del mediastino superior?
Timoma, bocio extratorcico y enfermedad de Hodgkin.
A qu corresponden las masas del mediastino anterior?
Quiste dermoide, quiste pleuropericrdico, quiste broncognico y redu-
plicacin esofgica.
237
A qu corresponden las masas del mediastino posterior?
Neurofibroma y feocromocitoma (con sndrome de Horner).
Qu es la osteoartropata pulmonar hipertrfica y con qu se le relaciona?
Es un sndrome con artralgias de las mano, los tobillos, las muecas y las
rodillas asociado a mesotelioma. Hay que diferenciarlo de la osteroartropa-
ta hipertrfica que se encuentra en la EPOC (dedos en palillo de tambor).
Cules son los factores de riesgo relacionados con la tromboembolia pul-
monar?
Estasis venosa, estados de hipercoagulabilidad, obesidad y fractura re-
ciente de la cadera.
Cul es la teraputica profilctica para pacientes en riesgo de sufrir TEP?
5,000 U de heparina no fraccionada cada 8 horas. Se busca idealmente
lograr un tiempo de tromboplastina parcial de 31.5 a 36.
Cul es el cuadro clnico de la tromboembolia pulmonar?
Trax rgido, dolor pleurtico, hemoptisis, broncoespasmo, friccin pleu-
ral, ritmo de galope, desdoblamiento fijo del segundo ruido y cianosis.
Cul es la imagen tpica radiolgica en los casos de TEP?
Arteria pulmonar abultada y segmento oligohmico.
A qu se llama joroba de Hampton?
10% de los casos de tromboembolia pulmonar evoluciona a infarto pul-
monar que cursa con hemoptisis, dolor pleurtico y fiebre de bajo grado.
La joroba de Hampton es un signo radiogrfico que sugiere infarto pul-
monar, y consiste en una consolidacin densa y homognea en forma de
cua, de base orientada hacia la pleura y punta en forma convexa orien-
tada hacia el hilio, lo que le confiere una forma de joroba.
Qu se observa en el electrocardiograma practicado en pacientes con TEP
(patrn de esfuerzo del ventrculo derecho)?
Eje a la derecha.
S anormal en D-I.
Q anormal en D-III.
P pulmonar.
Bloqueo de la rama derecha del Haz de His.
T invertida en V-1 y V-3.
238
Qu cambios gasomtricos se observan en los pacientes con TEP?
La TEP causa hipoventilacin, por lo que inicialmente la PaCO
2
est
disminuida y el pH se eleva de manera compensatoria provocando alca-
losis respiratoria aguda. Tambin existe hipoxia (PaO
2
menor de 80%) y
una diferencia alvolo-arterial aumentada.
Qu es el dmero-D?
Es un producto de la degradacin de la fibrina que se utiliza para apoyar
el diagnstico de TEP cuando es mayor a 500 ng/ml.
Qu condiciones predisponen a la tromboembolia pulmonar?
Ciruga abdominal baja, ciruga del territorio leofemoral, y en pacientes
con infarto del miocardio, enfermedad vascular cerebral, cncer o con fa-
lla cardiaca, postrados por tiempo prolongado.
Cul es la imagen tpica del infarto pulmonar?
Imagen en cua.
Cul es la presin normal de la circulacin pulmonar?
25/8 mmHg
Cul es el estmulo ms importante para la vasoconstriccin pulmonar?
La hipoxia.
Cules son los signos de hipertensin pulmonar?
Desdoblamiento del segundo ruido, reforzamiento pulmonar y chasquido
sistlico de expulsin (P2 fijo).
Cul es el diagnstico diferencial de la hipertensin pulmonar?
Hipertensin pulmonar poscapilar, cortocircuito intracardiaco, TEP y ci-
rrosis heptica (por el secuestro de lquidos).
Cul es el tratamiento de la hipertensin pulmonar primaria?
Nifedipino, diltiacem y prostaciclina.
Qu es el SIRPA?
Es el sndrome de insuficiencia respiratoria progresiva del adulto. Presen-
ta inicio sbito de disnea, hipoxemia e infiltrado en alas de mariposa
(con PCP <18) 12 a 18 horas despus del evento desencadenante.
Cules son los factores de riesgo relacionados con el SIRPA?
Sepsis, contusin pulmonar, choque, pseudoahogamiento y mltiples
transfusiones.
239
Cules son las alteraciones gasomtricas de los pacientes con hipoventila-
cin crnica?
La obesidad mrbida es una causa de hipoventilacin crnica. Se observa
PaCO
2
aumentada con pH normal.
Qu es el sndrome de Pickwick?
Es el sndrome en el que hay obesidad mrbida con hipoventilacin y
cambios gasomtricos acordes. Tambin hay somnolencia, apetito exce-
sivo y policitemia.
Cul es la terapia mdica para la fibrosis pulmonar?
D-penicilamina o ciclofosfamida ms prednisona.

240
241
NEUROLOGA







Dnde se encuentra los estriosomas?
En los ganglios basales.
Dnde se encuentran las neuronas piramidales?
En la corteza y en el hipocampo.
Dnde se encuentra las clulas de Renshaw?
En la sustancia gris.
Cules son las capas del cerebelo?
Piamadre, molecular, de Purkinje y granulosa.
Cmo se diferencia una disfuncin parasimptica de un estado hipersim-
ptico?
En los dos hay taquipnea, taquicardia y midriasis. El estado hipersimp-
tico se caracteriza por diaforesis marcada, a diferencia de la disfuncin
parasimptica.
Cmo se compone la va aferente del reflejo pupilar?
Comienza en las clulas ganglionares de la retina, sigue el curso del ner-
vio ptico hasta el ncleo pretectorial del mesencfalo; se decuza en la
comisura posterior y llega al ncleo de Edinger-Westphal.
Cul ncleo media la constriccin pupilar?
El ncleo de Edinger-Westphal (ncleo parasimptico preganglionar del
tercer par).
Cul es la va de la dilatacin pupilar?
Est compuesta por tres neuronas.
La de primer orden sale del hipotlamo, discurre por la porcin cer-
vical de la mdula y hace sinapsis preganglionar ipsilateral a nivel de
T1-T2.
242
La de segundo orden parte de T1-T2 siguiendo el tronco simptico
hasta el ganglio cervical superior.
La neurona de tercer orden sigue el trayecto de la cartida para llegar
finalmente a la rbita e inervar el msculo liso del iris.
Qu es el sndrome de Horner?
Es un sndrome provocado por la interrupcin de la va simptica del ojo
en cualquiera de sus puntos, lo que produce ptosis, miosis y anhidrosis.
Qu signo clnico provoca el dao al ncleo de Edinger Westphal?
Pupilas grandes no reactivas. Generalmente traduce una lesin pontina
de mal pronstico.
Qu signo clnico provoca el dao en la va que va del ojo al hipotlamo?
Provoca pupilas pequeas reactivas.
Qu signo provoca el dao al puente o al VI par?
Incapacidad para la abduccin del ojo.
Qu signos provoca el dao en el fascculo longitudinal medial o en el III
par?
Incapacidad para la aduccin del ojo y desviacin de la mirada hacia
afuera y abajo.
Cules son las vas somatosensoriales de la cara?
El ncleo trigminoespinal.
El ncleo ventral medial posterior del tlamo (ispsilateral).
La rama posterior de la cpsula interna (contralateral).
Cul es el ncleo propioceptivo de la cara?
El sensorial principal del trigmino.
Cul es el ncleo de la sensacin tctil de la cara?
El mesenceflico del trigmino.
Cules tipos existen de parlisis facial?
La parlisis facial central puede ser unilateral o bilateral y respeta los
msculos inervados por la rama superior del facial (frente y orbicular
de los prpados), que tiene una inervacin sinrgica por ambos he-
misferios cerebrales.
La parlisis facial perifrica siempre es unilateral y total. Esto quiere
decir que afecta los msculos inervados tanto por la rama tmporofa-
cial como por la crvicofacial.
243
Qu es la va piramidal?
Es la va crticoespinal, que parte del rea cuatro de Broadman en la cor-
teza cerebral y pasa por la cpsula interna, los pednculos, el tallo cere-
bral y entra en la mdula donde es relevada por una neurona de segundo
orden, que es la que tiene contacto con el msculo que finalmente va a
inervar. En resumen, es la va motora voluntaria.
Cmo se integra el sndrome piramidal?
Prdida de la motilidad voluntaria.
Anomalas de la motilidad refleja, como la exageracin de los reflejos
profundos o clono; presencia de reflejos especiales, como el de Ba-
binski, y aparicin de movimientos sincinticos.
Contractura piramidal despus de algunos das; primero existe flaccidez.
Puede haber anomalas de la sensibilidad.
Qu se observa en el sndrome de neurona motora inferior?
Fasciculaciones y reflejos disminuidos.
Cules son los sndromes en los que se observa hemipleja?
Hemipleja cortical, faciobraquial, ipsilateral a la lesin.
Hemipleja capsular, contralateral total.
Hemipleja alterna superior, peduncular o sndrome de Weber
Hemipleja alterna media, protuberancial o sndrome de Millard-
Gubler
Hemipleja alterna inferior, bulbar lateral o sndrome de Wallemberg.
Qu es el sndrome de Weber?
Es un sndrome secundario a una lesin de la va crticoespinal a nivel de
los pednculos, que presenta alteracin del tercer par craneal ipsilateral a
la lesin (globo ocular dirigido hacia abajo y afuera, ptosis y midriasis) y
hemipleja cruzada de los miembros torcico y plvico.
Qu es el sndrome de Millard-Gubler?
Es un sndrome secundario a una lesin en la va crticoespinal a nivel de
la protuberancia, que causa parlisis facial total de tipo perifrico, ipsila-
teral a la lesin, y hemipleja del lado opuesto.
Cmo se presenta el sndrome de Wallemberg izquierdo?
Afecta el hipogloso izquierdo y provoca desviacin de la lengua hacia
la izquierda y parlisis del la izquierdo del velo del paladar.
244
Afecta la pirmide izquierda provocando hemipleja derecha.
Afecta al lemnisco medial izquierdo y provoca disminucin de la
propiocepcin y el tacto del lado derecho.
Afecta el ncleo vestibular y causa prdida del equilibrio, nistagmo y
vrtigo y trastornos auditivos.
Cmo se encuentra el ojo cuando hay parlisis del tercer par craneal (mo-
tor ocular comn?
Divergente y deprimido.
Qu se debe sospechar cuando hay parlisis del cuarto par?
Aneurisma de la comunicante posterior.
Cmo se comprueba la integridad del cuarto par?
Existe rotacin interna al intentar la mirada hacia abajo.
Cul es el cuadro clnico de la parlisis del cuarto par?
Diplopia vertical (al mirar hacia abajo) y desviacin del ojo hacia arriba.
Cul es el cuadro clnico de la parlisis del VI par (motor ocular externo)?
Estrabismo convergente sin abduccin.
A qu se le llama va extrapiramidal?
A la va motora de los ganglios basales.
A qu se le llama va piramidal?
A la va crticoespinal o neurona motora superior
Cules son los signos para evaluar la va piramidal?
Los signos de Babinsky, Openheimer, Scheafer y Bing.
Cules son los sntomas del tallo?
Diplopia, disartria y vrtigo.
Cul es el sntoma extrapiramidal ms comn?
La acatisia. Se caracteriza por imposibilidad para permanecer quieto; hay
movimientos y marcha constantes
Cules son las manifestaciones focales de una lesin frontal?
Hemiplejia progresiva, y convulsiones focales o generalizadas,
Alteraciones mentales (falta de atencin y prdida de la motivacin)
y marcha atxica si se afecta ms el cuerpo calloso.
Si la lesin se localiza en la base de los lbulos, puede haber anosmia.
Existe afasia de expresin si se afecta el hemisferio dominante.
245
Cules son las manifestaciones focales de una lesin parietal?
Se alteran la estereognosia y la discriminacin entre dos puntos de
manera contralateral a la lesin.
Es caracterstica la negacin de la enfermedad (anosognosia)
Cuando est afectado el hemisferio dominante aparecen agnosia de
los dedos, agrafia y alteraciones del habla.
Cules son las manifestaciones focales de una lesin en el lbulo temporal?
La lesin en el hemisferio no dominante es silente.
La lesin temporal profunda causa hemianopsia contralateral.
Puede haber crisis convulsivas con auras olfatorias y visuales complejas.
Los tumores superficiales causan afasia mixta.
Cules son las manifestaciones focales de una lesin en el lbulo occipital?
Defecto visual en el cuadrante contralateral o hemianopsia con pre-
servacin de la mcula.
Las crisis convulsivas van precedidas de auras centelleantes.
Cules son las manifestaciones focales de las lesiones subcorticales?
Cpsula interna: hemipleja contralateral.
Tlamo: deterioro sensitivo cutneo contralateral.
Ganglios basales: atetosis, temblores extraos o posturas distnica.
(parkinsonismo).
Cul es la diferencia entre una lesin del putamen y una del territorio de
la cartida?
Ambas cursan con hemiparesia y hemianestesia contralaterales. En las
lesiones del putamen hay disminucin del estado de conciencia, y en las
del territorio de la cartida hay afasia.
Qu es el sistema crticobulbar?
Es el sistema que relaciona las funciones voluntarias con las races de los
nervios craneales que se encuentran en el puente y el bulbo. La rama pos-
terior de la cpsula interna es parte de esta va.
Qu ocasiona una lesin extensa de la rama posterior de la cpsula inter-
na (va corticobulbar)?
Parlisis facial central contralateral, hemipleja espasmdica contralate-
ral, desviacin contralateral de la lengua, hipertona e hiperreflexia con-
tralateral.
246
Cul es el sndrome de la cartida primitiva?
Amaurosis, ambliopa, hemipleja y falta de puso maxilar o soplo a ese
nivel.
Qu signos se encuentran en el sndrome de la cerebral anterior?
Cuando se afecta la izquierda, hay afasia y apraxia.
Si se afecta a nivel del origen hay hemipleja de la pierna y pie opuestos.
La afecin a nivel de la arteria de Heubner causa parlisis facial, de la
lengua y del miembro superior opuesto.
Si se afecta a nivel distal de la arteria de Heubner se presenta el refle-
jo de prensin palmar.
Existe abulia o mutismo acintico.
Qu signos constituyen el sndrome de la cerebral media?
Parlisis cortical, que afecta la hemicara y el brazo contralaterales.
Afasia central (sndrome de Gertsman), que aparece si se afecta el
hemisferio dominante y cursa con agnosia de los dedos, confusin
izquierda-derecha, alexia y acalculia.
Hemipleja capsular, si ocurre a nivel de la cpsula interna. La hemi-
pleja es contralateral total.
Si se afecta la zona lentculoestriada hay hemipleja y hemianestesia y
no hay afasia ni hemianopsia.
Hemipleja braquial con afasia, si se afecta la zona silviana.
Qu es la afasia?
Es la dificultad para hablar, escribir y leer, es decir, el defecto o la prdi-
da de todas las funciones del lenguaje.
Qu es la afasia no fluida de Broca?
Es la afasia que ocurre por dao en el rea frontal del cerebro, en torno a
la cisura de Silvio, con habla telegrfica y comprensin intacta.
Qu es la afasia fluida de Wernicke?
Es la afasia que ocurre por dao a la parte posterior del cerebro, en torno
al lbulo temporal, con habla fluida pero sin comprensin de lo que se
dice o se escucha.
Qu se debe investigar en un paciente que sufre sncope (desvanecimiento)?
Si el desvanecimiento es de origen vestibular o de origen vascular
(vahdo o presncope).
247
Cuando es de origen hemodinmico se acompaa de una respuesta
autonmica como puede ser la palidez de tegumentos y la diaforesis
con piel fra.
Si es vestibular, hay que diferenciar si es central (acompaado por di-
plopia, disartria, disfagia y vrtigo, que son sntomas del tallo) o peri-
frico (slo se acompaa de tinnitus y prdida de la audicin), o bien,
si es de origen cardiaco.
Cmo se clasifican los TCE?
Leve (menor): no hay prdida de la conciencia, puede haber dolor de
cabeza, desmayo, vmito en una ocasin, no se encuentra focalizado,
hay dificultad en la concentracin. Se debe a una concusin y no hay
datos de hematoma en la TAC.
Moderado (intermedio): el paciente no se encuentra comatoso pero s en
estado confusional. Se debe a una contusin y hay hematoma en la
TAC. Puede haber prdida de memoria, abulia, focalizacin, confusin,
vmito repetido, diabetes inspida. Se trata con glucocorticoides.
Grave: el paciente se encuentra en estado comatoso, debe sospechar-
se lesin cervical asociada y hay que inmovilizar.
Qu es una concusin?
Es cuando un golpe causa movimiento del cerebro dentro del crneo.
Qu es una contusin?
Es la desaceleracin del cerebro contra el crneo, ya sea en el punto de
impacto o en el opuesto. Puede verse como una hiperdensidad heterog-
nea y hemorragia en la TAC.
Qu dficits neurolgicos se producen tras una fractura del esfenoides?
Se provoca seccin del nervio ptico con ceguera total del lado afectado
y pupilas no reactiva. El reflejo consensual est conservado.
Qu nervios se lesionan en la fractura de base de crneo?
I, II, III y IV.
Cul es el cuadro clnico del hematoma subdural y epidural?
Dolor de cabeza unilateral, pupila agrandada ipsilateral estupor o coma.
Aparece en minutos u horas. El hematoma epidural produce dficit neu-
rolgico y estado comatoso de manera rpida. Los signos pupilares y el
estupor son ms frecuentes en el subdural.
248
Cmo se presentan el hematoma subdural y el hematoma epidural en los
ancianos y en los alcohlicos?
De manera subaguda durante das o semanas con somnolencia, dolor de
cabeza, confusin y hemiparesia leve.
Cul es el comportamiento tpico del hematoma epidural?
Existe un intervalo lcido de horas o minutos antes de caer en coma.
Cul es la caracterstica tomogrfica de un hematoma epidural?
Que la hemorragia se observa en forma lenticular.
Qu es la presin de perfusin cerebral y cual es el valor normal?
Es la diferencia entre la presin sistmica y la intracerebral y debe ser <de
20 a 30.
A qu fractura se asocia el hematoma epidural?
A la del temporal.
Cul es la causa principal de hemorragia subaracnoidea?
El aneurisma saculado que generalmente se encuentra en la comunicante
anterior, en la cerebral media o en alguna de sus ramas.
Cul es la diferencia clnica entre la hemorragia subaracnoidea y la intra-
cerebral?
Las hemorragias parenquimatosas tienen signos de focalizacin im-
portantes; las subaracnoideas no.
La hemorragia intraparenquimatosa tiene como antecedente la hiper-
tensin y la subaracnoidea las malformaciones arteriovenosas.
Cules son los estmulos que provocan vasoconstriccin y vasodilatacin
cerebral?
El aumento de la presin artirial provoca vasoconstriccin cerebral,
mientras que el aumento de CO
2
y la disminucin de O
2
causan vasodila-
tacin.
Cules son las medidas para disminuir la presin intracraneana?
No se utiliza restriccin de lquidos; debe mantenerse la normocapnia, ya
que la hipercapnia (y la vasodilatacin subsecuente) aumenta la presin y
habr que evaluar la necesidad de practicar una ventriculostoma.
Qu es un sndrome de herniacin?
Es el que se provoca por el paso de tejido cerebral a travs de aberturas
intracranales fijas como la hoz del cerebelo.
249
Qu tipos existen de herniacin transtentorial?
Los signos de ambos se deben al que el tejido que protruye comprime es-
tructuras vitales del tallo.
Herniacin central: de manera bilateral, los lbulos temporales pro-
truyen a travs de la escotadura tentorial. Hay pupilas fijas en posi-
cin media, alteraciones de la respiracin, prdida de los reflejos
oculoenceflicos (rotacin de la cabeza) y oculovestibulares (estimu-
lacin calrica) y parlisis motora bilateral (rigidez de descerebracin
o flacidez).
Herniacin temporal: de manera asimtrica, el tejido de uno de los
lbulos temporales protruye. Existe parlisis precoz del tercer par
(pupila dilatada y fija, unilateral, y parlisis extraocular). En algunas
ocasiones puede haber paro cardiorrespiratorio brusco.
Cules son los signos clnicos cuando el tallo cerebral est intacto?
Respuesta pupilar normal.
Ojos de mueca.
Respuesta culovestibular normal.
Reflejo corneal normal.
Cmo se evala clnicamente una prueba con agua fra en el odo izquierdo?
Un paciente normal presenta movimientos oculares de componente rpi-
do hacia la derecha y componente lento hacia la izquierda
Qu se debe pensar si al instilar agua caliente en un odo no se obtiene
respuesta?
Se debe presumir una lesin en el VII par, o en el laberinto, causada por
traumatismo o por Schwanoma del acstico.
Cules son los criterios para hacer el diagnstico de muerte cerebral?
Pupila midritica no reactiva.
Ausencia de los reflejos del tallo.
Ausencia de automatismo respiratorio.
Electrocardiograma plano.
Ausencia de antecedente de hipotermia o intoxicacin.
250
En qu porcentaje se relaciona, los fenmenos trombtico cerebrales a
ataques isqumicos previos?
En 30 a 50 por ciento de los fenmenos trombticos hay historia de
TIAS.
Qu es un infarto lacunar?
Son lesiones pequeas, por lo general menores de cinco milmetros de
dimetro, que se presentan en la distribucin de las arterias penetrantes
cortas en ganglios basales, puente de Varolio, cerebelo, extremo anterior
de la cpsula interna y con menor frecuencia en la sustancia blanca pro-
funda del cerebro.
Cules son las caractersticas del accidente vascular cerebral hemorrgico?
Es sbito, con dficits focales, cefalea intensa, efecto de masa y alteracin
del estado mental.
Cules son las tres causas principales de muerte despus de un accidente
vascular cerebral?
Son, en orden de frecuencia: neumona, tromboembolia pulmonar y car-
diopata isqumica.
En cules casos de enfermedad vascular cerebral se indica la anticoagula-
cin?
En el accidente isqumico que est en evolucin.
Cul es la respuesta fisiolgica al aumento de la presin intracraneal ele-
vada?
Hipotensin arterial y bradicardia (respuesta de Cushing).
Cmo se integra el sndrome de hipertensin endocraneana?
Cefalea, vmito, anomalas oculares (parlisis abducens), papiledema y
sntomas vegetativos (bradicardia, hipotensin) y bradipsiquia.
Cundo son tiles los esteroides para tratar el edema cerebral?
Cuando el edema vasgeno como el de las neoplasias, pero no cuando es
citotxico como en los casos de enfermedad vascular cerebral.
Cul es el orden de aparicin de los sntomas de hipertensin intracra-
neana?
Alteracin del estado de conciencia, hiperventilacin y signos de hernia-
cin.
251
Cmo es el electrocardiograma durante las crisis de ausencia?
Se caracteriza por las espigas de 3Hz.
Cules son los efectos adversos de la fenitona?
Ataxia, mareo, nusea, hiperplasia gingival y alteraciones gastrointestinal.
Cul es el mecanismo de accin de la fenitona?
Bloquea los canales de sodio y calcio.
Cul es el mecanismo de accin de la carbamazepina?
Bloquea nicamente los canales de sodio.
Cules son los efectos adversos de la carbamazepina?
Ataxia, vrtigo, visin borrosa, depresin respiratoria y coma.
Para qu tipo de crisis convulsivas se utilizan la gabapentina y la lamotri-
gina?
Para tratar las crisis parciales, ya sean simples o complejas.
Cules son los frmacos de eleccin para tratar las crisis de ausencia?
El fenobarbital y la etosuccimida.
Cules son los nuevos frmacos para el tratamiento de las crisis parciales
y crisis con generalizacin secundaria?
Topiramato: bloquea los canales de sodio, aumenta la accin del ci-
do gamma-aminobutrico (GABA) en sus receptores y bloquea algu-
nos receptores de glutamato.
Vigabatrina: acta inhibiendo la GABA-transaminasa, por lo que
aumenta los niveles de cerebrales de GABA.
Cules son las teoras sobre el origen de la migraa?
La del vasoespasmo y la de la disfuncin trigminovascular.
Cul es la nica migraa con aura?
La de la arteria basilar.
Qu es la cefalea en racimos (cluster headache)?
Es una cefalea predominante que se presenta en varones. Hay dolor pe-
riorbitario intenso todos los das durante varias semanas que puede
acompaarse de congestin nasal ipsilateral, epfora, ojo rojo y sndrome
de Horner.
Cules son las formas de terapia para la migraa?
Para la migraa aguda: antiinflamatorios no esteroideos, ergotamina
o sumatriptn.
252
Como profilcticos de la migraa: amitriptilina, propranolol, vera-
pamilo o nifedipino.
Sobre qu receptores actan los medicamentos antimigraosos?
Los abortivos de la migraa actan sobre los 5HT tipo 1.
Los profilcticos de la migraa actan sobre los 5HT tipo 2.
Cul es el tratamiento de la cefalea por tensin?
Amitriptilina: 100 mg.
Qu neurotransmisores estn asociados a las siguientes enfermedades?
Parkinson, Alzheimer, esquizofrenia: dopamina.
Alzheimer: acetilcolina.
Depresin: serotonina.
Ansiedad: cido gamma-aminobutrico.
Qu es la demencia?
Es la disminucin de la funcin intelectual y cognitiva sin alteracin con-
comitante del estado de conciencia.
Cules son los dos tipos de demencia?
La demencia senil (Alzheimer) y la demencia multiinfarto.
Dnde ocurre el dao caracterstico en quienes padecen enfermedad de
Alzheimer?
En el hipocampo.
Qu pacientes tienen ms riesgo de padecer Alzheimer?
Los que expresan la protena APOE 4/4.
Cul es la caracterstica anatomopatolgica de la enfermedad de Alzhei-
mer?
Maraas neurofibrilares intracelulares y placas neurticas extracelulares.
Qu medicamentos se utilizan en los casos de enfermedad de Alzheimer?
La tacrina es el nico medicamento autorizado para el tratamiento. Para
disminuir la progresin de los sntomas se utiliza el IFN y el copolmero 1.
Dnde ocurre el dao caracterstico en la enfermedad de Parkinson?
En la sustancia nigra pars compacta y en el locus coeruleus. Son caractersti-
cos los cuerpos de Lewy.
Cules son las vas neurolgicas de los movimientos coreiformes?
La va indirecta o sistema paleoestriado, que est formada por el globo
plido y la sustancia nigra. Su funcin consiste en inhibir el tono
253
muscular y agilizar los movimientos. Una lesin en esta va provoca
un sndrome hipertnico-parkinsoniano.
La va directa o sistema neoestriado, que est compuesta por el pu-
tamen y el ncleo caudado. Su funcin consiste en aumentar el tono
muscular e inhibir el movimiento. Una lesin en este sistema provoca
un sndrome hipercintico atetsico.
Cul es la diferencia entre el temblor de origen cerebeloso y el que se ob-
serva en quienes padecen la enfermedad de Parkinson?
El temblor cerebeloso aumenta al intentar movimientos voluntarios; el
parkinsoniano disminuye.
Cul es el mecanismo de accin de la levodopa?
Es un precursor metablico de la dopamina. Ya que la enfermedad de
Parkinson resulta de una cantidad insuficiente de dopamina en regiones
especficas del cerebro, puede ayudar a aliviar los sntomas.
Cmo acta la carbidopa?
Acta potenciando la accin de la levodopa, ya que es un inhibidor de la
enzima dopa-descarboxilasa en los tejidos perifricos, por lo que levodopa
(que cruza fcilmente la barrera hematoenceflica) alcanza el sistema
nervioso central.
Cmo acta la bromocriptina?
Es un derivado de la ergotamina (alcaloide con accin vasoconstrictora);
es un agonista de los receptores de dopamina. Se usa en pacientes que no
responden a la levodopa.
Cmo acta la amantadina?
Tiene accin como antiparkinsoniano. Parece actuar aumentando la sn-
tesis, liberacin y recaptura de dopamina.
Cmo actan el pramipexol y el ropinirol?
Son agonistas dopa no ergotamnicos, por lo que no acentan el vasoes-
pasmo perifrico y pueden usarse solos o en combinacin con levodopa.
Qu es el tolcapone?
Es un derivado del nitrocatecol que representa una nueva clase de medi-
camento antiparkinsoniano. Su funcin es inhibir selectiva y reversible-
254
mente la accin de la catecol-O-metiltransferasa (COMT), lo que disminuye
la metilacin de la levodopa a 3-O-metildopa, aumenta la recaptura de
levodopa en el SNC y, finalmente, incrementa las concentraciones cere-
brales de dopamina.
Cmo se integra el sndrome cerebeloso?
En el sndrome cerebeloso se encuentran afectadas las tres funciones ce-
rebelares principales, que son la coordinacin del movimiento (causa asi-
nergia), la regulacin del tono muscular y el equilibrio.
Qu medicamentos tienen interaccin con la levodopa?
La piridoxina aumenta su metabolismo perifrico y junto con los inhibidores
de la mono-aminooxidasa causa arterial por exceso de catecolaminas.
Cules son los efectos adversos de la levodopa?
Anorexia, nusea, vmito y taquicardia (por estimulacin dopaminrgica).
Qu frmacos pueden causar parkinsonismo?
Las fenotiacinas y butirofenonas (neurolpticos).
Qu es el sndrome de Wernicke-Korsacoff?
Es un sndrome provocado por la deficiencia de tiamina (principalmente
por consumo de alcohol), que cursa con ataxia, confusin y oftamoplejia
(VI par bilateral).
Qu se observa en la resonancia magntica de un paciente con sndrome
de Wernicke-Korsacoff?
Reforzamiento de los cuerpos mamilares.
Cul es ms frecuente, el EVC hemorrgico o el trombtico?
El trombtico en 85% de los casos.
Cul es la lesin anatomopatolgica que se encuentra en los casos de la
esclerosis mltiple?
Inflamacin crnica, desmielinizacin y gliosis.
Cul es la caracterstica patolgica de la esclerosis mltiple?
Afecta la mielina y respeta el cilindroeje.
Cul es el principal agente causal de la meningitis infecciosa del adulto?
El neumococo.
Cul es la presentacin clnica de la meningitis bacteriana?
Fiebre, cefalea, meningismo y disfuncin de los pares IV, VI y VII.
255
Cmo se integra el sndrome menngeo?
Hay sndrome de hipertensin endocraneana y signos de irritacin de ra-
ces como rigidez de nuca y signos de Kernig y Brundzinski.
Cul es la base para el diagnstico de meningitis bacteriana?
El anlisis del lquido cafalorraqudeo (LCR).
Cules son las caractersticas del LCR en la meningitis bacteriana?
Pleocitosis neutroflica, hipoglucorraquia, protenas elevadas y aumento
de la presin de apertura.
Qu se debe pensar si el LCR es negativo a bacterias y existe monocitosis?
Se debe hacer diagnstico diferencial entre listeria, tuberculosis, infeccin
viral, sfilis y enfermedad de Lyme.
Cules son las contraindicaciones de la puncin lumbar?
Efecto de masa o hipertensin endocraneana.
Trombocitopenia (menos de 50,000).
Tiempo de protrombina mayor de 3 segundos.
Cul es la dosis de penicilina til para el tratamiento de la meningitis
neumoccica y meningoccica?
24,000,000 U cada 6 horas.
Cul es el tratamiento profilctico de la meningitis meningoccica?
Rifampicina: 600 mg c/12 horas durante 2 das.
Cul es el tratamiento de la meningitis del neonato y del mayor de 50
aos?
La combinacin ampicilina-cefotaxima.
Cul es el tratamiento de la meningitis meningoccica en el nio de 3-6
aos?
La combinacin vancomicina-cefotaxima.
Por qu no se da dexametasona los casos de meningitis meningoccica?
Porque reduce la penetrancia de la vancomicina al SNC.
Cul es el efecto de la vancomicina en la meningitis meningoccica?
Aumenta la penetrancia del antibitico; puede usarse para contrarrestar
el efecto de la dexametasona.
Cules son las tres causas de absceso cerebral?
Un foco contiguo, un foco distante o postrauma quirrgico.
256
En orden de frecuencia cules son las reas afectadas por el absceso cere-
bral?
Frontal, temporal, tmporoparietal, parietal, cerebelar y occipital.
Cul es la presentacin clnica del absceso cerebral?
Cefalea, desorientacin, convulsiones y fiebre.
Focalizacin.
Sndrome de hipertensin endocraneana.
Cul es el tratamiento emprico del el absceso?
Penicilina 20-24 millones IV cada 6 horas.
Cmo es la sintomatologa y el LCR en la meningitis viral?
Cuadro clnico: fiebre, cefalea y meningismo. LCR: normoglucemia, pro-
tenas ligeramente elevadas, pleocitosis linfoctica.
Qu es la leucoencefalopata multifocal progresiva?
Es un trastorno del sistema nervioso central que se observa en los pacien-
tes inmunodeficientes; afecta a los oligodendrocitos y es provocado por el
virus JC (VJC).
Cules son los sndromes neurocutneos?
Neurofibromatosis tipo 1 (enfermedad de Von Recklinghausen), neurofi-
bromatosis Tipo 2, esclerosis tuberosa (sndrome de Bourneville) y sn-
drome de Von Hippel Lindau.
Cul es el sndrome centromedular?
Existe prdida de las sensaciones con topografa en capa, debilidad
pronunciada en los brazos y debilidad leve en las piernas.
Cmo se clasifican las lesiones extramedulares?
En intradurales y extradurales. Las intradurales generalmente son malig-
nas y las extradurales benignas.
Qu causa la lesin de C-3 y C-5?
Cuadripleja ms prdida del automatismo respiratorio.
Cmo se presenta la lesin a nivel de C-4 y C-5?
Cuadripleja con automatismo respiratorio.
Cmo se presenta la lesin de C-5 a C-6?
El hombro se encuentra respetado; se pierden los reflejos bicipital y bra-
quiorradial.
257
Cul es la inervacin de las cuerdas vocales verdaderas?
El nervio larngeo inferior inerva al msculo cricoaritenoideo posterior,
que se encarga de dar movilidad a las cuerdas vocales verdaderas.
Cul es la presentacin clnica de una lesin del nervio torcico largo?
Se afecta el msculo serrato anterior y se produce la llamada escpula
alada.
Qu causa la lesin del nervio radial?
Afecta los extensores del brazo y antebrazo y causa una mueca pndula.
Cules son las funciones del nervio citico poplteo?
Su rama tibial anterior es responsable de la dorsiflexin del pie y su rama
musculocutnea inerva el compartimiento lateral de la pierna, que es res-
ponsable de la eversin del pie.
Con qu pruebas se evala la integridad de las races L-4 y L-5?
Caminar sobre los talones evala a L-4 y sobre las puntas, a L-5.
Cmo inicia la mielitis transversa?
Como un dolor en la espalda o el cuello seguido de dficit neurolgico,
con paraplejia sensoriomotora que se detecta fcilmente como un nivel
con pruebas de sensibilidad o movilidad habituales. Generalmente, en la
historia clnica se encuentra que fue precedida por una infeccin por vi-
rus o micoplasma.
Qu ramas forman el plexo sacro?
El tronco lumbosacro y S-1, S-2 y S-3.
Cul es el tumor ms comn de la mdula espinal?
El ependimoma.
Cules son los tumores extramedulares ms comunes?
El neurofibroma, el meningioma y los tumores metastsicos.
Cul es el tratamiento de la neuropata diabtica?
Los inhibidores de la aldosa-reductasa.
Cmo actan la actina y la miosina en la contraccin muscular?
Cuando se proporciona energa en forma de ATP a una molcula de
miosina, sta cambia su estructura plegando uno de sus extremos, que
est unido a la actina, con lo que la actina se mueve sobre la miosina.
Qu enfermedades se presentan con debilidad muscular proximal?
La miastenia gravis, el sndrome de Eaton-Lambert, la esclerosis lateral
amiotrfica, esclorosis mltiple y polimiositis.
258
Cmo se evala clnicamente la debilidad muscular proximal?
Si el paciente no puede realizar actividades como levantarse de la seden-
tacin sin ayuda o no puede levantar los brazos para peinarse, entonces
existe debilidad muscular proximal.
Cmo se evala la debilidad muscular distal?
Pidiendo al paciente que apriete con su mano los dedos del explorador.
Subjetivamente se valorar la presencia o no de debilidad. Una historia
de dejar caer las cosas frecuentemente de las manos tambin apoya el
diagnstico.
Cul es el diagnstico diferencial en los estados de debilidad muscular?
Hipotiroidismo: debilidad muscular y creatincinasa elevada.
Hipertiroidismo: debilidad muscular y creatincinasa normal.
Cushing: debilidad muscular distal y proximal y creatincinasa normal.
Enfermedades musculares inflamatorias: dermatomiositis y polimio-
sitis.
Enfermedades neurolgicas: miastenia gravis y esclerosis lateral
amiotrfica.
Bacteriana: borreliosis de Lyme.
Parasitaria: toxoplasmosis y triquinosis.
Medicamentosa: AZT, D-penicilamina, clorpropamida, clorproma-
zina, etctera.
Cul es la diferencia entre el sndrome de Guillain-Barr y el botulismo?
En ambos hay ausencia de reflejos sin cambios sensoriales.
En el primero, la parlisis es ascendente y hay elevacin de protenas
en el lquido cefalorraqudeo.
En el segundo, la parlisis es descendente y comienza afectando los
nervios craneales; el lquido cefalorraqudeo es normal.
Cul es el tratamiento del botulismo?
Medidas de apoyo para mejorar la debilidad muscular.
Cul es la diferencia entre polimialgia y polimiositis?
Las dos se presentan con dolor muscular, pero la primera no causa debi-
lidad muscular proximal.
259
Cul es el cuadro clnico de la miastenia gravis?
Dificultad para tragar, dificultad para hablar, visin doble y debilidad de
miembros.
Qu diferencia hay entre el sndrome miastnico de Eaton-Lambert (EL) y
la miastenia gravis?
En la MG hay anticuerpos antirreceptor de acetilcolina y en el EL el
anticuerpo es anticanal de calcio presinptico (cuya funcin consiste
en liberar calcio para que a su vez las vesculas de acetilcolina sean
liberadas).
En el primero, la fuerza de la contraccin aumenta o permanece igual
tras la neuroestimulacin repetida, y en el segundo disminuye.
El sndrome miastnico respeta los msculos oculares y extraoculares.
Con qu se relaciona el sndrome Eaton-Lambert?
Con el cncer pulmonar de clulas pequeas.
Qu es el sndrome de Miller-Fisher?
Es un cuadro clnico similar al del sndrome de Guillain-Barr con par-
lisis descendente que comienza afectando los nervios craneales (hay que
hacer diagnstico diferencial con botulismo).
Cul es el tratamiento para de la miastenia gravis (MG) y el sndrome de
Eaton-Lambert (EL)?
Para la MG se utiliza algn inhibidor de la colinesterasa, preferente-
mente la piridostigmina o mestinn.
Para el EL se utiliza la guanidina.
Cul es la diferencia de accin entre la fisostigmina y la neostigmina?
La fisostigmina s logra el acceso al SNC. Una ventaja es que gracias a
esta accin previene el delirio producido por los muscarnicos como la
escopolamina y metoclopramida.
Cmo se lleva a cabo el diagnstico de una miopata?
Cuando coexiste la elevacin de la creatincinasa con electromiografa su-
gestiva, elevada, EMG sugestiva, biopsia de msculo positiva y estudio
gentico positivo.
Cul es el gen afectado en la distrofia muscular de Duchenne?
El gen distrofina no detectable.
260
Cul es el gen afectado en la distrofia muscular de Becker?
El gen distrofina detectable.
Cul es el gen afectado en la distrofia miotnica?
Es un trastorno autosmico dominante en el que se afecta el gen mioto-
nin-proten-kinasa en el cromosoma 19.
Qu son las distrofias no miotnicas?
Son canalopatas que se presentan con relajacin muscular prolongada
despus de actividad muscular constante o de estimulacin mecnica.

261
NUTRICIN, METABOLISMO,
LQUIDOS Y ELECTROLITOS







A qu se le llama dieta de lquidos claros?
Es una dieta predominante en agua, con 500-1000 kcal de carbohidratos,
con electrolitos y sin fibra.
Cmo se calculan los requerimientos dietticos diarios segn el peso del
paciente?
En los pacientes que tienen peso adecuado se utiliza el peso real, y en los
pacientes obesos se utiliza el peso ideal.
Cules son los requerimientos diarios del paciente hospitalizado?
Kcal 2,000 y 60 g de protenas.
Cul es el marcador clnico de un aumento de protenas en la dieta?
El nitrgeno ureico en sangre (BUN) srico se eleva.
En quines est indicada la restriccin de protenas de la dieta?
En los hepatpatas y en los nefrpatas.
Cmo puede evaluarse clnicamente el peso del paciente?
Puede evaluarse como peso relativo, que se calcula como peso actual/peso
deseable x 100, o bien, como ndice de masa corporal (IMC), que se calcula
como peso/estatura en m
2
. El IMC normal es de 20 a 25 kg/m
2
.
Cul es la medicin antropomtrica ms utilizada para evaluar la compo-
sicin corporal?
La composicin corporal (reservas de grasa y musculoesqueltica) se eva-
la con mayor frecuencia midiendo el pliegue cutneo del trceps y la circun-
ferencia muscular de la parte media del brazo.
Cul es el estudio de laboratorio ms utilizado para evaluar la desnutricin?
La albmina srica.
262
Cules son las dos variantes clnicas de la desnutricin protico-calrica?
El kwashiorkor, que es causado por una deficiencia de protenas en pre-
sencia de energa adecuada y el marasmo, ocasionado por la deficiencia
protico-energtica combinada.
Cmo se define la obesidad?
Existe obesidad cuando el peso relativo es mayor a 120% y el ndice de
masa corporal es mayor a 27 kg/m
2
.
Cules son los tipos de obesidad de acuerdo con la distribucin de la grasa
corporal?
Existe la obesidad de la parte superior del cuerpo, en la que el exceso de grasa
se encuentra alrededor de la cintura y el costado, y la obesidad de la parte
inferior, en la que el tejido adiposo se acumula en los muslos y regiones
glteas. Los individuos con obesidad de la parte superior tienen un riesgo
mayor para la salud, y los que presentan un ndice cintura-cadera mayor de
1 en varones y menor de 0.85 en mujeres, tienen un riesgo incrementado
para padecer diabetes mellitus, enfermedades vascular cerebral, corona-
riopata y muerte temprana.
Qu es la leptina?
La leptina es una protena que tiene un receptor especfico en el hipot-
lamo, cuya funcin consiste en controlar el apetito. Un defecto en la pro-
duccin o reconocimiento de la leptina pueden ser causa de obesidad.
Cules son los frmacos aprobados para el tratamiento de la obesidad?
Los agentes anorexignicos y los inhibidores de la lipasa.
Cules son los frmacos anorexignicos aprobados?
La fentermina y la sibutramina son agentes similares a la anfetamina, que
actan inhibiendo la recaptura de serotonina, norepinefrina y dopamina
(en el caso de la sibutramina), o bien, aumentando la liberacin de do-
pamina (fentermina).
Cules son los efectos adversos de los anorexignicos?
Debido a que incrementan los niveles de aminas vasoactivas puede existir
boca seca, aumento de la presin arterial y de la frecuencia respiratoria.
En el caso de la sibutramina, no debe utilizarse en pacientes que estn ba-
jo tratamiento con inhibidores de la monoaminoexidasa (IMAO) (fluoxe-
263
tina), agonistas de la serotonina (sumatriptn), litio o dextrometorfn
pues sus acciones son similares y los efectos se ven potenciados.
Cmo actan los inhibidores de la lipasa?
El orlistat es el nico frmaco aprobado de este grupo. Acta inhibiendo
las lipasas gstrica y pancretica, disminuye la absorcin de grasas en
30%. Esta indicado solamente en pacientes con un IMC mayor de 30 o
en pacientes con IMC mayor de 27 pero con otros factores de riesgo co-
mo diabetes, dislipidemia o hipertensin.
Cules son las vitaminas hidrosolubles?
La tiamina (vitamina B-1), la rivoflavina (vitamina B-2), la biotina, la
niacina, la piridoxina (vitamina B-6), la cobalamina (vitamina B-12), el
cido flico y el cido ascrbico (vitamina C).
Qu es el beri-beri y cuntos tipos clnicos existen?
Es la deficiencia de tiamina (B-1) y existen la variedad hmeda y la seca.
Qu es el beri-beri hmedo?
El beri-beri con afeccin cardiovascular.
Por qu el beri beri hmedo es una cardiopata de alto gasto?
Porque provoca vasodilatacin perifrica, excesiva que obliga al incre-
mento del gasto cardiaco.
Qu es el beri-beri seco?
Es la presentacin neurolgica de la enfermedad. Tiene una presentacin
central (Wernicke-Korsakoff) y una perifrica (neuropata simtrica mo-
tora y sensitiva, con dolor).
Cul es la dosis de tiamina para el tratamiento del beri-beri?
50-100 mg/da hasta que haya signos de mejora o el olor de la orina in-
dique que se ha saturado de tiamina. Despus se reduce la dosis a 5-10
mg/da (que es la dosis para deficiencias leves).
En quines debe sospecharse beri-beri?
En los alcohlicos crnicos.
Cules son las reacciones que requieren cobalamina como cofactor?
La reaccin de conversin de homocistena a metionina y la de metilma-
lonil-coenzima A a succinil-coenzima A.
Cules son las acciones de la niacina en hiperlipidemias?
Aumenta las lipoprotenas de alta densidad y disminuye las lipoprotenas
de baja densidad y las de muy baja densidad.
264
Qu es la pelagra?
Es la deficiencia de niacina que cursa con dermatitis (mucosas secas,
obscuras y escamosas), diarrea y demencia (insomnio e irritabilidad, que
progresan hasta alucinaciones y psicosis).
Cules son los efectos adversos de la niacina?
Rubor de la piel, irritacin gstrica, aumento de enzimas hepticas, hi-
perglucemia y gota.
Cul es la funcin de la vitamina C?
La vitamina C es necesaria para formar colgena; si se carece de ella, se
produce osteopenia y falta de cierre de las heridas, as como hemorragia.
Qu es el escorbuto?
Es la deficiencia de cido ascrbico (vitamina C), que se manifiesta con
hemorragias perifoliculares y en astilla, hemartrosis y hemorragias subpe-
risticas.
De dnde se obtiene la biotina?
Es una vitamina sintetizada por los microorganismos intestinales.
Para qu sirve la biotina?
Es un intermediario en las reacciones de transferencia de CO
2
(carboxila-
cin). La ovidina, que es una protena encontrada en el huevo, puede in-
hibirla y causar intoxicacin.
Cmo acta la piridoxina?
La piridoxina y sus subproductos (piridoxal, piridoxamina y sus steres 5
fosfato) estn estrictamente implicados en el metabolismo intermedio. Es
de particular importancia el fosfato de piridoxal, que es la coenzima princi-
pal para el metabolismo de los aminocidos y la sntesis del hem.
A qu se debe principalmente la deficiencia de piridoxina?
Las causas principales son el alcoholismo y la accin de frmacos anta-
gonistas (isoniacida, anticonceptivos, cicloserina y penicilamina.)
Cules son los sntomas generales de las deficiencias de las vitaminas B?
Ulceras orales, queilitis, debilidad e irritabilidad. La deficiencia grave
causa neuropata perifrica, anemia y convulsiones.
Qu etiologa tiene la deficiencia de Vitamina A?
Ocurre en estados de malabsorcin de grasas y en lactantes.
265
Cules son las manifestaciones iniciales de la deficiencia de vitamina A?
Ceguera nocturna, placas blancas en la conjuntiva y xerosis.
Cules son las manifestaciones tardas de la deficiencia de vitamina A?
Queratomalacia, endoftalmitis y ceguera.
Cul es la dosis para el tratamiento de la deficiencia de vitamina A?
30,000 UI al da.
Cul es la accin de la vitamina D?
Aumentar la absorcin intestinal de calcio.
Cules son los tipos de raquitismo?
Nutricional, por falta de vitamina D. Se puede encontrar calcio srico
bajo, protenas bajas, hormona paratiroidea aumentada y fosfatasa
alcalina elevada.
Por metabolismo anormal de la vitamina D, como en la ausencia de
alfa1-hidroxilasa.
Hiperfosfatemia ligada a X.
Raquitismo de la prematurez.
Cul es el cuadro clnico del raquitismo?
Existe rosario raqutico, arqueamiento de las piernas, aumento del espa-
cio articular en las muecas y rodillas, craneotabes (crneo blando), fon-
tanelas abiertas y metfisis ocupadas o ensuciadas.
Cul es el tratamiento del raquitismo?
Se utilizan 5,000 a 10,000 U de vitamina D diariamente durante 2 a 3
meses.
Cul es la estructura qumica de las lipoprotenas?
Estn compuestas por una fraccin protenica llamada apolipoprotena o
apoprotena, que puede ser desde 1 hasta 60% del total de la molcula, se-
gn el tipo de lipoprotena y por diferentes cantidades de los tres lpidos
principales (triglicridos, colesterol y fosfolpidos).
A qu se le llama lipoprotenas beta, pre-beta y alfa?
Son los nombres alternativos para las lipoprotenas de baja densidad
(LDL), las lipoprotenas de muy baja densidad (VLDL) y las lipoprote-
nas de alta densidad (HDL), respectivamente.
Cul es la funcin de los quilomicrones?
Transportar los triglicridos desde su lugar de absorcin en el intestino
hasta el torrente circulatorio.
266
Cul es la funcin de las VLDL?
Transportar los triglicridos desde el hgado a la circulacin.
Qu sucede cuando los quilomicrones y las VLDL reaccionan con la lipo-
protenlipasa para liberar triglicridos?
Los quilomicrones se convierten en quilomicrones remanentes y las VLDL
se convierten en lipoprotenas remanentes de densidad intermedia (IDL).
Cul es la importancia de los triglicridos plasmticos?
Son los principales lpidos transportados en la sangre. Diariamente se
transportan entre 70 y 150 gramos de ellos mientras que se transportan
slo 1 o 2 gramos de colesterol o fosfolpidos.
Cul es la funcin de las HDL?
Las HDL remueven el colesterol de los quilomicrones, de las VLDL y de
las membranas celulares y lo llevan al hgado y a los macrfagos donde
lo entregan por medio de la apolipoprotenas AI.
Con qu valores de laboratorio se hace el diagnstico de dislipidemia?
Colesterol total menor de 240, HDL menor de 35.
En que lipoprotenas se encuentran las siguientes APO?
APO E: VLDL, LDL, HDL y IDL.
APO D: HDL.
APO B48: Quilomicrones.
Qu es la APO-CIII?
Es un inhibidor de la lipoproten-lipasa.
Cul es la hiperlipoproteinemia tipo I?
Tambin se conoce como hiperquilomicromemia familiar. El plasma presen-
ta una capa cremosa de sobrenadante despus de ser refrigerado. Clni-
camente presenta pancreatitis o xantomas eruptivos en zonas de presin
y lipemia retinalis. Es causada por deficiencia de LPL o de APO-CII. Se
trata solo con dieta baja en grasas.
Cul es la hiperlipoproteinemia tipo II-A?
Tambin es conocida como hipercolesterolemia familiar. Existen cifras ele-
vadas de LDL y cifras normales de VLDL; por tanto, el colesterol est
elevado y los triglicridos son normales. Las LDL no refractan la luz, as
que el plasma es traslcido. La causa es una disminucin en los recepto-
267
res de LDL. Los pacientes presentan aterosclerosis y enfermedad corona-
ria acelerada, xantomas tendinosos en el tendn de Aquiles, rotulianos y
de los extensores de los dedos. La variedad heterocigota se trata con co-
lestiramina y estatinas. Debe agregarse niacina al tratamiento de la varie-
dad homocigota.
Cul es la hiperlipoproteinemia II-B?
Tambin llamada hiperlipidemia familiar combinada o mixta. Cursa con ni-
veles altos de LDL y VLDL; por tanto, el colesterol y los triglicridos es-
tn elevados. La presentacin clnica es la misma que en la tipo II-A. La
causa es un aumento en la produccin heptica de VLDL. La teraputica
requiere triple esquema de frmacos: secuestradores de cidos biliares,
inhibidores 3-hidroxi-3metil-glutaril-coenzima A (HMG CoA).
Cul es la hiperlipoproteinemia tipo III?
Tambin conocida como disbetalipoproteinemia familiar. Se debe al acmu-
lo de IDL debido a la sobreproduccin o a la presencia de una apolipo-
protena (APO) E mutante. El plasma es turbio y con una ligera capa de
quilomicrones. Clnicamente presenta aterosclerosis de los vasos corona-
rios y perifricos y xantomas eruptivos, tendinosos y tuberosos. El trata-
miento incluye prdida de peso, dieta baja en colesterol, estatinas,
fibratos y niacina.
Cul es la hiperlipoproteinemia tipo IV?
Es la hipertrigliceridemia familiar en la que hay niveles altos de VLDL con
cifras normales de LDL, y se asocia con obesidad, intolerancia a la glu-
cosa o diabetes e hiperuricemia. Tambin se observa durante la terapia
estrognica, en el tercer trimestre del embarazo y en el alcoholismo. El
tratamiento incluye prdida de peso, restricciones dietticas que involu-
cran carbohidratos, grasa y alcohol. La terapia farmacolgica puede no
ser necesaria si se siguen las recomendaciones; cuando es necesaria, re-
quiere niacina, fibratos y estatinas.
Cul es la hiperlipoproteinemia tipo V?
Es la hipertrigliceridemia familiar mixta, en la que se encuentra elevados los
quilomicrones y las VLDL. Clnicamente hay xantomas eruptivos. Ocu-
rre en pacientes obesos y diabticos y el tratamiento es el mismo que en
los tipos III y IV.
268
Cules son las dislipidemias que pueden debutar con pancreatitis?
Las de tipo I , IV y V.
Cules son los efectos de la niacina en las dislipidemias?
Inhibe la sntesis de cidos grasos a partir de triglicridos en el tejido adi-
poso; por tanto, no se forman VLDL en el hgado y no se forman LDL
en la periferia. Son tiles cuando estn elevadas las VLDL y LDL con
niveles bajos de HDL baja.
Cules son los efectos adversos de la niacina?
Enrojecimiento de la piel (flushing) y prurito.
Cmo actan los fibratos?
Aumentan la actividad de las LPL sobre las VLDL y los quilomicrones
(transportadores de triglicridos y antgenos), incrementan el catabolismo
no esplcnico de las VLDL y probablemente aumentan las HDL; por tan-
to, se usan cuando estn elevadas las VLDL y LDL y las HDL, bajas.
Cules son los efectos de la niacina y los fibratos sobre las HDL?
Los dos provocan un aumento de las HDL (considerablemente ms la
primera).
Cul es un efecto adverso de los fibratos?
Pueden causar malignidad, miositis y miopata. Tambin potencian la
accin de los cumarnicos.
Cmo actan la colestiramina y el colestipol?
El hgado sintetiza cidos biliares a partir del colesterol y stos se reab-
sorben en el intestino. Estas resinas fijan el colesterol de las sales biliares
impidiendo su reabsorcin.
En qu patologa se utilizan la colestiramina y el colestipol?
Son de eleccin para las dislipidemias tipo II-A y II-B.
Cules son sus efectos adversos de la colestiramina y el colestipol?
Disminuyen la absorcin de las vitaminas liposolubles (ADEK), de la
niacina y del cido flico.
Cmo actan los inhibidores de la HMG-CoA reductasa (estatinas)?
Actan inhibiendo el paso de HMG a mevalonato y de ste a colesterol.
Adems de inhibir la HMG-reductasa, aumentan los receptores para
LDL e incrementan su metabolismo.
269
Cules son los efectos secundarios de las estatinas (a nivel bioqumico)?
Afectan a la coenzima Q, al hem y a las protenas de membrana.
Cul es el mecanismo comn de las estatinas y la colestiramina para dis-
minuir las HDL?
Aumentan la recaptura de LDL en el hgado, ya que incrementan la acti-
vidad de las APO-B y E.
Cules son las principales enfermedades por defecto en el metabolismo li-
sosomal?
Las enfermedades por atesoramiento se deben a defectos en la funcin de los
lisosomas que deben degradar las macromolculas a sus unidades bsicas
(aminocidos, monosacridos y cidos grasos). Las principales son las li-
pidosis, las gangliosidosis, las mucopolisacaridosis y las leucodistrofias.
Qu es la enfermedad de Fabry?
Es la lipidosis familiar por acmulo de glucoesfingolpidos transmitida de
manera ligada al sexo y causada por la deficiencia de alfa-galactosidasa.
Los depsitos de lpidos producen angioqueratomas en la piel.
Qu es la enfermedad de Gaucher?
Es la lipidosis familiar autosmica recesiva causada por acmulo de glu-
cocerebrsidos, y que se debe a una deficiencia de la beta-glucosidasa. Se
manifiesta por hepatoesplenomegalia, pigmentacin de la piel, lesiones
esquelticas y pingculas. Es la lipidosis ms observada en la clnica.
Qu es la enfermedad de Niemann-Pick?
Es la lipidosis familiar causada por acmulo de esfingomielina, y que se de-
be a la deficiencia de esfingomielinasa cida. Se divide en Tipo A (es la lipi-
dosis con muerte infantil) y la lipidosis infantil hepatopulmonar (Tipo B).
Qu es la enfermedad de Tay-Sachs?
Es la gangliosidosis neuronal tipo GM2 debida a deficiencia de hexo-
saminidasa, caracterizada por un comienzo muy temprano, retraso en el
desarrollo, parlisis, demencia y muerte a los 3 o 4 aos.
Qu es la enfermedad de Refsum?
Es una enfermedad debida a acmulo de cido fitnico, y que causa prin-
cipalmente sntomas neurolgicos como neuropata perifrica, ataxia ce-
rebelosa, retinitis pigmentaria; adems produce alteraciones en los huesos
y la piel.
270
Cules son las caractersticas clnicas de las mucopolisacaridosis?
Afeccin multiorgnica, hepatoesplenomegalia y anormalidades seas.
Qu es el sndrome de Hurler?
Es un cuadro clnico compatible con mucopolisacaridosis. Los datos
principales son: facies caracterstica, disminucin de la alfa-L-
iduronidasa y acumulacin de heparn-sulfato y dermatn-sulfato.
Cul es la enfermedad de San Filippo?
Es un cuadro clnico de mucopolisacaridosis, degeneracin grave del sis-
tema nervioso y afeccin multiorgnica leve.
Qu es el sndrome de Hunter?
Es una variedad del sndrome de Hurler, que aparece entre los 2 y los 4
aos.
Qu es el sndrome de Morquio?
Es una enfermedad que se presenta con cuadro clnico de mucopolisaca-
ridosis, enanismo, depsitos corneales y displasia espondiloepifisiaria.
Qu son las leucodistrofias?
Son desrdenes debidos a atesoramiento y que afectan la sustancia blan-
ca cerebral.
Qu es la enfermedad de Krabbe?
Es la leucodistrofia globoide secundaria a depsitos de galactosilcerebr-
sido debidos a la deficiencia de la enzima galactosilceramidasa.
Qu son las glucogenosis?
Son enfermedades causadas por atesoramiento de glucgeno, que pueden
tener una variedad muscular y otra heptica.
Qu son las porfirias?
Son un grupo de alteraciones en la biosntesis del hem. Pueden ser hepticas
o eritropoyticas, segn el lugar de expresin del defecto enzimtico. Las
principales caractersticas clnicas de las porfirias son la fotosensibilidad y
los sntomas neurolgicos como dolor abdominal, trastornos de la moti-
lidad intestinal, disestesias y parlisis muscular o respiratoria.
Qu es la porfiria aguda intermitente?
Es una enfermedad autosmica dominante que aparece por deficiencia
de la uroporfobilingeno sintetasa, cuyo sntoma inicial es un dolor ab-
271
dominal que simula ser quirrgico, mismo que se desencadena con ma-
yor frecuencia por el uso de substancias y que aumenten la actividad del
citocromo P450.
Cules son las reglas de THUMB?
Por cada aumento de 100 g de glucosa, el sodio disminuye 1.6mEq.
El potasio aumenta 0.6 mEq por cada 0.1 de disminucin del pH.
El calcio disminuye 0.8 mEeq por cada gramo que disminuye la al-
bmina.
Por cada aumento de 10 mEq de CO
2
, el pH disminuye 0.8 unidades.
Por cada 10 mEq de HCO
3
, el pH cambia 0.15 unidades directamente.
Qu propiedad del epitelio tubular permite que se lleve a cabo el transpor-
te de solutos?
La diferencia en la distribucin de las bombas y otras unidades a ambos
lados de la membrana es lo que hace posible el transporte de solutos en el
tbulo.
Qu clulas estn capacitadas para excretar hidrogeniones a la luz de una
cavidad?
Las clulas del tbulo contorneado distal, del tbulo proximal y del tbu-
lo colector, as como las clulas parietales gstricas.
De dnde proviene el hidrgeno que se secreta a la luz tubular?
Las clulas generan CO
2
como producto de desecho; la enzima anhidrasa
carbnica lo une al H
2
O para formar H
2
CO
3,
que fcilmente se disocia en
HCO
3
-
y H
+
.
Cul es el comportamiento del sodio dentro de la nefrona?
El sodio se filtra en grandes cantidades hacia la luz tubular, pero es
transportado activamente fuera de ella en todas las porciones exepto del
asa de Henle, en condiciones normales, se resorbe de 96 a 99%.
Cules son los diferentes tipos de hiponatremia?
Isosmolar (facticia): por hiperlipemia, hiperproteinemia e hiperglu-
cemia. H.
Hiposmolar: hipovolmica, euvolmica e hipervolmica.
Hiperosmolar: secundaria a manitol o hiperglucemia.
272
Cules son las causas de la hiponatremia hiposmolar?
Hipovolmica: cualquiera en la que se pierda sodio ya sea renal o ex-
trarrenal, lo que se evala midiendo el sodio urinario.
Euvolmica: sndrome de secrecin inadecuada de hormona antidiu-
rtica (SIADH), potomana y polidipsia psicgena.
Hipervolmica: insuficiencia cardiaca congestivo-venosa (ICCV) y
hepatopata.
Cules son las causas de hipernatremia?
Puede deberse a deshidratacin, en cuyo caso la osmolaridad urinaria
est aumentada (400 mOsm).
Puede deberse a diabetes inspida central o nefrognica, en la que el
paciente slo pierde agua y en la que la osmolaridad urinaria est
disminuda (menos de 250 mOsm).
Cules son las causas de la hiperkalemia?
Sangrado gastrointestinal, transfusin, acidosis y falla renal.
Cules son las causas de las acidosis hiperkalmica?
Prdida de HCO
3
por el rin (como en la acidosis tubular renal
(ATR) tipo 2).
Prdida gastrointestinal de HCO
3
por diarrea o por fstula del intes-
tino delgado.
Cules son las causas de la acidosis hipoclormica?
Prdida de cloro por vmito repetido.
Disminucin de la excrecin de HCO
3
debida al uso de esteroides y
diurticos.
A qu se deben la hiperkalemia y la hipocalcemia provocadas por transfu-
sin?
Entre ms viejo es un paquete globular, aumenta la posibilidad de hi-
perkalemia secundaria a la lisis celular que libera potasio del interior de
las clulas.
Los paquetes contienen citrato que funciona como quelante del cal-
cio, lo que sucede a partir de que se administran cinco paquetes glo-
bulares.
273
Qu alteraciones plasmticas genera la hipoproteinemia (disglobuli-
nemia)?
Aumento de colesterol y de sodio.
Qu alteraciones provoca la hipocolesterolemia?
Aumento de protenas y de sodio.
En qu segmento del tbulo se encuentra el sistema colector de cido?
En el contorneado proximal.
Cules son las alteraciones hidroelectrolticas de los siguientes padeci-
mientos?
Diabetes inspida: existe prdida de agua, por lo que los electrolitos se
elevan uniformemente.
SIADH: se acumula agua, por lo que los electrolitos decaen unifor-
memente.
Diabetes descompensada: el sodio desciende 1.4 por cada 100 mg de
glucosa aumentada y el CO
2
es bajo por acidosis leve (ligera compen-
sacin respiratoria).
Sndrome androgenital perdedor de sal: existe hiponatremia, hiperka-
lemia e hipocloremia.
Cules son los sndromes de Bartter y Gitelman?
Son sndromes que cursan con prdida de potasio por la va renal, alcalo-
sis metablica y poliuria. El sndrome de Bartter se debe a una falla de la
bomba NaKCl en la rama ascendente del asa de Henle y el de Gitelman
se debe a falla de la bomba NaCl en el tbulo contorneado distal.
Cmo se calcula el anion gap (brecha aninica)?
Na- (Cl+HCO
3
)
Cul es el mecanismo fisiopatolgico que explica la acidosis en la hiper-
cloremia?
El exceso de cloro srico provoca entrada de HCO
3
a la clula.
Cules son las causas de la acidosis hiperclormica con anion gap normal?
La prdida gastrointestinal de bicarbonato y la acidosis tubular renal.
Cules son los cuatro tipos de acidosis tubular renal (ATR)?
I. clsica: hipokalemia e hipercloremia secundarias a un defecto selectivo
de la secrecin de hidrgeno en la nefrona distal.
274
II. proximal: existe un defecto selectivo de la capacidad del tbulo pro-
ximal para reabsorber adecuadamente el HCO
3;
coexiste con otros defec-
tos de absorcin, por lo que puede haber aminoaciduria, fosfaturia y
glucosuria.
III. insuficiencia glomerular: por descenso de la velocidad de filtracin
glomerular se produce una falla en el sistema generador de NH
3;
existe
hipercloremia y normokalemia.
IV. hipoaldosteronismo hiporreninmico: existe hipercloremia e hiperka-
lemia.
Cmo est el anion gap en la acidosis lctica?
La acidosis lctica es una acidosis de anion gap elevado que se corrige f-
cilmente con la administracin de lquidos.
Cules son los cambios gasomtricos cuando existe acidosis respiratoria
aguda?
pH disminuido y CO
2
aumentado.
Cules son los cambios gasomtricos que se encuentran cuando existe hi-
poventilacin crnica?
La hipoventilacin crnica causa retencin de CO
2,
los que conduce a
acidosis respiratoria crnica con compensacin metablica.
Cules son los cambios gasomtricos que se observan en los casos de aci-
dosis metablica aguda?
pH normal-bajo y CO
2
en lmites bajos por la compensacin respiratoria.

275
OFTALOMOGA







Cul es la inervacin de los msculos del iris?
El sistema simptico inerva las fibras radiales que se encargan de la mi-
driasis, mientras que el parasimptico inerva el constrictor de la pupila.
Qu estructuras conforman la vea?
El iris, el cuerpo ciliar y la coroides.
Cul es la funcin del msculo constrictor del cuerpo ciliar?
Provoca la relajacin de las fibras de la znula y un aumento del dime-
tro nteroposterior del cristalino.
Cules son las capas de la retina?
Epitelio.
Conos y bastones.
Nuclear externa.
Plexiforme externa.
Nuclear interna.
Plexiforme interna.
Clulas ganglionares.
Fibras del nervio ptico.
Cul es el mecanismo retiniano que permite la visin nocturna?
El ingreso de sodio a las clulas retinianas, gracias al canal de cationes no
selectivo, permite ver durante la noche.
Cules son los tipos de orzuelo?
El que se encuentra en la superficie conjuntival.
El que se encuentra en el borde ciliar.
Cundo est indicado el drenaje del orzuelo?
Si en 48 horas no ha mejorado con compresas.
276
Qu es un chalazin?
Es una inflamacin granulomatosa del absceso estafiloccico (orzuelo).
Qu serotipos de chlamydia causan tracoma?
El A y el C.
Qu serotipos de chlamydia causan la conjuntivitis de inclusin?
El D y K.
Cul es la conjuntivitis viral ms frecuente y cul es su tratamiento.
Es la fiebre faringoconjuntival por adenovirus tipo 3; se administran go-
tas de sulfas y se aplican compresas para prevenir una complicacin bac-
teriana.
Cules son los agentes etiolgicos de la queratitis bacteriana?
Moraxella catarrhalis, neumococo y Pseudomonas.
Qu es una pingcula?
Es una mancha amarilla en la conjuntiva; lo ms frecuente es que se ubi-
que en el lado nasal.
Qu es un pterigin?
Es una carnosidad triangular con vrtice hacia el iris y que se ubica del
lado nasal.
Cul es el cuadro clnico de la queratitis?
Dolor, fotofobia, lagrimeo e inyeccin circuncorneal.
Cmo se observa la lcera corneal por herpes?
Se requiere fluorescena y es una lcera dendrtica.
Cul es el tratamiento de la lcera corneal por herpes?
Trifluouridina, idoxuridina o deoxuridina ms un ciclopljico.
Cules son las caractersticas de la uvetis anterior?
Hay fenmeno de Tindall, precipitados querticos y pupila pequea (por
sinequias posteriores).
Cul es la diferencia entre la uvetis anterior y la posterior?
La anterior es dolorosa; la posterior no lo es.
Cules son complicaciones del uso de esteroides oftlmicos?
Catarata, lcera corneal, queratitis y glaucoma.
Cules son los factores predisponentes al desprendimiento retiniano?
Ser mayor de 50 aos, antecedente de extraccin de catarata y miopa.
277
Cul es el signo patognomnico del desprendimiento retiniano?
El paciente describe que su visin ha sido tapada por una cortina.
Cules son causas de oclusin de la arteria central de la retina?
Embolia, trombosis de una arteria con aterosclerosis y arteritis temporal.
Cmo se presenta la oclusin de la arteria central de la retina (OACR)?
Ceguera unilateral sbita e indolora.
Cules son los hallazgos clnicos en los casos de OACR?
Reflejo fotomotor disminuido y consensual conservado, dilatacin pupi-
lar, el fondo de ojo plido y opaco con una fvea como mancha de color
rojo cereza y arterias atenuadas y exanges en vagn de ferrocarril.
Cul es el tratamiento de la OACR?
Disminucin de la presin ocular mediante masaje sobre los prpados y
aspirar CO
2
5-10% en O
2
para aliviar el espasmo retiniano.
Cules son las causas de oclusin de la vena central de la retina?
Ocurre principalmente en ancianos y en pacientes jvenes. La causa es
frecuentemente reumatolgica (arteritis de clulas gigantes). La diabetes,
la hipertensin y la viscosidad sangunea aumentada son factores predis-
ponentes.
Cul es el cuadro clnico de la oclusin de la vena central de la retina
(OVCR)?
Disminucin paulatina de la visin, unilateral e indolora; fondo de ojo
congestionado y edematoso, con venas tortuosas. Al cabo de semanas de
la obstruccin, la neovascularizacin causa rubeosis iridis con glaucoma
secundario.
Cmo se diagnostica la OVCR?
Por medio de la angiografa fluorescenica.
Cul es el tratamiento de la OVCR?
No existe uno especfico. Para tratar la neovascularizacin secundaria, se
puede utilizar fotocoagulacin.
Cules son los tipos de retinopata diabtica?
No proliferativa: se observa aumento de la permeabilidad capilar, mi-
croaneurismas (puntos rojos en el polo posterior de la retina), hemorra-
gias (en punto y en mancha), edema (que se diagnostica mejor por la
278
angiografa fluorescenica) y exudados. Es la ms comn; aparecice
en 90-95% de los casos.
Proliferativa: se caracteriza por neoformacin de vasos dentro de la re-
tina, que se extiende hasta el vtreo, y fibrosis que puede causar des-
prendimiento de la retina en etapas avanzadas.
Cules son los tipos de exudado?
Blandos: tambin conocidos como algodonosos o cotonosos, son de-
bidos a anoxia (deficiencia vascular aguda).
Duros: de coloracin amarilla, causados por edema crnico (capila-
res lesionados).
Cules son las causas de la retinopata hipertensiva?
La hipertensin esencial crnica, la hipertensin maligna y la toxemia del
embarazo.
Cul es la clasificacin de Keith-Wagener-Barker para la retinopata hi-
pertensiva?
Grado 1: constriccin de las arteriolas retinianas.
Grado 2: constriccin y esclerosis de las arteriolas retinianas (hilos de
plata).
Grado 3: exudados y hemorragias aadidos a las alteraciones vascu-
lares.
Grado 4: hipertensin maligna (papiledema).
Quines padecen glaucoma de ngulo cerrado (glaucoma agudo)?
Los hipermtropes y los asiticos.
Qu factor puede desencadenar el glaucoma agudo?
Una midriasis prolongada (por ejemplo, en el caso de la permanencia
prolongada en un lugar oscuro, como en una sala de cine).
Cul es el cuadro clnico del glaucoma agudo?
Dolor extremo, visin borrosa, ojo rojo y halos alrededor de las luces.
Puede aparecer nusea y dolor abdominal.
Cul es el tratamiento del glaucoma agudo?
Pilocarpina.
Inhibidores de la anhidrasa carbnica (acetazolamida 500 mg ).
Iridotoma.
279
Cmo es el glaucoma de ngulo abierto (crnico)?
Hay una prdida gradual de la visin perifrica, un acopamiento de la
papila y no hay luces con halo.
Cul es el tratamiento del glaucoma crnico?
Timolol (o betaxolol): 1 gota cada 12 horas durante 3 a 6 semanas.
Pilocarpina: 4 gotas al da.
Adrenalina: 2 gotas al da.

280
281
ONCOLOGA








Cules son los patrones de tincin?
La eosina es un colorante cido que se observa de color azul; se une a
componentes celulares alcalinos que contengan protenas. La hematoxi-
lina es un colorante alcalino de color rojo.
Por orden de importancia, cules son las neoplasias malignas ms impor-
tantes en mujeres y hombres?
Mujeres: pulmn, mama, colon.
Hombres: pulmn, prstata y colon.
Cules son las neoplasias con patrn de herencia autosmica dominante?
Tumor de Wilms, Feocromocitoma, cncer medular de tiroides y retino-
blastoma.
Cul es la utilidad de la vacuna BCG (Bacilo de Calmette-Gurin) en los
pacientes con cncer?
Se utiliza como modificador de la respuesta y se usa para el cncer vesical.
Para qu se usa la interleucina 2 (IL2) en oncologa?
Para tratar el cncer de clulas renales y el melanoma.
Cules son los principales marcadores tumorales?
Calcitonina: cncer medular de tiroides.
Ca 19-9: cncer pancretico, colorrectal, gstrico y ovrico mucinoso.
Ca 15-3: cncer mamario.
Ca 125: ovrico, endometrial, hepatocarcinoma, pulmonar y pan-
cretico.
Alfa fetoprotena: tumor gonadal no seminomatoso; cncer hepatoce-
lular y gastrointestinal.
282
Gonadotropina corinica humana (Beta-HGC): tumor testicular se-
minomatoso.
Antgeno carcinoembrionario (CEA): mamario, colorrectal, gstrico,
pulmonar, tiroideo y pancretico.
Tiroglobulina: cncer tiroideo papilar y folicular.
Antgeno prosttico especfico (APE): cncer prosttico.
Cul es la etiologa y tratamiento del cncer de laringe?
90% de los cnceres larngeos son epidermoides y secundarios a alcoho-
lismo o tabaquismo. Los estudios T1 y T2 se tratan con radioterapia y los
T3 y T4, con laringectoma total.
Cules son los cuatro tipos de melanoma?
Lntigo maligno, acral lentiginoso, nodular y superficial diseminado.
Cul es el tipo de melanoma ms frecuente?
El superficial diseminado.
Qu factor aumenta 5 a 10 veces el cncer tiroideo?
La radiacin.
Cules son los cuatro tipos de cncer tiroideo?
Papilar, folicular, medular y anaplsico.
Cules son la consistencia de la glndula tiroides segn el tipo de cncer?
Firme: es ms probablemente maligno.
Blanda: ms probablemente benigno
Cul es la incidencia del cncer tiroideo en relacin con la nodularidad?
Ndulo nico: 20%.
Ndulo mltiple: 40%.
Qu relacin guarda el cncer tiroideo con el tamao del ndulo?
Cuando hay aparicin rpida o aumento rpido de tamao es probable-
mente maligno.
Cul es un hallazgo clnico que se asocia a cncer tiroideo?
Parlisis larngea ipsilateral al ndulo.
Por qu los ndulos fros son probablemente malignos?
Tanto el adenoma como el cncer son hipofuncionantes. An as, 40% de
los cnceres puede captar cierta cantidad.
283
Cmo se interpreta el comportamiento de los ndulos tiroideos segn los
hallazgos clnicos?
Si el ndulo es francamente caliente, se puede descartar maligni-
dad; cuando el ndulo es fro, casi siempre es maligno.
Los ndulos qusticos raramente son malignos; cuando son slidos,
un bajo porcentaje es benigno.
Cul es el tratamiento de eleccin del cncer tiroideo?
El quirrgico.
Cules son las caractersticas y la incidencia del cncer papilar?
La incidencia es 80% en general. Las mujeres son dos veces ms afecta-
das que los hombres. Si est encapsulado es de buen pronstico.
Cules son las caractersticas y la incidencia del cncer folicular?
La incidencia es menos de 10%. Forma parte del sndrome NEMII (neo-
plasia endocrina mltiple tipo II). Su pronstico depende de la etapa.
Cursa con calcitonina y calciuria elevadas.
Cules son las caractersticas y la incidencia del cncer anaplsico?
Su incidencia es menos de 10%. Afecta ms a las personas de 50 a 70
aos. Su pronstico es pobre.
A qu se le llama ndulo pulmonar solitario?
Es una lesin pulmonar bien circunscrita, que mide hasta 5 cm de dimetro.
Qu caractersticas sugieren la probabilidad de ndulo pulmonar solitario
benigno?
Sin cambios en 2 aos.
Se observan calcificaciones densas, concntricas o en palomitas de
maz.
Es menor de 1 cm.
Cul es el protocolo para el estudio de ndulo pulmonar solitario?
Primero, placa simple del trax; despus, tomografa y, por ltimo, biopsia.
Cul es el cncer pulmonar ms frecuente y su localizacin?
Es el adenocarcinoma de localizacin perifrica (alejado de los bronquios
principales).
Cul es el tumor pulmonar benigno ms comn?
El hamartoma (imagen en palomitas de maz).
284
El adenoma bronquial es maligno o benigno?
Es maligno.
Cules son los cnceres broncognicos centrales detectables por la citolo-
ga del esputo?
El cncer de clulas escamosas y el de clulas pequeas.
Cul es el sndrome de la vena cava superior?
Se debe a obstruccin del vaso y cursa con edema, hiperemia y cianosis
de los miembros torcicos, el cuello y la cara. Tambin se presenta ma-
reo, estupor, sncope, edema cerebral y larngeo.
Cundo aparecen los sndromes de Horner y de la vena cava superior?
En el cncer intratorcico (broncognico principalmente) cuando la le-
sin provoca un efecto de masa y aparece en 5% de los casos.
En el cncer broncognico qu significan los estadios T1 y T2?
Menor y mayor y de 3 cm, respectivamente.
En el cancer broncognico que significan los estadios T3 y T4?
Extensin a la pared torcica sin y con afectacin mediastnica respecti-
vamente.
Cules son los sndromes paraneoplsicos del cncer pulmonar de clulas
pequeas?
Los sndromes de Eaton-Lambert, Cushing, secrecin inadecuada de
hormona antidiurtica (SIADH) y la degeneracin cerebelar subaguda.
Cul es el rgimen estndar de quimioterapia para el cncer de clulas pe-
queas?
Ciclofosfamida, adriamicina y vincristina (CAV).
Cules se consideran neoplasias pulmonares malignas de bajo grado?
Los tumores carcinoides, dentro de los que se encuentra el carcinoma de
glndulas bronquiales.
Qu porcentaje de los adenomas bronquiales representa el carcinoide?
80-90%.
De dnde procede el carcinoide bronquial y cul es su tipo celular?
Procede de las clulas madre bronquiales. Se conforma de clulas tipo
Kultchinsky.
Cmo se clasifica histolgicamente el hamartoma pulmonar?
Como adenocondroma.
285
Cules son los hallazgos clnicos en los casos de carcinoide bronquial?
Tos, atelectasia, infeccin recurrente y hemoptisis.
Qu es el mesotelioma y con qu se relaciona?
Es un tumor de la pleura o del peritoneo asociado a asbestosis.
Qu es el sndrome de Pancoast?
Es un sndrome secundario al tumor del sulcus pulmonale superior, que
cursa con ronquera (afeccin del larngeo recurrente) y parlisis del he-
midiafragma del lado afectado.
Cules son los tres tumores benignos del hgado?
El hemangioma, el adenoma hepatocelular y la hiperplasia nodular focal.
En quines incide ms el cncer heptico?
En hombres (2:1) y en mayor de 50 aos.
Qu factores se asocian al cncer hepatocelular?
Hepatis B, 80%; hepatitis C, 50%; cirrosis macronodular, solventes, ni-
trosaminas, pesticidas rganoclorados y la aflotoxina de Aspergillus.
Cul es la presentacin clnica del cncer hepatocelular?
Malestar, fiebre, ictericia, hepatomegalia (88%), prdida de peso (85%),
masa abdominal dolorosa (50%) y hallazgos compatibles con cirrosis
(60%).
Cul es la sobrevida del cncer heptico resecable?
De 3 a 5 aos (20%)
Cul es la sobrevida en los casos de cncer heptico no resecable?
De 4 meses.
Cul es el cncer heptico ms frecuente en adulto?
El hepatocelular.
Cul es el cncer heptico ms frecuente en nios?
El hepatoblastoma.
Cul es la incidencia del colangiocarcinoma?
5-30% en personas de 60-70 aos.
En qu rgano se asientan con ms frecuencia las metstasis y cmo se de-
tectan?
En el hgado; la deteccin se hace con determinaciones de amino-
transferasa de aspartato (AST) y fosfata alcalina (ALP).
286
Cul es el tratamiento ms efectivo de las metstasis hepticas?
El quirrgico.
Cules son los factores predisponentes para el cncer del colon?
Poliposis, enfermedad inflamatoria, enfermedad hereditaria y la dieta.
Qu medicamentos son auxiliares en la prevencin primaria del cncer del
colon?
La aspirina y los AINES (piroxicam y sulindaco). El sulindaco reduce la
incidencia de plipos adenomatosos rectales cuando se utiliza a dosis de
150 mg/da.
Cules son los dos tipos de carcinoma colorrectal hereditario no polipode
(CCHNP)?
Lynch I y II.
Cules son los antioncogenes del cncer del colon?
Gen APC y cromosoma 17p53.
Cules son los genes afectados en los casos de CCHNP?
hMSH1 y el hMSH2.
Cmo se tratan las neoplasias del tercio inferior del recto?
Con reseccin abdminoperineal de Miles.
Qu quimioterapia se usa en los casos de cncer colorrectal (adyuvante)?
5-fluoruracilo (5FU) ms levamisol o 5FU ms leucovorn.
Cul es la clasificacin de Dukes del cncer colorrectal?
A: Confinado a la pared del colon.
B: Penetra la serosa y los tejidos perirrectales.
C: Metstasis linfticas.
D: Metstasis distantes.
Cmo se monitorea el cncer colorrectal?
Con la determinacin de los niveles del antgeno carcinoembrionario
(CEA). Si se incrementan, estn indicados los estudios de imagen como
radiografa o la tomografa.
Cul es la localizacin ms frecuente del cncer del pncreas?
La cabeza del pncreas; se presenta como ictericia indolora.
Cul es la causa ms frecuente de vescula palpable (signo de Courvoi-
sier)?
El cncer del pncreas.
287
Cul es el tratamiento del cncer del pncreas resecable?
El procedimiento de Whipple.
Cul es la clasificacin histolgica del cncer endometrial?
Adenocarcinomas (65%) y adenoacantomas (22%). stos se dividen en
grados segn su diferenciacin: grado 1, diferenciado; grado 2, modera-
damente indiferenciado y grado 3, indiferenciado.
Cul es la etapa I del cncer endometrial?
En esta etapa el cncer se encuentra confinado al cuerpo uterino.
Tipo A: cuando slo afecta al endometrio y puede ser grado 1, 2 o 3.
Tipo B: cuando afecta menos de la mitad del miometrio con o sin in-
vasin vascular.
Tipo C: cuando afecta ms de la mitad del miometrio.
Cul es la etapa II del cncer endometrial?
En esta etapa, el tumor invade el crvix, pero no se extiende ms all del
tero.
Tipo A: cuando invade la mucosa cervical.
Tipo B: cuando invade el estroma cervical.
Cules son las etapas III y IV del cncer endometrial?
En la etapa III hay invasin a los ovarios y al peritoneo; en la etapa IV
existe invasin a vejiga, el recto y rganos distantes.
Cules son las modalidades de tratamiento para el cncer endometrial se-
gn la etapa correspondiente, de acuerdo con la clasificacin de Ginecolo-
ga y Obstetricia (FIGO)?
IAG1: Histerectoma extrafacial (HE) + Salpingooforectoma bilate-
ral (SOB).
IAG2: HE + SOB + Linfadenectoma plvica y para-artica (LPP).
IAG3: HE + SOB + LPP + Radioterapia (RT).
IB1 (sin invasin vascular): HE + SOB + LPP.
IB2 (con invasin vascular): HE + SOB + LPP + RT.
IIA: HE + SOB + RT.
IIB: HE + SOB + RT + Braquiterapia.
III y IV: todas las modalidades de tratamiento: quimioterapia, radio-
terapia y hormonoterapia si el tumor es positivo a receptores.
288
Cul es el cuadro clnico del cncer crvicouterino?
Se presenta con hemorragia uterina disfuncional (HUD), exudado ftido,
ulceracin cervical y manchado poscoital.
Cul es el tratamiento para la neoplasia cervical intraepitelial en etapa I
(NIC I)?
Se prefiere el uso de asa diatrmica y criociruga, ya que con ellas se pue-
den obtener muestras de tejido para anlisis.
Cul es el tratamiento con radioterapia para el cncer crvicouterino?
La dosis de radioterapia es de 4,500 a 5,000 cGy y su complicacin ms
temprana es la diarrea.
Cules son los principales tumores ovricos?
Seroso: se asemeja al tejido tubrico y presenta cuerpos de psamoma.
Mucinoso: similar al tejido endocervical y del colon. El marcador
tumoral es el cncer 19-9.
Endometrial: semejante al tejido endometrial.
De clulas germinales: contiene estructuras extra e intraembrionarias.
Disgerminoma: aparecen cordones de linfocitos infiltrando el estro-
ma; es hormonalmente activo, hiperestrognico y radiosensible (co-
mo el seminoma).
Seno endodrmico: tiene tejido de saco vitelino con diferenciacin
extraembrionaria y cuerpos de Schiller-Duval (clulas tumorales con
vaso central).
Teratoma inmaduro: tejido de las tres capas embrionarias, preponde-
rantemente neuroectodrmico.
De clulas granulosas: se observan cuerpos de Call-Exner (clulas
granulosas rodeadas de material protenico), que secretan inhibina.
De celulas de Sertoli-Leydig: secretores de andrgenos.
Cules son los genes asociados al cncer ovrico?
K-RAS, HER 2 NEU, P53 y BRCA1.
Cul es el cuadro clnico del cncer del ovario?
Hay sntomas gastrointestinales como distensin, malestar general, masa
o tumor en los cuadrantes inferiores, dolor lumbar y ascitis.
289
Cules son los marcadores del tumor de clulas germinales del ovario?
La deshidrogenasa lctica, alfa-fetoprotena (alfa-FP) y gonadotropina
corinica humana.
Cul es el marcador para el cncer ovrico en general?
El CA 125.
Cul es el rgimen de quimioterapia para el cncer ovrico?
Carboplatino ms paclitaxel.
Cul es la neoplasia benigna de la mama ms frecuente?
El fibroadenoma mamario se presenta en pacientes menores de 30 aos,
con consistencia de caucho, relativamente mvil, separado, no hipersen-
sible y de 1 a 5 cm. En las mujeres mayores de 30 aos debe pensarse en
mastopata fibroqustica y carcinoma mamario.
Cul es el gen del cncer de la mama?
C17P53 y C17BRCA1.
Por orden de frecuencia, cules son los tipos celulares del cncer mama-
rio?
Ductal infiltrante, medular, inflamatorio y papilar.
Cul es el protocolo para el estudio de las tumoraciones mamarias?
Primero la mamografa; si la lesin es sospechosa, se realiza la biopsia
por aspiracin con aguja fina (BAAF).
Cundo est indicado practicar la BAAF y aguja de calibre ancho en pre-
sencia de tumores de mama?
La BAAF tiene su indicacin precisa en las lesiones qusticas, y la de cen-
tro de aguja, en las lesiones slidas.
Cundo est indicada la biopsia abierta en caso de tumores de mama?
Cuando en la mamografa se observan calcificaciones o cuando hay des-
carga sanguinolenta.
Cul es la conducta a seguir cuando se encuentra cncer lobular de mama
in situ?
Est indicada la conducta espectante y no la quirrgica.
Cmo se presenta un cistosarcoma filoides?
Se presenta con los mismos datos clnicos del fibroadenoma mamario; pue-
de alcanzar gran tamao, y si no se le extirpa adecuadamente, recurre.
http://booksmedicos.org
290
Qu marcador tumoral se utiliza en los casos de cncer de mama?
CA15-3. Se utiliza slo para investigar recadas y nunca como prueba de de-
teccin o diagnstico pues tambin se eleva cuando existe cncer heptico.
Cul es la terapia de combinacin para el cncer de mama?
Ciclofosfamida, adriamicina, metotrexate y 5 fluoruracilo
Cul es la estirpe ms comn del cncer prosttico?
El adenocarcinoma.
Cul es la localizacin ms frecuente del cncer prosttico?
Primero en la zona perifrica, y luego en la periuretral.
Con base en la determinacin del antgeno prosttico especfico (APE)
quines tienen mayor probabilidad de supervivencia?
Quienes tienen menos de 4 ng/ml.
Cmo se clasifica el cncer prosttico segn los niveles de APE?
Se clasifica como confinado cuando el nivel de APE es menor de 3 ng/ml
y no confinado cuando el valor es mayor de 30 ng/ml.
Cul es la sobrevida media de los pacientes con cncer prosttico?
De dos a dos aos y medio.
Cul es el tratamiento del cncer prosttico no metastsico?
Prostatectoma radical y radiacin; determinar peridicamente los niveles
de APE para monitoreo.
Cul es el tratamiento del cncer prosttico con metstasis?
Agonistas de la hormona liberadora de la hormona luteinizante (LHRH),
orquiectoma y estrgenos, ya sea por separado o combinados con el fin
de disminuir la produccin de testosterona.
Cules son los frmacos antiandrognicos que ms se utilizan?
El ketoconazol, el finasteride para la hiperplasia prosttica benigna
(HPB), la ciproterona (para el hirsutismo) y la flutamida (para el cncer
prosttico).
Qu factores predisponen al cncer de la vejiga?
Edad de 60-70 aos, ciclofosfamida, tabaquismo, esquistomiasis y el uso
de anilinas.
Cul es la forma ms comn de presentacin del cncer de vejiga?
Hematuria indolora con sntomas irritativos.
291
Cules son los sitos donde se localizan ms frecuentemente las metstasis
del cncer prosttico?
Hueso y ganglios retroperitoneales.
Cules son las localizaciones ms frecuentes de la metstasis del cncer
vesical?
Los ganglios plvicos, los pararticos y el hgado.
Cul es el tratamiento del cncer transicional superficial de la vejiga?
Reseccin transuretral ms tiotepa, mitomicina C, doxorrubicina o vacu-
na (BCG).
Cul es el tratamiento del cncer vesical con metstasis?
Vincristina, metotrexate, cisplatino, taxol y gemcitabina.
Qu significan las siguientes categoras en el cncer prosttico?
T1a, T1b: hallazgo incidental con reseccin menor de 5% y mayor de
5%, respectivamente.
T2a, T2b: menor de 1.5 cm y mayor de 1.5 cm respectivamente.
T3: Traspasa la cpsula y llega al cuello de la vescula o las vesculas se-
minales.
Por lo que respecta al cncer vesical, qu significan estos trminos?
T1: invade la submucosa.
T2: invade el msculo superficial.
T3a: invade el msculo profundo.
T4b: invade la grasa perivesical.
En relacin con el cncer prosttico y vesical qu significan estos trmi-
nos?
N1: metstasis menores de 2 cm.
N2: metstasis de 2-5 cm o metstasis mltiples.
N3: metstasis mayores de 5 cm.
Cul es la triada clsica del cncer renal?
Masa en un flanco, hematuria y dolor (pero es raro ver la presentacin
clsica).
Cules son los datos paraneoplsicos del cncer renal?
Anemia: menos frecuente que la eritrocitosis.
Eritrocitosis: el tumor puede producir eritropoyetina en exceso.
292
Hipercalcemia: calcio srico elevado (ms de 10.5); pueden ser varios
mecanismos.
Hipertensin: se produce renina en exceso.
Fiebre.
Sndrome de Stouffer: es la disfuncin heptica no metastsica. Apa-
rece galactorrea, virilizacin o feminizacin, sndrome de Cushing y
disfibrinogenemia adquirida.
Cules son los estadios del cncer renal segn la clasificacin de Robson?
I/A: dentro de la cpsula.
II/B: fuera de la cpsula y dentro de la fascia de Gerota.
III/C: invade los ganglios regionales, la vena renal, la vena cava.
Cul es el tratamiento de los estadios I y II del cncer renal?
Nefrectoma radical con linfadenectoma.
De qu lado es ms frecuente el varicocele?
Del izquierdo.
Cul es la frecuencia por edad de los cnceres testiculares?
Seminomatosos de los 35-60 aos.
Linfoma en mayores de 70 aos.
Teratoma del saco de Yolk en nios.
Cul es el diagnstico diferencial del cncer testicular?
Cuando el aumento de volumen es indoloro, es ms probable que se trate
de cncer; si hay dolor puede deberse a epididimitis, hemorragia o infarto.
Cul es el tratamiento del cncer testicular seminomatoso?
Quimioterapia y radioterapia.
Qu significan las categoras IIA, IIB y IIC en el cncer testicular?
Invasin retroperitoneal microscpica, invasin retroperitoneal mediana
y masa palpable, respectivamente.
Cul es el tratamiento en las etapas I, IIA y IIB del cncer testicular?
Orquiectoma radical con reseccin retroperitoneal.
Cules son las neoplasias que invaden los ganglios plvicos y paraarti-
cos?
El cncer de la vejiga y el del cuerpo uterino.
293
Cules son las dos presentaciones de los tumores cerebrales?
Presencia subaguda de sntomas focales
Desrdenes no focales, como la cefalea, demencia y alteracin de la
personalidad o de la marcha.
Cules son los signos de tumor cerebral en la TAC?
Efecto de masa, refuerzo del contraste y edema de patrn vasognico,
principalmente en la sustancia blanca.
Cul es la neoplasia intracraneal primaria ms frecuente?
El astrocitoma, 50% (glioma, oligodendroglioma).
Cmo se mide la proliferacin intracraneal de los tumores cerebrales?
Con el ndice de captacin de bromodeoxuridina (BudR), que es un an-
logo de la timidina.
Cules son las caractersticas del astrocitoma de grado bajo?
Es ms frecuente en nios; su tipo celular es el astrocitoma piloctico, y
se trata con radioterapia.
Cules son las caractersticas del astrocitoma de grado alto?
Es ms frecuente en adultos; generalmente su localizacin es supratento-
rial y el tratamiento es quirrgico en los menores de 65 aos.
Dnde se encuentran con ms frecuencia los ependimomas?
Son mas frecuentes en los nios. Se ubican mayormente en los ventrcu-
los y de stos el ms afectado es el cuarto.
Cules son otros nombres del schwanoma?
Neuroma, neurinoma y neurilemoma.
Dnde aparece ms frecuentemente el schwanoma?
En el VIII par (neurinoma del acstico). Se presenta con mareo, prdida
de la audicin y tinnitus.
Cul es el tratamiento del meningioma y el schwanoma?
Ciruga y radioterapia.
Dnde se asientan los tumores metastsicos intracraneales?
En la unin gris-blanca en los territorios de la cerebral media y posterior.
Cules son los pasos para la conversin de purinas y pirimidinas a prote-
nas?
Sntesis de purinas y pirimidinas ribonucletidos deoxirribonucle-
tidos DNA RNA protenas.
294
Cules son las fases del ciclo celular?
M: mitosis.
G0: cuando la clula est en reposo.
G1: sntesis de enzimas que se necesitan para la sntesis de DNA.
S: replicacin del DNA.
G2: sntesis de componentes celulares requeridos para la mitosis.
Cmo se clasifican los antineoplsicos?
Antimetabolitos: metotrexato, 5- fluoruracilo (5FU), fludarabina y ci-
tarabina.
Antibiticos: bleomicina, daunorrubicina, dactinomicina y plicami-
cina.
Alquilantes: carmustina, lomustina, mecloretaminay estreptozocina.
Inhibidores de microtbulos: navelbina, paclitaxel, vinblastina y vin-
cristina.
Hormonas esteroides y antagonistas: aminogluteimida, estrgenos,
groserelina, leuprolide, prednisona y tamioxifn.
Otros: aspariginasa, cisplatino, carboplatino, etopsido, interfern y
procarbacina.
Qu antineoplsicos son ciclo-celular especficos?
Son aquellos efectivos contra neoplasias que se replican rpidamente,
como las hematolgicas. En este grupo se encuentran los antimetabolitos,
la bleomicina, los alcaloides de la vinca (vincristina, vinblastina) y el eto-
psido.
Cmo actan los alquilantes?
Se unen covalentemente al DNA en las clulas proliferantes y no prolife-
rantes.
Cmo actan los antimetabolitos?
Interfieren con la biodisponibilidad de precursores de nucletidos (puri-
nas y pirimidinas) inhibiendo su sntesis o compitiendo con ellos en la
sntesis de DNA y RNA.
Qu medicamentos requieren la utilizacin de rescates de cido folnico
(leucovorn)?
El metotrexate y la pirimetamina.
295
Qu enfermedades no neoplsicas se tratan con ciclofosfamida?
El sndrome nefrtico, la artritis reumatoide y la fibrosis pulmonar.
Qu medicamento se adjunta a la terapia con ciclofosfamida para prevenir
la cistitis?
El mercaptonato sulfonato de sodio (MESNA).
Qu frmacos quimioterpicos provocan ms neutropenia?
La ciclofosfamida, el metotrexate, 5-fluoruracilo y la doxorrubicina.
Cmo acta el 5-fluoruracilo?
El timidilato es el nico precursor del DNA que no se forma a partir de las
purinas y pirimidinas. El 5-fluoruracilo acta inactivndolo.

296
297
OTORRINOLARINGOLOGA







Cul es el rango de audibilidad del odo humano?
Detecta sonidos que van de 0 a 140 dB y de 20 a 20,000 Hz.
Cul es el plexo de Kiesselbach?
Es el plexo del tabique nasal anterior.
Con qu se asocia la rinitis alrgica?
Con plipos nasales.
Cul es el tratamiento de los plipos nasales pequeos?
Esteroides inhalados o ingeridos.
Qu rganos afecta ms la granulomatosis de Wegener?
Nariz y los senos paranasales.
Cundo est indicada la adenoidectoma?
Cuando la obstruccin provoca apnea del sueo o corpulmonale.
En la prueba de Weber cmo se percibe el sonido en los casos de sordera
de conduccin?
Ms fuerte en el lado afectado, por razones no bien comprendidas. En el
paciente con sordera neurosensorial unilateral el sonido se oye en el odo
normal.
Cmo se interpreta la prueba de Rinne?
Es una prueba que compara la transmisin del sonido por las vas area y
sea. Normalmente la duracin de la conduccin area es mayor que la
sea. En las sorderas de conduccin esta relacin se invierte. En las sor-
deras neurosensoriales la relacin permanece, pero la duracin del soni-
do disminuye.
A qu se le llama sordera neurosensorial?
Cuando es incierto si la causa es una lesin en el odo interno o en el VIII
par.
298
Cules son las causas de sordera sensorial?
Traumatismo tico, laberintitis, frmacos ototxicos y enfermedad de
Mnire.
Cules son las causas de sordera neural?
Tumores del ngulo pontocerebeloso y una gran variedad de enfermeda-
des neurolgicas.
Cmo se evala clnicamente el vrtigo?
Con la prueba de Romberg y la presencia de nistagmo. Cuando la lesin
es perifrica el nistagmo es horizontal con un componente rotatorio.
Cmo se tratan el mareo de la laberintitis y el secundario a movimiento?
La laberintitis se trata con difenidol, y el mareo por movimiento, con an-
tihistamnicos como el dimenhidrinato.
Cules son los agentes causales de la otitis externa?
Grmenes gram positivos Proteus, Pseudomonas y hongos.
Qu es la enfermedad de Mnire?
Tambin se le llama hidropesa endolinftica. Es una enfermedad que pro-
duce vrtigo intenso y episdico, con duracin de 1 hasta 8 horas acom-
paado de sordera sensorial y causado por distensin del canal
endolinftico.
Cul es el tratamiento de la enfermedad de Mnire?
Hidroclorotiazida, para disminuir la presin endolinftica.
Qu es la laberintitis?
Es un vrtigo grave de inicio agudo, con duracin hasta de una semana.
Cul es el tratamiento del vrtigo?
Cuando es grave, se utiliza diacepam; cuando es moderado, se usan anti-
histamnicos como el dimenhidrinato.
Cmo se presenta la sinusitis maxilar?
Con dolor maxilar en los caninos e incisivos.
Cmo se presenta la sinusitis etmoidal?
Con dolor en la pared antero-lateral de la nariz, que se irradia a la rbita.
Cmo se presenta la sinusitis esfenoidal?
Con cefalea justo a la mitad de la cabeza.
Cul es el tratamiento de eleccin para la sinusitis?
Descongestionantes (pseudoefedrina: 60-120mg 3 o 4 veces al da) +
amoxicilina.
299
Cules son las causas de la parlisis unilateral de las cuerdas vocales?
El cncer de tiroides, el tumor de Pancoast y el sndrome del agujero yu-
gular.
Cul es el origen ms frecuente de la epistaxis?
En 90% de los casos, la epistaxis tiene origen anterior por lo que el tapo-
namiento nasal anterior suele ser suficiente para controlarla.
Cul es el tratamiento en caso de sangrado nasal activo?
Se colocan torundas impregnadas con solucin de xilocana y adrenalina
al 1 o 2% dentro de la cavidad nasal, en el sitio de la epistaxis, y se hace
compresin sobre ambas alas de la nariz durante 10 minutos. Si la hemo-
rragia cede, debe observarse al paciente durante 30 minutos ms para ad-
vertir recurrencias.
Cules son las indicaciones de taponamiento nasal posterior?
El sangrado nasal posterior y el sangrado en focos mltiples no visibles,
como en los casos de discrasias sanguneas y traumatismos.

300
301
PEDIATRA Y GENTICA








Cules son los hitos del desarrollo?
El reflejo natatorio y el reflejo de marcha desaparecen entre los 4 a 6
meses.
La fontanela anterior cierra entre los 17 a los 19 meses.
La primera denticin ocurre a los 6-8 meses.
La fontanela posterior se cierra al nacimiento o en la etapa neonatal.
El recin nacido duerme hasta 20 horas al da.
En el 1 y 2 da, el recin nacido sigue con los ojos un objeto y tiene
poca reaccin al dolor.
El atrapamiento del pulgar es signo de dao neurolgico.
El nio responde con una sonrisa a las 6-8 semanas.
El nio cambia de decbito lateral al supino y realiza pinza gruesa en
el 4 mes.
El nio levanta la cabeza y el tronco a los 5 meses.
El nio se sienta sin apoyo a los 6 meses.
El nio pasa objetos de una mano a otra a los 6-7 meses.
El nio tiene temor a los extraos a los 6 meses.
La ansiedad de separacin en el nio ocurre de los 6 a los 8 meses.
El nio puede decir tres palabras dislabas, gatear y realizar la pinza
pulgar-ndice a los 10 meses.
El nio se mantiene parado sin apoyo a los 12 meses.
El nio comienza a caminar sin apoyo a los 13-14 meses.
El nio comprende el significado de no a los 12 meses.
302
El nio tiene sentido de pertenencia y conducta oposicionista-
negativista a los 18-24 meses.
El nio consigue el control de sus esfnteres a los 24 meses.
Cules son las reglas del crecimiento?
El nio: pierde 5-10% del peso al nacimiento.
Recupera el peso en 7-10 das.
Pesa 500 g ms y mide 3 cm ms el primer mes.
Aumenta 750 g y 3 cm mensualmente entre el segundo y cuarto mes.
Aumenta 500 g y 2 cm por mes de los 5-8 meses.
Aumenta 250-300 g y 1 cm por mes de los 8-12 meses.
Ha duplicado el peso a los 4-5 meses.
Ha triplicado el peso en el primer ao.
Ha cuadruplicado su peso a los 4-5 aos.
Cules son los promedios de peso del nacimiento a los 10 aos?
Nacimiento: 3.5 kg.
1 ao: 10 kg.
5 aos: 20 kg.
10 aos: 30 kg.
Cules son los promedios de talla del nacimiento a los 4 aos?
Nacimiento: 51 cm.
1 ao: 76 cm.
3 aos: 90 cm.
4 aos: 101 cm.
Cundo ocurre la hiperplasia del tejido linfoide?
De los 6 a los 10 aos.
Cundo se completa la primera denticin?
A los 2 aos, con 20 dientes.
Cundo se inicia la denticin definitiva?
A los 6 aos.
A qu edad se diagnostica el pie plano?
A los 4 aos.
Cunto aumenta el permetro ceflico durante el primer ao?
1 cm por mes.
303
Cul es la tasa de crecimiento de los 2 aos a la pubertad?
2.25 kg y 5 cm por ao.
En quines se indica el uso de plantilla correctora?
En aquellos pacientes que no tienen arco a la bipedestacin, pero que s
lo tienen al explorarlos en sedentacin.
A qu edad se considera el diagnstico de enuresis?
A los 5 aos en los varones y a los 6 en las mujeres.
Qu tipo de trastorno es la enuresis nocturna?
Se le considera una disomnia no REM de las etapas 3 y 4.
Qu exmenes se realizan para el diagnstico de enuresis?
EGO y examen neurolgico.
Qu frmacos se utilizan para el tratamiento de la enuresis?
Imipramina y desmopresina.
Cules son los indicadores menores de retraso en el aprendizaje?
Sincinesias, disdiadococinesia y presencia de movimientos coreiformes.
Qu anomalas se asocian al transtorno de dficit de atencin (TDA)?
Malformaciones en el odo, epicanto y clinodactilia.
Cul es el tratamiento para el TDA en los menores de 6 aos?
La dextroanfetamina. El metilfenidato slo se utiliza en los nios mayo-
res de 6 aos.
Cul es el cuadro clnico del autismo?
Patrones estereotpicos de conducta, poca o nula interaccin social y po-
ca o nula comunicacin.
Cul es el IQ normal?
Mayor de 70.
Qu complicaciones provoca el trauma obsttrico en el producto?
Cput sucedneo, cfalohematoma, hemorragia difusa del cuero cabellu-
do, moldeamiento del crneo, fractura de huesos largos y lesin de ner-
vios perifricos (parlisis Erb-Klumpke).
Cmo se diferencia al cfalohematoma del cput sucedneo y de una he-
morragia difusa?
En el cput slo se observa edema, sin cambios en la coloracin y puede
abarcar ms de una sutura. El cfalohematoma es una coleccin de san-
304
gre subperistica; por lo tanto, est contenido en la zona correspondiente
a una sola sutura y es de consistencia dura. Se alivia en 24 a 48 horas. La
hemorragia difusa abarca ms de una zona de sutura, es blanda y la piel
involucrada se muestra azul; se asocia a anomalas de la coagulacin y
puede ser tan extensa que haga disminuir el hematocrito, causar hipovo-
lemia y, finalmente, producir ictericia por la hemlisis del gran cogulo.
Qu es la parlisis de Erb-Duchenne?
Es la que resulta de un traumatismo en las races de C-5 y C-6 y que ge-
nera incapacidad de los movimientos del hombro e incapacidad de fle-
xionar el antebrazo.
Qu es la parlisis de Klumpke?
Es la que resulta de la lesin de las races de C-7, C-8 y T-1, con debilidad
de la mano, sndrome de Horner y parlisis del hemidiafragma corres-
pondiente.
Cul es la incidencia de la gestacin mltiple?
1 a 1.3%
Qu tipo de gemelos se asocian a la fertilizacin in vitro?
Los dicigticos.
Cul es la complicacin ms comn del embarazo gemelar?
El parto pretrmino.
Cul es el mtodo de eleccin para el diagnstico de gemelos idnticos?
Solo las pruebas de HLA son precisas, ya que an siendo gemelos mono-
cigticos, pueden ser bicorinicos.
Cules son las complicaciones que ocurren en la madre a causa del emba-
razo mltiple?
Hipertensin en el embarazo; hiperemesis gravdica.
Qu complicaciones ocurren en los productos del embarazo mltiple?
Hemorragia intracraneal, sndrome de distrs respiratorio del recin na-
cido (SDRRN), y peso bajo al nacer.
Cmo se clasifica el grado de desarrollo intrauterino?
Como pequeo, adecuado y grande para la edad gestacional, segn el pe-
so, la longitud y el permetro ceflico.
Cundo un recin nacido es pequeo para la edad gestacional?
Cuando tiene menos del percentil 10 de peso para una edad especfica.
305
Cundo un recin nacido tiene bajo peso y muy bajo peso al nacer?
Cuando al nacimiento pesa menos de 2500 gr y menos de 1500 gr respec-
tivamente.
Cules son los factores de riesgo relacionados con el peso bajo al nacer?
La madre pesa menos de 45 kg.
El aumento de peso durante el embarazo fue menor de 4.5 kg.
La presencia de crvicovaginitis durante el embarazo.
El consumo de tabaco, alcohol o drogas durante el embarazo.
La edad de la madre, si es menor de 16 o mayor de 35 aos.
Los periodos intergensicos cortos.
Cules son las causas de retraso del crecimiento intrauterino?
Cualquiera que provoque enfermedad de pequeos vasos (como en la diabe-
tes mellitus, la hipertensin arterial y la enfermedad renal), la toxoplas-
mosis, la infeccin por citomegalovirus, la rubola, el alcohol, la cocana
y el tabaco.
Cundo un recin nacido es grande para la edad gestacional?
Cuando est por arriba del percentil 90 de peso para una edad especfica.
Cul es la deficiencia congnita de disacaridasas?
La de lactasa-isomaltasa.
A qu edad funcionan la sucrasa, maltasa e isomaltasa?
Desde las 12 semanas; alcanzan el mximo a las 34 semanas.
A qu edad funciona la lactasa?
Desde las 37 semanas (en recin nacido a trmino).
Cul es el contenido de la leche para el prematuro en Cal?
20-23 Cal por cada onza (30 ml).
Cmo se alimenta a un recin nacido de menos de 34 semanas?
Con sonda orogstrica.
Qu tipo de alimentacin se utiliza para un nio con un peso al nacer me-
nor de 1,000 g?
Alimentacin gstrica continua o transpilrica.
Cules son los requerimientos proteicos de un recin nacido a trmino?
2-3 gramos/kg/da.
Cul es la edad ms comn de presentacin de la deficiencia de lactasa?
De los 8 a los 15 aos.
306
Cmo se manifiesta la deficiencia de lactasa?
Como diarrea osmtica, heces cidas y meteorismo.
Cmo se presenta la intolerancia a protenas de la leche o de soya?
Como enterocolitis o enteropata.
Qu dao provoca en el intestino la intolerancia a protenas de leche o de
soya?
Atrofia de vellosidades en el intestino delgado y colitis en el grueso.
Cmo evala el estado nutricional la clasificacin de Gmez?
Evala el estado nutricional tomando en cuenta el peso como un por-
centaje del peso normal promedio para la edad en los menores de 5 aos.
Cmo evala el estado nutricional la clasificacin de Waterlow?
Lo evala con una relacin entre el peso y la talla.
Cul es la clasificacin de Gmez?
Normal: 90-110% del peso normal.
Desnutricin de 1er. grado: 76-90%.
Desnuticin de 2 grado 61-75%.
Desnutricin de 3 grado menos de 60%.
Cules son las indicaciones para realizar el diagnstico prenatal?
Edad materna menor de 35 aos, alfa-fetoprotenas baja, antecedente de
un hijo afectado, anomalas cromosmicas conocidas, estado cromos-
mico incierto del hijo previo, alteracin cromosmica o mendeliana de
los padres.
Cul es el periodo indicado para realizar la amniocentesis?
A las 16 semanas de gestacin.
Cundo debe realizarse la biopsia de vellosidades corinicas?
De las 10 a las 12 semanas de gestacin.
Cmo se clasifican los defectos del nacimiento?
Ambientales: los teratgenos (qumicos o biolgicos) son agentes que
alteran el crecimiento, la organognesis o la fisiologa celular en el
producto, y corresponden al 10%
Genticos: causan anomalas aisladas, anomalas del desarrollo y
sndromes reconocibles. Estas anomalas pueden ser heredadas o apa-
recer de novo.
307
Multifactoriales: resultan de una combinacin de factores genticos o
ambientales. Cuando varios genes son los causantes, se denomina
anomala polignica.
Espordicos: debidos a accidentes que ocurren durante la gestacin,
como la oclusin de un vaso, u otros que fomenten un desarrollo
inadecuado de la anatoma del producto. Tiene una incidencia de
1%.
Cules son los signos de enfermedad congnita del tejido conectivo?
Dilatacin de la raz artica y articulaciones elsticas.
Cules son los signos de las anomalas congnitas del desarrollo de las c-
lulas musculares?
Se encuentran evidencias de que los tendones y ligamentos son laxos.
Cules son los signos de enfermedad sea congnita?
Huesos largos y anchos en exceso.
Cules son los signos de enfermedad neurolgica congnita?
Contracturas y falta de flexin.
Qu signos indican alteracin de los factores de crecimiento?
Talla alta y exceso de tejido subcutneo.
Qu es un dismorfismo?
Es una anomala estructural del desarrollo que puede ser una malforma-
cin, una deformacin o una disrupcin. Puede ser medible objetivamen-
te como en el caso del hipertelorismo que es un aumento en la distancia
interpupilar normal, o bien, apreciable subjetivamente como en el caso de
los pliegues palmares anormales.
Qu es un sndrome de malformacin?
Es un patrn reconocible de anormalidades que se presume se deben a
una causa comn. Las anormalidades pueden ser internas o externas, es-
tructurales o funcionales.
Cules son las posibles etiologas de los sndromes de malformacin?
Teratognicas, espordicas, de un solo gen y cromosmicas.
A qu se le llama tiempo de exposicin al teratgeno?
Es el periodo comprendido entre las 8 y las 12 semanas durante las cuales
se lleva a cabo la organognesis y, por tanto, pudiera verse afectada. Una
308
exposicin al teratgeno antes de la implantacin (7 a 10 das) slo resul-
tara en la prdida del embrin sin efectos posteriores.
Qu agente es la causa ms frecuente de malformaciones congnitas de
origen infeccioso?
El citomegalovirus que causa retraso en el crecimiento, prdida de la au-
dicin y anomalas del sistema nervioso central.
Cules son las malformaciones ms comnmente asociadas a la rubola?
El glaucoma, la catarata, el microftalmos y la uvetis.
Cules son los tres tipos de desrdenes cromosmicos?
El numrico, el debido a un defecto estructural y mosaicismo.
Qu es el imprinting?
Es el fenmeno por el cual la expresin de un gen se modifica sugn pro-
ceda del padre o de la madre.
Qu es la disoma uniparental?
Sucede cuando los cromosomas de un mismo par pertenecen al mismo
progenitor.
Cul es el patrn de transmisin de los defectos autosmicos recesivos?
El defecto se expresa cuando ambos padres son portadores. Si solo uno es
portador, entonces el hijo tambin ser portador.
Cules son las caractersticas de transmisin de los defectos autosmicos
dominantes?
25% de la progenie es normal.
50% son homocigotos.
25% son heterocigotos.
Cules son los defectos cromosmicos estructurales ms frecuentes?
Cri du chat: es una delecin en el 5p.
Tumor de Wilms-aniridia: delecin en el 11p.
Prader Willi: delecin en el 15q.
Cul es la incidencia de los principales desrdenes cromosmicos?
Sndrome de Down 1:700.
Sndrome de Klinefelter 1:800.
Sndrome de Turner 1:2500.
Trisoma 18 1:8000.
309
Trisoma 13 1:4000-1:10,000.
Cri du chat 1:50,000.
Cules son las enfermedades ms frecuentes ligadas a X?
La ceguera al color, la distrofia muscular de Duchenne y el sndrome de
X frgil.
Qu enfermedades se deben a expansin de trinucletido?
La distrofia miotnica, la ataxia de Friedrich, la enfermedad de Hunting-
ton, la ataxia espinocerebelar, el sndrome de x frgil y la atrofia muscu-
lar espinal bulbar.
Cules son las principales enfermedades por defecto en el metabolismo de
aminocidos?
La fenilcetonuria, la homocistinuria, la alcaptonuria y la acidemia isova-
lrica, entre otras.
Cul es la incidencia de la fenilcetonuria y de la homocistinuria?
1 en 1,200 y 1 en 100,000, respectivamente.
Qu errores del metabolismo requieren deteccin oportuna?
La homocistinuria y la fenilcetonuria.
Qu es la fenilcetonuria?
Es una deficiencia de fenilalanina hidroxilosa que cursa con aumento de
catabolitos de la fenilalanina y produce retraso mental. El dao se puede
prevenir evitando consumir alimentos que contengan fenilalanina.
Qu es la alcaptonuria?
Es una enfermedad causada por la deficiencia de la cido homogentsico-
oxidasa, transmitida de manera autosmica recesiva.
Qu es la ocronosis?
Es la coloracin opaca de los cartlagos del cuerpo principalmente en los
de la oreja por depsito de cido homogentsico.
Qu es la acidemia isovalrica?
Es una enfermedad debida al aumento del cido isovalrico (derivado de
la leucina).
Qu es la galactosemia?
Es la deficiencia congnita de galactosa-1-uridyl transferasa. Al consumir
galactosa se producen en el recin nacido: insuficiencia heptica (aumen-
to de las bilirrubinas y del tiempo de protrombina y disminucin de la
310
glucosa), alteracin de la funcin tubular (acidosis, glucosuria y aminoa-
ciduria) y cataratas.
Cules son algunas enfermedades peroxismicas?
La adrenoleucodistrofia neonatal, la enfermedad de Refsum y la condro-
displasia rizomlica punteada, entre otras.
Cules son los defectos del ciclo de la urea?
La hiperamonemia, la deficiencia de ornitintrascarbamilasa y la deficien-
cia de carbamoyl fosfato sintetasa I.
Cules son los sndromes que cursan con acidosis lctica?
MELAS: miopata, encefalopata, lactic acidosis, stroke (acidosis lcti-
ca, enfermedad vascular cerebral).
MERRF: myotonic epilepsy with red ragged fibers (epilepsia mioclnica
con fibras rojas rasgadas).
Kerns-Sayre: oftalmoplejia y degeneracin retiniana.
Cules son las nicas anomalas enzimticas que se transmiten de manera
autosmica dominante?
La enfermedad de Gilbert, la deficiencia de alfa-1-antitripsina (alfa1AT)
y porfiria intermitente aguda.
Qu enfermedades produce la deficiencia de alfa1AT en el adulto?
EPOC tipo enfisema en el 60%, cirrosis y carcinoma hepatocelular.
Qu enfermedad produce la deficiencia de alfa-1-antitripsina en el recin
nacido?
Hepatitis metablica neonatal, que conduce a cirrosis e hipertensin portal.
Cules son las anomalas cardiacas asociadas a desrdenes cromosmi-
cos?
Sndrome de Down: defecto de cojinetes endocrdicos y defectos ven-
triculares septales.
Sndrome de Turner: coartacin artica y aorta bicspide.
Que se debe sospechar si la fontanela posterior es mayor de 5 cm en el re-
cin nacido?
Hipotiroidismo congnito.
Qu es la enfermedad de Wilson?
311
Es la toxicosis hereditaria por cobre (autosmica recesiva), en la que hay
concentraciones hepticas de cobre elevadas 10 a 50 veces, dficit de ce-
ruloplasmina, mitocondrias hepticas anmalas y esteatosis heptica. Las
manifestaciones pueden aparecer en cualquier momento desde los 5 a los
50 aos, con anillo de Kayser-Fleischer (coloracin dorada o verde ama-
rillenta de la crnea), problemas de comportamiento, alteraciones del
movimiento y anemia hemoltica.
Cul es el defecto gentico en la enfermedad de Wilson?
Est afectado el cromosoma 13 en el gen ATP 7B.
Cul es el gen defectuoso en los casos de sndrome de Marfan?
El gen de fibrilina que se encuentra en el brazo largo del cromosoma 15
(15q).
Cul es el diagnstico diferencial del sndrome de Marfan?
Debe hacerse con la homocistinuria principalmente.
Cules son los genes defectuosos en los pacientes con osteognesis imper-
fecta?
Los dos genes que codifican la alfa colgena: COL 1A1 y COL 1A2.
Cul es el cuadro clnico de la osteognesis imperfecta?
Esclerticas azules, osteopenia, dentinognesis imperfecta, historia de
sordera e imagen de palomitas de maz en las metfisis.
Qu es la enfermedad de Hirschprung?
Es una agenesia de los plexos mientricos en el colon distal, el cual se en-
cuentra contrado siempre e impide el paso del bolo fecal.
En quin se debe sospechar enfermedad de Hirschprung?
En un neonato que tiene retraso en el paso del meconio, pobre alimenta-
cin, vmito biliar y distensin abdominal. El diagnstico debe hacerse
antes de los tres meses mediante biopsia rectal.
Qu defectos producen las fallas en el desarrollo mlleriano?
Defectos en la porcin superior de la vagina, en el cuello uterino, el tero
o las trompas de Falopio.
Qu defectos producen las fallas en el desarrollo del seno urogenital?
Agenesia uterina y vaginal.
Cules son los sndromes de feminizacin?
Deficiencia de colesterol-desmolasa.
312
Deficiencia de 3beta-desoxiesteroide-deshidrogenasa.
Deficiencia de 17-hidroxilasa.
Deficiencia de 5 alfa-reductasa.
Cules son los sndromes de virilizacin?
Deficiencia de 21-hidroxilasa.
Deficiencia de 11-hidroxilasa.
Deficiencia de 3-beta-hidroxiesteroide-deshidrogenasa.
Cules son las causas de pubertad retrasada en mujeres y hombres?
Constitucional, falla testicular/ovrica e hipogonadismo hipogonadotr-
pico.
Cul es el cuadro clnico de la deficiencia de 17 alfa-hidroxilasa?
Son varones hipertensos, con infantilismo sexual y azoospermia. El in-
fantilismo sexual se debe a la ausencia de cortisol, andrgenos o estrge-
nos. La hipertensin se presenta porque existe mayor produccin de
mineralocorticoides.
Qu hormona estimula la produccin de testosterona?
La hormona luteinizante acta sobre las clulas de Leydig para producir
testosterona.
Qu es el sndrome de feminizacin testicular?
Es un estado de refractariedad de los tejidos perifricos a la dihidrotestos-
terona, lo cual da como resultado un paciente con fenotipo femenino, pe-
ro con testculos con localizacin intraabdominal.
Cules son las causas de la falla ovrica/ testicular primaria?
El sndrome de Turner y el sndrome de Klinefelter, respectivamente.
Dnde se encuentra el defecto del sndorme de X frgil?
En el brazo largo del cromosoma X (Xq28).
Cmo se hace el diagnstico de sndrome de X frgil?
Demostrando que el triplete CGG est repetido ms de 200 veces en el
gen FMR1. Afecta a uno de cada 4,000 varones.
Cules son las variantes citogenticas del sndrome de Klinefelter?
80% presenta cariotipo 47XXY; otros son el 48XXXY, el 48XXYY y el
49XXXXY.
Cules son los cariotipos de los llamados superhombre y supermujer?
313
XYY y XXX, respectivamente.
A qu se asocia el cariotipo XYY?
A conducta antisocial.
Cul es la hernia congnita ms frecuente?
La hernia diafragmtica psterolateral de Bochdalek.
Cul es la pentaloga de Cantrell?
Hernia diafragmtica, esternn hendido, pericardio ausente, onfalocele y
defectos intracardiacos.
Qu es el onfalocele?
Es una herniacin de las vsceras dentro del cordn umbilical. Puede
afectarse el intestino, el hgado o ambos.
Qu es la gastrosquisis?
Es una unin defectuosa en la lnea media que deja pasar al intestino ha-
cia el exterior.
Cmo se diagnostica prenatalmente el onfalocele?
Con ultrasonido.
Qu es el complejo VATER?
Son defectos Vertebrales, Anales, Traqueoesofgicos, Radiales y Renales.
Cul es el tipo ms frecuente de fstula traqueoesofgica?
La atresia esofgica proximal con fstula traqueoesofgica distal (tipo IV)
se presenta en 85% de los casos. La segunda en frecuencia es la atresia sin
fstulas; se presenta en 8%.
Qu signos acompaan a la atresia esofgica?
Abdomen escafoide, salivacin excesiva e imposibilidad para pasar una
sonda nasogstrica. Si la atresia es pura no hay aire en el estmago.
Qu es la estenosis pilrica?
Es una hipertrofia de la capa muscular a nivel del ploro, que cursa clni-
camente sin vmito al nacimiento, sigue una etapa en que inicia y au-
menta gradualmente el vmito y finalmente aparece el vmito no biliar
en proyectil. Se observa de los 2 meses a los 2 aos del nacimiento.
Cmo se diagnostica la estenosis pilrica?
Con el hallazgo de una masa abdominal superior derecha en un nio con
historia de vmito. El ultrasonido es el procedimiento diagnstico de
314
eleccin; un ploro mayor de 15 mm y una pared pilrica de grosor ma-
yor a 4 mm son altamente sugestivos.
Qu anomalas se asocian a la atresia intestinal?
Trisoma 21, VATER y pncreas anular.
Cules son los dos hallazgos en y la atresia duodenal?
Vmitos biliares y signo de la doble burbuja.
A qu se asocia el 10% de las atresias yeyuno-ileo-clicas?
A la fibrosis qustica.
Cul es la diferencia radiolgica entre el ileo meconial y la atresia de ye-
yuno-ileo?
En la primera slo hay distensin e imagen en burbujas de jabn,
mientras que en la segunda hay distensin con niveles hidroareos.
Cmo se presenta el divertculo de Meckel?
82% como hemorragia indolora, 10% como obstruccin y 6% como di-
verticulitis (hay que hacer diagnstico diferencial con apendicitis).
Cul es la anomala intestinal congnita ms frecuente?
El divertculo de Meckel
Cundo aparece la displasia de la cadera?
Es frecuente tras un parto de nalgas y su incidencia es de 1 en 100 con
predominio en las mujeres.
Qu es la malformacin de Arnold-Chiari?
Es la protrusin del cerebelo hacia el canal espinal con distorsin de la
mdula espinal.
Cul es el cuadro clnico de la atresia de las coanas?
El recin nacido se presenta cianosis que slo se alivia con el llanto; exis-
te imposibilidad para colocar una sonda nasogstrica.
Qu se debe sospechar al detectar estridor larngeo en un recin nacido?
Que hay laringomalacia.
Cul es el cuadro clnico del hipotiroidismo congnito?
Distrs respiratorio, cianosis, ictericia, rechazo del alimento, llanto ron-
co, hernia umbilical y retraso en el crecimiento seo.
Cmo se diagnostica la hipocalcemia del neonato?
315
Con un Calcio total menor de 7 y un Calcio ionizado menor de 3-3.5,
con signos clnicos de hipocalcemia como el mioclono.
Cul es una causa frecuente de la hipocalcemia del neonato?
La hiperfosfatemia que se provoca cuando se le alimenta con leche de
vaca.
Cundo se considera hipoglucemia en el neonato?
Cuando la glucemia es menor de 20 mg/dl en el neonato pretrmino,
menor de 30 mg en el neonato de trmino y menor de 45 despus del pe-
riodo neonatal. Se presenta con nerviosismo, hipotona, apnea, taquipnea
o convulsiones.
Cules son los factores de riesgo relacionados con la hipoglucemia neonatal?
Diabetes materna: crea un estado de hiperinsulinemia en el neonato,
que provoca episodios de hipoglucemia.
Prematurez: no hay madurez de las enzimas gluconeognicas y las
reservas de glucgeno son bajas.
Retardo en el crecimiento
Asfixia perinatal y estrs por fro,
Sepsis y anormalidades metablicas o endocrinas como la galactose-
mia o el panhipopituitarismo.
Qu anomalas se pueden presentar en los hijos de madres diabticas?
Regresin caudal, macrosomia, muerte fetal, parto pretrmino, hipoglu-
cemia y policitemia.
Qu es la regresin caudal?
Es la falta de desarrollo del tejido nervioso y seo de la columna inferior
relacionado con la presencia de diabetes en la madre.
Cul es la dosis de vitamina K utilizable en los neonatos?
0.5 a 1 miligramo.
Qu es la enfermedad hemorrgica del recin nacido?
La secundaria a niveles bajos de factores K dependientes y un metabo-
lismo heptico acelerado (induccin microsomal por fenitona o por fe-
nobarbital) o administracin de cumarnicos a la madre. Dentro de los
primeros tres das pueden presentarse hemorragias en el cuero cabelludo,
las mucosas, el rea de circuncisin o en el ombligo.
316
Qu es la enfermedad hemoltica del recin nacido?
Tambin conocida como eritroblastosis fetalis, es la causa ms frecuente de
anemia neonatal. Ocurre cuando los anticuerpos de la madre cruzan la
barrera placentaria para atacar a los antgenos de las clulas de la sangre
del producto, especficamente a los dos grupos principales de antgenos,
que son el sistema ABO (leve) y el sistema Rho (D) (grave). La anemia in
tero puede ser tan severa que conlleve hidrops fetalis.
Cul es el hematocrito de un recin nacido a trmino?
50- 55%
Cules son las manifestaciones clnicas del hidrops fetalis?
La hipoxia prolongada por anemia en el producto provoca acidosis. Se
identifica por la triada de anemia, anasarca e hipoproteinemia.
Cules son causas de hidrops fetalis?
La anemia severa, alfa-talasemia in tero, incompatibilidad de grupo,
transfusin gemelo-gemelo y el cierre prematuro del foramen oval.
Cul es el tratamiento del hidrops fetalis?
Corregir la anemia, tratar la falla cardiaca congestiva y tratar el distrs
respiratorio.
Cules son las causas del cierre del formen oval?
Aumento de la presin pulmonar, aumento de la oxemia y disminucin
de la prostaglandina E
2
.
Qu es el sndrome de hiperviscosidad del recin nacido?
Tambin llamado policitemia (Hto mayor de 65%). Ocurre en los recin
nacido postrmino hijos de madres diabticas y con pinzamiento tardo
del cordn. Lucen pletricos, rubicundos y con acrocianosis. Los snto-
mas son irritabilidad, letargo o convulsiones. Adems puede observarse
ictericia y cardiomegalia.
Cules desrdenes genticos cursan con policitemia?
El sndrome de Down y el sndrome de Beckwidth-Wiedeman.
Cules son las etapas del desarrollo pulmonar?
Pseudoglandular (5-16 semanas): se forman desde los bronquios has-
ta los bronquolos terminales.
317
Canalicular (16-25 semanas): se forman los sacos alveolares.
Alveolar (25-40 semanas): aumenta el nmero de sacos alveolares y
aparece el surfactante.
Cul es el componente ms importante del factor surfactante?
La fosfatidilcolina.
Cul es el ndice lecitina-esfingomielina?
Es la relacin entre ambas, que se mide directamente de una muestra de
lquido amnitico. Lo normal es de 2:1, y tiene un valor predictivo sobre
la madurez pulmonar del producto al momento de tomar la muestra.
Cules son las dos causas principales de hipoplasia pulmonar?
La primera es la disminucin de lquido amnitico y la segunda es una
lesin ocupante del trax (hernia diafragmtica).
Cul es la diferencia clnica y teraputica entre la taquipnea transitoria
del recin nacido (TTRN) y el sndrome de distrs respiratorio del recin
nacido (SDRRN)?
La TTRN se debe a un drenaje linftico insuficiente del lquido pul-
monar (generalmente causado por un mecanismo anormal de parto y
realizacin de cesrea) que muestra ligera disnea, sin retracciones ni
cianosis. Se resuelve en pocos das con oxgeno suplementario.
El SDRRN o sndrome de membrana hialina se debe a una produc-
cin insuficiente de factor surfactante generalmente debida a corta
edad gestacional. En las primeras tres horas de vida extrauterina hay
disnea con cianosis y retracciones. La terapia comprende oxgeno su-
plementario y reposicin de factor surfactante con alguno de los
anlogos sintticos.
Cul es la imagen radiolgica del pulmn hmedo (TTRN)?
Hiperinsuflacin, trama parahiliar densa, corazn con borde deshilacha-
do y lquido en las fisuras.
Cul es la imagen en los diferentes tipos de sndrome de distrs respiratorio?
Tipo 1: retculo granular con mayores zonas de aireacin.
Tipo 2: retculo granular con mayores zonas de atelectasia.
Tipo 3: retculo granular con mayores zonas de ateleutasia y bronco-
grama intenso.
318
Tipo 4: atelectasias difusas que borran la silueta cardiaca.
Cules son las complicaciones del SDRRN?
Hemorragias intraventriculares, neumotrax espontneo, conducto arte-
rioso permeable, enterocolitis necrosante, displasia broncopulmonar y re-
tinitis de la prematurez.
A qu se le llama displasia broncopulmonar del recin nacido?
Es una enfermedad que resulta del uso de oxgeno y ventilacin mecni-
ca en los infantes con SDRRN. Se caracteriza por la necesidad de perpe-
tuar la terapia con oxgeno ms all de los 28 das de vida.
Qu es el sndrome de aspiracin de meconio?
En los productos postrmino, la hipoxia es un estmulo que por va vagal
media la liberacin de meconio al lquido amnitico, que posteriormente
se deposita en las vas respiratorias y en el tubo digestivo. Los depsitos
de meconio producen signos de distrs respiratorio y signos radiogrficos
que demuestran hiperinflacin con reas de infiltrado difuso.
Qu es la enterocolitis necrosante del recin nacido?
El tejido intestinal previamente afectado por la hipoxia es daado ade-
ms por el sobrecrecimiento bacteriano al iniciar la dieta del recin naci-
do. Se presenta con distensin abdominal, sangre en las heces, acidosis,
hipotensin o choque. Los casos leves se autolimitan al descontinuar la
va oral e instalar el antibitico adecuado. Los casos graves con hemorra-
gia franca requieren reseccin intestinal.
Qu es le retinopata de la prematurez?
Tambin se le conoce como fibroplasia retrolental y se debe a la inmadurez
de los vasos retinianos. Los factores que contribuyen a su aparicin son
la hiperoxia (en el tratamiento de padecimientos respiratorios), hipercap-
nia, hipoxia intermitente y sepsis.
Cules son las principales causas de ictericia fisiolgica del recin nacido?
Puede deberse a actividad retardada de la glucoronil-transferasa, aumen-
to de la carga de bilirrubina en los hepatocitos y disminucin de la depu-
racin plasmtica de bilirrubina.
Cules son las causas de ictericia no fisiolgica del recin nacido?
319
Hemlisis: inmune (incompatibilidad de grupo) y no inmune (esfero-
citosis, hemoglobinopatas y deficiencias enzimticas).
Acmulo de sangre extravascular: cfalohematoma.
Circulacin enteroheptica aumentada: obstruccin intestinal.
Alimentacin con seno materno.
Desrdenes del metabolismo de la bilirrubina: sndromes de Lucey-
Driscoll, Crigler-Najjar y de Gilbert
Desorden metablico: hipotiroidismo, panhipopituitarismo y galacto-
semia.
Sepsis bacteriana.
Qu enzima produce biliverdina?
La hem-oxidasa.
Qu enzima produce bilirrubina?
La biliverdn-transferasa.
Qu es el sndrome de Gilbert?
Es la deficiencia de uridn-difosfoglicerasa, que produce hiperbilirrubi-
nemia crnica congnita con niveles de 2-5 mg/dl. La deficiencia provo-
ca un defecto en la captacin de bilirrubina.
Qu es el sndrome de Crigler-Najjar?
Es una deficiencia hereditaria de glucoronil-transferasa.
Tipo I: hiperbilirrubinemia grave; mueren por kernctero en el primer
ao de vida.
Tipo II: cursa con bilirrubinas menores de 20; alcanzan la edad adul-
ta.
Qu es el sndrome de Dubin-Johnson?
Es un sndrome con ictericia provocada por un defecto en la excrecin de
bilirrubinas, en el que el hgado se encuentra pigmentado.
Qu es el sndrome de Rotor?
Es un defecto en la excrecin de la bilirrubina, en el que el hgado no est
pigmentado.
Cul es el nivel normal de bilirrubina en el recin nacido?
La total, menos de 13 y la directa, de 1.5-2; nunca aumentan ms de 5 en
24 horas.
Cundo se indica la fototerapia?
Cuando la bilirrubina total va de 12 a 15 mg/dl.
320
Cundo se indica la exanguinotransfusin?
Cuando la bilirrubina total alcanza 20 mg/dl
Cules son los defectos cardiacos congnitos ms comunes?
La comunicacin interventricular (CIV) que ocurre en 26% de los casos,
seguida por la persistencia del conducto arterioso (PCA), que se presenta
en 10% de los casos y la coartacin de la aorta, en 8%. La traspocisin de
los grandes vasos tiene una incidencia de 5%.
Cules son los hallazgos clnicos en los casos de coartacin de la aorta?
Pulsos amplios en la mitad superior del cuerpo y soplo en el pex o a lo
largo del borde esternal izquierdo e interescapular alto; radiografa de t-
rax con el signo del 3 y el signo de Roessler.
Cules son las caractersticas clnicas de la comunicacin interauricular
(CIA)?
Levantamiento ventricular derecho, P
2
continuo y soplo pulmonar en el
tercero y cuarto espacios.
Cules son los tres tipos de CIA?
Defecto ostium secundum: Aparece en 80% de los casos y el defecto se
ubica a la mitad del tabique interauricular.
Defecto ostium primum: es el siguiente en frecuencia y se ubica en la
parte inferior del tabique interauricular.
Defecto tipo seno venoso: es el menos frecuente y se ubica en la parte
superior del tabique interauricular.
Qu se observa en el electrocardiograma de un paciente con defecto ostium
primum y de cojinetes cardiacos?
Desviacin del eje a la extrema izquierda.
Qu es el sndrome de Eisenmenger?
Es un sndrome que puede ocurrir en presencia de cualquier tipo de corto
circuito intracardiaco. Normalmente provocan un corto circuito izquier-
da-derecha. Cuando la cardiopata es avanzada desarrolla hipertensin
en el circuito pulmonar, que eventualmente se torna mayor que la pre-
sin de las cavidades izquierdas y revierte el flujo a derecha-izquierda.
Dnde se ausculta el soplo en los pacientes con CIA?
En el borde esternal derecho, y es una evidencia del aumento de la circu-
lacin derecha.
321
Cules son causas de conducto arterioso persistente?
Hipoxia fetal o neonatal y aumento de los niveles de prostaglandinas.
Qu es la anomala de Ebstein?
Es un defecto congnito de la tricspide, que se encuentra implantada
dentro del ventrculo derecho ms abajo de lo normal.
Cul es el cuadro clnico de la CIV o PCA no complicada?
Hay estigmas de falla cardiaca congestiva, fallo de medro e infecciones
respiratorias bajas frecuentes.
Cul es la causa principal de PCA?
La hipoxia aguda o crnica previene el cierre. Los nios con PCA son
nios de trmino que presentan cianosis e hipoxia progresivas.
Cules son las cardiopatas ductodependientes (dependen de la PCA o
CIV para que el paciente pueda seguir viviendo)?
La atresia pulmonar y la atresia tricuspdea.
Qu se encuentra en la exploracin de la CIV no complicada?
Un soplo mesoesternal, mesosistlico en el borde izquierdo.
Qu se encuentra en la exploracin de los pacientes que presentan CIV
con sndrome de Eisenmenger?
El ventrculo derecho est aumentado de tamao, hay S2 fuerte y desdo-
blada y un soplo producido por la insuficiencia pulmonar.
Cules son los datos electrocardiogrficos que se observan en la hiperten-
sin ventricular derecha?
Patrn S-1, Q-3.
Ondas R mayores o iguales de 20 mv.
Ondas T positivas en las derivaciones precordiales derechas (V-1, V-
3).
Qu se observa en las placas radiogrficas cuando la CIV es muy impor-
tante?
Aumento de tamao del corazn y de la arteria pulmonar.
Cules son los grmenes patgenos que producen la endocarditis de vl-
vula daada y de los usuarios de drogas IV?
S. viridans y S. Aureus, respectivamente.
Cules son las cinco Ts de las cardiopatas ciangenas?
322
Tetraloga de Fallot, Tronco arterioso persistente, atresia Tricuspideopul-
monar, Trasposicin de grandes vasos y drenaje venoso Total anmalo.
Qu es la tetraloga de Fallot?
Es una cardiopata caracterizada por la presencia de estenosis pulmonar,
comunicacin interventricular, hipertrofia del ventrculo derecho y ca-
balgamiento de la aorta. Radiolgicamente se observa un corazn en bota y
clnicamente se presentan crisis de hipoxia.
Cul es el situs normal (situs solitus)?
Cuando el pex se encuentra a la izquierda junto con el bazo y el ven-
trculo derecho a la derecha con el hgado.
Cul es la posicin de la arteria pulmonar en los situs?
Anterior y a la izquierda en el situs normal, y anterior y a la dercha en el
situs inverso.
Cules son los hallazgos anatmicos en las trasposiciones de los grandes
vasos, derecha e izquierda?
La aorta se encuentra anteriormente ya sea izquierda o derecha y nace
del ventrculo derecho.
Cmo se usa el sistema de tres letras para describir la trasposicin de los
grandes vasos?
Primera letra: (S) solitus, (I) inversus. Indican la posicin de la aur-
cula.
Segunda letra: (D) dextro, (L) levo. Indican la posicin del ventrculo.
Tercera letra: (D) dextro, (L) levo. Indican la posicin de la trasposi-
cin.
Cules son los microorganismos ms frecuentemente responsables de las
infecciones perinatales?
Estreptococos del grupo B, E. Coli, Klebsiella, Chlamydia y Neisseria.
Cul es el periodo de incubacin de la oftalma por Chlamydia y por go-
nococo?
5 a 15 das y 1 a 21 das, respectivamente.
Cul es el tratamiento profilctico para evitar la oftalma por gonococo?
Se usa una preparacin con 1% de nitrato de plata, 1% de tetraciclina y
0.5% de eritromicina.
323
Cules son los principales factores de riesgo relacionados con la sepsis
neonatal?
Favorecen la infeccin: rotura prematura de las membranas, co-
rioamniotis y fiebre materna.
Debilitan al producto: parto prematuro y bajo peso al nacer.
Cul es el tratamiento emprico de la sepsis neonatal?
Ampicilina-gentamicina durante 10 das (excepto en los casos de menin-
gitis y osteomielitis)
Qu agentes causan frecuentemente la meningitis asptica del recin na-
cido?
Los virus coxsackie y ECHO
Cules son los tres exantemas benignos del recin nacido?
El eritema txico neonatorum, la miliaria y la melanosis pustulosa.
Qu es la bronquiolitis?
Es la infeccin del tracto respiratorio de los menores de 2 aos, que tiene
una incidencia mxima a los 6 meses.
Cul es la causa nmero uno de la bronquiolitis y la neumona en meno-
res de un ao?
El virus sincicial respiratorio. El tratamiento se lleva a cabo con rivavirina.
Cul es la imagen radiolgica de la bronquiolitis?
Hiperinflacin con reas de atelectasia.
Con qu enfermedad se asocia el uso de miel en la dieta de los nios?
Con el botulismo.
Qu es el sndrome de Reye?
Es la hepatitis fulminante provocada por utilizar cido acetilasaliclico
durante algunas infecciones virales.
Cul es la vasculitis que aparece en los nios despus de la faringitis esptrep-
toccica?
La prpura de Henoch-Schnlein.
Cul es la diferencia entre la candidiosis y la dermatitis del paal?
Las lesiones eczematosas son muy similares en ambas, pero la dermati-
tis del paal respeta los pliegues cutneos, a diferencia de la candidiosis.
Cmo se presenta un examema estafiloccico?
324
Con bulas y eritema superficiales.
Cul es el segundo tumor slido ms frecuente en la infancia?
El neuroblastoma. Se genera en la cresta neural y tiene una incidencia de 7%.
Cules son las dos presentaciones del neuroblastoma?
Abdominal (70% de los casos) y torcica.
Cundo se debe sospechar la presencia de un retinoblastoma en el recin
nacido?
Cuando el reflejo del fondo de ojo no tiene la coloracin roja habitual
(leukocoria).
Cules son los tipos de hidrocefalia?
Comunicante (por disminucin de la absorcin del lquido cefalorraqu-
deo) y no comunicante.
Cules son las causas de hidrocefalia no comunicante en el neonato?
Malformacin, infeccin y hemorragia intraventricular.
Cules son causas de hidrocefala no comunicante en el nio mayor?
Las masas ocupantes o tumores son las causas ms frecuentes.
Qu anomala cromosmica se asocia al tumor de Wilms?
La delecin del 11p13.
Qu es el sndrome de WAGR?
Es la nemotecnia para tumor de Wilms, Aniridia, malformacion Geni-
tourinaria y Retraso psicomotor.
Con qu se asocia el sndrome de Beckwith-Wiedemann?
Con tumor de Wilms y hemihipertrofia.
Cul es el tumor seo infantil ms frecuente?
El sarcoma osteognico.
Dnde aparece con ms frecuencia el sarcoma osteognico?
Es proximal a la rodilla (en el fmur distal y la tibia proximal).
Cul es la imagen radiolgica del sarcoma osteognico?
Se observa la imagen en rayos de sol.
Qu es el sarcoma de Ewing?
Es un sarcoma no diferenciado que aparece en los huesos.
Cul es la imagen radiolgica del sarcoma de Ewing?
325
La destruccin ltica es el signo ms frecuente, pero puede haber mlti-
ples capas de neoformacin reactiva de hueso subperistico que determi-
nan la imagen de piel de cebolla.

326
PSIQUIATRA







A qu se le llama cluster A de personalidad?
Al paranoide, esquizoide y esquizotpico. Son personas raras y excntri-
cas que guardan distancia emocional con otros.
A qu se le llama cluster B de personalidad?
Al antisocial, border, histrinico y narcisita. Son personas de conducta
impulsiva, excesivamente emocional y errtica.
A qu se le llama cluster C de personalidad?
Al evasivo, obsesivo-compulsivo y dependiente. Los rasgos ms consis-
tentes son de naturaleza ansiosa y temerosa.
Cundo se diagnostica un trastorno de ajuste?
Cuando hay evidencias clnicas de ansiedad o depresin que son clara-
mente secundarias a un estrs identificable, generalmente cuando los su-
cesos rebasan la capacidad de adaptacin del individuo.
Cules son las formas de presentacin del trastorno de ajuste?
Ansiedad, depresin, un sntoma fsico y el inicio de una aventura amo-
rosa o el consumo de alcohol o tabaco. Las respuestas subjetivas son el
temor (a que se repita el acontecimiento), la ira (por la frustracin) y la
pena (en cuanto a la falta de capacidad para afrontar el problema).
Cules son los trastornos de ansiedad?
Los principales son el trastorno de ansiedad generalizada, el trastorno de
angustia y el trastorno obsesivo-compulsivo.
Qu frmacos se utilizan para tratar los trastornos de ansiedad?
En el trastorno de ansiedad generalizada, las benzodiacepinas y la
buspirona constituyen los frmacos de eleccin.
En los casos de crisis de angustia pueden utilizarse benzodiacepinas e
inhibidores selectivos de la recaptura de serotina (SSRI).
327
En los casos de trastorno obsesivo-compulsivo, se utilizan frmacos se-
rotoninrgicos como la clorimipramina y la fluoxetina (SSRI). La buspi-
rona puede potenciar los efectos antiobsesivos de estos frmacos.
Cul es el mecanismo de accin de las benzodiacepinas?
Se unen a receptores especficos adjuntos al receptor de cido gamma-
aminobutrico (GABA) que aumentan la afinidad por este neurotransmi-
sor y hacen ms frecuente la apertura de los canales de cloro activados
por el GABA, lo que da como resultado final una hiperpolarizacin de la
clula y la inhibicin de la actividad neuronal (despolarizacin).
Cules son los cuatro usos clnicos de las benzodiacepinas?
Desrdenes de ansiedad, desrdenes musculares, crisis epilpticas y tras-
tornos del sueo.
Cul es la benzodiacepina indicada para tratar la angustia de separacin
patolgica?
El clonacepam.
Cul es la caracterstica del pensamiento paranoide?
Las ideas delirantes (creencia falsa).
En qu padecimiento son ms frecuentes las alucinaciones auditivas?
En la esquizofrenia.
En qu padecimiento son ms frecuentes las alucinaciones visuales?
En el trastorno mental orgnico.
Cules son los sntomas positivos en la esquizofrenia?
Alucinaciones auditivas, delirio y trastornos del pensamiento formal.
Cul es el tratamiento de eleccin de la esquizofrenia?
El haloperidol es el primer frmaco, seguido por la clozapina, en los ca-
sos de refractariedad o cuando se presentan extrapiramidalismo o disci-
nesias tardas.
Qu es la discinesia tarda?
Es la aparicin de corea o tics de manera secundaria al uso de neurolpticos.
Qu es el delirio?
Es un trastorno global transitorio de la atencin.
Qu es la demencia?
Es un trastorno caracterizado por disfuncin crnica de las funciones
mentales superiores.
328
Cules son las demencias frontoparietal y frontotemporal?
La demencia de Pick y la de Alzheimer, respectivamente.
Cules son los sntomas negativos?
Inhabilidad de relaciones personales, pobreza de habla y afecto restringido.
Qu frmacos son inhibidores selectivos de la recaptura de serotonina
(SSRI)?
La fluoxetina, paroxetina y sertralina.
Cul es el antidepresivo utilizado en los casos de trastorno de pnico?
La paroxetina.
Qu frmacos pertenecen al grupo de las fenotiacinas?
La clorpromacina, flufenacina, tioridacina y trifluoperacina.
Sobre qu receptores acta la clorpromacina?
Sobre receptores muscarnicos, alfa-adrenrgicos e histamnicos (H-1).
Qu otros antipsicticos se usan comnmente?
El haloperidol (butirofenona) la risperidona (benzisoxazol).
Cmo actan las fenotiacinas y las butirofenonas?
Son bloqueadores de receptores de dopamina tipo 2 (D2)
Cmo actan la risperidona y la clozapina (dibenzotiacepina)?
Es un bloqueador de receptores de dopamina tipo 4 (D4).
Qu efectos produce el bloqueo central dopa?
Puede causar hipersecrecin de las hormonas de la hipfisis anterior.
Cmo acta la risperidona?
Es bloqueador D2 y de la serotonina (5-HT
2)
.
Cmo acta la olanzapina (tienbenzodiacepina)?
Es un bloqueador 5HT
2
, D1, D2, D4, y bloqueador muscarnico.
Cul es el orden de potencia de los antipsicticos?
En orden decreciente: haloperidol, tiotixeno, clorpromacina, tioridacina
y clozapina.
Qu es el sndrome neurolptico maligno?
Es un estado similar a la catatonia, con aumento de la presin, signos ex-
trapiramidales e hiperpirexia.
Qu frmacos son tiles para tratar el sndrome neurolptico maligno
(SNM)?
Los agonistas dopa (bromocriptina y amantadina).
http://booksmedicos.org
329
Cules son las indicaciones de la amantadina?
Profilaxis contra el virus de la influenza.
Enfermedad de Parkinson (por ser agonista Dopa).
Esquizofrenia.
Qu otras complicaciones aparte del SNM provoca el tratamiento antipsi-
ctico?
Distona aguda, parkinsonismo y discinesia tarda.
A qu se llama parkinsonismo?
Al cuadro clnico constituido por facies de mscara, apata, dificultad pa-
ra iniciar (o lentitud de) los movimientos.
Qu padecimientos se asocian con depresin?
La migraa, la epilepsia, el hipotiroidismo y la insuficiencia adrenal.
Qu exmenes de laboratorio se realizan en el estudio de un paciente con
depresin?
Pruebas de funcionamiento tiriodeo, prueba de supresin con dexameta-
sona y pruebas de funcionamiento heptico.
Cmo se interpreta la prueba con dexametasona en los pacientes que pa-
decen depresin?
La prueba es altamente sensible y utilizada para diagnosticar la depresin
mayor. En los pacientes deprimidos no hay supresin (prueba positiva), y
en estos casos se prev que habr mejor respuesta a los frmacos o al tra-
tamiento electro-convulsivo. Una prueba negativa no descarta la posibili-
dad de depresin.
Cmo se afecta la secrecin de hormona estimulante del tiroides (TSH) en
los casos de depresin?
Cuando existe depresin mayor, la liberacin de TSH tras la administra-
cin de factor liberador hipotalmico est disminuida o lentificada.
Cmo se presenta la depresin atpica?
Puede haber hipersomnia, ingestin excesiva de alimentos, hipersensibi-
lidad al rechazo y letargia.
Cules son los tres tipos de antidepresivos?
Tricclicos, SSRI e inhibidores de la monoaminooxidasa (IMAO).
Cmo actan los antidepresivos tricclicos?
Son bloqueadores de la recaptura de serotonina y noradrenalina (a dife-
rencia de los SSRI); por tanto, tienen efectos catecolaminrgicos. Secun-
330
dariamente son bloqueadores de los receptores de acetilcolina, por lo que
causan efectos anticolinrgicos.
Cul es el frmaco de eleccin para el tratamiento de los trastornos bipo-
lares?
El litio.
Qu frmacos se utilizan para tratar un episodio manaco?
Los neurolpticos, las benzodiacepinas y el litio.
Cules son los efectos adversos del litio?
Disminuye la sensibilidad renal a hormona antidiurtica, por lo que
causa poliuria y polidipsia.
Sntomas gastrointestinales.
Tremor fino.
Cmo es el paciente obsesivo compulsivo?
Inteligente, organizado y concienzudo.
Cules son los estados disociativos?
Las fugas, el sonambulismo, la personalidad mltiple y la despersonali-
zacin.
Qu efectos producen la sobredosis de ansiolticos?
Depresin respiratoria, hipotensin y coma.
Cul es la triada indicativa del abuso de sustancias?
Dependencia psicolgica, dependencia fisiolgica y tolerancia.
Qu parmetros de laboratorio estn alterados en los pacientes alcohlicos?
Aumento del volumen corpuscular medio, elevacin de las sustancias
que abarcan las pruebas de funcionamiento heptico, incremento de los
triglicridos y aumento del cido rico srico.
Cundo aparece el delirium tremens despus de la supresin alcohlica?
Despus de 25 a 72 horas sin ingerir bebidas alcohlicas.
Cul es la triada del sndrome de Wernicke-Korsakoff?
Confusin, ataxia y alteracin del VI par.
Cules son los efectos de la nicotina?
Aumenta el nivel srico de noradrenalina y serotonina.
Qu frmaco es un auxiliar en el tratamiento del tabaquismo?
El antidepresivo bupropiona, a dosis de 150 a 300 mg/da.
331
Cundo se inicia el tratamiento del sndrome de supresin de narcticos?
Cuando hay sntomas de grado 2 (midriasis, piloereccin y alteraciones
cardiovasculares), con 10 miligramos de metadona. Puede utilizarse clo-
nidina para controlar los efectos cardiovasculares.
Por qu el abuso de cocana causa destruccin del tabique nasal?
Porque provoca vasoconstriccin prolongada.
Cules son las complicaciones producidas por el consumo de cocana?
El colapso cardiovascular, el infarto, las arritmias y el ataque isqumico
transitorio (TIAS).
























332
COMPENDIO DE SNDROMES Y ENFERMEDADES








Alcaptonuria: defecto innato de la cido homogentsico-oxidasa.
Ocronosis: depsitos de cido homogentsico en los cartlagos, que los ha-
cen verse obscuros.
Homocistinuria: defecto autosmico recesivo de la cistationina-beta-
sintasa que se presenta entro los 10 a los 20 aos, con trombosis arte-
riales y venosas sin factores de riesgo.
Enfermedad de Gaucher: autosmica recesiva + deficiencia de beta-
glucocerebrosidasa + anemia + trombocitopenia + crisis seas simila-
res a las de la anemia de clulas falciformes + hiperesplenismo + c-
lulas de Gaucher en la mdula sea.
Enfermedad de Hand-Schuller-Christian: granuloma eosinoflico de la hi-
pfisis posterior + diabetes inspida.
Enfermedad de Huntington: corea + demencia + cromosoma 4 afectado.
Enfermedad de Osler-Weber-Rendu: hemorragia-telangiectasia hereditaria
asociada a los genes HHT 1 y 2.
Enfermedad de von Gierke: glucogenosis tipo IA que afecta la glucosa-6-
fosfatasa.
Enfermedad de von Hippel-Lindaw: Sndrome neurocutneo que consiste
en hemangioblastomatosis retinocerebelar + feocromocitoma + Ca
de clulas renales + neoplasia de islotes.
Enfermedad de von Willebrand: anomala cuantitativa (Tipo I) o cualitati-
va (tipo II) del factor de Von Willebrand secretado por el endotelio
vascular.
Enfermedad de Wiscott-Aldrich: Inmunodeficiencia + trombocitopenia.
333
Esclerosis mltiple: zonas de gliosis (cicatrizacin) en la sustancia blanca
enceflica y espinal + sntomas sensoriales (hormigueo, entumeci-
miento) y neuromusculares episdicas (paraparesia espstica, diplo-
pa) + etiologa autoinmune.
Macroglobulinemia de Waldenstrm: malignidad de clulas B + IgM ele-
vada + sndrome de hiperviscosidad + eritrocitos apilados (rouleaux).
Sndrome de ataxia-telangiectasia: deficiencia de clulas T y B + dermati-
tis + deterioro neurolgico.
Sndrome de Beckwidth-Wiedemann: hemihipertrofia corporal + tumor de
Wilms.
Sndrome de Churg-Strauss: es una variante de la poliarteritis nodosa con
predileccin por el pulmn. El cuadro est dominado por una granu-
lomatosis alrgica pulmonar, asma y eosinofilia.
Sndrome de CREST: sndrome que viene de las siglas en ingls: Calcinosis,
Raynaud, Esofagic disorder, Sclerodactilia y Telangiectasia.
Sndrome de Cronkhite-Canada: poliposis adenomatosa familiar (FAP) +
alopecia + hiperpigmentacin + aplasia de uas.
Sndrome de Diamond-Blackfan: aplasia pura de eritrocitos.
Sndrome de DiGiorge: aplasia tmica y de clulas B + facies caracterstica
+ anomala del arco artico + enfermedad cardiaca + hipocalcemia.
Sndrome de Felty: artritis reumatoide + esplenomegalia + granulocitope-
nia.
Sndrome de Gardner: FAP + osteomas + tumores de tejidos blandos.
Sndrome de Gilles de la Tourrete: mltiples tics motores y fnicos +
inicio antes de los 21 aos + evolucin progresiva.
Sndrome de Horner: miosis, ptosis, enoftalmos y prdida de la sudacin
del lado afectado por lesin del simptico cervical.
Sndrome de Nelson: adenoma hipofisario posadrenalectoma bilateral.
Sndrome de Peutz-Jeghers: FAP + hipopigmentacin de mucosas + pli-
pos hamartomatosos.
Sndrome de Plummer: tirotoxicosis + bocio nodular.
Sndrome de Plumer-Vinson: Adherencias esofgicas en el tercio proxi-
mal, con disfagia secundaria a ferropenia.
334
Sndrome de Rokitansky-Kunster-Hauser: anomala mlleriana caracteri-
zada por agenesia del tero y anexos.
Sndrome de Sipple (neoplasia, endcrina mltiple tipo II-A): adenoma ti-
roideo + feocromocitoma + hiperparatiroidismo.
Sndrome de Trousseau: tromboembolia pulmonar, trombosis venosa su-
perficial, trombosis venosa profunda y endocarditis marntica.
Sndrome de Turcot: FAP + tumores malignos del sistema nervioso cen-
tral.
Sndrome WAGR: Wilms + Aniridia + Genitourinarias + Retraso.
Sndrome de Verter-Morrison: Es el VIPoma o clera pancretica. Diarrea
+ hipokalemia + aclorhidria.
Sndrome de Werner (noeplasia endcrina mltipletipo I): Adenoma pitui-
tario + adenoma pancretico + adenoma paratiroideo
335
336
GLOSARIO DE ABREVIATURAS







5HT: 5 hidroxitriptamina (serotonina)
1AT: alfa 1-antitripsina.
FP: alfa-fetoproteina.
A5-ASA: cido 5-aminosaliclico.
ABVD: rgimen quimioteraputico. Son las siglas para Adriamicina (do-
xorrubicina), Bleomicina, Vincristina y Dacarbacina.
Ac: anticuerpo.
ACF: anemia de clulas falciformes.
ACP: fosfatasa cida.
AF: angiografa fluorescenica.
Ag: antgeno.
ADH: hormona antidiurtica.
ALP: fosfatasa alcalina.
ALT: aminotransferasa de alanina.
ANA: anticuerpos antincleo.
ANCA: anticuerpos anticitoplasma de neutrfilo.
APE: antgeno prosttico especfico.
APO: apolipoprotena.
AR: artritis reumatoide.
ARJ: artritis reumatoide juvenil.
AS: anemia sideroblstica.
ASA: cido acetilsaliclico.
ASCA: anticuerpos antisacaromices.
AST: aminotransferasa de aspartato.
ATLPC: angioplasta transluminal percutnea.
337
ATR: acidosis tubular renal.
BAAF: biopsia por aspiracin con aguja fina.
BCG: bacilo de Calmette y Guerin.
BRQT: braquiterapia.
HGC: gonadotropina corinica humana.
CBP: cirrosis biliar primaria.
CCHNP: carcinoma colorrectal hereditario no polipoide.
CCK: colecistocinina.
CCP: cncer de clulas pequeas.
CID: coagulacin intravascular diseminada.
CMCH: concentracin media de hemoglobina corpuscular.
CMV: citomegalovirus.
CNS: sistema nerviosos central.
CPS: ciclos por segundo.
CRF: factor liberador de cortisol.
CRP: proteina C Reactiva.
CPRP: colangiografa pancretica retrgrada endoscpica.
CUCI: colitis ulcerativa crnica inespecfica.
DEA: dehidroepiandrosterona.
DFH: difenilhidantoinato.
DG6PD: deficiencia de glucosa-6-fosfato deshidrogenasa.
DLCO
2
: capacidad de difusin del CO
2.

DPKA: deficiencia de piruvatocinasa.
EA: espondilitis anquilosante.
EAP: enfermedad acidopptica.
EBHA: estreptococo beta-hemoltico del grupo A de Lancefield.
EEG: electroencefalograma.
EEI: esfnter esofgico inferior.
EF: exploracin fsica.
EH: enfermedad de Hodgkin.
EL: sndrome de Eaton-Lambert
ELA: esclerosis lateral amiotrfica.
EM: esclerosis mltiple.
ERCP: colangiografia pancretica retrograda endoscpica.
338
ETEC: Escherichia coli enterotoxignica.
EVC: enfermedad vascular cerebral.
FA: fibrilacin auricular.
FAL: fosfatasa alcalina leucocitaria.
FAP: poliposis adenomatosa familiar.
FC: frecuencia cardiaca.
FENa: fraccin excretada de sodio.
FEV1: volumen espiratorio forzado del primer segundo.
FIO2: fraccin inspirada de oxgeno.
FNT: factor de necrosis tumoral alfa.
FR: factor reumatoide.
FSH: hormona folculoestimulante.
FVC: capacidad vital forzada.
FVW: factor VonWillebrand.
GI: gastrointestinal.
GTP: gammaglutamiltranspeptidasa.
GM-CSF: factor estimulante de colonias granulocito-monocito.
GN: glomerulonefritis.
GnRH: hormona liberadora de gonadotropinas.
HAIN: hidracida del cido nicotnico (isoniacida).
Hb: hemoglobina.
HCM: hemoglobina corpuscular media.
HE: histerectoma extrafascial.
HIV: virus de la inmunodeficiencia humana.
HLA: antgeno leucocitario humano.
HMWK: ciningeno de alto peso molecular.
HPB: hiperplasia prosttica benigna.
HPTH: hiperparatiroidismo.
HTDA: hemorragia del tubo digestivo alto.
Hto: hematocrito.
HUD: hemorragia uterina disfuncional.
IAM: infarto agudo del miocardio.
IGF-1: factor de crecimiento semejante a la insulina.
339
IMAO: inhibidores de la monoamiooxidasa.
IRI: radioinmunoanlisis de insulina.
ITL: ndice de tiroxina libre.
KDa: kilodaltones.
KS: cetosteroides.
LATS: estimulador tiroideo de larga accin.
LB: linfocito B.
LCR: lquido cefalorraqudeo.
LES: lupus eritematoso sistmico.
LED: lupus eritematoso discoide.
LH: hormona luteinizante.
LHRH: hormona liberadora de hormona luteinizante.
LLA: leucemia linfoctica aguda.
LLC: leucemia linfoctica crnica.
LMA: leucemia mieloctica aguda.
LMC: leucemia mielocitica crnica.
LNH: linfoma no Hodgkin.
Lpm: latidos por minuto.
LPP: linfadenectoma plvica y parartica.
LPL: lipoprotenlipasa.
LT: linfocito T.
MHC: complejo mayor de histocompatibilidad.
MO: mdula sea.
MODY: mature onset diabetes of the young (diabetes juvenil de inicio tar-
do).
MOPP: rgimen quimioteraputico. Son las siglas para Mecloretamina, On-
covin (vincristina), Procarbacina y Prednisona.
NEM: neoplasia endocrina mltiple.
NPS: ndulo pulmonar solitario.
NTA: necrosis tubular aguda.
OA: osteoartritis.
OACR: oclusin de la arteria central de la retina.
OHCS: hidroxicorticosteroides.
OHKS: hidroxicetosteroides.
340
PAF: poliposis adenomatosa familiar.
PB19: parvovirus B19.
PCA: persistencia del conducto arterioso.
PCr: creatinina plasmtica.
PCR: reaccin en cadena de polimerasa.
PFC: plasma fresco congelado.
PFH: pruebas de funcin heptica.
PGE: prostaglandina E.
PGI
2
: proastaglandina I2, prostaciclina.
PHS: prpura de Henoch-Schnlein.
PIG: periodo intergensico.
PL: puncin lumbar.
PM: peso molecular.
PMN: polimorfonuclear.
PNa: sodio plasmtico.
POMC: proopiomelanocortina.
PPD: derivado proteico purificado.
PTI: prpura trombocitopnica idioptica.
PTOG: prueba de tolerancia oral a la glucosa.
RBC: clula roja de la sangre (eritrocito).
RCIU: retraso del crecimiento intrauterino.
RCT: receptor de clula T.
RMN: resonancia magntica nuclear.
RPR: prueba de reagina rpida.
RT: radioterapia.
RTU: reseccin transuretral.
RVR: respuesta ventricular rpida.
Sat Hb: saturacin de hemoglobina.
SDR: sndrome de distrs respiratorio.
SHU: sndrome hemoltico urmico.
SIADH: sndrome de secrecin inadecuada de hormona antidiurtica.
SNC: sistema nervioso central.
SNM: sndrome neurolptico maligno.
SOB: salpingooforectoma bilateral.
341
SSRI: inhibidores selectivos de la recaptura de serotonina.
TAC: tomografa axial computada.
TAM: tensin arterial media.
TCE: traumatismo craneoenceflico.
TCS: tejido celular subcutneo.
TDA: trastorno de dficit de atencin.
TEP: tromboembolia pulmonar.
TGV: trasposicin de los grandes vasos.
TIAS: ataque isqumico transitorio.
TTRN: taquipnea transitoria del recin nacido.
TXA
2
: tromboxano A2.
T
3
RU: ndice de recaptura de T
3.

TAD: trastorno autosmico dominante.
TEP: tromboembolia pulmonar.
UCr: creatinina urinaria.
UDPG: uridindifosfoglicerasa.
UNa: sodio urinario.
VCM: volumen corpuscular medio.
VEB: virus de Epstein-Barr.
VFG: velocidad de filtracin glomerular.
VIP: pptido intestinal vasoactivo.
VMA: acido vanililmandlico.
VSR: virus sincicial respiratorio.
342
BIBLIOGRAFA


PRIMERA PARTE

Arber, Hctor. Desarrollo de la memoria. Curso interactivo en CD. ILVEM,
Mxico.
Arber, Hctor. Aprenda a aprender. Curso interactivo en CD. ILVEM, Mxico.
Arber, Hctor. Lectura veloz. Curso interactivo en CD. ILVEM, Mxico.
Bouisquie, Georges. Cmo organizar su trabajo. Segunda edicin. Ediciones
Deusto, Espaa, 1972.
Buzan, Tony. Cmo utilizar su mente con un mximo rendimiento. Segunda edi-
cin. Planeta Mexicana, Mxico, 1991.
Goleman, Daniel. La inteligencia emocional. 37 edicin. Vergara, Mxico,
2003.
Hernndez Santiago, Ren Gastn. El xito en tus estudios. Segunda edicin.
Editorial Trillas, Mxico, 1982
Ibarra, Luz Mara. Aprende mejor con gimnasia cerebral. Octava edicin. Garnik
Ediciones, Mexico, 1999.
James, Muriel. Nacidos para triunfar: anlisis transaccional con experimentos Ges-
talt. Primera edicin. Fondo Educativo Interamericano S.A., Mxico,
1976.
Segal, Jeanne. Su inteligencia emocional aprenda a incrementarla y usarla. Primera
edicin. Grijalbo Mondadori, Barcelona, 2001.
Silva, Jos. El mtodo Silva de control mental. Sptima edicin. Editorial Diana,
Mxico, 1994

SEGUNDA PARTE

Adler, Stephen N. A Pocket Manual of Differential Diagnosis. Cuarta edicin.
Lipincott Williams & Wilkins, Philadelphia, Pennsylvania, 2000.
Barton, Thomas K. Appleton & Lange Repaso para el USMLE Paso 1. Tercera
edicin. McGraw Hill, Mxico, 2001.
343
Beck, William W. NMS Obstetrics and Gynecology. Cuarta edicin. Lipincott
Williams & Wilkins, Philadelphia, Pennsylvania, 2000
Behrman, Kliegman. Nelson Compendio de Pediatra. Tercera edicin. McGraw
Hill, Espaa, 1999.
Berkow Robert. El Manual Merck. Novena edicin. Harcourt Brace, Espaa,
1997.
Chan, Carlyle H. Appleton & Lange Repaso para el USMLE Paso 2. Tercera edi-
cin, McGraw Hill, Mxico, 2001.
Dworkin, Paul H. NMS Pediatrics. Cuarta edicin. Lipincott Williams & Wil-
kins, Philadelphia, Pennsylvania, 2000
Fauci, Anthony S. Harrisons Principles of Internal Medicine. 14 edicin.
McGraw Hill, Estados Unidos, 1998,
INNSZ. Manual de Teraputica Mdica y Procedimientos de Urgencias. Cuarta
edicin. McGraw Hill, Mxico, 2000.
Jarrell, Bruce E. NMS surgery. Cuarta edicin. Lipincott Williams & Wilkins,
Philadelphia, Pennsylvania, 2000
Lederman, Robert J. Tarascon internal medicine & critical care pocketbook. Se-
gunda edicin. Tarascon Publishing, Estados Unidos, 2001.
Mycek, Mary J. Lipincotts ilustrated rewiews: pharmacology. Segunda edicin.
Lipincott Williams & Wilkins, Philadelphia, Pennsylvania, 2000.
Wyers, Allen R. NMS medicine. Cuarta edicin. Lipincott Williams & Wil-
kins, Philadelphia, Pennsylvania, 2000.
Rolak, Loren A. Secretos de la neurologa. Segunda edicin. McGraw Hill, M-
xico, 2000.
Surs Batll, Juan. Semiologa mdica y tcnica eploratoria. Sptima edicin.
Masson S.A., Barcelona, 1998.
Sodi Pallares, D. Electrocardiografa clnica anlisis deductivo. Primera edicin,
Mndez Editores, Mxico, 2000.
Tierney, Lawrence M. Diagnstico clnico y tratamiento. 35 edicin. Manual
Moderno, Mxico, 2000.
Wallach, Jacques, Interpretation of diagnostic rests. Sptima edicin. Lipincott
Williams & Wilkins, Philadelphia, Pennsylvania, 2000.
Zollo, Anthony J. Secretos de la medicina. Segunda edicin. McGraw Hill In-
teramericana, Mxico, 2001.
344
NDICE



Presentacin 5

Parte I 9

Captulo 1/ Introduccin 11
Captulo 2/ Aspectos tcnicos 19
Visualizacin y lectura veloz 19
Actividad elctrica y memoria 22
Inteligencia emocional 28
Economa del rendimiento 30
Captulo 3/ Un programa de estudios 31
Planeacin de la sesin de estudio 32
Planeacin a largo plazo 35
Algunas cosas que es preciso considerar 37

Parte 2 41

Cardiologa 43
Ciruga 63
Dermatologa 73
Endocrinologa 77
Gastroenterologa 91
Ginecologa y obstetricia 105
Hematologa 115
Infectologa 137
Inmunologa y reumatologa 159
Intoxicaciones y accidentes 181
Uronefrologa 187
Neumologa 201
345
Neurologa 215
Nutricin, metabolismo, lquidos y electrolitos 235
Oftalmologa 249
Oncologa 255
Otorrinolaringologa 271
Pediatra y gentica 275
Psiquiatra 299
Compendio de sndromes y enfermedades 305
Glosario de abreviaturas 309
Bibliografa 315

346













Manual para Examen Nacional
de Residencias Mdicas
de Carlos Gonzlez Parra
se termin de imprimir en agosto de 2005
en los talleres de Formacin Grfica S.A. de C.V.
con domicilio en Matamoros 112,
col. Ral Romero de Ciudad Nezahualcyotl,
Estado de Mxico y con nmero de telfono 57 97 60 60.
En la edicin, la composicin y el diseo son de Jos Luis Olazo Garca,
la revisin tcnica de Orlando Escrcega Alarcn y el cuidado de la DGFE.
El tiraje consta de 1000 ejemplares.

You might also like